Goldman's Cecil Medicine (Vol 1-2), 25th Edition [PDF]

  • 0 0 0
  • Gefällt Ihnen dieses papier und der download? Sie können Ihre eigene PDF-Datei in wenigen Minuten kostenlos online veröffentlichen! Anmelden
Datei wird geladen, bitte warten...
Zitiervorschau

QUICK REFERENCE (QR) VIDEO ACCESS The images below are QR codes. Each code corresponds to a video from the Goldman-Cecil Medicine 25 collection. For fast and easy video access, right from your mobile device, follow these instructions. The videos are also available on Expertconsult.com.

What You Need

• A mobile device, such as a smartphone or tablet, equipped with a camera and Internet access • A QR code reader application (If you do not already have a reader installed on your mobile device, look for free versions in your app store.)

How It Works

• Open the QR code reader application on your mobile device. • Point the device’s camera at the code and scan. • Each code opens an individual video player for instant viewing—no log-on required.

Confusion Assessment Method (CAM) Chapter 28, Video 28-1 – Marcos Mialnez, Jorge G. Ruiz, and Rosanne M. Leipzig

Standard Echocardiographic Views: Four-Chamber Image Plane Chapter 55, Video 55-1D – Catherine M. Otto

Interlaminar Epidural Steroid Injection Chapter 30, Video 30-1 – Ali Turabi

Dilated Cardiomyopathy: Long Axis View Chapter 55, Video 55-2A – Catherine M. Otto

Standard Echocardiographic Views: Long Axis Image Plane Chapter 55, Video 55-1A – Catherine M. Otto

Dilated Cardiomyopathy: Short Axis View Chapter 55, Video 55-2B – Catherine M. Otto

Standard Echocardiographic Views: Short Axis Image Plane Chapter 55, Video 55-1B – Catherine M. Otto

Dilated Cardiomyopathy: Apical Four-Chamber View Chapter 55, Video 55-2C – Catherine M. Otto

Standard Echocardiographic Views: Short Axis Image Plane Chapter 55, Video 55-1C – Catherine M. Otto

Three-Dimensional Echocardiography Chapter 55, Video 55-3 – Catherine M. Otto

4

Quick Reference (QR) Video Access

Stress Echocardiography: Normal Reaction Chapter 55, Video 55-4A – Catherine M. Otto

Perimembranous Ventricular Septal Defect Chapter 69, Video 69-2 – Ariane J. Marelli

Stress Echocardiography: Normal Reaction Chapter 55, Video 55-4B – Catherine M. Otto

Coronary Stent Placement Chapter 74, Video 74-1 – Paul S. Teirstein

Stress Echocardiography: Proximal Stenosis of the Left Anterior Descending Coronary Artery Chapter 55, Video 55-4C – Catherine M. Otto

Guidewire Passage Chapter 74, Video 74-2 – Paul S. Teirstein

Stress Echocardiography: Proximal Stenosis of the Left Anterior Descending Coronary Artery Chapter 55, Video 55-4D – Catherine M. Otto

Delivering the Stent Chapter 74, Video 74-3 – Paul S. Teirstein

Pericardial Effusion: Parasternal Long Axis Chapter 55, Video 55-5A – Catherine M. Otto

Inflating the Stent Chapter 74, Video 74-4 – Paul S. Teirstein

Pericardial Effusion: Parasternal Short Axis Chapter 55, Video 55-5B – Catherine M. Otto

Final Result Chapter 74, Video 74-5 – Paul S. Teirstein

Pericardial Effusion: Apical Four-Chamber Views Chapter 55, Video 55-5C – Catherine M. Otto

Superficial Femoral Artery (SFA) Stent Procedure Chapter 79, Video 79-1 – Christopher J. White

Secundum Atrial Septal Defect Chapter 69, Video 69-1 – Ariane J. Marelli

Orthotopic Bicaval Cardiac Transplantation Chapter 82, Video 82-1 – Y. Joseph Woo

Quick Reference (QR) Video Access

Wheezing Chapter 87, Video 87-1 – Jeffrey M. Drazen

Endoscopic Mucosal Resection Using Saline Lift Polypectomy of a Colon Adenoma Followed by Closure of the Mucosal Defect with Clips Chapter 193, Video 193-3 – Douglas O. Faigel

VATS Wedge Resection Chapter 101, Video 101-1 – Malcolm M. DeCamp

Endoscopic View of Rectal Cancer Chapter 193, Video 193-4 – Douglas O. Faigel

Ventilation of an Ex Vivo Rat Lung Chapter 105, Video 105-1 – Arthur S. Slutsky, George Volgyesi, and Tom Whitehead

Endoscopic Ultrasound Chapter 193, Video 193-5 – Douglas O. Faigel

Renal Artery Stent Chapter 125, Video 125-1 – Renato M. Santos and Thomas D. DuBose, Jr.

Laparoscopic Roux-en-Y Gastric Bypass Chapter 220, Video 220-1 – James M. Swain

Interpretation of a Computed Tomographic Colonography Chapter 133, Video 133-1 – David H. Kim

Pituitary Surgery Chapter 224, Video 224-1 – Ivan Ciric

Donor Liver Transportation–Donor and Recipient Chapter 154, Video 154-1 – Igal Kam, Thomas Bak, and Michael Wachs

Skin Testing Chapter 251, Video 251-1 – Larry Borish

Snare Polypectomy of a Colon Adenoma Chapter 193, Video 193-1 – Douglas O. Faigel

Nasal Endoscopy Chapter 251, Video 251-2 – Larry Borish

Laparascopic-Assisted Double Balloon Enteroscopy with Polypectomy of a Jejunal Adenoma Followed by Surgical Oversew of the Polypectomy Site Chapter 193, Video 193-2 – Douglas O. Faigel

Hip Arthroscopy Osteochondroplasty Chapter 276, Video 276-1 – Bryan T. Kelly

5

6

Quick Reference (QR) Video Access

Cervical Provocation Chapter 400, Video 400-1 – Richard L. Barbano

Left Rolandic Seizure Chapter 403, Video 403-2 – Samuel Wiebe

Spurling Maneuver Chapter 400, Video 400-2 – Richard L. Barbano

Left Temporal Complex Partial Seizure Chapter 403, Video 403-3 – Samuel Wiebe

Cervical Distraction Test Chapter 400, Video 400-3 – Richard L. Barbano

Left Temporal Complex Partial Seizure Postictal Confusion Chapter 403, Video 403-4 – Samuel Wiebe

Straight Leg Raise Chapter 400, Video 400-4 – Richard L. Barbano

Left Temporal Complex Partial Seizure Chapter 403, Video 403-5 – Samuel Wiebe

Contralateral Straight Leg Raise Chapter 400, Video 400-5 – Richard L. Barbano

Supplementary Sensory-Motor Seizure Chapter 403, Video 403-6 – Samuel Wiebe

Seated Straight Leg Raise Chapter 400, Video 400-6 – Richard L. Barbano

Right Posterior Temporal Seizure - Dramatic Frontal Semiology Chapter 403, Video 403-7 – Samuel Wiebe

Discectomy Chapter 400, Video 400-7 – Jason H. Huang

Right Mesial Frontal Seizure Chapter 403, Video 403-8 – Samuel Wiebe

Absence Seizure Chapter 403, Video 403-1 – Samuel Wiebe

Nonconvulsive Status Epilepticus Chapter 403, Video 403-9 – Samuel Wiebe

Quick Reference (QR) Video Access

GTC Seizure Tonic Phase Chapter 403, Video 403-10 – Samuel Wiebe

Minimally Conscious State Chapter 404, Video 404-3 – James L. Bernat and Eelco F. M. Wijdicks

GTC Seizure Clonic Phase Chapter 403, Video 403-11 – Samuel Wiebe

Akinetic Mutism Chapter 404, Video 404-4 – James L. Bernat and Eelco F. M. Wijdicks

Myoclonic Facial Seizure Chapter 403, Video 403-12 – Samuel Wiebe

Early Parkinson’s Disease Chapter 409, Video 409-1 – Anthony E. Lang

Tonic Seizure Lennox Gastaut Chapter 403, Video 403-13 – Samuel Wiebe

Freezing of Gait in Parkinson’s Disease Chapter 409, Video 409-2 – Anthony E. Lang

Atonic Seizure Lennox Gastaut Chapter 403, Video 403-14 – Samuel Wiebe

Gunslinger Gait in Progressive Supranuclear Palsy Chapter 409, Video 409-3 – Anthony E. Lang

Reflex Auditory Seizure Chapter 403, Video 403-15 – Samuel Wiebe

Supranuclear Gaze Palsy in Progressive Supranuclear Palsy Chapter 409, Video 409-4 – Anthony E. Lang

Four Score Chapter 404, Video 404-1 – James L. Bernat and Eelco F. M. Wijdicks

Applause Sign in Progressive Supranuclear Palsy Chapter 409, Video 409-5 – Anthony E. Lang

Persistent Vegetative State Chapter 404, Video 404-2 – James L. Bernat and Eelco F. M. Wijdicks

Apraxia of Eyelid Opening in Progressive Supranuclear Palsy Chapter 409, Video 409-6 – Anthony E. Lang

7

8

Quick Reference (QR) Video Access

Cranial Dystonia in Multiple System Atrophy Chapter 409, Video 409-7 – Anthony E. Lang

Hemiballism Chapter 410, Video 410-3 – Anthony E. Lang

Anterocollis in Multiple System Atrophy Chapter 409, Video 409-8 – Anthony E. Lang

Blepharospasm Chapter 410, Video 410-4 – Anthony E. Lang

Stridor in Multiple System Atrophy Chapter 409, Video 409-9 – Anthony E. Lang

Oromandibular Dystonia Chapter 410, Video 410-5 – Anthony E. Lang

Alien Limb Phenomenon in Corticobasal Syndrome Chapter 409, Video 409-10 – Anthony E. Lang

Cervical Dystonia Chapter 410, Video 410-6 – Anthony E. Lang

Myoclonus in Corticobasal Syndrome Chapter 409, Video 409-11 – Anthony E. Lang

Writer’s Cramp Chapter 410, Video 410-7 – Anthony E. Lang

Levodopa-Induced Dyskinesia in Parkinson’s Disease Chapter 409, Video 409-12 – Anthony E. Lang

Embouchure Dystonia Chapter 410, Video 410-8 – Anthony E. Lang

Essential Tremor Chapter 410, Video 410-1 – Anthony E. Lang

Sensory Trick in Cervical Dystonia Chapter 410, Video 410-9 – Anthony E. Lang

Huntington’s Disease Chapter 410, Video 410-2 – Anthony E. Lang

Generalized Dystonia Chapter 410, Video 410-10 – Anthony E. Lang

Quick Reference (QR) Video Access

Tics Chapter 410, Video 410-11 – Anthony E. Lang

Limb Symptoms and Signs Chapter 419, Video 419-1 – Pamela J. Shaw

Tardive Dyskinesia Chapter 410, Video 410-12 – Anthony E. Lang

Bulbar Symptoms and Signs Chapter 419, Video 419-2 – Pamela J. Shaw

Hemifacial Spasm Chapter 410, Video 410-13 – Anthony E. Lang

Normal Swallowing Chapter 419, Video 419-3 – Pamela J. Shaw

Wernickes Encephalopathy Eye Movements: Before Thiamine Chapter 416, Video 416-1 – Barbara S. Koppel

Charcot-Marie-Tooth Disease Exam and Walk Chapter 420, Video 420-1 – Michael E. Shy

Wernickes Encephalopathy Eye Movements: After Thiamine Chapter 416, Video 416-2 – Barbara S. Koppel

9

This page intentionally left blank

GOLDMAN-CECIL MEDICINE

This page intentionally left blank

GOLDMAN-CECIL MEDICINE 25TH EDITION Volume II

EDITED BY

LEE GOLDMAN, MD

Harold and Margaret Hatch Professor Executive Vice President and Dean of the Faculties of Health Sciences and Medicine Chief Executive, Columbia University Medical Center Columbia University New York, New York

ANDREW I. SCHAFER, MD

Professor of Medicine Director, Richard T. Silver Center for Myeloproliferative Neoplasms Weill Cornell Medical College New York, New York

1600 John F. Kennedy Blvd. Ste. 1800 Philadelphia, PA 19103-2899

GOLDMAN-CECIL MEDICINE, 25TH EDITION

ISBN: 978-1-4557-5017-7 Volume 1 Part Number: 9996096564 Volume 2 Part Number: 9996096629

International Edition (IE):

ISBN: 978-0-323-28800-2 IE Volume 1 Part Number: 9996118347 IE Volume 2 Part Number: 9996118282

Copyright © 2016, 2012, 2008, 2004, 2000, 1996, 1991, 1988, 1982, 1979, 1975, 1971, 1963, 1959, 1955 by Saunders, an imprint of Elsevier Inc. Copyright 1951, 1947, 1943, 1940, 1937, 1933, 1930, 1927 by Saunders, an imprint of Elsevier Inc. Copyright renewed 1991 by Paul Beeson. Copyright renewed 1979 by Russell L. Cecil and Robert F. Loeb. Copyright renewed 1987, 1975, 1971, 1965, 1961, 1958, 1955 by Elsevier Inc. All rights reserved. No part of this publication may be reproduced or transmitted in any form or by any means, electronic or mechanical, including photocopying, recording, or any information storage and retrieval system, without permission in writing from the Publisher. Details on how to seek permission, further information about the Publisher’s permissions policies and our arrangements with organizations such as the Copyright Clearance Center and the Copyright Licensing Agency, can be found at our website: www.elsevier.com/permissions. This book and the individual contributions contained in it are protected under copyright by the Publisher (other than as may be noted herein).

Notices Knowledge and best practice in this field are constantly changing. As new research and experience broaden our understanding, changes in research methods, professional practices, or medical treatment may become necessary. Practitioners and researchers must always rely on their own experience and knowledge in evaluating and using any information, methods, compounds, or experiments described herein. In using such information or methods they should be mindful of their own safety and the safety of others, including parties for whom they have a professional responsibility. With respect to any drug or pharmaceutical products identified, readers are advised to check the most current information provided (i) on procedures featured or (ii) by the manufacturer of each product to be administered, to verify the recommended dose or formula, the method and duration of administration, and contraindications. It is the responsibility of practitioners, relying on their own experience and knowledge of their patients, to make diagnoses, to determine dosages and the best treatment for each individual patient, and to take all appropriate safety precautions. To the fullest extent of the law, neither the Publisher nor the authors, contributors, or editors assume any liability for any injury and/or damage to persons or property as a matter of products liability, negligence or otherwise, or from any use or operation of any methods, products, instructions, or ideas contained in the material herein. Library of Congress Cataloging-in-Publication Data Goldman’s Cecil medicine.   Goldman-Cecil medicine / [edited by] Lee Goldman, Andrew I. Schafer.—25th edition.    p. ; cm.   Cecil medicine   Preceded by Goldman’s Cecil medicine / [edited by] Lee Goldman, Andrew I. Schafer. 24th ed. c2012.   Includes bibliographical references.   ISBN 978-1-4557-5017-7 (hardcover, 2 vol set : alk. paper)—ISBN 978-0-323-28800-2 (international edition : alk. paper)—ISBN 978-9996096563 (volume 1 : alk. paper)—ISBN 9996096564 (volume 1 : alk. paper)—ISBN 978-9996096624 (volume 2 : alk. paper)—ISBN 9996096629 (volume 2 : alk. paper)   I.  Goldman, Lee (Physician), editor.  II.  Schafer, Andrew I., editor.  III.  Title.  IV.  Title: Cecil medicine.   [DNLM:  1.  Medicine.  WB 100]   RC46   616—dc23 2014049904 Executive Content Strategist: Kate Dimock Senior Content Development Manager: Maureen Iannuzzi Publishing Services Manager: Anne Altepeter Senior Project Manager: Cindy Thoms Design Specialist: Paula Catalano Printed in the United States of America Last digit is the print number:  9  8  7  6  5  4  3  2  1

ASSOCIATE EDITORS Mary K. Crow, MD

Joseph P. Routh Professor of Rheumatic Diseases in Medicine Weill Cornell Medical College Physician-in-Chief and Benjamin M. Rosen Chair in Immunology and Inflammation Research Hospital for Special Surgery New York, New York

James H. Doroshow, MD Bethesda, Maryland

Jeffrey M. Drazen, MD

Distinguished Parker B. Francis Professor of Medicine Harvard Medical School Senior Physician Brigham and Women’s Hospital Boston, Massachusetts

Robert C. Griggs, MD

Professor of Neurology, Medicine, Pediatrics, and Pathology and Laboratory Medicine University of Rochester School of Medicine and Dentistry Rochester, New York

Donald W. Landry, MD, PhD

Samuel Bard Professor of Medicine Chair, Department of Medicine Physician-in-Chief Columbia University Medical Center New York, New York

Wendy Levinson, MD Professor of Medicine Chair Emeritus Department of Medicine University of Toronto Toronto, Ontario, Canada

Anil K. Rustgi, MD

T. Grier Miller Professor of Medicine and Genetics Chief of Gastroenterology American Cancer Society Professor University of Pennsylvania Perelman School of Medicine Philadelphia, Pennsylvania

W. Michael Scheld, MD

Bayer-Gerald L. Mandell Professor of Infectious Diseases Professor of Medicine Clinical Professor of Neurosurgery Director, Pfizer Initiative in International Health University of Virginia Health System Charlottesville, Virginia

Allen M. Spiegel, MD

Dean Albert Einstein College of Medicine Bronx, New York

This page intentionally left blank

PREFACE In the 90 years since the first edition of the Cecil Textbook of Medicine was published, almost everything we know about internal medicine has changed. Progress in medical science is now occurring at an ever-accelerating pace, and it is doing so within the framework of transformational changes in clinical practice and the delivery of health care at individual, social, and global levels. This textbook and its associated electronic products incorporate the latest medical knowledge in multiple formats that should appeal to students and seasoned practitioners regardless of how they prefer to access this rapidly changing information. Even as Cecil’s specific information has changed, however, we have remained true to the tradition of a comprehensive textbook of medicine that carefully explains the why (the underlying pathophysiology of disease) and the how (now expected to be evidence-based from randomized controlled trials and meta-analyses). Descriptions of physiology and pathophysiology include the latest genetic advances in a practical format that strives to be useful to the nonexpert. Medicine has entered an era when the acuity of illness and the limited time available to evaluate a patient have diminished the ability of physicians to satisfy their intellectual curiosity. As a result, the acquisition of information, quite easily achieved in this era, is often confused with knowledge. We have attempted to address this dilemma with a textbook that not only informs but also stimulates new questions and gives a glimpse of the future path to new knowledge. Grade A evidence is specifically highlighted in the text and referenced at the end of each chapter. In addition to the information provided in the textbook, the Cecil website supplies expanded content and functionality. In many cases, the full articles referenced in each chapter can be accessed from the Cecil website. The website is also continuously updated to incorporate subsequent Grade A information, other evidence, and new discoveries. The sections for each organ system begin with a chapter that summarizes an approach to patients with key symptoms, signs, or laboratory abnormalities associated with dysfunction of that organ system. As summarized in E-Table 1-1, the text specifically provides clear, concise information regarding how a physician should approach more than 100 common symptoms, signs, and laboratory abnormalities, usually with a flow diagram, a table, or both for easy reference. In this way, Cecil remains a comprehensive text to guide diagnosis and therapy, not only for patients with suspected or known diseases but also for patients who may have undiagnosed abnormalities that require an initial evaluation. Just as each edition brings new authors, it also reminds us of our gratitude to past editors and authors. Previous editors of Cecil include a short but remarkably distinguished group of leaders of American medicine: Russell Cecil, Paul Beeson, Walsh McDermott, James Wyngaarden, Lloyd H. Smith,

Jr., Fred Plum, J. Claude Bennett, and Dennis Ausiello. As we welcome new associate editors—Mary K. Crow, James H. Doroshow, and Allen M. Spiegel—we also express our appreciation to William P. Arend, James O. Armitage, David R. Clemmons, and other associate editors from the previous editions on whose foundation we have built. Our returning associate editors—Jeffrey M. Drazen, Robert C. Griggs, Donald W. Landry, Wendy Levinson, Anil K. Rustgi, and W. Michael Scheld—continue to make critical contributions to the selection of authors and the review and approval of all manuscripts. The editors, however, are fully responsible for the book as well as the integration among chapters. The tradition of Cecil is that all chapters are written by distinguished experts in each field. We are also most grateful for the editorial assistance in New York of Maribel Lim and Silva Sergenian. These individuals and others in our offices have shown extraordinary dedication and equanimity in working with authors and editors to manage the unending flow of manuscripts, figures, and permissions. We also thank Cassondra Andreychik, Ved Bhushan Arya, Cameron Harrison, Karen Krok, Robert J. Mentz, Gaétane Nocturne, Patrice Savard, Senthil Senniappan, Tejpratap Tiwari, and Sangeetha Venkatarajan, who contributed to various chapters, and we mourn the passing of Morton N. Swartz, MD, co-author of the chapter on “Meningitis: Bacterial, Viral, and Other” and Donald E. Low, MD, author of the chapter “Nonpneumococcal Streptococcal Infections, Rheumatic Fever.” At Elsevier, we are most indebted to Kate Dimock and Maureen Iannuzzi, and also thank Maria Holman, Gabriela Benner, Cindy Thoms, Anne Altepeter, Linda McKinley, Paula Catalano, and Kristin Koehler, who have been critical to the planning and production process under the guidance of Mary Gatsch. Many of the clinical photographs were supplied by Charles D. Forbes and William F. Jackson, authors of Color Atlas and Text of Clinical Medicine, Third Edition, published in 2003 by Elsevier Science Ltd. We thank them for graciously permitting us to include their pictures in our book. We have been exposed to remarkable physicians in our lifetimes and would like to acknowledge the mentorship and support of several of those who exemplify this paradigm— Eugene Braunwald, Lloyd H. Smith, Jr., Frank Gardner, and William Castle. Finally, we would like to thank the Goldman family—Jill, Jeff, Abigail, Mira, Samuel, Daniel, Robyn, Tobin, and Dashel—and the Schafer family— Pauline, Eric, Melissa, Nathaniel, Pam, John, Evan, Samantha, Kate, and Sean, for their understanding of the time and focus required to edit a book that attempts to sustain the tradition of our predecessors and to meet the needs of today’s physician. LEE GOLDMAN, MD ANDREW I. SCHAFER, MD

This page intentionally left blank

CONTRIBUTORS Charles S. Abrams, MD Professor of Medicine, Pathology, and Laboratory Medicine, University of Pennsylvania School of Medicine; Director, PENN-Chop Blood Center for Patient Care & Discovery, Hospital of the University of Pennsylvania, Philadelphia, Pennsylvania Thrombocytopenia Frank J. Accurso, MD Professor of Pediatrics, University of Colorado School of Medicine; Attending Physician, Children’s Hospital Colorado, Aurora, Colorado Cystic Fibrosis Ronald S. Adler, MD, PhD Professor of Radiology, New York University School of Medicine; Department of Radiology, NYU Langone Medical Center, New York, New York Imaging Studies in the Rheumatic Diseases Cem Akin, MD, PhD Associate Professor, Harvard Medical School; Attending Physician, Director, Mastocytosis Center, Brigham and Women’s Hospital, Department of Medicine, Division of Rheumatology, Immunology, and Allergy, Boston, Massachusetts Mastocytosis Allen J. Aksamit, Jr., MD Professor of Neurology, Mayo Clinic College of Medicine, Consultant in Neurology, Mayo Clinic, Rochester, Minnesota Acute Viral Encephalitis Qais Al-Awqati, MB ChB Robert F. Loeb Professor of Medicine, Jay I. Meltzer Professor of Nephrology and Hypertension, Professor of Physiology and Cellular Biophysics, Division of Nephrology, Columbia University, College of Physicians and Surgeons, New York, New York Structure and Function of the Kidneys Ban Mishu Allos, MD Associate Professor of Medicine, Division of Infectious Diseases, Associate Professor, Preventive Medicine, Vanderbilt University School of Medicine, Nashville, Tennessee Campylobacter Infections David Altshuler, MD, PhD Professor of Genetics and of Medicine, Harvard Medical School, Massachusetts General Hospital; Professor of Biology (Adjunct), Massachusetts Institute of Technology, Boston and Cambridge, Massachusetts The Inherited Basis of Common Diseases

Larry J. Anderson, MD Professor, Division of Infectious Disease, Department of Pediatrics, Emory University School of Medicine and Children’s Healthcare of Atlanta, Atlanta, Georgia Coronaviruses Aśok C. Antony, MD Chancellor’s Professor of Medicine, Indiana University School of Medicine; Attending Physician, Indiana University Health Affiliated Hospitals and Richard L. Roudebush Veterans Affairs Medical Center, Indianapolis, Indiana Megaloblastic Anemias Gerald B. Appel, MD Professor of Medicine, Division of Nephrology, Department of Medicine, Columbia University College of Physicians and Surgeons, New York, New York Glomerular Disorders and Nephrotic Syndromes Frederick R. Appelbaum, MD Executive Vice President and Deputy Director, Fred Hutchinson Cancer Research Center; President, Seattle Cancer Care Alliance; Professor, Division of Medical Oncology, University of Washington School of Medicine, Seattle Washington The Acute Leukemias Suneel S. Apte, MBBS, DPhil Staff, Cleveland Clinic Lerner College of Medicine at Case Western Reserve University, Cleveland, Ohio Connective Tissue Structure and Function James O. Armitage, MD The Joe Shapiro Professor of Internal Medicine, University of Nebraska Medical Center, Omaha, Nebraska Approach to the Patient with Lymphadenopathy and Splenomegaly, Non-Hodgkin Lymphomas M. Amin Arnaout, MD Professor of Medicine, Departments of Medicine and Developmental and Regenerative Biology, Harvard Medical School; Physician and Chief Emeritus, Division of Nephrology, Massachusetts General Hospital, Boston, Massachusetts Cystic Kidney Diseases Robert M. Arnold, MD Leo H. Criep Professor of Clinical Care, Chief, Section of Palliative Care and Medical Ethics, University of Pittsburgh; Medical Director, UPMC Palliative and Supportive Care Institute, Pittsburgh, Pennsylvania Care of Dying Patients and Their Families

Michael Aminoff, MD, DSc Professor, Department of Neurology, University of California San Francisco, San Francisco, California Approach to the Patient with Neurologic Disease

David Atkins, MD, MPH Director, Health Services Research and Development, Veterans Health Administration, Washington, D.C. The Periodic Health Examination

Jeffrey L. Anderson, MD Professor of Internal Medicine, University of Utah School of Medicine; Vice-Chair for Research, Department of Internal Medicine, Associate Chief of Cardiology and Director of Cardiovascular Research, Intermountain Medical Center, Intermountain Healthcare, Salt Lake City, Utah ST Segment Elevation Acute Myocardial Infarction and Complications of Myocardial Infarction

John P. Atkinson, MD Chief, Division of Rheumatology, Internal Medicine, Washington University School of Medicine in St. Louis, St. Louis, Missouri Complement System in Disease

x

Contributors

Bruce R. Bacon, MD Endowed Chair in Gastroenterology, Professor of Internal Medicine, Co-Director, Saint Louis University Liver Center; Director, Saint Louis University Abdominal Transplant Center, Saint Louis University School of Medicine, St. Louis, Missouri Iron Overload (Hemochromatosis) Larry M. Baddour, MD Professor of Medicine, Chair, Division of Infectious Diseases, Mayo Clinic, Rochester, Minnesota Infective Endocarditis Grover C. Bagby, MD Professor of Medicine and Molecular and Medical Genetics, Knight Cancer Institute at Oregon Health and Science University and Portland VA Medical Center, Portland, Oregon Aplastic Anemia and Related Bone Marrow Failure States Barbara J. Bain, MBBS Professor in Diagnostic Haematology, Imperial College London; Honorary Consultant Haematologist, St. Mary’s Hospital, London, United Kingdom The Peripheral Blood Smear Dean F. Bajorin, MD Attending Physician and Member, Medicine, Memorial Hospital, Memorial Sloan Kettering Cancer Center; Professor of Medicine, Weill Cornell Medical College, New York, New York Tumors of the Kidney, Bladder, Ureters, and Renal Pelvis

Stephen G. Baum, MD Chairman of Medicine, Mount Sinai Beth Israel Hospital; Professor of Medicine and of Microbiology and Immunology, Albert Einstein College of Medicine, New York, New York Mycoplasma Infections Daniel G. Bausch, MD, MPH&TM Associate Professor, Department of Tropical Medicine, Tulane University Health Sciences Center, New Orleans, Louisiana Viral Hemorrhagic Fevers Arnold S. Bayer, MD Professor of Medicine, David Geffen School of Medicine at University of California Los Angeles; LA Biomedical Research Institute; Vice Chair for Academic Affairs, Department of Medicine, Harbor-UCLA Medical Center, Los Angeles, California Infective Endocarditis Hasan Bazari, MD Associate Professor of Medicine, Harvard Medical School, Department of Medicine, Clinical Director, Nephrology, Program Director, Internal Medicine Residency Program, Massachusetts General Hospital, Boston, Massachusetts Approach to the Patient with Renal Disease John H. Beigel, MD National Institute of Allergy and Infectious Diseases, National Institutes of Health, Bethesda, Maryland Antiviral Therapy (Non-HIV)

Robert W. Baloh, MD Professor of Neurology, University of California Los Angeles School of Medicine, Los Angeles, California Neuro-Ophthalmology; Smell and Taste; Hearing and Equilibrium

George A. Beller, MD Professor of Medicine, University of Virginia Health System, Charlottesville, Virginia Noninvasive Cardiac Imaging

Jonathan Barasch, MD, PhD Professor of Medicine and Pathology and Cell Biology, Department of Medicine, Division of Nephrology, Columbia University College of Physicians & Surgeons, New York, New York Structure and Function of the Kidneys

Robert M. Bennett, MD Professor of Medicine, Oregon Health and Science University, Portland, Oregon Fibromyalgia, Chronic Fatigue Syndrome, and Myofascial Pain

Richard L. Barbano, MD, PhD Professor of Neurology, University of Rochester, Rochester, New York Mechanical and Other Lesions of the Spine, Nerve Roots, and Spinal Cord Elizabeth Barrett-Connor, MD Professor of Community and Family Medicine, University of California San Diego, San Diego, California Menopause John R. Bartholomew, MD Section Head, Vascular Medicine, Cardiovascular Medicine, Cleveland Clinic, Professor of Medicine, Cleveland Clinic Lerner College of Medicine of Case Western Reserve University, Cleveland, Ohio Other Peripheral Arterial Diseases Mary Barton, MD, MPP Vice President, Performance Measurement, National Committee for Quality Assurance, Washington, D.C. The Periodic Health Examination Robert C. Basner, MD Professor of Medicine, Columbia University Medical Center; Director, Columbia University Cardiopulmonary Sleep and Ventilatory Disorders Center, Columbia University College of Physicians and Surgeons, New York, New York Obstructive Sleep Apnea

Joseph R. Berger, MD Professor of Neurology, Chief of the Multiple Sclerosis Division, Department of Neurology, Perelman School of Medicine, University of Pennsylvania, Philadelphia, Pennsylvania Cytomegalovirus, Epstein-Barr Virus, and Slow Virus Infections of the Central Nervous System; Neurologic Complications of Human Immunodeficiency Virus Infection; Brain Abscess and Parameningeal Infections Paul D. Berk, MD Professor of Medicine, Department of Medicine, Columbia University College of Physicians and Surgeons, New York, New York Approach to the Patient with Jaundice or Abnormal Liver Tests Nancy Berliner, MD Professor of Medicine, Harvard Medical School; Chief, Division of Hematology, Brigham and Women’s Hospital, Boston, Massachusetts Leukocytosis and Leukopenia James L. Bernat, MD Louis and Ruth Frank Professor of Neuroscience, Professor of Neurology and Medicine, Geisel School of Medicine at Dartmouth, Hanover, New Hampshire; Department of Neurology, Dartmouth-Hitchcock Medical Center, Lebanon, New Hampshire Coma, Vegetative State, and Brain Death Philip J. Bierman, MD Professor, Department of Internal Medicine, University of Nebraska Medical Center, Omaha, Nebraska Approach to the Patient with Lymphadenopathy and Splenomegaly; Non-Hodgkin Lymphomas

Contributors

xi

Michael R. Bishop, MD Professor of Medicine, Director, Hematopoietic Cellular Therapy Program, Section of Hematology and Oncology, Department of Medicine, University of Chicago, Chicago, Illinois Hematopoietic Stem Cell Transplantation

William E. Boden, MD Professor of Medicine, Albany Medical College; Chief of Medicine, Albany Stratton VA Medical Center; Vice-Chairman, Department of Medicine, Albany Medical Center, Albany, New York Angina Pectoris and Stable Ischemic Heart Disease

Bruce R. Bistrian, MD, PhD, MPH Professor of Medicine, Beth Israel Deaconess Medical Center; Professor of Medicine, Harvard Medical School, Boston, Massachusetts Nutritional Assessment

Jean Bolognia, MD Professor of Dermatology, Yale Medical School; Attending Physician, Yale-New Haven Hospital, New Haven, Connecticut Infections, Hyperpigmentation and Hypopigmentation, Regional Dermatology, and Distinctive Lesions in Black Skin

Joseph J. Biundo, MD Clinical Professor of Medicine, Tulane Medical Center, New Orleans, Louisiana Bursitis, Tendinitis, and Other Periarticular Disorders and Sports Medicine Adrian R. Black, PhD Assistant Professor, Director of Tissue Sciences for the Eppley Institute, The Eppley Institute for Research in Cancer and Allied Diseases, University of Nebraska Medical Center, Omaha, Nebraska Cancer Biology and Genetics Charles D. Blanke, MD Professor of Medicine, Oregon Health and Science University, Portland, Oregon Neoplasms of the Small and Large Intestine Joel N. Blankson, MD, PhD Associate Professor, Johns Hopkins University School of Medicine, Baltimore, Maryland Immunopathogenesis of Human Immunodeficiency Virus Infection Martin J. Blaser, MD Muriel and George Singer Professor of Medicine, Professor of Microbiology, Director, Human Microbiome Program, New York University Langone Medical Center, New York, New York Acid Peptic Disease; Human Microbiome William A. Blattner, MD Professor and Associate Director, Institute of Human Virology, School of Medicine, University of Maryland; Professor of Medicine, School of Medicine, University of Maryland; Professor and Head, Division of Cancer Epidemiology, Department of Epidemiology and Public Health, School of Medicine, University of Maryland, Baltimore, Maryland Retroviruses Other Than Human Immunodeficiency Virus Thomas P. Bleck, MD Professor of Neurological Sciences, Neurosurgery, Internal Medicine, and Anesthesiology, Associate Chief Medical Officer (Critical Care), Rush Medical College, Chicago, Illinois Arboviruses Affecting the Central Nervous System Joel A. Block, MD The Willard L. Wood MD Professor and Director, Division of Rheumatology, Rush University Medical Center, Chicago, Illinois Osteoarthritis Henk Blom, MD Laboratory of Clinical Biochemistry and Metabolism, Department of General Pediatrics, Adolescent Medicine and Neonatology, University Medical Centre Freiburg, Head of Laboratory/Clinical Biochemical Geneticist, Freiburg, Germany Homocystinuria and Hyperhomocysteinemia Olaf A. Bodamer, MD Medical Genetics, University of Miami Hospital, Miami, Florida Approach to Inborn Errors of Metabolism

Robert A. Bonomo, MD Chief, Medical Service, Louis Stokes Cleveland VA Medical Center; Professor of Medicine, Pharmacology, Biochemistry, Molecular Biology, and Microbiology, Case Western Reserve University School of Medicine, Cleveland, Ohio Diseases Caused by Acinetobacter and Stenotrophomonas Species Larry Borish, MD Professor of Medicine, Allergy, and Clinical Immunology, University of Virginia Health System, Charlottesville, Virgina Allergic Rhinitis and Chronic Sinusitis Patrick J. Bosque, MD Associate Professor of Neurology, University of Colorado Denver School of Medicine; Neurologist, Denver Health Medical Center, Denver, Colorado Prion Diseases David J. Brenner, PhD, DSc Higgins Professor of Radiation Biophysics, Center for Radiological Research, Columbia University Medical Center, New York, New York Radiation Injury Itzhak Brook, MD, MSc Professor of Pediatrics and Medicine, Georgetown University, Georgetown University Medical Center, Washington, D.C. Diseases Caused by Non–Spore-Forming Anaerobic Bacteria; Actinomycosis Enrico Brunetti, MD Assistant Professor of Infectious Diseases, University of Pavia; Attending Physician, Division of Infectious and Tropical Diseases, IRCCS San Matteo Hospital Foundation; Co-Director, WHO Collaborating Centre for Clinical Management of Cystic Echinococcosis, Pavia, Italy Cestodes David M. Buchner, MD, MPH Shahid and Ann Carlson Khan Professor in Applied Health Sciences, Department of Kinesiology and Community Health, University of Illinois at Urbana-Champaign, Champaign, Illinois Physical Activity Pierre A. Buffet, MD, PhD Research Unit Head, Erythrocyte Parasite Pathogenesis Research Team INSERM–University Paris 6, CIMI–Paris Research Center, University Pierre and Marie Curie; Associate Professor of Parasitology, Faculty of Medicine, University Pierre and Marie Curie, Pitié-Salpêtrière Hospital, Paris, France Leishmaniasis H. Franklin Bunn, MD Professor of Medicine, Harvard Medical School; Physician, Brigham and Women’s Hospital, Boston, Massachusetts Approach to the Anemias David A. Bushinsky, MD John J. Kuiper Distinguished Professor of Medicine, Chief, Nephrology Division, University of Rochester School of Medicine; Associate Chair for Academic Affairs in Medicine, University of Rochester Medical Center, Rochester, New York Nephrolithiasis

xii

Contributors

Vivian P. Bykerk, MD Associate Professor of Medicine, Weill Cornell Medical College; Associate Attending Physician, Hospital for Special Surgery, New York, New York Approach to the Patient with Rheumatic Disease Peter A. Calabresi, MD Professor of Neurology and Director of the Richard T. Johnson Division of Neuroimmunology and Neuroinfectious Diseases, Johns Hopkins University; Director of the Multiple Sclerosis Center, Johns Hopkins Hospital, Baltimore, Maryland Multiple Sclerosis and Demyelinating Conditions of the Central Nervous System David P. Calfee, MD, MS Associate Professor of Medicine and Healthcare Policy and Research, Weill Cornell Medical College; Chief Hospital Epidemiologist, New YorkPresbyterian Hospital/Weill Cornell Medical Center, New York, New York Prevention and Control of Health Care–Associated Infections Douglas Cameron, MD, MBA Professor of Ophthalmology and Visual Neurosciences, University of Minnesota, Minneapolis, Minnesota Diseases of the Visual System Michael Camilleri, MD Atherton and Winifred W. Bean Professor, Professor of Medicine, Pharmacology, and Physiology, College of Medicine, Mayo Clinic, Consultant, Division of Gastroenterology and Hepatology, Mayo Clinic, Rochester, Minnesota Disorders of Gastrointestinal Motility Grant W. Cannon, MD Thomas E. and Rebecca D. Jeremy Presidential Endowed Chair for Arthritis Research, Associate Chief of Staff for Academic Affiliations, George E. Wahlen VA Medical Center, Salt Lake City, Utah Immunosuppressing Drugs Including Corticosteroids Maria Domenica Cappellini, MD Professor of Internal Medicine, University of Milan, Fondazione IRCCS Ca’ Granda Ospedale Maggiore Policlinico, University of Milan, Milan, Italy The Thalassemias Blase A. Carabello, MD Professor of Medicine, Chairman, Department of Cardiology, Mount Sinai Beth Israel Heart Institute, New York, New York Valvular Heart Disease Edgar M. Carvalho, MD Professor of Medicine and Clinical Immunology, Faculdade de Medicina da Bahia, Universidade Federal da Bahia and Escola Bahiana de Medicina e Saúde Pública, Salvador, Bahia, Brazil Schistosomiasis (Bilharziasis) William H. Catherino, MD, PhD Professor and Research Head, Department of Obstetrics and Gynecology, Uniformed Services University of the Health Sciences Division of Reproductive Endocrinology and Infertility; Program in Reproductive and Adult Endocrinology, Eunice Kennedy Shriver National Institute of Child Health and Human Development, National Institutes of Health, Bethesda, Maryland Ovaries and Development; Reproductive Endocrinology and Infertility Jane A. Cauley, DrPH Professor of Epidemiology, University of Pittsburgh Graduate School of Public Health, Vice Chair of the Department of Epidemiology, Pittsburgh, Pennsylvania Epidemiology of Aging: Implications of the Aging of Society

Naga P. Chalasani, MD David W. Crabb Professor and Director, Division of Gastroenterology and Hepatology, Indiana University School of Medicine, Indianapolis, Indiana Alcoholic and Nonalcoholic Steatohepatitis Henry F. Chambers, MD Professor of Medicine, University of California San Francisco School of Medicine; Director, Clinical Research Services, Clinical and Translational Sciences Institute, San Francisco, California Staphylococcal Infections William P. Cheshire, Jr., MD Professor of Neurology, Mayo Clinic, Jacksonville, Florida Autonomic Disorders and Their Management Ilseung Cho, MD, MS Assistant Professor of Medicine, Division of Gastroenterology, Department of Medicine, New York University, New York, New York Human Microbiome Arun Chockalingam, PhD Professor of Epidemiology and Global Health, Director, Office of Global Health Education and Training; Dalla Lana Faculty of Public Health, University of Toronto, Toronto, Ontario, Canada Global Health David C. Christiani, MD Professor of Medicine, Harvard Medical School; Physician, Pulmonary and Critical Care, Massachusetts General Hospital; Elkan Blout Professor of Environmental Genetics, Environmental Health, Harvard School of Public Health, Boston, Massachusetts Physical and Chemical Injuries of the Lung David H. Chu, MD, PhD Director, Contact Dermatitis, Division of Dermatology and Cutaneous Surgery, Scripps Clinic Medical Group, La Jolla, California Structure and Function of the Skin Theodore J. Cieslak, MD Pediatric Infectious Diseases, Clinical Professor of Pediatrics, University of Texas Health Science Center at San Antonio; Department of Pediatrics, Fort Sam Houston, Texas Bioterrorism Carolyn Clancy, MD Interim Under Secretary for Health, Veterans Administration, Washington, D.C. Measuring Health and Health Care David R. Clemmons, MD Kenan Professor of Medicine, University of North Carolina School of Medicine; Attending Physician, Medicine, UNC Hospitals, Chapel Hill, North Carolina Approach to the Patient with Endocrine Disease David Cohen, MD Professor of Medicine, Division of Nephrology; Medical Director, Kidney and Pancreas Transplantation, Columbia University Medical Center, New York, New York Treatment of Irreversible Renal Failure Jeffrey Cohen, MD Chief, Laboratory of Infectious Diseases, National Institute of Allergy and Infectious Diseases, National Institutes of Health, Bethesda, Maryland Varicella-Zoster Virus (Chickenpox, Shingles)

Contributors

xiii

Myron S. Cohen, MD Associate Vice Chancellor for Global Health, Director, UNC Institute for Global Health and Infectious Diseases, Chief, Division of Infectious Diseases, Yeargan-Bate Eminent Professor of Medicine, Microbiology, and Immunology and Epidemiology, Chapel Hill, North Carolina Approach to the Patient with a Sexually Transmitted Infection; Prevention of Human Immunodeficiency Virus Infection

Mary K. Crow, MD Joseph P. Routh Professor of Rheumatic Diseases in Medicine, Weill Cornell Medical College; Physician in Chief and Benjamin M. Rosen Chair in Immunology and Inflammation Research, Hospital for Special Surgery, New York, New York The Innate Immune Systems; Approach to the Patient with Rheumatic Disease; Systemic Lupus Erythematosus

Steven P. Cohen, MD Professor of Anesthesiology and Critical Care Medicine and Physical Medicine and Rehabilitation, Johns Hopkins School of Medicine, Baltimore, Maryland, and Uniformed Services University of the Health Sciences, Bethesda, Maryland; Director, Pain Research, Walter Reed National Military Medical Center, Bethesda, Maryland Pain

John A. Crump, MB ChB, MD, DTM&H McKinlay Professor of Global Health, Centre for International Health, University of Otago, Dunedin, New Zealand Salmonella Infections (Including Enteric Fever)

Steven L. Cohn, MD Professor of Clinical Medicine, University of Miami Miller School of Medicine; Medical Director, UHealth Preoperative Assessment Center; Director, Medical Consultation Service, University of Miami Hospital, Miami, Florida Preoperative Evaluation Robert Colebunders, MD Emeritus Professor, Institute of Tropical Medicine, Antwerp, Belgium Immune Reconstitution Inflammatory Syndrome in HIV/AIDS Joseph M. Connors, MD Clinical Professor, University of British Columbia; Clinical Director, BC Cancer Agency Centre for Lymphoid Cancer, Vancouver, British Columbia, Canada Hodgkin Lymphoma Deborah J. Cook, MD, MSc Professor of Medicine, Clinical Epidemiology and Biostatistics, McMaster University, Hamilton, Ontario, Canada Approach to the Patient in a Critical Care Setting Kenneth H. Cowan, MD, PhD Director, Fred & Pamela Buffett Cancer Center; Director, The Eppley Institute for Research in Cancer and Allied Diseases; Professor of Medicine, University of Nebraska Medical Center, Omaha, Nebraska Cancer Biology and Genetics Joseph Craft, MD Paul B. Beeson Professor of Medicine and Immunobiology, Section Chief, Rheumatology, Program Director, Investigative Medicine, Department of Internal Medicine, Yale University School of Medicine, New Haven, Connecticut The Adaptive Immune Systems Jill Patricia Crandall, MD Professor of Clinical Medicine, Division of Endocrinology and Diabetes Research Center, Albert Einstein College of Medicine, Bronx, New York Diabetes Mellitus Simon L. Croft, BSc, PhD Professor of Parasitology, Faculty of Infectious and Tropical Diseases, London School of Hygiene and Tropical Medicine, London, United Kingdom Leishmaniasis Kristina Crothers, MD Associate Professor, Department of Medicine, Division of Pulmonary and Critical Care, University of Washington School of Medicine, Seattle, Washington Pulmonary Manifestations of Human Immunodeficiency Virus and Acquired Immunodeficiency Syndrome

Mark R. Cullen, MD Professor of Medicine, Department of Medicine, Stanford University School of Medicine, Stanford, California Principles of Occupational and Environmental Medicine Charlotte Cunningham-Rundles, MD, PhD Professor of Medicine and Pediatrics, Icahn School of Medicine at Mount Sinai, New York, New York Primary Immunodeficiency Diseases Inger K. Damon, MD, PhD Director, Division of High Consequence Pathogens and Pathology, Centers for Disease Control and Prevention, Atlanta, Georgia Smallpox, Monkeypox, and Other Poxvirus Infections Troy E. Daniels, DDS, MS Professor Emeritus of Oral Pathology and Pathology, University of California San Francisco, San Francisco, California Diseases of the Mouth and Salivary Glands Nancy E. Davidson, MD Hillman Professor of Oncology, University of Pittsburgh; Director, University of Pittsburgh Cancer Institute and UPMC CancerCenter, Pittsburgh, Pennsylvania Breast Cancer and Benign Breast Disorders Lisa M. DeAngelis, MD Chair, Department of Neurology, Memorial Sloan-Kettering Cancer Center; Professor of Neurology, Weill Cornell Medical College, New York, New York Tumors of the Central Nervous System Malcolm M. DeCamp, MD Fowler McCormick Professor of Surgery, Feinberg School of Medicine, Northwestern University; Chief, Division of Thoracic Surgery, Northwestern Memorial Hospital, Chicago, Illinois Interventional and Surgical Approaches to Lung Disease Carlos del Rio, MD Hubert Professor and Chair and Professor of Medicine, Hubert Department of Global Health, Rollins School of Public Health and Department of Medicine, Emory University School of Medicine, Atlanta, Georgia Prevention of Human Immunodeficiency Virus Infection Patricia A. Deuster, PhD, MPH Professor and Director, Consortium for Health and Military Performance, Department of Military and Emergency Medicine, Uniformed Services University of the Health Sciences, Bethesda, Maryland Rhabdomyolysis Robert B. Diasio, MD William J. and Charles H. Mayo Professor, Molecular Pharmacology and Experimental Therapeutics and Oncology, Mayo Clinic, Rochester, Minnesota Principles of Drug Therapy

xiv

Contributors

David J. Diemert, MD Associate Professor, Department of Microbiology, Immunology, and Tropical Medicine, School of Medicine and Health Sciences, The George Washington University, Washington, D.C. Intestinal Nematode Infections; Tissue Nematode Infections Kathleen B. Digre, MD Professor of Neurology, Ophthalmology, Director, Division of Headache and Neuro-Ophthalmology, University of Utah, Salt Lake City, Utah Headaches and Other Head Pain James H. Doroshow, MD Bethesda, Maryland Approach to the Patient with Cancer; Malignant Tumors of Bone, Sarcomas, and Other Soft Tissue Neoplasms John M. Douglas, Jr., MD Executive Director, Tri-County Health Department, Greenwood Village, Colorado Papillomavirus Jeffrey M. Drazen, MD Distinguished Parker B. Francis Professor of Medicine, Harvard Medical School; Senior Physician, Brigham and Women’s Hospital, Boston, Massachusetts Asthma Stephen C. Dreskin, MD, PhD Professor of Medicine and Immunology, Division of Allergy and Clinical Immunology, Department of Medicine, University of Colorado Denver, School of Medicine, Aurora, Colorado Urticaria and Angioedema W. Lawrence Drew, MD, PhD Professor Emeritus, Laboratory Medicine and Medicine, University of California San Francisco, San Francisco, California Cytomegalovirus George L. Drusano, MD Professor and Director, Institute for Therapeutic Innovation, College of Medicine, University of Florida, Lake Nona, Florida Antibacterial Chemotherapy Thomas D. DuBose, Jr., MD Emeritus Professor of Internal Medicine and Nephrology, Wake Forest School of Medicine, Winston-Salem, North Carolina Vascular Disorders of the Kidney F. Daniel Duffy, MD Professor of Internal Medicine and Steve Landgarten Chair in Medical Leadership, School of Community Medicine, University of Oklahoma College of Medicine, Tulsa, Oklahoma Counseling for Behavior Change Herbert L. DuPont, MD, MACP Mary W. Kelsey Chair and Director, Center for Infectious Diseases, University of Texas School of Public Health; H. Irving Schweppe Chair of Internal Medicine and Vice Chairman, Department of Medicine, Baylor College of Medicine; Chief of Internal Medicine, St. Luke’s Hospital System, Houston, Texas Approach to the Patient with Suspected Enteric Infection Madeleine Duvic, MD Professor and Deputy Chairman, Department of Dermatology, The University of Texas MD Anderson Cancer Center, Houston, Texas Urticaria, Drug Hypersensitivity Rashes, Nodules and Tumors, and Atrophic Diseases

Kathryn M. Edwards, MD Sarah H. Sell and Cornelius Vanderbilt Chair in Pediatrics, Vanderbilt University School of Medicine; Director, Vanderbilt Vaccine Research Program, Monroe Carrell Jr. Children’s Hospital at Vanderbilt, Nashville, Tennessee Parainfluenza Viral Disease N. Lawrence Edwards, MD Professor of Medicine, Vice Chairman, Department of Medicine, University of Florida; Chief, Section of Rheumatology, Medical Service, Malcom Randall Veterans Affairs Medical Center, Gainesville, Florida Crystal Deposition Diseases Lawrence H. Einhorn, MD Distinguished Professor, Department of Medicine, Division of Hematology/Oncology, Livestrong Foundation Professor of Oncology, Indiana University School of Medicine, Indianapolis, Indiana Testicular Cancer Ronald J. Elin, MD, PhD A.J. Miller Professor and Chairman, Department of Pathology and Laboratory Medicine, University of Louisville School of Medicine, Louisville, Kentucky Reference Intervals and Laboratory Values George M. Eliopoulos, MD Professor of Medicine, Harvard Medical School; Physician, Division of Infectious Diseases, Beth Israel Deaconess Medical Center, Boston, Massachusetts Principles of Anti-Infective Therapy Perry Elliott, MD Professor in Inherited Cardiovascular Disease, Institute of Cardiovascular Science, University College London, London, United Kingdom Diseases of the Myocardium and Endocardium Jerrold J. Ellner, MD Professor of Medicine, Boston University School of Medicine; Chief, Section of Infectious Diseases, Boston Medical Center, Boston, Massachusetts Tuberculosis Dirk M. Elston, MD Director, Ackerman Academy of Dermatopathology, New York, New York Arthropods and Leeches Ezekiel J. Emanuel, MD, PhD Vice Provost for Global Initiatives, Diane V.S. Levy and Robert M. Levy University Professor, Chair, Department of Medical Ethics and Health Policy, University of Pennsylvania, Philadelphia, Pennsylvania Bioethics in the Practice of Medicine Joel D. Ernst, MD Director, Division of Infectious Diseases and Immunology, Jeffrey Bergstein Professor of Medicine, Professor of Medicine, Pathology, and Microbiology, New York University School of Medicine; Attending Physician, New York University Langone Medical Center, New York, New York Leprosy (Hansen Disease) Gregory T. Everson, MD Professor of Medicine, Director of Hepatology, University of Colorado School of Medicine, Aurora, Colorado Hepatic Failure and Liver Transplantation Amelia Evoli, MD Associate Professor of Neurology, Catholic University, Agostino Gemelli University Hospital, Rome, Italy Disorders of Neuromuscular Transmission

Contributors Douglas O. Faigel, MD Professor of Medicine, Mayo Clinic, Chair, Division of Gastroenterology and Hepatology, Scottsdale, Arizona Neoplasms of the Small and Large Intestine

Manuel A. Franco, MD, PhD Director of Postgraduate Programs, School of Sciences, Pontificia Universidad Javeriana, Bogota, Colombia Rotaviruses, Noroviruses, and Other Gastrointestinal Viruses

Matthew E. Falagas, MD, MSc, DSc Director, Alfa Institute of Biomedical Sciences, Athens, Greece; Adjunct Associate Professor of Medicine, Tufts University School of Medicine, Boston, Massachusetts; Chief, Department of Medicine and Infectious Diseases, Iaso General Hospital, Iaso Group, Athens, Greece Pseudomonas and Related Gram-Negative Bacillary Infections

David O. Freedman, MD Professor of Medicine and Microbiology, University of Alabama at Birmingham; Director, Gorgas Center for Geographic Medicine, Birmingham, Alabama Approach to the Patient before and after Travel

Gary W. Falk, MD, MS Professor of Medicine, Division of Gastroenterology, University of Pennsylvania Perelman School of Medicine, Philadelphia, Pennsylvania Diseases of the Esophagus Gene Feder, MBBS, MD Professor, Centre for Academic Primary Care, School of Social and Community Medicine, University of Bristol; General Practitioner, Helios Medical Centre, Bristol, United Kingdom Intimate Partner Violence David J. Feller-Kopman, MD Director, Bronchoscopy and Interventional Pulmonology, Associate Professor of Medicine, The Johns Hopkins University, Baltimore, Maryland Interventional and Surgical Approaches to Lung Disease Gary S. Firestein, MD Dean and Associate Vice Chancellor of Translational Medicine, University of California San Diego School of Medicine, La Jolla, California Mechanisms of Inflammation and Tissue Repair Glenn I. Fishman, MD Director, Leon H. Charney Division of Cardiology, Vice-Chair for Research, Department of Medicine, William Goldring Professor of Medicine, New York University School of Medicine, New York, New York Principles of Electrophysiology Lee A. Fleisher, MD Robert D. Dripps Professor and Chair, Anesthesiology and Critical Care, Professor of Medicine, University of Pennsylvania Perelman School of Medicine, Philadelphia, Pennsylvania Overview of Anesthesia Paul W. Flint, MD Professor and Chair, Otolaryngology, Head and Neck Surgery, Oregon Health and Science University, Portland, Oregon Throat Disorders Evan L. Fogel, MD, MSc Professor of Clinical Medicine, Indiana University School of Medicine, Indianapolis, Indiana Diseases of the Gallbladder and Bile Ducts Marsha D. Ford, MD Adjunct Professor of Emergency Medicine, School of Medicine, University of North Carolina-Chapel Hill; Director, Carolinas Poison Center, Carolinas HealthCare System, Charlotte, North Carolina Acute Poisoning Chris E. Forsmark, MD Professor of Medicine, Chief, Division of Gastroenterology, Hepatology, and Nutrition, University of Florida, Gainesville, Florida Pancreatitis Vance G. Fowler, Jr., MD, MHS Professor of Medicine, Duke University Medical Center, Durham, North Carolina Infective Endocarditis

xv

Martyn A. French, MD Professor in Clinical Immunology, School of Pathology and Laboratory Medicine, University of Western Australia, Perth, Australia Immune Reconstitution Inflammatory Syndrome in HIV/AIDS Karen Freund, MD, MPH Professor of Medicine, Associate Director, Tufts Clinical and Translational Science Institute, Tufts University School of Medicine, Tufts Medical Center, Boston, Massachusetts Approach to Women’s Health Cem Gabay, MD Professor of Medicine, Head, Division of Rheumatology, University Hospitals of Geneva, Geneva, Switzerland Biologic Agents Kenneth L. Gage, PhD Chief, Entomology and Ecology Activity, Centers for Disease Control and Prevention, Division of Vector-Borne Diseases, Bacterial Diseases Branch, Fort Collins, Colorado Plague and Other Yersinia Infections John N. Galgiani, MD Professor of Medicine, Valley Fever Center for Excellence, University of Arizona, Tucson, Arizona Coccidioidomycosis Patrick G. Gallagher, MD Professor of Pediatrics, Pathology, and Genetics, Yale University School of Medicine; Attending Physician, Yale–New Haven Hospital, New Haven, Connecticut Hemolytic Anemias: Red Blood Cell Membrane and Metabolic Defects Leonard Ganz, MD Director of Cardiac Electrophysiology, Heritage Valley Health System, Beaver, Pennsylvania Electrocardiography Hasan Garan, MD Director, Cardiac Electrophysiology, Dickinson W. Richards, Jr. Professor of Medicine, Columbia University Medical Center, New York, New York Ventricular Arrhythmias Guadalupe Garcia-Tsao, MD Professor of Medicine, Yale University School of Medicine; Chief, Digestive Diseases, VA Connecticut Healthcare System, West Haven, Connecticut Cirrhosis and Its Sequelae William M. Geisler, MD, MPH Professor of Medicine, University of Alabama at Birmingham, Birmingham, Alabama Diseases Caused by Chlamydiae Tony P. George, MD Division of Brain and Therapeutics, Department of Psychiatry, University of Toronto; Schizophrenia Division, The Centre for Addiction and Mental Health, Toronto, Ontario, Canada Nicotine and Tobacco

xvi

Contributors

Lior Gepstein, MD, PhD Edna and Jonathan Sohnis Professor in Medicine and Physiology, Rappaport Faculty of Medicine and Research Institute, Technion–Israel Institute of Technology, Rambam Health Care Campus, Haifa, Israel Gene and Cell Therapy

Larry B. Goldstein, MD Professor of Neurology, Director, Duke Stroke Center, Neurology, Duke University; Staff Neurologist, Durham VA Medical Center, Durham, North Carolina Approach to Cerebrovascular Diseases; Ischemic Cerebrovascular Disease

Susan I. Gerber, MD Team Lead, Respiratory Viruses/Picornaviruses, Division of Viral Diseases/Epidemiology Branch, National Center for Immunization and Respiratory Diseases, Centers for Disease Control and Prevention, Atlanta, Georgia Coronaviruses

Lawrence T. Goodnough, MD Professor of Pathology and Medicine, Stanford University; Director, Transfusion Service, Stanford University Medical Center, Stanford, California Transfusion Medicine

Dale N. Gerding, MD Professor of Medicine, Loyola University Chicago Stritch School of Medicine, Research Physician, Edward Hines, Jr. VA Hospital, Hines, Illinois Clostridial Infections Morie A. Gertz, MD Consultant, Division of Hematology, Mayo Clinic, Rochester, Minnesota; Roland Seidler, Jr. Professor of the Art of Medicine in Honor of Michael D. Brennan, MD, Professor of Medicine, Mayo Clinic, College of Medicine, Rochester, Minnesota Amyloidosis Gordon D. Ginder, MD Professor, Internal Medicine, Director, Massey Cancer Center, Virginia Commonwealth University, Richmond, Virginia Microcytic and Hypochromic Anemias Jeffrey S. Ginsberg, MD Professor of Medicine, McMaster University, Member of Thrombosis and Atherosclerosis Research Institute, St. Joseph’s Healthcare Hamilton, Hamilton, Ontario, Canada Peripheral Venous Disease Geoffrey S. Ginsburg, MD, PhD Director, Duke Center for Applied Genomics and Precision Medicine; Professor of Medicine, Pathology and Biomedical Engineering, Duke University, Durham, North Carolina Applications of Molecular Technologies to Clinical Medicine Michael Glogauer, DDS, PhD Professor, Faculty of Dentistry, University of Toronto, Toronto, Ontario, Canada Disorders of Phagocyte Function John W. Gnann, Jr., MD Professor of Medicine, Department of Medicine, Division of Infectious Diseases, Medical University of South Carolina, Charleston, South Carolina Mumps Matthew R. Golden, MD, MPH Professor of Medicine, University of Washington, Director, HIV/STD Program, Public Health–Seattle & King County, Seattle, Washington Neisseria Gonorrhoeae Infections Lee Goldman, MD Harold and Margaret Hatch Professor, Executive Vice President and Dean of the Faculties of Health Sciences and Medicine, Chief Executive, Columbia University Medical Center, Columbia University, New York, New York Approach to Medicine, the Patient, and the Medical Profession: Medicine as a Learned and Humane Profession; Approach to the Patient with Possible Cardiovascular Disease Ellie J.C. Goldstein, MD Clinical Professor of Medicine, David Geffen School of Medicine at University of California Los Angeles, Los Angeles, California; Director, R.M. Alden Research Laboratory, Santa Monica, California Diseases Caused by Non–Spore-Forming Anaerobic Bacteria

Eduardo H. Gotuzzo, MD Professor of Medicine, Director, Alexander von Humboldt Tropical Medicine Institute, Universidad Peruana Cayetano Heredia; Chief Physician, Department of Infectious, Tropical, and Dermatologic Diseases, National Hospital Cayetano Heredia, Lima, Peru Cholera and Other Vibrio Infections; Liver, Intestinal, and Lung Fluke Infections Deborah Grady, MD, MPH Professor of Medicine, University of California San Francisco, San Francisco, California Menopause Leslie C. Grammer, MD Professor of Medicine, Northwestern University Feinberg School of Medicine; Attending Physician, Northwestern Memorial Hospital, Chicago, Illinois Drug Allergy F. Anthony Greco, MD Medical Director, Sarah Cannon Cancer Center, Nashville, Tennessee Cancer of Unknown Primary Origin Harry B. Greenberg, MD Professor, Departments of Medicine and Microbiology and Immunology, Stanford University School of Medicine, Stanford, California Rotaviruses, Noroviruses, and Other Gastrointestinal Viruses Steven A. Greenberg, MD Associate Professor of Neurology, Harvard Medical School; Associate Neurologist, Brigham and Women’s Hospital, Boston, Massachusetts Inflammatory Myopathies Robert C. Griggs, MD Professor of Neurology, Medicine, Pediatrics, and Pathology and Laboratory Medicine, University of Rochester School of Medicine and Dentistry, Rochester, New York Approach to the Patient with Neurologic Disease Lev M. Grinberg, MD, PhD Professor, Chief, Department of Pathology, Ural Medical University; Chief Researcher of the Ural Scientific Research Institute of Phthisiopulmonology, Chief Pathologist of Ekaterinburg, Ekaterinburg, Russia Anthrax Daniel Grossman, MD Vice President for Research, Ibis Reproductive Health, Oakland, California; Assistant Clinical Professor, Bixby Center for Global Reproductive Health, Department of Obstetrics, Gynecology and Reproductive Sciences, University of California San Francisco, San Francisco, California Contraception Lisa M. Guay-Woodford, MD Hudson Professor of Pediatrics, The George Washington University; Director, Center for Translational Science, Director, Clinical and Translational Institute at Children’s National, Children’s National Health System, Washington, D.C. Hereditary Nephropathies and Developmental Abnormalities of the Urinary Tract

Contributors Richard L. Guerrant, MD Thomas H. Hunter Professor of International Medicine, Founding Director, Center for Global Health, Division of Infectious Diseases and International Health, University of Virginia School of Medicine, University of Virginia Health Sciences Center, Charlottesville, Virginia Cryptosporidiosis Roy M. Gulick, MD, MPH Gladys and Roland Harrison Professor of Medicine, Medicine/Infectious Diseases, Weill Cornell Medical College; Attending Physician, New York– Presbyterian Hospital, New York, New York Antiretrovial Therapy of HIV/AIDS Klaus D. Hagspiel, MD Professor of Radiology, Medicine, and Pediatrics, Chief, Noninvasive Cardiovascular Imaging, University of Virginia Health System, Charlottesville, Virginia Noninvasive Cardiac Imaging John D. Hainsworth, MD Chief Scientific Officer, Sarah Cannon Research Institute, Nashville, Tennessee Cancer of Unknown Primary Origin Anders Hamsten, MD, PhD Professor of Cardiovascular Diseases, Center for Molecular Medicine and Department of Cardiology, Karolinska University Hospital, Department of Medicine, Karolinska Institute, Stockholm, Sweden Atherosclerosis, Thrombosis, and Vascular Biology Kenneth R. Hande, MD Professor of Medicine and Pharmacology, Vanderbilt/Ingram Cancer Center, Vanderbilt University School of Medicine, Nashville, Tennessee Neuroendocrine Tumors and the Carcinoid Syndrome H. Hunter Handsfield, MD Professor Emeritus of Medicine, University of Washington Center for AIDS and STD, Seattle, Washington Neisseria Gonorrhoeae Infections Göran K. Hansson, MD, PhD Professor of Cardiovascular Research, Center for Molecular Medicine at Karolinska University Hospital, Department of Medicine, Karolinska Institute, Stockholm, Sweden Atherosclerosis, Thrombosis, and Vascular Biology Raymond C. Harris, MD Professor of Medicine, Ann and Roscoe R. Robinson Chair in Nephrology, Chief, Division of Nephrology, Vanderbilt University School of Medicine, Nashville, Tennessee Diabetes and the Kidney Stephen Crane Hauser, MD Associate Professor of Medicine, Internal Medicine, Division of Gastroenterology and Hepatology, Mayo Clinic College of Medicine, Rochester, Minnesota Vascular Diseases of the Gastrointestinal Tract Frederick G. Hayden, MD Stuart S. Richardson Professor of Clinical Virology and Professor of Medicine, University of Virginia School of Medicine; Staff Physician, University of Virginia Health System, Charlottesville, Virginia Influenza Douglas C. Heimburger, MD, MS Professor of Medicine, Associate Director for Education and Training, Vanderbilt University School of Medicine, Vanderbilt Institute for Global Health, Nashville, Tennessee Nutrition’s Interface with Health and Disease

xvii

Erik L. Hewlett, MD Professor of Medicine and of Microbiology, Immunology, and Cancer Biology, University of Virginia School of Medicine, University of Virginia Health System, Charlottesville, Virginia Whooping Cough and Other Bordetella Infections Richard J. Hift, PhD, MMed School of Clinical Medicine, University of KwaZulu-Natal, Durban, South Africa The Porphyrias David R. Hill, MD, DTM&H Professor of Medical Sciences, Director of Global Public Health, Frank H. Netter MD School of Medicine at Quinnipiac University, Hamden, Connecticut Giardiasis Nicholas S. Hill, MD Professor of Medicine, Tufts University School of Medicine; Chief, Division of Pulmonary, Critical Care, and Sleep Medicine, Tufts Medical Center, Boston, Massachusetts Respiratory Monitoring in Critical Care L. David Hillis, MD Professor and Chair, Department of Medicine, University of Texas Health Science Center at San Antonio, San Antonio, Texas Acute Coronary Syndrome: Unstable Angina and Non-ST Elevation Myocardial Infarction Jack Hirsh, CM, MD, DSc Professor Emeritus, McMaster University, Hamilton, Ontario, Canada Antithrombotic Therapy Steven M. Holland, MD Chief, Laboratory of Clinical Infectious Diseases, National Institute of Allergy and Infectious Diseases, National Institutes of Health, Bethesda, Maryland The Nontuberculous Mycobacteria Steven M. Hollenberg, MD Professor of Medicine, Cooper Medical School of Rowan University; Director, Coronary Care Unit, Cooper University Hospital, Camden, New Jersey Cardiogenic Shock Edward W. Hook III, MD Professor and Director, Division of Infectious Diseases, University of Alabama at Birmingham, Birmingham, Alabama Granuloma Inguinale (Donovanosis); Syphilis; Nonsyphilitic Treponematoses David J. Hunter, MBBS, MPH, ScD Vincent L. Gregory Professor of Cancer Prevention, Harvard School of Public Health; Professor of Medicine, Harvard Medical School, Brigham and Women’s Hospital, Boston, Massachusetts The Epidemiology of Cancer Khalid Hussain, MBChB, MD, MSc Developmental Endocrinology Research Group, Clinical and Molecular Genetics Unit, Institute of Child Health, University College London, Department of Paediatric Endocrinology, Great Ormond Street Hospital for Children, London, United Kingdom Hypoglycemia/Pancreatic Islet Cell Disorders Steven E. Hyman, MD Director, Stanley Center for Psychiatric Research, Broad Institute, Distinguished Service Professor of Stem Cell and Regenerative Biology, Harvard University, Cambridge, Massachusetts Biology of Addiction

xviii

Contributors

Michael C. Iannuzzi, MD, MBA Chairman, Department of Internal Medicine, State University of New York Upstate Medical University, Syracuse, New York Sarcoidosis

Richard C. Jordan, DDS, PhD Professor of Oral Pathology, Pathology and Radiation Oncology, University of California San Francisco, San Francisco, California Diseases of the Mouth and Salivary Glands

Robert D. Inman, MD Professor of Medicine and Immunology, University of Toronto; Staff Rheumatologist, University Health Network, Toronto, Ontario, Canada The Spondyloarthropathies

Ralph F. Józefowicz, MD Professor, Neurology and Medicine, University of Rochester, Rochester, New York Approach to the Patient with Neurologic Disease

Sharon K. Inouye, MD, MPH Professor of Medicine, Harvard Medical School; Director, Aging Brain Center, Institute for Aging Research, Hebrew SeniorLife, Boston, Massachusetts Neuropsychiatric Aspects of Aging; Delirium or Acute Mental Status Change in the Older Patient

Stephen G. Kaler, MD Senior Investigator and Head, Section on Translational Neuroscience, Molecular Medicine Program, Eunice Kennedy Shriver National Institute of Child Health and Human Development, Bethesda, Maryland Wilson Disease

Geoffrey K. Isbister, MD, BSc Associate Professor, Clinical Toxicologist, Calvary Mater Newcastle, Callaghan, Senior Research Academic, School of Medicine and Public Health, University of Newcastle, New South Wales, Australia Envenomation Michael G. Ison, MD, MS Associate Professor in Medicine-Infectious Diseases and Surgery-Organ Transplantation, Northwestern University Feinberg School of Medicine, Chicago, Illinois Adenovirus Diseases Elias Jabbour, MD Associate Professor, Department of Leukemia, Division of Medicine, The University of Texas MD Anderson Cancer Center, Houston, Texas The Chronic Leukemias Michael R. Jaff, DO Professor of Medicine, Harvard Medical School, Chair, Institute for Heart, Vascular, and Stroke Care, Massachusetts General Hospital, Boston, Massachusetts Other Peripheral Arterial Diseases Joanna C. Jen, MD, PhD Professor of Neurology, University of California Los Angeles School of Medicine, Los Angeles, California Neuro-Ophthalmology; Smell and Taste; Hearing and Equilibrium Dennis M. Jensen, MD Professor of Medicine, David Geffen School of Medicine at University of California Los Angeles; Staff Physician, Medicine-GI, VA Greater Los Angeles Healthcare System; Key Investigator, Director, Human Studies Core & GI Hemostasis Research Unit, CURE Digestive Diseases Research Center, Los Angeles, California Gastrointestinal Hemorrhage Michael D. Jensen, MD Professor of Medicine, Endocrine Research Unit, Director, Obesity Treatment Research Program, Mayo Clinic, Rochester, Minnesota Obesity Robert T. Jensen, MD Chief, Cell Biology Section, Digestive Disease Branch, National Institute of Diabetes and Digestive and Kidney Diseases, National Institutes of Health, Clinical Center, Bethesda, Maryland Pancreatic Neuroendocrine Tumors Stuart Johnson, MD Professor of Medicine, Loyola University Chicago Stritch School of Medicine; Associate Chief of Staff for Research, Edward Hines, Jr. VA Hospital, Hines, Illinois Clostridial Infections

Moses R. Kamya, MB ChB, MMed, MPH, PhD Chairman, Department of Medicine, Makerere University College of Health Sciences, Kampala, Uganda Malaria Louise W. Kao, MD Associate Professor of Emergency Medicine, Department of Emergency Medicine, Indiana University School of Medicine, Indianapolis, Indiana Chronic Poisoning: Trace Metals and Others Steven A. Kaplan, MD E. Darracott Vaughan, Jr. Professor of Urology, Chief, Institute for Bladder and Prostate Health, Weill Cornell Medical College; Director, Iris Cantor Men’s Health Center, NewYork–Presbyterian Hospital, New York, New York Benign Prostatic Hyperplasia and Prostatitis Daniel L. Kastner, MD, PhD Scientific Director, National Human Genome Research Institute, National Institutes of Health, Bethesda, Maryland The Systemic Autoinflammatory Diseases Sekar Kathiresan, MD Associate Professor in Medicine, Harvard Medical School; Director, Preventive Cardiology, Massachusetts General Hospital, Boston, Massachusetts The Inherited Basis of Common Diseases David A. Katzka, MD Professor of and Consultant in Medicine, Gastroenterology, Mayo Clinic, Rochester, Minnesota Diseases of the Esophagus Debra K. Katzman, MD Professor of Pediatrics, Senior Associate Scientist, The Research Institute, The Hospital for Sick Children and University of Toronto, Toronto, Ontario, Canada Adolescent Medicine Carol A. Kauffman, MD Professor of Internal Medicine, University of Michigan Medical School; Chief, Infectious Diseases Section, Veterans Affairs Ann Arbor Healthcare System, Ann Arbor, Michigan Histoplasmosis; Blastomycosis; Paracoccidioidomycosis; Cryptococcosis; Sporotrichosis; Candidiasis Kenneth Kaushansky, MD Senior Vice President for Health Sciences, Dean, School of Medicine, Stony Brook University, Stony Brook, New York Hematopoiesis and Hematopoietic Growth Factors Keith S. Kaye, MD, MPH Professor of Medicine, Division of Infectious Diseases, Wayne State University School of Medicine, Detroit, Michigan Diseases Caused by Acinetobacter and Stenotrophomonas Species

Contributors Armand Keating, MD Professor of Medicine, Director, Division of Hematology, Epstein Chair in Cell Therapy and Transplantation, Professor, Institute of Biomaterials and Biomedical Engineering, University of Toronto, Toronto, Ontario, Canada Hematopoietic Stem Cell Transplantation Robin K. Kelley, MD Assistant Professor of Medicine, University of California San Francisco, Helen Diller Family Comprehensive Cancer Center, San Francisco, California Liver and Biliary Tract Cancers Morton Kern, MD Chief of Medicine, VA Long Beach Health Care System School of Medicine; Professor of Medicine, Associate Chief, Cardiology, University of California–Irvine, Irvine, California Catheterization and Angiography Gerald T. Keusch, MD Professor of Medicine and International Health and Public Health, Boston University School of Medicine, Boston, Massachusetts Shigellosis Fadlo R. Khuri, MD Professor and Chair, Hematology and Medical Oncology, Deputy Director, Winship Cancer Institute, Emory University, Atlanta, Georgia Lung Cancer and Other Pulmonary Neoplasms David H. Kim, MD Vice Chair of Education, Professor of Radiology, Section of Abdominal Imaging, University of Wisconsin School of Medicine and Public Health, Madison, Wisconsin Diagnostic Imaging Procedures in Gastroenterology

xix

Kevin M. Korenblat, MD Associate Professor of Medicine, Department of Medicine, Washington University School of Medicine, St. Louis, Missouri Approach to the Patient with Jaundice or Abnormal Liver Tests Bruce R. Korf, MD, PhD Wayne H. and Sara Crews Finley Chair in Medical Genetics, Professor and Chair, Department of Genetics, University of Alabama at Birmingham, Birmingham, Alabama Principles of Genetics Neil J. Korman, MD, PhD Professor, Dermatology, Case Western Reserve University School of Medicine, University Hospitals Case Medical Center, Cleveland, Ohio Macular, Papular, Vesiculobullous, and Pustular Diseases Mark G. Kortepeter, MD, MPH Associate Dean for Research, Associate Professor of Preventive Medicine and Medicine, Consultant to the Army Surgeon General for Biodefense; Office of the Dean, Edward Hébert School of Medicine, Uniformed Services University of the Health Sciences, Bethesda, Maryland Bioterrorism Joseph A. Kovacs, MD Senior Investigator and Head, AIDS Section, Critical Care Medicine Department, National Institutes of Health, Bethesda, Maryland Pneumocystis Pneumonia Thomas O. Kovacs, MD Professor of Medicine, Division of Digestive Diseases, David Geffen School of Medicine at University of California Los Angeles, Los Angeles, California Gastrointestinal Hemorrhage

Matthew Kim, MD Instructor of Medicine, Harvard Medical School; Associate Physician, Brigham and Women’s Hospital, Boston, Massachusetts Thyroid

Monica Kraft, MD Professor of Medicine, Duke University School of Medicine; Chief, Division of Pulmonary, Allergy, and Critical Care Medicine, Duke University Medical Center, Durham, North Carolina Approach to the Patient with Respiratory Disease

Louis V. Kirchhoff, MD, MPH Professor, Departments of Internal Medicine (Infectious Diseases) and Epidemiology, University of Iowa Health Care; Staff Physician, Medical Service, Department of Veterans Affairs Medical Center, Iowa City, Iowa Chagas Disease

Christopher M. Kramer, MD Ruth C. Heede Professor of Cardiology, Professor of Radiology, Director, Cardiovascular Imaging Center, University of Virginia Health System, Charlottesville, Virginia Noninvasive Cardiac Imaging

David S. Knopman, MD Professor of Neurology, Mayo Clinic College of Medicine, Rochester, Minnesota Regional Cerebral Dysfunction: Higher Mental Function; Alzheimer Disease and Other Dementias

Donna M. Krasnewich, MD, PhD Program Director, National Institute of General Medical Sciences, National Institutes of Health, Bethesda, Maryland The Lysosomal Storage Diseases

Tamsin A. Knox, MD, MPH Associate Professor of Medicine, Nutrition/Infection Unit, Tufts University School of Medicine, Boston, Massachusetts Gastrointestinal Manifestions of HIV and AIDS D.P. Kontoyiannis, MD, ScD Professor, Department of Infectious Diseases, Infection Control and Employee Health, The University of Texas MD Anderson Cancer Center, Houston, Texas Mucormycosis; Mycetoma Barbara S. Koppel, MD Professor of Clinical Neurology, New York Medical College, Chief of Neurology, Metropolitan Hospital Center, New York City Health and Hospital Corporation, New York, New York Nutritional and Alcohol-Related Neurologic Disorders

Peter J. Krause, MD Senior Research Scientist in Epidemiology, Medicine, and Pediatrics, Yale School of Public Health and Yale School of Medicine, New Haven, Connecticut Babesiosis and Other Protozoan Diseases John F. Kuemmerle, MD Chair, Division of Gastroenterology, Hepatology, and Nutrition, Professor of Medicine, and Physiology and Biophysics, Center for Digestive Health, Virginia Commonwealth University, Richmond, Virginia Inflammatory and Anatomic Diseases of the Intestine, Peritoneum, Mesentery, and Omentum Ernst J. Kuipers, MD, PhD Professor of Medicine, Department of Gastroenterology and Hepatology, Chief Executive Officer, Erasmus MC University Medical Center, Rotterdam, The Netherlands Acid Peptic Disease

xx

Contributors

Paul W. Ladenson, MD Professor of Medicine, Pathology, Oncology, and Radiology and Radiological Sciences, John Eager Howard Professor of Endocrinology and Metabolism, University Distinguished Service Professor, The Johns Hopkins University School of Medicine; Physician and Division Director, The Johns Hopkins Hospital, Baltimore, Maryland Thyroid Daniel Laheru, MD Ian T. MacMillan Professorship in Clinical Pancreatic Research, Medical Oncology, Johns Hopkins University School of Medicine, Baltimore, Maryland Pancreatic Cancer Donald W. Landry, MD, PhD Samuel Bard Professor of Medicine, Chair, Department of Medicine, Physician-in-Chief, NewYork-Presbyterian Hospital/Columbia University Medical Center, New York, New York Approach to the Patient with Renal Disease Anthony E. Lang, MD Director, Division of Neurology, Jack Clark Chair for Research in Parkinson’s Disease, University of Toronto; Director, Morton and Gloria Shulman Movement Disorders Clinic and the Edmond J. Safra Program in Parkinson’s Disease and the Lily Safra Chair in Movement Disorders, Toronto Western Hospital, Toronto, Ontario, Canada Parkinsonism; Other Movement Disorders

Gary R. Lichtenstein, MD Professor of Medicine, Perelman School of Medicine at the University of Pennsylvania, Director, Center for Inflammatory Bowel Disease, University of Pennsylvania, Philadelphia, Pennsylvania Inflammatory Bowel Disease Henry W. Lim, MD Chairman and C.S. Livingood Chair, Department of Dermatology, Henry Ford Hospital; Senior Vice President for Academic Affairs, Henry Ford Health System, Detroit, Michigan Eczemas, Photodermatoses, Papulosquamous (Including Fungal) Diseases, and Figurate Erythemas Aldo A.M. Lima, MD, PhD Professor of Medicine and Pharmacology, School of Medicine, Federal University of Ceará, Fortaleza, Ceará, Brazil Cryptosporidiosis; Amebiasis Geoffrey S.F. Ling, MD, PhD Professor of Neurology, Uniformed Services University of the Health Sciences, Bethesda, Maryland Traumatic Brain Injury and Spinal Cord Injury William C. Little, MD Patrick Lehan Professor of Cardiovascular Medicine, Chair, Department of Medicine, University of Mississippi Medical Center, Jackson, Mississippi Pericardial Diseases

Richard A. Lange, MD, MBA President and Dean, Paul L. Foster School of Medicine, Texas Tech University Health Sciences Center El Paso, El Paso, Texas Acute Coronary Syndrome: Unstable Angina and Non-ST Elevation Myocardial Infarction

Donald M. Lloyd-Jones, MD, ScM Senior Associate Dean, Chair, Department of Preventive Medicine, Eileen M. Foell Professor of Preventive Medicine and Medicine, Northwestern University Feinberg School of Medicine, Chicago, Illinois Epidemiology of Cardiovascular Disease

Frank A. Lederle, MD Core Investigator, Center for Chronic Disease Outcomes Research, Minneapolis VA Medical Center; Professor of Medicine, University of Minnesota School of Medicine, Minneapolis, Minnesota Diseases of the Aorta

Bennett Lorber, MD Thomas M. Durant Professor of Medicine and Professor of Microbiology and Immunology, Temple University School of Medicine, Philadelphia, Pennsylvania Listeriosis

Thomas H. Lee, MD, MSc Senior Physician, Department of Medicine, Brigham and Women’s Hospital; Chief Medical Officer, Press Ganey, Boston, Massachusetts Using Data for Clinical Decisions

Donald E. Low, MD† Nonpneumococcal Streptococcal Infections, Rheumatic Fever

William M. Lee, MD Professor of Internal Medicine, University of Texas Southwestern Medical Center, Dallas, Texas Toxin- and Drug-Induced Liver Disease James E. Leggett, MD Associate Professor, Department of Medicine, Oregon Health and Science University; Infectious Diseases, Department of Medical Education, Providence Portland Medical Center, Portland, Oregon Approach to Fever or Suspected Infection in the Normal Host Stuart Levin, MD Professor of Medicine, Emeritus Chairman, Department of Medicine, Rush University Medical Center, Chicago, Illinois Zoonoses Stephanie M. Levine, MD Professor of Medicine, Division of Pulmonary Diseases and Critical Care Medicine, The University of Texas Health Science Center San Antonio, South Texas Veterans Health Care System, San Antonio, Texas Alveolar Filling Disorders

Daniel R. Lucey, MD, MPH Adjunct Professor, Microbiology and Immunology, Georgetown University Medical Center, Washington, D.C. Anthrax James R. Lupski, MD, PhD Cullen Professor of Molecular and Human Genetics, Professor of Pediatrics, Baylor College of Medicine and Texas Children’s Hospital, Houston, Texas Gene, Genomic, and Chromosomal Disorders Jeffrey M. Lyness, MD Senior Associate Dean for Academic Affairs, Professor of Psychiatry and Neurology, University of Rochester School of Medicine and Dentistry, Rochester, New York Psychiatric Disorders in Medical Practice Bruce W. Lytle, MD Chair, Heart and Vascular Institute, Professor of Surgery, Thoracic and Cardiovascular Surgery, Cleveland Clinic, Cleveland, Ohio Interventional and Surgical Treatment of Coronary Artery Disease



Deceased.

Contributors C. Ronald MacKenzie, MD Assistant Attending Physician, Department of Medicine-Rheumatology, C. Ronald MacKenzie Chair in Ethics and Medicine, Hospital for Special Surgery, Associate Professor of Clinical Medicine and Medical Ethics, Weill Cornell Medical College of Cornell University, New York, New York Surgical Treatment of Joint Disease Harriet L. MacMillan, MD, MSc Professor, Departments of Psychiatry and Behavioural Neurosciences, and Pediatrics, Chedoke Health Chair in Child Psychiatry, Offord Centre for Child Studies, McMaster University, Hamilton, Ontario, Canada Intimate Partner Violence Robert D. Madoff, MD Professor of Surgery, Stanley M. Goldberg, MD, Chair, Colon and Rectal Surgery, University of Minnesota, Minneapolis, Minnesota Diseases of the Rectum and Anus Frank Maldarelli, MD, PhD Head, Clinical Retrovirology Section, HIV Drug Resistance Program, National Cancer Institute, National Institutes of Health, Bethesda, Maryland Biology of Human Immunodeficiency Viruses Atul Malhotra, MD Chief of Pulmonary and Critical Care, Kenneth M. Moser Professor of Medicine, Director of Sleep Medicine, University of California San Diego, La Jolla, California Disorders of Ventilatory Control Mark J. Manary, MD Helene B. Roberson Professor of Pediatrics, Washington University School of Medicine; Attending Physician, St. Louis Children’s Hospital, St. Louis, Missouri; Adjunct Professor, Children’s Nutrition Research Center, Baylor College of Medicine, Houston, Texas; Senior Lecturer in Community Health, University of Malawi College of Medicine, Blantyre, Malawi Protein-Energy Malnutrition Donna Mancini, MD Professor of Medicine, Department of Medicine, Division of Cardiology, Columbia University College of Physicians and Surgeons, Center for Advanced Cardiac Care, Columbia University Medical Center, New York, New York Cardiac Transplantation Lionel A. Mandell, MD Professor of Medicine, Faculty of Health Sciences, McMaster University, Hamilton, Ontario, Canada Streptococcus Pneumoniae Infections Peter Manu, MD Professor of Medicine and Psychiatry, Hofstra North Shore–LIJ School of Medicine at Hofstra University, Hempstead, New York; Adjunct Professor of Clinical Medicine, Psychiatry and Behavioral Sciences, Albert Einstein College of Medicine, Bronx, New York; Director of Medical Services, Zucker Hillside Hospital, Glen Oaks, New York Medical Consultation in Psychiatry Ariane Marelli, MD, MPH Professor of Medicine, McGill University, Director, McGill Adult Unit for Congenital Heart Disease, Associate Director, Academic Affairs and Research, Cardiology, McGill University Health Centre, Montreal, Québec, Canada Congenital Heart Disease in Adults Xavier Mariette, MD, PhD Professor, Rheumatology, Université Paris-Sud, AP-HP, Le Kremlin Bicêtre, France Sjögren Syndrome

xxi

Andrew R. Marks, MD Wu Professor and Chair, Department of Physiology and Cellular Biophysics, Founding Director, Helen and Clyde Wu Center for Molecular Cardiology, Columbia University College of Physicians and Surgeons, New York, New York Cardiac Function and Circulatory Control Kieren A. Marr, MD Professor of Medicine and Oncology, The Johns Hopkins University, Director, Transplant and Oncology Infectious Diseases, Baltimore, Maryland Approach to Fever and Suspected Infection in the Compromised Host Thomas J. Marrie, MD Dean, Faculty of Medicine, Dalhousie University; Professor of Medicine, Capital District Health Authority, Halifax, Nova Scotia, Canada Legionella Infections Paul Martin, MD Professor of Medicine and Chief, Division of Hepatology, Miller School of Medicine, University of Miami, Miami, Florida Approach to the Patient with Liver Disease Joel B. Mason, MD Professor of Medicine and Nutrition, Tufts University; Staff Physician, Divisions of Gastroenterology and Clinical Nutrition, Tufts Medical Center, Boston, Massachusetts Vitamins, Trace Minerals, and Other Micronutrients Henry Masur, MD Chief, Critical Care Medicine Department, Clinical Center, National Institutes of Health, Bethesda, Maryland Infectious and Metabolic Complications of HIV and AIDS Eric L. Matteson, MD, MPH Professor of Medicine, Mayo Clinic College of Medicine, Consultant, Divisions of Rheumatology and Epidemiology, Mayo Clinic, Rochester, Minnesota Infections of Bursae, Joints, and Bones Michael A. Matthay, MD Professor, Departments of Medicine and Anesthesia, University of California San Francisco, San Francisco, California Acute Respiratory Failure Toby A. Maurer, MD Professor of Dermatology, University of California San Francisco; Chief of Dermatology, San Francisco General Hospital, San Francisco, California Skin Manifestations in Patients with Human Immunodeficiency Virus Infection Emeran A. Mayer, MD, PhD Professor of Medicine, Physiology, and Psychiatry, Division of Digestive Diseases, Department of Medicine, University of California Los Angeles, Los Angeles, California Functional Gastrointestinal Disorders: Irritable Bowel Syndrome, Dyspepsia, Chest Pain of Presumed Esophageal Origin, and Heartburn Stephan A. Mayer, MD Director, Institute for Critical Care Medicine, Icahn School of Medicine at Mount Sinai, New York, New York Hemorrhagic Cerebrovascular Disease Stephen A. McClave, MD Professor of Medicine, Director of Clinical Nutrition, University of Louisville School of Medicine, Louisville, Kentucky Enteral Nutrition F. Dennis McCool, MD Professor of Medicine, The Warren Alpert Medical School of Brown University; Medical Director of Sleep Center, Memorial Hospital of Rhode Island, Pawtucket, Rhode Island Diseases of the Diaphragm, Chest Wall, Pleura, and Mediastinum

xxii

Contributors

Charles E. McCulloch, PhD Professor of Biostatistics, Department of Epidemiology and Biostatistics, University of California San Francisco, San Francisco, California Statistical Interpretation of Data

Ernest Moy, MD, MPH Medical Officer, Center for Quality Improvement and Patient Safety Agency for Healthcare Research and Quality, Rockville, Maryland Measuring Health and Health Care

William J. McKenna, MD Professor of Cardiology, Institute of Cardiovascular Science, University College London, London, United Kingdom Diseases of the Myocardium and Endocardium

Atis Muehlenbachs, MD, PhD Infectious Diseases Pathology Branch, Centers for Disease Control and Prevention, Atlanta, Georgia Leptospirosis

Vallerie McLaughlin, MD Kim A. Eagle, MD, Endowed Professor of Cardiovascular Medicine, Director, Pulmonary Hypertension Program, University of Michigan, Ann Arbor, Michigan Pulmonary Hypertension

Andrew H. Murr, MD Professor and Chairman, Roger Boles, MD Endowed Chair in Otolaryngology Education, Department of Otolaryngology-Head and Neck Surgery, University of California San Francisco School of Medicine, San Francisco, California Approach to the Patient with Nose, Sinus, and Ear Disorders

John J.V. McMurray, MB, MD Professor of Cardiology, Institute of Cardiovascular and Medical Sciences, University of Glasgow, Glasgow, Scotland, United Kingdom Heart Failure: Management and Prognosis Kenneth R. McQuaid, MD Professor of Clinical Medicine, Marvin H. Sleisenger Endowed Chair, Vice Chairman, University of California San Francisco; Chief, Medical Services and Gastroenterology, San Francisco VA Medical Center, San Francisco, California Approach to the Patient with Gastrointestinal Disease Marc Michel, MD Professor of Internal Medicine, Head of the Unit of Internal Medicine at Henri Mondor University Hospital, National Referral Center for Adult’s Immune Cytopenias, Creteil, France Autoimmune and Intravascular Hemolytic Anemias Jonathan W. Mink, MD, PhD Frederick A. Horner, MD Endowed Professor in Pediatric Neurology, Professor of Neurology, Neurobiology & Anatomy, Brain & Cognitive Sciences, and Pediatrics, Chief, Division of Child Neurology, Vice Chair, Department of Neurology, University of Rochester, Rochester, New York Congenital, Developmental, and Neurocutaneous Disorders William E. Mitch, MD Gordon A. Cain Chair in Nephrology, Director of Nephrology, Baylor College of Medicine, Houston, Texas Chronic Kidney Disease Mark E. Molitch, MD Martha Leland Sherwin Professor of Endocrinology, Division of Endocrinology, Metabolism, and Molecular Medicine, Northwestern University Feinberg School of Medicine, Chicago, Illinois Neuroendocrinology and the Neuroendocrine System; Anterior Pituitary Bruce A. Molitoris, MD Professor of Medicine, and Cellular and Integrative Physiology Director, Indiana Center for Biological Microscopy, Indiana University, Indianapolis, Indiana Acute Kidney Injury Jose G. Montoya, MD Professor of Medicine, Division of Infectious Disease and Geographic Medicine, Stanford University School of Medicine, Stanford, California; Director, Palo Alto Medical Foundation Toxoplasma Serology Laboratory, National Reference Center for the Study and Diagnosis of Toxoplasmosis, Palo Alto, California Toxoplasmosis Alison Morris, MD, MS Associate Professor of Medicine, Clinical Translational Science, and Immunology, Division of Pulmonary, Allergy, and Critical Care Medicine, University of Pittsburgh School of Medicine, Pittsburgh, Pennsylvania Pulmonary Manifestations of Human Immunodeficiency Virus and Acquired Immunodeficiency Syndrome

Daniel M. Musher, MD Professor of Medicine, Molecular Virology, and Microbiology, Distinguished Service Professor, Baylor College of Medicine, Infectious Disease Section, Michael E. DeBakey Veterans Affairs Medical Center, Houston, Texas Overview of Pneumonia Robert J. Myerburg, MD Professor of Medicine and Physiology, Division of Cardiology, Department of Medicine, American Heart Association Chair in Cardiovascular Research, University of Miami Miller School of Medicine, Miami, Florida Approach to Cardiac Arrest and Life-Threatening Arrhythmias Sandesh C.S. Nagamani, MD Assistant Professor, Department of Molecular and Human Genetics, Director, Clinic for Metabolic and Genetic Disorders of Bone, Baylor College of Medicine and Texas Children’s Hospital, Houston, Texas Gene, Genomic, and Chromosomal Disorders Stanley J. Naides, MD Medical Director and Interim Scientific Director, Immunology, Quest Diagnostics Nichols Institute, San Juan Capistrano, California Arboviruses Causing Fever and Rash Syndromes Yoshifumi Naka, MD, PhD Professor of Surgery, Department of Surgery, Columbia University College of Physicians and Surgeons, New York, New York Cardiac Transplantation Theodore E. Nash, MD Principal Investigator, Clinical Parasitology Section, Laboratory of Parasitic Diseases, National Institute of Allergy and Infectious Diseases, National Institutes of Health, Bethesda, Maryland Giardiasis Avindra Nath, MD Chief, Section of Infections of the Nervous System, National Institute of Neurological Disorders and Stroke, National Institutes of Health, Bethesda, Maryland Cytomegalovirus, Epstein-Barr Virus, and Slow Virus Infections of the Central Nervous System; Neurologic Complications of Human Immunodeficiency Virus Infection; Meningitis: Bacterial, Viral, and Other; Brain Abscess and Parameningeal Infections Eric G. Neilson, MD Vice President for Medical Affairs and Lewis Landsberg Dean, Northwestern University Feinberg School of Medicine, Northwestern Memorial Hospital, Chicago, Illinois Tubulointerstitial Nephritis

Contributors Lawrence S. Neinstein, MD Professor of Pediatrics and Medicine, Keck School of Medicine of USC; Executive Director, Engemann Student Health Center, Division Head of College Health, Assistant Provost, Student Health and Wellness, University of Southern California, Los Angeles, California Adolescent Medicine Lewis S. Nelson, MD Professor of Emergency Medicine, Director, Fellowship in Medical Toxicology, New York University School of Medicine; Attending Physician, New York University Langone Medical Center and Bellevue Hospital Center, New York, New York Acute Poisoning Eric J. Nestler, MD, PhD Nash Family Professor and Chair, Department of Neuroscience, Director, The Friedman Brain Institute, Icahn School of Medicine at Mount Sinai, New York, New York Biology of Addiction Anne B. Newman, MD, MPH Professor of Epidemiology, The University of Pittsburgh Graduate School of Public Health; Chair, Department of Epidemiology, Director, University of Pittsburgh Center for Aging and Population Health, Pittsburgh, Pennsylvania Epidemiology of Aging: Implications of the Aging of Society Thomas B. Newman, MD, MPH Professor, Epidemiology & Biostatistics and Pediatrics, University of California San Francisco, San Francisco, California Statistical Interpretation of Data William L. Nichols, MD Associate Professor, Medicine and Laboratory Medicine, Mayo Clinic College of Medicine; Staff Physician, Special Coagulation Laboratory, Comprehensive Hemophilia Center, and Coagulation Clinic, Mayo Clinic, Rochester, Minnesota Von Willebrand Disease and Hemorrhagic Abnormalities of Platelet and Vascular Function Lindsay E. Nicolle, MD Professor of Internal Medicine and Medical Microbiology, University of Manitoba, Health Sciences Centre, Winnipeg, Manitoba, Canada Approach to the Patient with Urinary Tract Infection Lynnette K. Nieman, MD Senior Investigator, Program on Reproductive and Adult Endocrinology, Eunice Kennedy Shriver National Institute of Child Health and Human Development, Bethesda, Maryland Approach to the Patient with Endocrine Disease; Adrenal Cortex; Polyglandular Disorders Dennis E. Niewoehner, MD Professor of Medicine, University of Minnesota; Staff Physician, Minneapolis Veterans Affairs Health Care System, Minneapolis, Minnesota Chronic Obstructive Pulmonary Disease S. Ragnar Norrby, MD, PhD Director General, Swedish Institute for Infectious Disease Control, Solna, Sweden Approach to the Patient with Urinary Tract Infection Susan O’Brien, MD Professor, Department of Leukemia, Division of Medicine, The University of Texas MD Anderson Cancer Center, Houston, Texas The Chronic Leukemias Christopher M. O’Connor, MD Professor of Medicine and Chief, Division of Cardiology, Director, Duke Heart Center, Durham, North Carolina Heart Failure: Pathophysiology and Diagnosis

xxiii

Francis G. O’Connor, MD, MPH Professor and Chair, Military and Emergency Medicine, Medical Director, Uniformed Services University Consortium for Health and Military Performance, Bethesda, Maryland Disorders Due to Heat and Cold; Rhabdomyolysis Patrick G. O’Connor, MD, MPH Professor and Chief, General Internal Medicine, Yale University School of Medicine, New Haven, Connecticut Alcohol Abuse and Dependence James R. O’Dell, MD Bruce Professor and Vice Chair of Internal Medicine, Chief, Division of Rheumatology, University of Nebraska Medical Center and Omaha VA Nebraska–Western Iowa Health Care System, Omaha, Nebraska Rheumatoid Arthritis Anne E. O’Donnell, MD Professor of Medicine, Chief, Division of Pulmonary, Critical Care, and Sleep Medicine, Georgetown University Medical Center, Washington, D.C. Bronchiectasis, Atelectasis, Cysts, and Localized Lung Disorders Jae K. Oh, MD Professor of Medicine, Director, Echocardiography Core Laboratory and Pericardial Clinic, Division of Cardiovascular Diseases, Co-Director, Integrated Cardiac Imaging, Division of Cardiovascular Diseases, Mayo Clinic, Rochester, Minnesota Pericardial Diseases Jeffrey E. Olgin, MD Gallo-Chatterjee Distinguished Professor of Medicine, Chief, Division of Cardiology, Co-Director, Heart and Vascular Center, University of California San Francisco, San Francisco, California Approach to the Patient with Suspected Arrhythmia Walter A. Orenstein, MD Professor of Medicine, Pediatrics, and Global Health, Emory University School of Medicine, Atlanta, Georgia Immunization Douglas R. Osmon, MD, MPH Professor of Medicine, Mayo Clinic College of Medicine; Consultant, Division of Infectious Diseases, Mayo Clinic, Rochester, Minnesota Infections of Bursae, Joints, and Bones Catherine M. Otto, MD J. Ward Kennedy-Hamilton Endowed Chair in Cardiology, Professor of Medicine, University of Washington School of Medicine; Director, Heart Valve Clinic, University of Washington Medical Center, Seattle, Washington Echocardiography Mark Papania, MD, MPH Medical Epidemiologist, Division of Viral Diseases, Measles, Mumps, Rubella, and Herpes Virus Laboratory Branch, Centers for Disease Control and Prevention, Atlanta, Georgia Measles Peter G. Pappas, MD Professor of Medicine, University of Alabama at Birmingham, Birmingham, Alabama Dematiaceous Fungal Infections Pankaj Jay Pasricha, MD Director, The Johns Hopkins Center for Neurogastroenterology; Professor of Medicine and Neurosciences, The Johns Hopkins School of Medicine; Professor of Innovation Management, Johns Hopkins Carey Business School, Baltimore, Maryland Gastrointestinal Endoscopy

xxiv

Contributors

David L. Paterson, MD Professor of Medicine, University of Queensland Centre for Clinical Research, Royal Brisbane and Women’s Hospital Campus, Brisbane, Queensland, Australia Infections Due to Other Members of the Enterobacteriaceae, Including Management of Multidrug Resistant Strains Carlo Patrono, MD Professor and Chair of Pharmacology, Department of Pharmacology, Catholic University School of Medicine, Rome, Italy Prostaglandin, Aspirin, and Related Compounds Jean-Michel Pawlotsky, MD, PhD Professor of Medicine, The University of Paris-Est; Director, National Reference Center for Viral Hepatitis B, C, and Delta and Department of Virology, Henri Mondor University Hospital; Director, Department of Molecular Virology and Immunology, Institut Mondor de Recherche Biomédicale, Créteil, France Acute Viral Hepatitis; Chronic Viral and Autoimmune Hepatitis Richard D. Pearson, MD Professor of Medicine and Pathology, University of Virginia School of Medicine and University of Virginia Health System, Charlottesville, Virginia Antiparasitic Therapy Trish M. Perl, MD, MSc Professor of Medicine and Pathology, The Johns Hopkins School of Medicine; Professor of Epidemiology, Johns Hopkins Bloomberg School of Public Health; Infectious Diseases Specialist and Senior Epidemiologist, The Johns Hopkins Hospital and Health System, Baltimore, Maryland Enterococcal Infections Adam Perlman, MD, MPH Associate Professor, Department of Medicine, Duke University Medical Center; Executive Director, Duke Integrative Medicine, Duke University Health System, Durham, North Carolina Complementary and Alternative Medicine William A. Petri, Jr., MD, PhD Wade Hampton Frost Professor, Departments of Medicine, Pathology, Microbiology, Immunology, and Cancer Biology, School of Medicine, University of Virginia; Chief, Division of Infectious Diseases and International Health, University of Virginia Hospitals, Charlottesville, Virginia Relapsing Fever and Other Borrelia Infections; African Sleeping Sickness; Amebiasis Marc A. Pfeffer, MD, PhD Dzau Professor of Medicine, Harvard Medical School; Senior Physician, Cardiovascular Division, Brigham and Women’s Hospital, Boston, Massachusetts Heart Failure: Management and Prognosis Perry J. Pickhardt, MD Professor of Radiology and Chief, Gastrointestinal Imaging, Section of Abdominal Imaging, University of Wisconsin School of Medicine and Public Health, Madison, Wisconsin Diagnostic Imaging Procedures in Gastroenterology David S. Pisetsky, MD, PhD Chief of Rheumatology, Medical Research Service, Durham VA Medical Center; Professor of Medicine and Immunology, Department of Medicine, Duke University Medical Center, Durham, North Carolina Laboratory Testing in the Rheumatic Diseases Marshall R. Posner, MD Professor of Medicine, Director of Head and Neck Medical Oncology, Director of the Office of Cancer Clinical Trials, The Tisch Cancer Institute, Icahn School of Medicine at Mount Sinai, New York, New York Head and Neck Cancer

Frank Powell, PhD Professor of Medicine, Chief of Physiology, University of California San Diego, La Jolla, California Disorders of Ventilatory Control Reed E. Pyeritz, MD, PhD William Smilow Professor of Medicine and Genetics and Vice Chair for Academic Affairs, Perelman School of Medicine at the University of Pennsylvania, Philadelphia, Pennsylvania Inherited Diseases of Connective Tissue Thomas C. Quinn, MD, MSc Associate Director for International Research, Head, Section of International HIV/AIDS Research, Division of Intramural Research, National Institute of Allergy and Infectious Diseases, National Institutes of Health; Professor of Medicine, Pathology, International Health, Molecular Microbiology and Immunology, and Epidemiology, The Johns Hopkins Medical Institutions, Baltimore, Maryland Epidemiology and Diagnosis of Human Immunodeficiency Virus Infection and Acquired Immunodeficiency Syndrome Jai Radhakrishnan, MD, MS Professor of Medicine, Division of Nephrology, Department of Medicine, Columbia University Medical Center; Associate Division Chief for Clinical Affairs, Division of Nephrology, New York-Presbyterian Hospital, New York, New York Glomerular Disorders and Nephrotic Syndromes Petros I. Rafailidis, MD, PhD, MSc Senior Researcher, Alfa Institute of Biomedical Sciences, Attending Physician, Department of Medicine and Hematology, Athens Medical Center, Athens Medical Group, Athens, Greece Pseudomonas and Related Gram-Negative Bacillary Infections Ganesh Raghu, MD Adjunct Professor of Medicine and Laboratory Medicine, University of Washington, Director, CENTER for Interstitial Lung Diseases at the University of Washington; Co-Director, Scleroderma Clinic, University of Washington Medical Center, Seattle, Washington Interstitial Lung Disease Margaret Ragni, MD, MPH Professor of Medicine and Clinical Translational Science, Department of Hematology/Oncology, University of Pittsburgh Medical Center; Director, Hemophilia Center of Western Pennsylvania, Pittsburgh, Pennsylvania Hemorrhagic Disorders: Coagulation Factor Deficiencies Srinivasa N. Raja, MD Professor of Anesthesiology and Neurology, Director, Division of Pain Medicine, The Johns Hopkins University School of Medicine, Baltimore, Maryland Pain S. Vincent Rajkumar, MD Professor of Medicine, Division of Hematology, Mayo Clinic, Rochester, Minnesota Plasma Cell Disorders Stuart H. Ralston, MB ChB, MD Professor of Rheumatology, Institute of Genetics and Molecular Medicine, Western General Hospital, The University of Edinburgh, Edinburgh, United Kingdom Paget Disease of Bone Didier Raoult, MD, PhD Professor, Aix Marseille Université, Faculté de Médecine; Chief, Hôpital de la Timone, Fédération de Microbiologie Clinique, Marseille, France Bartonella Infections; Rickettsial Infections

Contributors Robert W. Rebar, MD Professor, Department of Obstetrics and Gynecology, Western Michigan University Homer Stryker MD School of Medicine, Kalamazoo, Michigan Ovaries and Development; Reproductive Endocrinology and Infertility Annette C. Reboli, MD Founding Vice Dean, Professor of Medicine, Cooper Medical School of Rowan University, Cooper University Healthcare, Department of Medicine, Division of Infectious Diseases, Camden, New Jersey Erysipelothrix Infections

xxv

Karen Rosene-Montella, MD Professor and Vice Chair of Medicine, Director of Obstetric Medicine, The Warren Alpert Medical School of Brown University; Senior Vice President, Women’s Services and Clinical Integration, Lifespan Health System, Providence, Rhode Island Common Medical Problems in Pregnancy Philip J. Rosenthal, MD Professor, Department of Medicine, University of California San Francisco, San Francisco, California Malaria

K. Rajender Reddy, MD Professor of Medicine, Professor of Medicine in Surgery, Perelman School of Medicine at the University of Pennsylvania; Director of Hepatology, Director, Viral Hepatitis Center, Hospital of the University of Pennsylvania, Philadelphia, Pennsylvania Bacterial, Parasitic, Fungal, and Granulomatous Liver Diseases

Marc E. Rothenberg, MD, PhD Director, Division of Allergy and Immunology, Director, Cincinnati Center for Eosinophilic Disorders; Professor of Pediatrics, Cincinnati Children’s Hospital Medical Center, University of Cincinnati College of Medicine, Cincinnati, Ohio Eosinophilic Syndromes

Donald A. Redelmeier, MD Professor of Medicine, University of Toronto; Senior Scientist and Staff Physician, Sunnybrook Health Sciences Centre, Toronto, Ontario, Canada Postoperative Care and Complications

James A. Russell, MD Professor of Medicine, University of British Columbia; Associate Director, Intensive Care Unit, St. Paul’s Hospital, Vancouver, British Columbia, Canada Shock Syndromes Related to Sepsis

Susan E. Reef, MD Centers for Disease Control and Prevention, Atlanta, Georgia Rubella (German Measles)

Anil K. Rustgi, MD T. Grier Miller Professor of Medicine and Genetics, Chief of Gastroenterology, American Cancer Society; Professor, Perelman School of Medicine at the University of Pennsylvania, Philadelphia, Pennsylvania Neoplasms of the Esophagus and Stomach

Neil M. Resnick, MD Thomas P. Detre Endowed Chair in Gerontology and Geriatric Medicine, Professor of Medicine and Division Chief, Geriatrics, Associate Director, University of Pittsburgh Institute on Aging, University of Pittsburgh; Chief, Division of Geriatric Medicine and Gerontology, University of Pittsburgh Medical Center, Pittsburgh, Pennsylvania Incontinence David B. Reuben, MD Director, Multicampus Program in Geriatric Medicine and Gerontology; Chief, Division of Geriatrics, Archstone Professor of Medicine, David Geffen School of Medicine at University of California Los Angeles, Los Angeles, California Geriatric Assessment Emanuel P. Rivers, MD, MPH Professor and Vice Chairman of Emergency Medicine, Wayne State University; Senior Staff Attending, Critical Care and Emergency Medicine, Henry Ford Hospital, Detroit, Michigan Approach to the Patient with Shock Joseph G. Rogers, MD Professor of Medicine, Senior Vice Chief for Clinical Affairs, Division of Cardiology, Durham, North Carolina Heart Failure: Pathophysiology and Diagnosis Jean-Marc Rolain, PharmD, PhD Professor, Institut Hospitalo-Universitaire Méditerranée-Infection, Aix-Marseille Université, Marseille, France Bartonella Infections José R. Romero, MD Professor of Pediatrics, University of Arkansas for Medical Sciences, Horace C. Cabe Professor of Infectious Diseases; Director, Section of Pediatric Infectious Diseases, Arkansas Children’s Hospital, Little Rock, Arkansas Enteroviruses

Daniel E. Rusyniak, MD Professor of Emergency Medicine, Adjunct Professor of Neurology and Pharmacology and Toxicology, Department of Emergency Medicine, Indiana University School of Medicine, Indianapolis, Indiana Chronic Poisoning: Trace Metals and Others Robert A. Salata, MD Professor and Executive Vice Chair, Department of Medicine, Chief, Division of Infectious Diseases and HIV Medicine, Case Western Reserve University, University Hospitals Case Medical Center, Cleveland, Ohio Brucellosis Jane E. Salmon, MD Collette Kean Research Chair, Hospital for Special Surgery, Professor of Medicine, Weill Cornell Medical College, New York, New York Mechanisms of Immune-Mediated Tissue Injury Edsel Maurice T. Salvana, MD, DTM&H Associate Professor of Medicine, Section of Infectious Diseases, Department of Medicine, Philippine General Hospital; Director, Institute of Molecular Biology and Biotechnology, National Institutes of Health, University of the Philippines Manila, Manila, Philippines Brucellosis Renato M. Santos, MD Associate Professor, Cardiology, Wake Forest School of Medicine, Winston-Salem, North Carolina Vascular Disorders of the Kidney Michael N. Sawka, PhD Professor, School of Applied Physiology, Georgia Institute of Technology, Atlanta, Georgia Disorders Due to Heat and Cold Paul D. Scanlon, MD Professor of Medicine, Division of Pulmonary and Critical Care Medicine, Mayo Clinic, Rochester, Minnesota Respiratory Function: Mechanisms and Testing

xxvi

Contributors

Carla Scanzello, MD, PhD Assistant Professor of Medicine, Division of Rheumatology, Perelman School of Medicine at the University of Pennsylvania and Translational Musculoskeletal Research Center, Philadelphia Veterans Affairs Medical Center, Philadelphia, Pennsylvania Osteoarthritis Andrew I. Schafer, MD Professor of Medicine, Director, Richard T. Silver Center for Myeloproliferative Neoplasms, Weill Cornell Medical College, New York, New York Approach to Medicine, the Patient, and the Medical Profession: Medicine as a Learned and Humane Profession; Approach to the Patient with Bleeding and Thrombosis; Hemorrhagic Disorders: Disseminated Intravascular Coagulation, Liver Failure, and Vitamin K Deficiency; Thrombotic Disorders: Hypercoagulable States William Schaffner, MD Professor and Chair, Department of Preventive Medicine, Department of Health Policy; Professor of Medicine (Infectious Diseases), Vanderbilt University School of Medicine, Nashville, Tennessee Tularemia and Other Francisella Infections W. Michael Scheld, MD Bayer-Gerald L. Mandell Professor of Infectious Diseases, Professor of Medicine, Clinical Professor of Neurosurgery, Director, Pfizer Initiative in International Health, University of Virginia Health System, Charlottesville, Virginia Introduction to Microbial Disease: Host-Pathogen Interactions Manuel Schiff, MD Professor, Université Paris 7 Denis Diderot, Sorbonne Paris Cité, Head of Metabolic Unit/Reference Center for Inborn Errors of Metabolism, Robert Debré University Hospital, APHP, Paris, France Homocystinuria and Hyperhomocysteinemia Michael L. Schilsky, MD Associate Professor, Medicine and Surgery, Yale University School of Medicine, New Haven, Connecticut Wilson Disease Robert T. Schooley, MD Professor and Head, Division of Infectious Diseases, Executive Vice Chair for Academic Affairs, Department of Medicine, University of California San Diego, La Jolla, California Epstein-Barr Virus Infection David L. Schriger, MD, MPH Professor, Department of Emergency Medicine, University of California Los Angeles, Los Angeles, California Approach to the Patient with Abnormal Vital Signs Steven A. Schroeder, MD Distinguished Professor of Health and Healthcare and of Medicine, University of California San Francisco, San Francisco, California Socioeconomic Issues in Medicine Lynn M. Schuchter, MD Professor of Medicine, University of Pennsylvania; Chief, Hematology/ Oncology Division, Program Leader, Melanoma and Cutaneous Malignancies Program, Abramson Cancer Center, Hospital of the University of Pennsylvania, Philadelphia, Pennsylvania Melanoma and Nonmelanoma Skin Cancers Sam Schulman, MD, PhD Professor, Division of Hematology and Thromboembolism, Director of Clinical Thromboembolism Program, Department of Medicine, McMaster University, Hamilton, Ontario, Canada Antithrombotic Therapy

Lawrence B. Schwartz, MD, PhD Charles and Evelyn Thomas Professor of Medicine, Internal Medicine, Virginia Commonwealth University, Richmond, Virginia Systemic Anaphylaxis, Food Allergy, and Insect Sting Allergy Carlos Seas, MD Associate Professor of Medicine, Vice Director, Alexander von Humboldt Tropical Medicine Institute, Universidad Peruana Cayetano Heredia; Attending Physician, Department of Infectious, Tropical, and Dermatologic Diseases, National Hospital Cayetano Heredia, Lima, Peru Cholera and Other Vibrio Infections Steven A. Seifert, MD Professor of Emergency Medicine, University of New Mexico School of Medicine, Medical Director, New Mexico Poison and Drug Information Center, University of New Mexico Health Sciences Center, Albuquerque, New Mexico Envenomation Julian L. Seifter, MD Associate Professor of Medicine, Harvard Medical School; Senior Physician, Brigham and Women’s Hospital, Boston, Massachusetts Potassium Disorders; Acid-Base Disorders Duygu Selcen, MD Associate Professor of Neurology and Pediatrics, Department of Neurology, Mayo Clinic, Rochester, Minnesota Muscle Diseases Clay F. Semenkovich, MD Herbert S. Gasser Professor and Chief, Division of Endocrinology, Metabolism and Lipid Research, Washington University School of Medicine, St. Louis, Missouri Disorders of Lipid Metabolism Carol E. Semrad, MD Professor of Medicine, The University of Chicago Medicine, GI Section, Chicago, Illinois Approach to the Patient with Diarrhea and Malabsorption Harry Shamoon, MD Professor of Medicine and Associate Dean for Clinical and Translational Research, Albert Einstein College of Medicine; Director, Harold and Muriel Block Institute for Clinical and Translational Research at Einstein and Montefiore, Bronx, New York Diabetes Mellitus James C. Shaw, MD Associate Professor, Department of Medicine, University of Toronto; Head, Division of Dermatology, Department of Medicine, Women’s College Hospital, Toronto, Ontario, Canada Examination of the Skin and an Approach to Diagnosing Skin Diseases Pamela J. Shaw, DBE, MBBS, MD Professor of Neurology, University of Sheffield, Consultant Neurologist, Royal Hallamshire Hospital, Sheffield, United Kingdom Amyotrophic Lateral Sclerosis and Other Motor Neuron Diseases Robert L. Sheridan, MD Associate Professor of Surgery, Burn Service Medical Director, Boston Shriners Hospital for Children, Massachusetts General Hospital, Division of Burns, Harvard Medical School, Boston, Massachusetts Medical Aspects of Injuries and Burns Stuart Sherman, MD Professor of Medicine and Radiology, Director of ERCP, Indiana University School of Medicine, Indianapolis, Indiana Diseases of the Gallbladder and Bile Ducts

Contributors Michael E. Shy, MD Professor of Neurology, Pediatrics, and Physiology, University of Iowa, Iowa City, Iowa Peripheral Neuropathies

Frederick S. Southwick, MD Professor of Medicine, Division of Infectious Diseases, University of Florida and VF Health, Gainesville, Florida Nocardiosis

Ellen Sidransky, MD Chief, Section on Molecular Neurogenetics, Medical Genetics Branch, National Human Genome Research Institute, National Institutes of Health, Bethesda, Maryland The Lysosomal Storage Diseases

Allen M. Spiegel, MD Dean, Albert Einstein College of Medicine, Bronx, New York Principles of Endocrinology; Polyglandular Disorders

Richard M. Siegel, MD, PhD Clinical Director, National Institute of Arthritis, Musculoskeletal, and Skin Diseases, National Institutes of Health, Bethesda, Maryland The Systemic Autoinflammatory Diseases Robert F. Siliciano, MD, PhD Professor, The Johns Hopkins University School of Medicine, Howard Hughes Medical Institute, Baltimore, Maryland Immunopathogenesis of Human Immunodeficiency Virus Infection Michael S. Simberkoff, MD Chief of Staff, VA New York Harbor Healthcare System; Professor of Medicine, NYU School of Medicine, New York, New York Haemophilus and Moraxella Infections David L. Simel, MD, MHS Professor of Medicine, Duke University; Chief, Medical Service, Durham Veterans Affairs Medical Center, Durham, North Carolina Approach to the Patient: History and Physical Examination Kamaljit Singh, MD Associate Professor of Medicine, Attending Physician, Infectious Diseases, Rush University Medical Center, Chicago, Illinois Zoonoses Karl Skorecki, MD Annie Chutick Professor in Medicine, Rappaport Faculty of Medicine and Research Institute, Technion–Israel Institute of Technology; Director, Medical and Research Development, Rambam Health Care Campus, Haifa, Israel Gene and Cell Therapy; Disorders of Sodium and Water Homeostasis Itzchak Slotki, MD Associate Professor of Medicine, Hebrew University, Hadassah Medical School; Director, Division of Adult Nephrology, Shaare Zedek Medical Center, Jerusalem, Israel Disorders of Sodium and Water Homeostasis Arthur S. Slutsky, MD Professor of Medicine, Surgery, and Biomedical Engineering, University of Toronto; Vice President (Research), St. Michael’s Hospital, Keenan Research Centre, Li Ka Shing Knowledge Institute, Toronto, Ontario, Canada Acute Respiratory Failure; Mechanical Ventilation Eric J. Small, MD Professor of Medicine and Urology, Deputy Director and Director of Clinical Sciences, Helen Diller Family Comprehensive Cancer Center; Chief, Division of Hematology and Oncology, University of California San Francisco School of Medicine, San Francisco, California Prostate Cancer Gerald W. Smetana, MD Professor of Medicine, Harvard Medical School; Division of General Medicine and Primary Care, Beth Israel Deaconess Medical Center, Boston, Massachusetts Principles of Medical Consultation

xxvii

Robert F. Spiera, MD Professor of Clinical Medicine, Weill Cornell Medical College; Director, Scleroderma, Vasculitis, and Myositis Center, The Hospital for Special Surgery, New York, New York Polymyalgia Rheumatica and Temporal Arteritis Stanley M. Spinola, MD Professor and Chair, Department of Microbiology and Immunology, Professor of Medicine, Microbiology and Immunology, and Pathology and Laboratory Medicine, Indiana University School of Medicine, Indianapolis, Indiana Chancroid David Spriggs, MD Head, Division of Solid Tumor Oncology, Department of Medicine, Memorial Sloan Kettering Cancer Center; Professor of Medicine, Department of Medicine, Weill Cornell Medical College, New York, New York Gynecologic Cancers Paweł Stankiewicz, MD, PhD Department of Molecular and Human Genetics, Baylor College of Medicine, Houston, Texas Gene, Genomic, and Chromosomal Disorders Paul Stark, MD Professor Emeritus, University of California San Diego; Chief of Cardiothoracic Radiology, VA San Diego Healthcare System, San Diego, California Imaging in Pulmonary Disease David P. Steensma, MD Professor of Medicine, Harvard Medical School, Adult Leukemia Program, Dana-Farber Cancer Institute, Boston, Massachusetts Myelodysplastic Syndrome Martin H. Steinberg, MD Professor of Medicine, Pediatrics, and Pathology and Laboratory Medicine, Boston University School of Medicine; Director, Center of Excellence in Sickle Cell Disease, Boston Medical Center, Boston, Massachusetts Sickle Cell Disease and Other Hemoglobinopathies Theodore S. Steiner, MD Associate Professor, University of British Columbia; Associate Head, Division of Infectious Diseases, Vancouver General Hospital, Vancouver, British Columbia, Canada Escherichia Coli Enteric Infections David S. Stephens, MD Stephen W. Schwarzmann Distinguished Professor of Medicine, Emory University School of Medicine and Woodruff Health Sciences Center, Atlanta, Georgia Neisseria Meningitidis Infections David A. Stevens, MD Professor of Medicine, Stanford University Medical School; President, Principal Investigator, Infectious Diseases Research Laboratory, California Institute for Medical Research, San Jose and Stanford, California Systemic Antifungal Agents

xxviii

Contributors

James K. Stoller, MD, MS Chairman, Education Institute, Jean Wall Bennett Professor of Medicine, Cleveland Clinic Lerner College of Medicine; Staff, Respiratory Institute, Cleveland Clinic, Cleveland, Ohio Respiratory Monitoring in Critical Care John H. Stone, MD, MPH Professor of Medicine, Director, Clinical Rheumatology, Harvard Medical School, Massachusetts General Hospital, Boston, Massachusetts The Systemic Vasculitides Richard M. Stone, MD Professor of Medicine, Harvard Medical School, Clinical Director, Adult Leukemia Program, Dana-Farber Cancer Institute, Boston, Massachusetts Myelodysplastic Syndrome Raymond A. Strikas, MD, MPH Education Team Lead, Immunization Services Division, National Center for Immunization and Respiratory Diseases, Centers for Disease Control and Prevention, Atlanta, Georgia Immunization Edwin P. Su, MD Associate Professor of Clinical Orthopaedics, Orthopaedic Surgery, Weill Cornell University Medical College; Associate Attending Orthopaedic Surgeon, Adult Reconstruction and Joint Replacement, Hospital for Special Surgery, New York, New York Surgical Treatment of Joint Disease Roland W. Sutter, MD, MPH&TM Coordinator, Research, Policy and Product Development, Polio Operations and Research Department, World Health Organization, Geneva, Switzerland Diphtheria and Other Corynebacteria Infections Ronald S. Swerdloff, MD Professor of Medicine, David Geffen School of Medicine at University of California Los Angeles; Chief, Division of Endocrinology, Department of Medicine, Harbor-UCLA Medical Center, Torrance, California The Testis and Male Hypogonadism, Infertility, and Sexual Dysfunction Heidi Swygard, MD, MPH Associate Professor of Medicine, University of North Carolina at Chapel Hill, Chapel Hill, North Carolina Approach to the Patient with a Sexually Transmitted Infection Megan Sykes, MD Michael J. Friedlander Professor of Medicine, Director, Columbia Center for Translational Immunology, Columbia University Medical Center, New York, New York Transplantation Immunology Marian Tanofsky-Kraff, PhD Associate Professor, Department of Medical and Clinical Psychology, Uniformed Services University of Health Sciences, Bethesda, Maryland Eating Disorders Susan M. Tarlo, MBBS Professor of Medicine, Department of Medicine and Dalla Lana School of Public Health, University of Toronto, Respiratory Physician, University Health Network, Toronto Western Hospital and St. Michael’s Hospital, Toronto, Ontario, Canada Occupational Lung Disease Victoria M. Taylor, MD, MPH Professor of Medicine, University of Washington, Fred Hutchinson Cancer Research Center, Seattle, Washington Cultural Context of Medicine

Ayalew Tefferi, MD Professor of Medicine, Department of Hematology, Mayo Clinic, Rochester, Minnesota Polycythemia Vera, Essential Thrombocythemia, and Primary Myelofibrosis Paul S. Teirstein, MD Chief of Cardiology, Department of Medicine, Scripps Clinic, La Jolla, California Interventional and Surgical Treatment of Coronary Artery Disease Sam R. Telford III, ScD Professor, Tufts University Cummings School of Veterinary Medicine, North Grafton, Massachusetts Babesiosis and Other Protozoan Diseases Rajesh V. Thakker, MD May Professor of Medicine, University of Oxford; Radcliffe Department of Clinical Medicine, OCDEM, Churchill Hospital, Headington, Oxford, United Kingdom The Parathyroid Glands, Hypercalcemia, and Hypocalcemia Antonella Tosti, MD Professor of Clinical Dermatology, Department of Dermatology and Cutaneous Surgery, University of Miami, Miami, Florida Diseases of Hair and Nails Indi Trehan, MD, MPH, DTM&H Assistant Professor of Pediatrics, Washington University School of Medicine; Attending Physician, St. Louis Children’s Hospital, BarnesJewish Hospital, St. Louis, Missouri; Visiting Honorary Lecturer in Paediatrics and Child Health, University of Malawi College of Medicine; Consultant Paediatrician, Queen Elizabeth Central Hospital, Blantyre, Malawi Protein-Energy Malnutrition Ronald B. Turner, MD Professor of Pediatrics, University of Virginia School of Medicine, Charlottesville, Virginia The Common Cold Thomas S. Uldrick, MD Staff Clinician, HIV and AIDS Malignancy Branch, National Cancer Institute, Bethesda, Maryland Hematology and Oncology in Patients with Human Immunodeficiency Virus Infection Anthony M. Valeri, MD Professor of Medicine, Columbia University Medical Center; Director, Hemodialysis, Medical Director, Kidney and Pancreas Transplantation, New York-Presbyterian Hospital (CUMC); Director, Hemodialysis, Columbia University Dialysis Center, New York, New York Treatment of Irreversible Renal Failure John Varga, MD John and Nancy Hughes Professor of Medicine, Northwestern University Feinberg School of Medicine, Chicago, Illinois Systemic Sclerosis (Scleroderma) Bradley V. Vaughn, MD Professor of Neurology, Department of Neurology, University of North Carolina, Chapel Hill, North Carolina Disorders of Sleep Alan P. Venook, MD Professor of Medicine, University of California San Francisco, Helen Diller Family Comprehensive Cancer Center, San Francisco, California Liver and Biliary Tract Cancers Joseph G. Verbalis, MD Professor of Medicine, Georgetown University; Chief, Endocrinology and Metabolism, Georgetown University Hospital, Washington, D.C. Posterior Pituitary

Contributors Ronald G. Victor, MD Professor of Medicine, Burns and Allen Chair in Cardiology Research, Director, Hypertension Center, Associate Director, The Heart Institute, Cedars-Sinai Medical Center, Los Angeles, California Arterial Hypertension Angela Vincent, MBBS Professor of Neuroimmunology, University of Oxford; Honorary Consultant in Immunology, Oxford University Hospital Trust, Oxford, United Kingdom Disorders of Neuromuscular Transmission Robert M. Wachter, MD Professor and Associate Chairman, Department of Medicine, University of California San Francisco, San Francisco, California Quality of Care and Patient Safety Edward H. Wagner, MD, MPH Director Emeritus, MacColl Center for Health Care Innovation, Group Health Research Institute, Seattle, Washington Comprehensive Chronic Disease Management Edward E. Walsh, MD Professor of Medicine, University of Rochester School of Medicine and Dentistry; Head, Infectious Diseases, Rochester General Hospital, Rochester, New York Respiratory Syncytial Virus Thomas J. Walsh, MD Director, Transplantation-Oncology Infectious Diseases Program, Chief, Infectious Diseases Translational Research Laboratory, Professor of Medicine, Pediatrics, and Microbiology and Immunology, Weill Cornell Medical Center; Henry Schueler Foundation Scholar, Sharp Family Foundation Scholar in Pediatric Infectious Diseases, Adjunct Professor of Pathology, The Johns Hopkins University School of Medicine; Adjunct Professor of Medicine, The University of Maryland School of Medicine, Baltimore, Maryland Aspergillosis Jeremy D. Walston, MD Raymond and Anna Lublin Professor of Geriatric Medicine and Gerontology, The Johns Hopkins University School of Medicine, Baltimore, Maryland Common Clinical Sequelae of Aging Christina Wang, MD Professor of Medicine, David Geffen School of Medicine at University of California Los Angeles; Associate Director, UCLA Clinical and Translational Research Institute, Harbor-UCLA Medical Center, Torrance, California The Testis and Male Hypogonadism, Infertility, and Sexual Dysfunction Christine Wanke, MD Professor of Medicine and Public Health, Director, Division of Nutrition and Infection, Associate Chair, Department of Public Health, Tufts University School of Medicine, Boston, Massachusetts Gastrointestinal Manifestions of HIV and AIDS Stephen I. Wasserman, MD Professor of Medicine, University of California San Diego, La Jolla, California Approach to the Patient with Allergic or Immunologic Disease Thomas J. Weber, MD Associate Professor, Medicine/Endocrinology, Duke University, Durham, North Carolina Approach to the Patient with Metabolic Bone Disease; Osteoporosis Heiner Wedemeyer, MD Professor, Department of Gastroenterology, Hepatology, and Endocrinology, Hannover Medical School, Hannover, Germany Acute Viral Hepatitis

xxix

Geoffrey A. Weinberg, MD Professor of Pediatrics, University of Rochester School of Medicine and Dentistry; Director, Pediatric HIV Program, Golisano Children’s Hospital at University of Rochester Medical Center, Rochester, New York Parainfluenza Viral Disease David A. Weinstein, MD, MMSc Professor of Pediatric Endocrinology, Director, Glycogen Storage Disease Program, Division of Pediatric Endocrinology, University of Florida College of Medicine, Gainesville, Florida Glycogen Storage Diseases Robert S. Weinstein, MD Professor of Medicine, Department of Medicine, University of Arkansas for Medical Sciences; Staff Endocrinologist, Department of Medicine, Central Arkansas Veterans Health Care System, Little Rock, Arkansas Osteomalacia and Rickets Roger D. Weiss, MD Professor of Psychiatry, Harvard Medical School, Boston, Massachusetts; Chief, Division of Alcohol and Drug Abuse, McLean Hospital, Belmont, Massachusetts Drug Abuse and Dependence Martin Weisse, MD Chair, Pediatrics, Tripler Army Medical Center, Honolulu, Hawaii; Professor, Pediatrics, Uniformed Services University of the Health Sciences, Bethesda, Maryland Measles Jeffrey I. Weitz, MD Professor of Medicine and Biochemistry, McMaster University; Executive Director, Thrombosis and Atherosclerosis Research Institute, Hamilton, Ontario, Canada Pulmonary Embolism Samuel A. Wells, Jr., MD Medical Oncology Branch, National Cancer Institute, National Institutes of Health, Bethesda, Maryland Medullary Thyroid Carcinoma Richard P. Wenzel, MD, MSc Professor and Former Chairman, Internal Medicine, Virginia Commonwealth University, Richmond, Virginia Acute Bronchitis and Tracheitis Victoria P. Werth, MD Professor of Dermatology and Medicine, Hospital of the University of Pennsylvania and Philadelphia Veterans Administration Medical Center; Chief, Dermatology Division, Philadelphia Veterans Administration Medical Center, Philadelphia, Pennsylvania Principles of Therapy of Skin Diseases Sterling G. West, MD, MACP Professor of Medicine, University of Colorado School of Medicine; Associate Division Head for Clinical and Educational Affairs, University of Colorado Division of Rheumatology, Aurora, Colorado Systemic Diseases in Which Arthritis Is a Feature A. Clinton White, Jr., MD Paul R. Stalnaker Distinguished Professor and Director, Infectious Disease Division, Department of Internal Medicine, University of Texas Medical Branch, Galveston, Texas Cestodes Christopher J. White, MD Professor of Medicine, Ochsner Clinical School, University of Queensland School of Medicine; System Chairman of Cardiovascular Diseases, Ochsner Medical Center, New Orleans, Louisiana Atherosclerotic Peripheral Arterial Disease; Electrophysiologic Interventional Procedures and Surgery

xxx

Contributors

Perrin C. White, MD Professor of Pediatrics, The Audry Newman Rapoport Distinguished Chair in Pediatric Endocrinology, University of Texas Southwestern Medical Center, Chief of Endocrinology, Children’s Medical Center Dallas, Dallas, Texas Disorders of Sexual Development Richard J. Whitley, MD Distinguished Professor of Pediatrics, Loeb Eminent Scholar Chair in Pediatrics, Professor of Pediatrics, Microbiology, Medicine, and Neurosurgery, The University of Alabama at Birmingham, Birmingham, Alabama Herpes Simplex Virus Infections Michael P. Whyte, MD Professor of Medicine, Pediatrics, and Genetics, Division of Bone and Mineral Diseases, Washington University School of Medicine; MedicalScientific Director, Center for Metabolic Bone Disease and Molecular Research, Shriners Hospital for Children, St. Louis, Missouri Osteonecrosis, Osteosclerosis/Hyperostosis, and Other Disorders of Bone Samuel Wiebe, MD, MSc Professor of Clinical Neurosciences, University of Calgary; Co-Director, Calgary Epilepsy Program, Alberta Health Services, Foothills Medical Centre, Calgary, Alberta, Canada The Epilepsies Jeanine P. Wiener-Kronish, MD Henry Isaiah Dorr Professor of Research and Teaching in Anaesthesia and Anesthestist-in-Chief, Department of Anesthesia, Critical Care and Pain Medicine, Massachusetts General Hospital/Harvard Medical School, Boston, Massachusetts Overview of Anesthesia Eelco F.M. Wijdicks, MD, PhD Professor of Neurology, Division of Critical Care Neurology, Department of Neurology, Mayo Clinic, Rochester, Minnesota Coma, Vegetative State, and Brain Death David J. Wilber, MD George M. Eisenberg Professor of Medicine, Loyola Stritch School of Medicine; Director, Division of Cardiology, Director, Clinical Electrophysiology, Loyola University Medical Center, Maywood, Illinois Electrophysiologic Interventional Procedures and Surgery Beverly Winikoff, MD, MPH President, Gynuity Health Projects; Professor of Clinical Population and Family Health, Mailman School of Public Health, Columbia University, New York, New York Contraception Gary P. Wormser, MD Professor of Medicine and Chief, Division of Infectious Diseases, Department of Medicine, New York Medical College, Valhalla, New York Lyme Disease

Myron Yanoff, MD Professor and Chair, Ophthalmology, Drexel University College of Medicine, Philadelphia, Pennsylvania Diseases of the Visual System Robert Yarchoan, MD Branch Chief, HIV and AIDS Malignancy Branch, National Cancer Institute, Bethesda, Maryland Hematology and Oncology in Patients with Human Immunodeficiency Virus Infection Neal S. Young, MD Chief, Hematology Branch, NHLBI and Director, Trans-NIH Center for Human Immunology, Autoimmunity, and Inflammation, National Institutes of Health, Bethesda, Maryland Parvovirus William F. Young, Jr., MD, MSc Professor of Medicine, Mayo Clinic College of Medicine; Chair, Division of Endocrinology, Diabetes, Metabolism, and Nutrition, Mayo Clinic, Rochester, Minnesota Adrenal Medulla, Catecholamines, and Pheochromocytoma Alan S.L. Yu, MB, BChir Harry Statland and Solon Summerfield Professor of Medicine, Director, Division of Nephrology and Hypertension and the Kidney Institute, University of Kansas Medical Center, Kansas City, Kansas Disorders of Magnesium and Phosphorus Sherif R. Zaki, MD, PhD Chief, Infectious Diseases Pathology Branch, Centers for Disease Control and Prevention, Atlanta, Georgia Leptospirosis Mark L. Zeidel, MD Herman L. Blumgart Professor of Medicine, Harvard Medical School; Physician-in-Chief and Chairman, Department of Medicine, Beth Israel Deaconess Medical Center, Boston, Massachusetts Obstructive Uropathy Thomas R. Ziegler, MD Professor, Department of Medicine, Division of Endocrinology, Metabolism, and Lipids, Emory University School of Medicine, Atlanta, Georgia Malnutrition, Nutritional Assessment, and Nutritional Support in Adult Hospitalized Patients Peter Zimetbaum, MD Associate Professor of Medicine, Harvard Medical School; Director of Clinical Cardiology, Beth Israel Deaconess Medical Center, Boston, Massachusetts Cardiac Arrhythmias with Supraventricular Origin

CONTENTS VOLUME I

24 Geriatric Assessment

SECTION I: SOCIAL AND ETHICAL ISSUES IN MEDICINE

25 Common Clinical Sequelae of Aging

1 Approach to Medicine, the Patient, and the Medical Profession: Medicine as a Learned and Humane Profession

2 4 9 15

VICTORIA M. TAYLOR

5 Socioeconomic Issues in Medicine

17

STEVEN A. SCHROEDER

6 Global Health

19

ARUN CHOCKALINGAM

24 28 32

THOMAS B. NEWMAN AND CHARLES E. McCULLOCH

10 Using Data for Clinical Decisions

44

ROBERT M. WACHTER

13 Comprehensive Chronic Disease Management

46

EDWARD H. WAGNER

58 61 65 78 82 86

MARK G. KORTEPETER AND THEODORE J. CIESLAK

22 Chronic Poisoning: Trace Metals and Others

92

LOUISE W. KAO AND DANIEL E. RUSYNIAK

ANNE B. NEWMAN AND JANE A. CAULEY

162

GRANT W. CANNON

36 Biologic Agents and Signaling Inhibitors

169

CEM GABAY

37 Prostanoids, Aspirin, and Related Compounds

172

CARLO PATRONO

175 181

ADAM PERLMAN

40 Principles of Genetics

186

BRUCE R. KORF

41 Gene, Genomic, and Chromosomal Disorders

189

SANDESH C.S. NAGAMANI, PAWEŁ STANKIEWICZ, AND JAMES R. LUPSKI

42 The Inherited Basis of Common Diseases 43 Application of Molecular Technologies to Clinical Medicine

196 200

GEOFFREY S. GINSBURG

203

LIOR GEPSTEIN AND KARL SKORECKI

SECTION VII: PRINCIPLES OF IMMUNOLOGY AND INFLAMMATION 45 The Innate Immune System

SECTION IV: AGING AND GERIATRIC MEDICINE 23 Epidemiology of Aging: Implications of an Aging Society

156

ROGER D. WEISS

44 Regenerative Medicine, Cell, and Gene Therapies

DAVID J. BRENNER

21 Bioterrorism

149

SEKAR KATHIRESAN AND DAVID ALTSHULER

MARK R. CULLEN

20 Radiation Injury

145

PATRICK G. O’CONNOR

SECTION VI: GENETICS

RAYMOND A. STRIKAS AND WALTER A. ORENSTEIN

19 Principles of Occupational and Environmental Medicine

143

TONY P. GEORGE

54

DEBRA K. KATZMAN AND LAWRENCE S. NEINSTEIN

18 Immunization

133

ERIC J. NESTLER AND STEVEN E. HYMAN

39 Complementary and Alternative Medicine

DAVID M. BUCHNER

17 Adolescent Medicine

31 Biology of Addiction

52

DAVID ATKINS AND MARY BARTON

16 Physical Activity

30 Pain

SAM SCHULMAN AND JACK HIRSH

F. DANIEL DUFFY

15 The Periodic Health Examination

124

ROBERT B. DIASIO

38 Antithrombotic Therapy

SECTION III: PREVENTIVE AND ENVIRONMENTAL ISSUES 14 Counseling for Behavior Change

29 Principles of Drug Therapy

35 Immunosuppressing Drugs Including Corticosteroids 41

CAROLYN M. CLANCY AND ERNEST MOY

12 Quality of Care and Patient Safety

117

SECTION V: CLINICAL PHARMACOLOGY

34 Drugs of Abuse 37

THOMAS H. LEE

11 Measuring Health and Health Care

114

SHARON K. INOUYE

33 Alcohol Use Disorders

DAVID L. SCHRIGER

9 Statistical Interpretation of Data

28 Delirium or Acute Mental Status Change in the Older Patient

32 Nicotine and Tobacco

DAVID L. SIMEL

8 Approach to the Patient with Abnormal Vital Signs

27 Neuropsychiatric Aspects of Aging

STEVEN P. COHEN AND SRINIVASA N. RAJA

SECTION II: PRINCIPLES OF EVALUATION AND MANAGEMENT 7 Approach to the Patient: History and Physical Examination

110

SHARON K. INOUYE

ROBERT ARNOLD

4 Cultural Context of Medicine

26 Incontinence NEIL M. RESNICK

EZEKIEL J. EMANUEL

3 Care of Dying Patients and Their Families

106

JEREMY D. WALSTON

LEE GOLDMAN AND ANDREW I. SCHAFER

2 Bioethics in the Practice of Medicine

102

DAVID B. REUBEN

216

MARY K. CROW

100

46 The Adaptive Immune System JOSEPH CRAFT

220

xxxii

Contents

47 Mechanisms of Immune-Mediated Tissue Injury

226

JANE E. SALMON

48 Mechanisms of Inflammation and Tissue Repair

230 236 240

248 257 262

ANDREW R. MARKS

267

LEONARD GANZ

55 Echocardiography

274

CATHERINE M. OTTO

56 Noninvasive Cardiac Imaging

282

CHRISTOPHER M. KRAMER, GEORGE A. BELLER, AND KLAUS D. HAGSPIEL

57 Catheterization and Angiography

292 298 305 320 339 344

JEFFREY E. OLGIN

63 Approach to Cardiac Arrest and Life-Threatening Arrhythmias

352 356 367 374 381

RONALD G. VICTOR

68 Pulmonary Hypertension

397

VALLERIE McLAUGHLIN

69 Congenital Heart Disease in Adults

405

ARIANE J. MARELLI

70 Atherosclerosis, Thrombosis, and Vascular Biology

417

GÖRAN K. HANSSON AND ANDERS HAMSTEN

71 Angina Pectoris and Stable Ischemic Heart Disease

420

WILLIAM E. BODEN

72 Acute Coronary Syndrome: Unstable Angina and Non-ST Elevation Myocardial Infarction

432

PAUL S. TEIRSTEIN AND BRUCE W. LYTLE

526

MONICA KRAFT

84 Imaging in Pulmonary Disease

531

PAUL STARK

85 Respiratory Function: Mechanisms and Testing

539

86 Disorders of Ventilatory Control

545

87 Asthma

548

88 Chronic Obstructive Pulmonary Disease

555

89 Cystic Fibrosis

562

90 Bronchiectasis, Atelectasis, Cysts, and Localized Lung Disorders

566

ANNE E. O’DONNELL

571

92 Interstitial Lung Disease

575

93 Occupational Lung Disease

588

SUSAN M. TARLO

94 Physical and Chemical Injuries of the Lung

595

95 Sarcoidosis

603

MICHAEL C. IANNUZZI

96 Acute Bronchitis and Tracheitis

608

RICHARD P. WENZEL

97 Overview of Pneumonia

610

DANIEL M. MUSHER

98 Pulmonary Embolism

620

JEFFREY I. WEITZ

99 Diseases of the Diaphragm, Chest Wall, Pleura, and Mediastinum

627

F. DENNIS McCOOL

100 Obstructive Sleep Apnea

638

101 Interventional and Surgical Approaches to Lung Disease

642

DAVID J. FELLER-KOPMAN AND MALCOLM M. DeCAMP

441

JEFFREY L. ANDERSON

74 Interventional and Surgical Treatment of Coronary Artery Disease

83 Approach to the Patient with Respiratory Disease

ROBERT C. BASNER

RICHARD A. LANGE AND L. DAVID HILLIS

73 ST Segment Elevation Acute Myocardial Infarction and Complications of Myocardial Infarction

SECTION IX: RESPIRATORY DISEASES

DAVID C. CHRISTIANI

DAVID J. WILBER

67 Arterial Hypertension

519

DONNA MANCINI AND YOSHIFUMI NAKA

GANESH RAGHU

HASAN GARAN

66 Electrophysiologic Interventional Procedures and Surgery

82 Cardiac Transplantation

STEPHANIE M. LEVINE

PETER ZIMETBAUM

65 Ventricular Arrhythmias

511

JEFFREY S. GINSBERG

91 Alveolar Filling Disorders

ROBERT J. MYERBURG

64 Cardiac Arrhythmias with Supraventricular Origin

81 Peripheral Venous Disease

FRANK J. ACCURSO

GLENN I. FISHMAN

62 Approach to the Patient with Suspected Arrhythmia

504

MICHAEL R. JAFF AND JOHN R. BARTHOLOMEW

DENNIS E. NIEWOEHNER

WILLIAM J. McKENNA AND PERRY ELLIOTT

61 Principles of Electrophysiology

80 Other Peripheral Arterial Diseases

JEFFREY M. DRAZEN

JOHN J.V. McMURRAY AND MARC A. PFEFFER

60 Diseases of the Myocardium and Endocardium

497

ATUL MALHOTRA AND FRANK POWELL

CHRISTOPHER M. O’CONNOR AND JOSEPH G. ROGERS

59 Heart Failure: Management and Prognosis

492

PAUL D. SCANLON

MORTON KERN

58 Heart Failure: Pathophysiology and Diagnosis

483

CHRISTOPHER J. WHITE

DONALD M. LLOYD-JONES

54 Electrocardiography

78 Diseases of the Aorta 79 Atherosclerotic Peripheral Arterial Disease

LEE GOLDMAN

53 Cardiac Function and Circulatory Control

77 Pericardial Diseases

FRANK A. LEDERLE

SECTION VIII: CARDIOVASCULAR DISEASE

52 Epidemiology of Cardiovascular Disease

474

WILLIAM C. LITTLE AND JAE K. OH

JOHN P. ATKINSON

51 Approach to the Patient with Possible Cardiovascular Disease

76 Infective Endocarditis VANCE G. FOWLER, JR., ARNOLD S. BAYER, AND LARRY M. BADDOUR

MEGAN SYKES

50 Complement System in Disease

461

BLASE A. CARABELLO

GARY S. FIRESTEIN

49 Transplantation Immunology

75 Valvular Heart Disease

SECTION X: CRITICAL CARE MEDICINE 102 Approach to the Patient in a Critical Care Setting

650

DEBORAH J. COOK

456

103 Respiratory Monitoring in Critical Care JAMES K. STOLLER AND NICHOLAS S. HILL

652

Contents 104 Acute Respiratory Failure

655

MICHAEL A. MATTHAY AND ARTHUR S. SLUTSKY

105 Mechanical Ventilation

664 672 681 685 691

MICHAEL N. SAWKA AND FRANCIS G. O’CONNOR

110 Acute Poisoning

696

LEWIS S. NELSON AND MARSHA D. FORD

111 Medical Aspects of Injuries and Burns

711

ROBERT L. SHERIDAN

112 Envenomation

717

GEOFFREY K. ISBISTER AND STEVEN A. SEIFERT

113 Rhabdomyolysis

723

FRANCIS G. O’CONNOR AND PATRICIA A. DEUSTER

728

DONALD W. LANDRY AND HASAN BAZARI

115 Structure and Function of the Kidneys

737

QAIS AL-AWQATI AND JONATHAN BARASCH

116 Disorders of Sodium and Water Homeostasis

741

ITZCHAK SLOTKI AND KARL SKORECKI

117 Potassium Disorders

879

136 Disorders of Gastrointestinal Motility

884

137 Functional Gastrointestinal Disorders: Irritable Bowel Syndrome, Dyspepsia, Chest Pain of Presumed Esophageal Origin, and Heartburn

890

EMERAN A. MAYER

138 Diseases of the Esophagus

896

GARY W. FALK AND DAVID A. KATZKA

139 Acid Peptic Disease

908

ERNST J. KUIPERS AND MARTIN J. BLASER

140 Approach to the Patient with Diarrhea and Malabsorption

918

CAROL E. SEMRAD

141 Inflammatory Bowel Disease

935

GARY R. LICHTENSTEIN

SECTION XI: RENAL AND GENITOURINARY DISEASES 114 Approach to the Patient with Renal Disease

135 Gastrointestinal Hemorrhage

MICHAEL CAMILLERI

JAMES A. RUSSELL

109 Disorders Due to Heat and Cold

872

THOMAS O. KOVACS AND DENNIS M. JENSEN

STEVEN M. HOLLENBERG

108 Shock Syndromes Related to Sepsis

134 Gastrointestinal Endoscopy PANKAJ JAY PASRICHA

EMANUEL P. RIVERS

107 Cardiogenic Shock

866

DAVID H. KIM AND PERRY J. PICKHARDT

ARTHUR S. SLUTSKY

106 Approach to the Patient with Shock

133 Diagnostic Imaging Procedures in Gastroenterology

xxxiii

755

142 Inflammatory and Anatomic Diseases of the Intestine, Peritoneum, Mesentery, and Omentum

943

JOHN F. KUEMMERLE

143 Vascular Diseases of the Gastrointestinal Tract

951

STEPHEN CRANE HAUSER

144 Pancreatitis

959

CHRIS E. FORSMARK

145 Diseases of the Rectum and Anus

967

ROBERT D. MADOFF

JULIAN L. SEIFTER

118 Acid-Base Disorders

762

SECTION XIII: DISEASES OF THE LIVER, GALLBLADDER, AND BILE DUCTS

774

146 Approach to the Patient with Liver Disease

778

147 Approach to the Patient with Jaundice or Abnormal Liver Tests

JULIAN L. SEIFTER

119 Disorders of Magnesium and Phosphorus ALAN S.L. YU

120 Acute Kidney Injury BRUCE A. MOLITORIS

121 Glomerular Disorders and Nephrotic Syndromes

783

GERALD B. APPEL AND JAI RADHAKRISHNAN

122 Tubulointerstitial Nephritis

793

ERIC G. NEILSON

123 Obstructive Uropathy

799

MARK L. ZEIDEL

124 Diabetes and the Kidney

804

RAYMOND C. HARRIS

125 Vascular Disorders of the Kidney

807

THOMAS D. DUBOSE, JR. AND RENATO M. SANTOS

126 Nephrolithiasis

811

DAVID A. BUSHINSKY

127 Cystic Kidney Diseases

816

M. AMIN ARNAOUT

128 Hereditary Nephropathies and Developmental Abnormalities of the Urinary Tract

822

129 Benign Prostatic Hyperplasia and Prostatitis

827

STEVEN A. KAPLAN

130 Chronic Kidney Disease

833 841

149 Chronic Viral and Autoimmune Hepatitis

1000

JEAN-MICHEL PAWLOTSKY

150 Toxin- and Drug-Induced Liver Disease

1006

WILLIAM M. LEE

151 Bacterial, Parasitic, Fungal, and Granulomatous Liver Diseases

1011

K. RAJENDER REDDY

152 Alcoholic and Nonalcoholic Steatohepatitis

1019

NAGA P. CHALASANI

153 Cirrhosis and Its Sequelae

1023

GUADALUPE GARCIA-TSAO

1031

GREGORY T. EVERSON

155 Diseases of the Gallbladder and Bile Ducts

1038

EVAN L. FOGEL AND STUART SHERMAN

SECTION XIV: HEMATOLOGIC DISEASES 156 Hematopoiesis and Hematopoietic Growth Factors 157 The Peripheral Blood Smear 158 Approach to the Anemias

SECTION XII: GASTROINTESTINAL DISEASES KENNETH R. McQUAID

993

JEAN-MICHEL PAWLOTSKY

1050 1052

BARBARA J. BAIN

DAVID COHEN AND ANTHONY MICHAEL VALERI

132 Approach to the Patient with Gastrointestinal Disease

148 Acute Viral Hepatitis

KENNETH KAUSHANSKY

WILLIAM E. MITCH

131 Treatment of Irreversible Renal Failure

983

PAUL D. BERK AND KEVIN M. KORENBLAT

154 Hepatic Failure and Liver Transplantation

LISA M. GUAY-WOODFORD

976

PAUL MARTIN

1059

H. FRANKLIN BUNN

850

159 Microcytic and Hypochromic Anemias GORDON D. GINDER

1068

xxxiv

Contents

160 Autoimmune and Intravascular Hemolytic Anemias

1073

MARC MICHEL

161 Hemolytic Anemias: Red Blood Cell Membrane and Metabolic Defects

1080 1089 1095

MARTIN H. STEINBERG

164 Megaloblastic Anemias

1104

AŚOK C. ANTONY

165 Aplastic Anemia and Related Bone Marrow Failure States

1114

GROVER C. BAGBY

166 Polycythemia Vera, Essential Thrombocythemia, and Primary Myelofibrosis

1138 1142 1151 1154 1159

CHARLES S. ABRAMS

173 Von Willebrand Disease and Hemorrhagic Abnormalities of Platelet and Vascular Function

1167 1172 1181

ANDREW I. SCHAFER

176 Thrombotic Disorders: Hypercoagulable States

1185

ANDREW I. SCHAFER

177 Transfusion Medicine

1191

LAWRENCE T. GOODNOUGH

178 Hematopoietic Stem Cell Transplantation

1198

ARMAND KEATING AND MICHAEL R. BISHOP

1222

DAVID J. HUNTER

181 Cancer Biology and Genetics

1226

ADRIAN R. BLACK AND KENNETH H. COWAN

182 Myelodysplastic Syndromes

1233

DAVID P. STEENSMA AND RICHARD M. STONE

183 The Acute Leukemias

1239

FREDERICK R. APPELBAUM

184 The Chronic Leukemias

1246

SUSAN O’BRIEN AND ELIAS JABBOUR

185 Non-Hodgkin Lymphomas

1257

PHILIP J. BIERMAN AND JAMES O. ARMITAGE

186 Hodgkin Lymphoma

1268

JOSEPH M. CONNORS

187 Plasma Cell Disorders

1273

S. VINCENT RAJKUMAR

188 Amyloidosis MORIE A. GERTZ

1334 1339

ROBIN K. KELLEY AND ALAN P. VENOOK

1345

DEAN F. BAJORIN

1351

NANCY E. DAVIDSON

199 Gynecologic Cancers

1360

DAVID SPRIGGS

200 Testicular Cancer

1365

LAWRENCE H. EINHORN

201 Prostate Cancer

1367

ERIC J. SMALL

202 Malignant Tumors of Bone, Sarcomas, and Other Soft Tissue Neoplasms

1370

JAMES H. DOROSHOW

203 Melanoma and Nonmelanoma Skin Cancers 204 Cancer of Unknown Primary Origin

1373 1378

VOLUME II SECTION XVI: METABOLIC DISEASES 205 Approach to Inborn Errors of Metabolism

1384

OLAF A. BODAMER

206 Disorders of Lipid Metabolism

1389

CLAY F. SEMENKOVICH

207 Glycogen Storage Diseases

1397

DAVID A. WEINSTEIN

208 Lysosomal Storage Diseases 209 Homocystinuria and Hyperhomocysteinemia

1206

JAMES H. DOROSHOW

180 Epidemiology of Cancer

195 Pancreatic Neuroendocrine Tumors

1399

DONNA M. KRASNEWICH AND ELLEN SIDRANSKY

SECTION XV: ONCOLOGY 179 Approach to the Patient with Cancer

1332

DANIEL LAHERU

JOHN D. HAINSWORTH AND F. ANTHONY GRECO

MARGARET V. RAGNI

175 Hemorrhagic Disorders: Disseminated Intravascular Coagulation, Liver Failure, and Vitamin K Deficiency

194 Pancreatic Cancer

LYNN M. SCHUCHTER

WILLIAM L. NICHOLS

174 Hemorrhagic Disorders: Coagulation Factor Deficiencies

1320

CHARLES D. BLANKE AND DOUGLAS O. FAIGEL

198 Breast Cancer and Benign Breast Disorders

ANDREW I. SCHAFER

172 Thrombocytopenia

193 Neoplasms of the Small and Large Intestine

197 Tumors of the Kidney, Bladder, Ureters, and Renal Pelvis

MARC E. ROTHENBERG

171 Approach to the Patient with Bleeding and Thrombosis

1313

ANIL K. RUSTGI

1129

MICHAEL GLOGAUER

170 Eosinophilic Syndromes

1303

FADLO R. KHURI

196 Liver and Biliary Tract Cancers

JAMES O. ARMITAGE AND PHILIP J. BIERMAN

169 Disorders of Phagocyte Function

191 Lung Cancer and Other Pulmonary Neoplasms

1121

NANCY BERLINER

168 Approach to the Patient with Lymphadenopathy and Splenomegaly

1297

MARSHALL R. POSNER

ROBERT T. JENSEN

AYALEW TEFFERI

167 Leukocytosis and Leukopenia

190 Head and Neck Cancer

192 Neoplasms of the Esophagus and Stomach

MARIA DOMENICA CAPPELLINI

163 Sickle Cell Disease and Other Hemoglobinopathies

1287

LISA M. DeANGELIS

PATRICK G. GALLAGHER

162 The Thalassemias

189 Tumors of the Central Nervous System

1403

MANUEL SCHIFF AND HENK BLOM

210 The Porphyrias

1408

RICHARD J. HIFT

211 Wilson Disease

1416

STEPHEN G. KALER AND MICHAEL L. SCHILSKY

212 Iron Overload (Hemochromatosis)

1418

BRUCE R. BACON

SECTION XVII: NUTRITIONAL DISEASES 213 Nutrition’s Interface with Health and Disease

1426

DOUGLAS C. HEIMBURGER

214 Nutritional Assessment

1430

BRUCE R. BISTRIAN

215 Protein-Energy Malnutrition

1434

MARK J. MANARY AND INDI TREHAN

216 Enteral Nutrition

1437

STEPHEN A. McCLAVE

217 Malnutrition, Nutritional Assessment, and Nutritional Support in Adult Hospitalized Patients

1440

THOMAS R. ZIEGLER

1284

218 Vitamins, Trace Minerals, and Other Micronutrients JOEL B. MASON

1445

Contents 219 Eating Disorders

1455

MARIAN TANOFSKY-KRAFF

220 Obesity

1458

1468

DAVID R. CLEMMONS AND LYNNETTE K. NIEMAN

1471

ALLEN M. SPIEGEL

223 Neuroendocrinology and the Neuroendocrine System

1473

MARK E. MOLITCH

224 Anterior Pituitary

1480

MARK E. MOLITCH

225 Posterior Pituitary

1494

JOSEPH G. VERBALIS

226 Thyroid

1500

MATTHEW KIM AND PAUL W. LADENSON

227 Adrenal Cortex

1514

LYNNETTE K. NIEMAN

228 Adrenal Medulla, Catecholamines, and Pheochromocytoma

1521 1527

JILL CRANDALL AND HARRY SHAMOON

230 Hypoglycemia and Pancreatic Islet Cell Disorders

1548

KHALID HUSSAIN

231 Polyglandular Disorders

1555

LYNNETTE K. NIEMAN AND ALLEN M. SPIEGEL

232 Neuroendocrine Tumors and the Carcinoid Syndrome

1557

KENNETH R. HANDE

233 Disorders of Sexual Development

1560

PERRIN C. WHITE

234 The Testis and Male Hypogonadism, Infertility, and Sexual Dysfunction

SECTION XXI: DISEASES OF ALLERGY AND CLINICAL IMMUNOLOGY 249 Approach to the Patient with Allergic or Immunologic Disease 250 Primary Immunodeficiency Diseases 251 Allergic Rhinitis and Chronic Sinusitis 252 Urticaria and Angioedema 253 Systemic Anaphylaxis, Food Allergy, and Insect Sting Allergy 254 Drug Allergy

1568 1579 1584

ROBERT W. REBAR AND WILLIAM H. CATHERINO

1600

KAREN FREUND

1604

BEVERLY WINIKOFF AND DANIEL GROSSMAN

1610

KAREN ROSENE-MONTELLA

1623

DEBORAH GRADY AND ELIZABETH BARRETT-CONNOR

1629

GENE FEDER AND HARRIET L. MacMILLAN

255 Mastocytosis

SECTION XXII: RHEUMATIC DISEASES 256 Approach to the Patient with Rheumatic Disease

1636 1637

THOMAS J. WEBER

244 Osteomalacia and Rickets

1645

ROBERT S. WEINSTEIN

245 The Parathyroid Glands, Hypercalcemia and Hypocalcemia

257 Laboratory Testing in the Rheumatic Diseases

SAMUEL A. WELLS, JR.

1718

DAVID S. PISETSKY

258 Imaging Studies in the Rheumatic Diseases

1723

RONALD S. ADLER

1730

SUNEEL S. APTE

260 Inherited Diseases of Connective Tissue

1733

REED E. PYERITZ

262 Osteoarthritis

1739 1744

263 Bursitis, Tendinitis, and Other Periarticular Disorders and Sports Medicine

1749

JOSEPH J. BIUNDO

264 Rheumatoid Arthritis

1754

JAMES R. O’DELL

265 The Spondyloarthropathies

1762

ROBERT D. INMAN

266 Systemic Lupus Erythematosus

1769

MARY K. CROW

267 Systemic Sclerosis (Scleroderma)

1777 1785

XAVIER MARIETTE

269 Inflammatory Myopathies

1789

STEVEN A. GREENBERG

270 The Systemic Vasculitides

1793

JOHN H. STONE

271 Polymyalgia Rheumatica and Temporal Arteritis

1801

ROBERT F. SPIERA

1649

272 Infections of Bursae, Joints, and Bones

1662

273 Crystal Deposition Diseases

RAJESH V. THAKKER

246 Medullary Thyroid Carcinoma

1712

VIVIAN P. BYKERK AND MARY K. CROW

268 Sjögren Syndrome

THOMAS J. WEBER

243 Osteoporosis

1706

CEM AKIN

JOHN VARGA

SECTION XX: DISEASES OF BONE AND MINERAL METABOLISM 242 Approach to the Patient with Metabolic Bone Disease

1703

JOEL A. BLOCK AND CARLA SCANZELLO

SECTION XIX: WOMEN’S HEALTH

241 Intimate Partner Violence

1698

LESLIE C. GRAMMER

236 Reproductive Endocrinology and Infertility

240 Menopause

1693

STEPHEN C. DRESKIN

RICHARD M. SIEGEL AND DANIEL L. KASTNER

239 Common Medical Problems in Pregnancy

1687

LARRY BORISH

261 The Systemic Autoinflammatory Diseases

238 Contraception

1677

CHARLOTTE CUNNINGHAM-RUNDLES

ROBERT W. REBAR AND WILLIAM H. CATHERINO

237 Approach to Women’s Health

1674

STEPHEN I. WASSERMAN

259 Connective Tissue Structure and Function

RONALD S. SWERDLOFF AND CHRISTINA WANG

235 Ovaries and Pubertal Development

1667

LAWRENCE B. SCHWARTZ

WILLIAM F. YOUNG, JR.

229 Diabetes Mellitus

248 Osteonecrosis, Osteosclerosis/Hyperostosis, and Other Disorders of Bone MICHAEL P. WHYTE

SECTION XVIII: ENDOCRINE DISEASES 222 Principles of Endocrinology

1664

STUART H. RALSTON

MICHAEL D. JENSEN

221 Approach to the Patient with Endocrine Disease

247 Paget Disease of Bone

xxxv

1805

ERIC L. MATTESON AND DOUGLAS R. OSMON N. LAWRENCE EDWARDS

1811

xxxvi

Contents

274 Fibromyalgia, Chronic Fatigue Syndrome, and Myofascial Pain

1817

ROBERT M. BENNETT

275 Systemic Diseases in Which Arthritis Is a Feature

1823

STERLING G. WEST

276 Surgical Treatment of Joint Diseases

1828

C. RONALD MacKENZIE AND EDWIN P. SU

1837 1838

MARTIN J. BLASER AND ILSEUNG CHO

279 Principles of Anti-Infective Therapy

1843

GEORGE M. ELIOPOULOS

280 Approach to Fever or Suspected Infection in the Normal Host

1849

JAMES E. LEGGETT

281 Approach to Fever and Suspected Infection in the Compromised Host

1854 1861

1872

S. RAGNAR NORRBY AND LINDSAY E. NICOLLE

285 Approach to the Patient with a Sexually Transmitted Infection

1918 1920 1923 1924

MICHAEL S. SIMBERKOFF

1984

313 Whooping Cough and Other Bordetella Infections

1990 1993

THOMAS J. MARRIE

1996

JEAN-MARC ROLAIN AND DIDIER RAOULT

2001

EDWARD W. HOOK III

2002

STEPHEN G. BAUM

2007

WILLIAM M. GEISLER

2013

EDWARD W. HOOK III

2020

EDWARD W. HOOK III

2021

GARY P. WORMSER

322 Relapsing Fever and Other Borrelia Infections

2027

WILLIAM A. PETRI, JR.

323 Leptospirosis

2028

ATIS MUEHLENBACHS AND SHERIF R. ZAKI

324 Tuberculosis

2030

JERROLD J. ELLNER

325 The Nontuberculous Mycobacteria

2039

STEVEN M. HOLLAND

326 Leprosy (Hansen Disease)

2042

JOEL D. ERNST

1931

327 Rickettsial Infections

2046

DIDIER RAOULT

1934

328 Zoonoses

2056

STUART LEVIN AND KAMALJIT SINGH

1940

MATTHEW R. GOLDEN AND H. HUNTER HANDSFIELD

300 Haemophilus and Moraxella Infections

1982

KENNETH L. GAGE AND PAUL S. MEAD

321 Lyme Disease 1915

DAVID S. STEPHENS

299 Neisseria Gonorrhoeae Infections

312 Plague and Other Yersinia Infections

320 Nonsyphilitic Treponematoses 1913

ITZHAK BROOK

298 Neisseria Meningitidis Infections

1979

WILLIAM SCHAFFNER

319 Syphilis 1906

DALE N. GERDING AND STUART JOHNSON

297 Diseases Caused by Non–Spore-Forming Anaerobic Bacteria

1975

EDSEL MAURICE T. SALVANA AND ROBERT A. SALATA

318 Diseases Caused by Chlamydiae

ANNETTE C. REBOLI

296 Clostridial Infections

310 Brucellosis

1902

DANIEL R. LUCEY AND LEV M. GRINBERG

295 Erysipelothrix Infections

309 Shigellosis

317 Mycoplasma Infections

BENNETT LORBER

294 Anthrax

1971

JOHN A. CRUMP

1896

ROLAND W. SUTTER

293 Listeriosis

1968

KEITH S. KAYE AND ROBERT A. BONOMO

316 Granuloma Inguinale (Donovanosis)

TRISH M. PERL

292 Diphtheria and Other Corynebacterium Infections

307 Diseases Caused by Acinetobacter and Stenotrophomonas Species

1885

DONALD E. LOW

291 Enterococcal Infections

1962

MATTHEW E. FALAGAS AND PETROS I. RAFAILIDIS

315 Bartonella Infections

LIONEL A. MANDELL

290 Nonpneumococcal Streptococcal Infections and Rheumatic Fever

306 Pseudomonas and Related Gram-Negative Bacillary Infections

1881

HENRY F. CHAMBERS

289 Streptococcus Pneumoniae Infections

1960

DAVID L. PATERSON

314 Legionella Infections

GEORGE L. DRUSANO

288 Staphylococcal Infections

305 Infections Due to Other Members of the Enterobacteriaceae, Including Management of Multidrug-Resistant Strains

1876

DAVID O. FREEDMAN

287 Antibacterial Chemotherapy

1956

ERIK L. HEWLETT

HEIDI SWYGARD AND MYRON S. COHEN

286 Approach to the Patient before and after Travel

1953

BAN MISHU ALLOS

311 Tularemia and Other Francisella Infections 1868

HERBERT L. DUPONT

284 Approach to the Patient with Urinary Tract Infection

303 Campylobacter Infections

GERALD T. KEUSCH

DAVID P. CALFEE

283 Approach to the Patient with Suspected Enteric Infection

1950

EDUARDO GOTUZZO AND CARLOS SEAS

308 Salmonella Infections (Including Enteric Fever)

KIEREN A. MARR

282 Prevention and Control of Health Care–Associated Infections

302 Cholera and Other Vibrio Infections

THEODORE S. STEINER

W. MICHAEL SCHELD

278 The Human Microbiome

1949

STANLEY M. SPINOLA

304 Escherichia Coli Enteric Infections

SECTION XXIII: INFECTIOUS DISEASES 277 Introduction to Microbial Disease: Host-Pathogen Interactions

301 Chancroid

329 Actinomycosis

2060

ITZHAK BROOK

1946

330 Nocardiosis FREDERICK S. SOUTHWICK

2062

Contents 331 Systemic Antifungal Agents

2064

DAVID A. STEVENS

332 Histoplasmosis

2070 2072

366 Coronaviruses

2075

367 Measles

2076

368 Rubella (German Measles)

2078

369 Mumps

2079

370 Cytomegalovirus, Epstein-Barr Virus, and Slow Virus Infections of the Central Nervous System

CAROL A. KAUFFMAN

337 Sporotrichosis CAROL A. KAUFFMAN

338 Candidiasis CAROL A. KAUFFMAN

339 Aspergillosis

2083

THOMAS J. WALSH

340 Mucormycosis

2087

D.P. KONTOYIANNIS

341 Pneumocystis Pneumonia

2091

JOSEPH A. KOVACS

342 Mycetoma

2099

D.P. KONTOYIANNIS

343 Dematiaceous Fungal Infections

2101

PETER G. PAPPAS

344 Antiparasitic Therapy

2103

RICHARD D. PEARSON

345 Malaria

2107

PHILIP J. ROSENTHAL AND MOSES R. KAMYA

346 African Sleeping Sickness

2113

WILLIAM A. PETRI, JR.

347 Chagas Disease

2116

LOUIS V. KIRCHHOFF

348 Leishmaniasis

2120

353 Babesiosis and Other Protozoan Diseases 354 Cestodes

2147 2153

EDGAR M. CARVALHO AND ALDO A. M. LIMA

356 Liver, Intestinal, and Lung Fluke Infections

2155

EDUARDO GOTUZZO

357 Intestinal Nematode Infections

2159

DAVID J. DIEMERT

358 Tissue Nematode Infections

2164

DAVID J. DIEMERT

359 Arthropods and Leeches

2171

DIRK M. ELSTON

360 Antiviral Therapy (Non-HIV)

2177

EDWARD E. WALSH

2214

INGER K. DAMON

373 Papillomavirus

2219

JOHN M. DOUGLAS, JR.

374 Herpes Simplex Virus Infections

2223

RICHARD J. WHITLEY

375 Varicella-Zoster Virus (Chickenpox, Shingles)

2227

JEFFREY COHEN

376 Cytomegalovirus

2229

W. LAWRENCE DREW

377 Epstein-Barr Virus Infection

2232

ROBERT T. SCHOOLEY

378 Retroviruses Other Than Human Immunodeficiency Virus

2235

WILLIAM A. BLATTNER

379 Enteroviruses

2239

JOSÉ R. ROMERO

2244

MANUEL A. FRANCO AND HARRY B. GREENBERG

2247

DANIEL G. BAUSCH

2256

STANLEY J. NAIDES

2262

THOMAS P. BLECK

SECTION XXIV: HIV AND THE ACQUIRED IMMUNODEFICIENCY SYNDROME 384 Epidemiology and Diagnosis of Human Immunodeficiency Virus Infection and Acquired Immunodeficiency Syndrome

2272

THOMAS C. QUINN

385 Immunopathogenesis of Human Immunodeficiency Virus Infection

2278

JOEL N. BLANKSON AND ROBERT F. SILICIANO

386 Biology of Human Immunodeficiency Viruses

2280

FRANK MALDARELLI

387 Prevention of Human Immunodeficiency Virus Infection

2285

388 Antiretroviral Therapy of HIV/AIDS

2287

ROY M. GULICK

2185

RONALD B. TURNER

362 Respiratory Syncytial Virus

2212

CARLOS del RIO AND MYRON S. COHEN

JOHN H. BEIGEL

361 The Common Cold

372 Smallpox, Monkeypox, and Other Poxvirus Infections

383 Arboviruses Affecting the Central Nervous System

2142

2208

NEAL S. YOUNG

2138

A. CLINTON WHITE AND ENRICO BRUNETTI

355 Schistosomiasis (Bilharziasis)

371 Parvovirus

382 Arboviruses Causing Fever and Rash Syndromes

SAM R. TELFORD III AND PETER J. KRAUSE

2206

JOSEPH R. BERGER AND AVINDRA NATH

2135

WILLIAM A. PETRI, JR. AND ALDO A.M. LIMA

2204

JOHN W. GNANN, JR.

381 Viral Hemorrhagic Fevers

THEODORE E. NASH AND DAVID R. HILL

352 Amebiasis

2202

SUSAN E. REEF

2133

ALDO A.M. LIMA AND RICHARD L. GUERRANT

351 Giardiasis

2199

MARTIN WEISSE AND MARK PAPANIA

380 Rotaviruses, Noroviruses, and Other Gastrointestinal Viruses

JOSE G. MONTOYA

350 Cryptosporidiosis

2197

SUSAN I. GERBER AND LARRY J. ANDERSON

2125

SIMON L. CROFT AND PIERRE A. BUFFET

349 Toxoplasmosis

365 Adenovirus Diseases

2074

CAROL A. KAUFFMAN

336 Cryptococcosis

2191

MICHAEL G. ISON

CAROL A. KAUFFMAN

335 Paracoccidioidomycosis

364 Influenza FREDERICK G. HAYDEN

JOHN N. GALGIANI

334 Blastomycosis

2188

GEOFFREY A. WEINBERG AND KATHRYN M. EDWARDS

CAROL A. KAUFFMAN

333 Coccidioidomycosis

363 Parainfluenza Viral Disease

xxxvii

389 Infectious and Metabolic Complications of HIV and AIDS

2292

HENRY MASUR, LETHA M. HEALEY, AND COLLEEN HADIGAN

2187

390 Gastrointestinal Manifestations of HIV and AIDS TAMSIN A. KNOX AND CHRISTINE WANKE

2302

xxxviii

Contents

391 Pulmonary Manifestations of Human Immunodeficiency Virus and the Acquired Immunodeficiency Syndrome

2305

KRISTINA CROTHERS AND ALISON MORRIS

392 Skin Manifestations in Patients with Human Immunodeficiency Virus Infection

2318 2322 2328 2332

2338 2346

JEFFREY M. LYNESS

2356

KATHLEEN B. DIGRE

399 Traumatic Brain Injury and Spinal Cord Injury

2364

GEOFFREY S.F. LING

400 Mechanical and Other Lesions of the Spine, Nerve Roots, and Spinal Cord

2370 2382

DAVID S. KNOPMAN

402 Alzheimer Disease and Other Dementias

2388

DAVID S. KNOPMAN

403 The Epilepsies

2399

SAMUEL WIEBE

404 Coma, Vegetative State, and Brain Death

2409

JAMES L. BERNAT AND EELCO F.M. WIJDICKS

405 Disorders of Sleep

2415

BRADLEY V. VAUGHN

406 Approach to Cerebrovascular Diseases

2424

LARRY B. GOLDSTEIN

407 Ischemic Cerebrovascular Disease

2434 2445

STEPHAN A. MAYER

409 Parkinsonism

2454

ANTHONY E. LANG

410 Other Movement Disorders

2461

ANTHONY E. LANG

411 Multiple Sclerosis and Demyelinating Conditions of the Central Nervous System

2471

PETER A. CALABRESI

412 Meningitis: Bacterial, Viral, and Other

2480

AVINDRA NATH

413 Brain Abscess and Parameningeal Infections

2495

AVINDRA NATH AND JOSEPH BERGER

414 Acute Viral Encephalitis

2500

ALLEN J. AKSAMIT, JR.

415 Prion Diseases

2504

PATRICK J. BOSQUE

416 Nutritional and Alcohol-Related Neurologic Disorders

2506

BARBARA S. KOPPEL

417 Congenital, Developmental, and Neurocutaneous Disorders

WILLIAM P. CHESHIRE, JR.

424 Neuro-Ophthalmology

2573 2579

TROY E. DANIELS AND RICHARD C. JORDAN

426 Approach to the Patient with Nose, Sinus, and Ear Disorders

2585

ANDREW H. MURR

427 Smell and Taste

2592

ROBERT W. BALOH AND JOANNA C. JEN

428 Hearing and Equilibrium

2593

ROBERT W. BALOH AND JOANNA C. JEN

2601

PAUL W. FLINT

SECTION XXVII: MEDICAL CONSULTATION 430 Principles of Medical Consultation

2608

GERALD W. SMETANA

431 Preoperative Evaluation

2611

STEVEN L. COHN

432 Overview of Anesthesia

2617

JEANINE P. WIENER-KRONISH AND LEE A. FLEISHER

433 Postoperative Care and Complications

2621

DONALD A. REDELMEIER

434 Medical Consultation in Psychiatry

2625

PETER MANU

SECTION XXVIII: SKIN DISEASES 435 Structure and Function of the Skin

2632

436 Examination of the Skin and an Approach to Diagnosing Skin Diseases

2637

JAMES C. SHAW

437 Principles of Therapy of Skin Diseases

2657

VICTORIA P. WERTH

438 Eczemas, Photodermatoses, Papulosquamous (Including Fungal) Diseases, and Figurate Erythemas

2662

HENRY W. LIM

439 Macular, Papular, Vesiculobullous, and Pustular Diseases

2671

NEIL J. KORMAN

440 Urticaria, Drug Hypersensitivity Rashes, Nodules and Tumors, and Atrophic Diseases

2683

MADELEINE DUVIC

441 Infections, Hyperpigmentation and Hypopigmentation, Regional Dermatology, and Distinctive Lesions in Black Skin

2695

JEAN BOLOGNIA

442 Diseases of Hair and Nails

2703

ANTONELLA TOSTI

2512

APPENDIX-LABORATORY REFERENCE INTERVALS AND VALUES

2517

Reference Intervals and Laboratory Values

JONATHAN W. MINK

418 Autonomic Disorders and Their Management

2556

DAVID H. CHU

LARRY B. GOLDSTEIN

408 Hemorrhagic Cerebrovascular Disease

423 Diseases of the Visual System

429 Throat Disorders

RICHARD L. BARBANO

401 Regional Cerebral Dysfunction: Higher Mental Functions

2547

SECTION XXVI: EYE, EAR, NOSE, AND THROAT DISEASES

425 Diseases of the Mouth and Salivary Glands

ROBERT C. GRIGGS, RALPH F. JÓZEFOWICZ, AND MICHAEL AMINOFF

398 Headaches and Other Head Pain

422 Disorders of Neuromuscular Transmission

ROBERT W. BALOH AND JOANNA C. JEN

SECTION XXV: NEUROLOGY 397 Psychiatric Disorders in Medical Practice

2537

DUYGU SELCEN

MYRON YANOFF AND J. DOUGLAS CAMERON

ROBERT COLEBUNDERS AND MARTYN A. FRENCH

396 Approach to the Patient with Neurologic Disease

2527

MICHAEL E. SHY

AMELIA EVOLI AND ANGELA VINCENT

JOSEPH R. BERGER AND AVINDRA NATH

395 Immune Reconstitution Inflammatory Syndrome in HIV/AIDS

420 Peripheral Neuropathies 421 Muscle Diseases

THOMAS S. ULDRICK AND ROBERT YARCHOAN

394 Neurologic Complications of Human Immunodeficiency Virus Infection

2522

PAMELA J. SHAW

TOBY MAURER

393 Hematology and Oncology in Patients with Human Immunodeficiency Virus Infection

419 Amyotrophic Lateral Sclerosis and Other Motor Neuron Diseases

RONALD J. ELIN

2712

VIDEO CONTENTS This icon appears throughout the book to indicate chapters with accompanying video available on ExpertConsult.com. For quick viewing, use your smartphone to scan the QR codes in the front of the book. Aging and Geriatric Medicine Confusion Assessment Method (CAM) Video 28-1 – MARCOS MIALNEZ, JORGE G. RUIZ, AND ROSANNE M. LEIPZIG

Clinical Pharmacology Interlaminar Epidural Steroid Injection Video 30-1 – ALI TURABI

Cardiovascular Disease Standard Echocardiographic Views: Long Axis Image Plane Video 55-1A – CATHERINE M. OTTO Standard Echocardiographic Views: Short Axis Image Plane Video 55-1B – CATHERINE M. OTTO Standard Echocardiographic Views: Short Axis Image Plane Video 55-1C – CATHERINE M. OTTO Standard Echocardiographic Views: Four-Chamber Image Plane Video 55-1D – CATHERINE M. OTTO Dilated Cardiomyopathy: Long Axis View Video 55-2A – CATHERINE M. OTTO Dilated Cardiomyopathy: Short Axis View Video 55-2B – CATHERINE M. OTTO Dilated Cardiomyopathy: Apical Four-Chamber View Video 55-2C – CATHERINE M. OTTO Three-Dimensional Echocardiography Video 55-3 – CATHERINE M. OTTO Stress Echocardiography: Normal Reaction Video 55-4A – CATHERINE M. OTTO Stress Echocardiography: Normal Reaction Video 55-4B – CATHERINE M. OTTO Stress Echocardiography: Proximal Stenosis of the Left Anterior Descending Coronary Artery Video 55-4C – CATHERINE M. OTTO Stress Echocardiography: Proximal Stenosis of the Left Anterior Descending Coronary Artery Video 55-4D – CATHERINE M. OTTO Pericardial Effusion: Parasternal Long Axis Video 55-5A – CATHERINE M. OTTO Pericardial Effusion: Parasternal Short Axis Video 55-5B – CATHERINE M. OTTO Pericardial Effusion: Apical Four-Chamber Views Video 55-5C – CATHERINE M. OTTO Secundum Atrial Septal Defect Video 69-1 – ARIANE J. MARELLI Perimembranous Ventricular Septal Defect Video 69-2 – ARIANE J. MARELLI Coronary Stent Placement Video 74-1 – PAUL S. TEIRSTEIN Guidewire Passage Video 74-2 – PAUL S. TEIRSTEIN Delivering the Stent Video 74-3 – PAUL S. TEIRSTEIN Inflating the Stent Video 74-4 – PAUL S. TEIRSTEIN

Final Result Video 74-5 – PAUL S. TEIRSTEIN Superficial Femoral Artery (SFA) Stent Procedure Video 79-1 – CHRISTOPHER J. WHITE Orthotopic Bicaval Cardiac Transplantation Video 82-1 – Y. JOSEPH WOO

Respiratory Diseases Wheezing Video 87-1 – JEFFREY M. DRAZEN Technique for Use of a Metered-Dose Inhaler Video 87-2 – LESLIE HENDELES and the New England Journal of Medicine VATS Wedge Resection Video 101-1 – MALCOLM M. DeCAMP

Critical Care Medicine Ventilation of an Ex Vivo Rat Lung Video 105-1 – ARTHUR S. SLUTSKY, GEORGE VOLGYESI, AND TOM WHITEHEAD

Renal and Genitourinary Diseases Renal Artery Stent Video 125-1 – RENATO M. SANTOS AND THOMAS D. DUBOSE, JR. Interpretation of a Computed Tomographic Colonography Video 133-1 – DAVID H. KIM Donor Liver Transplantation—Donor and Recipient Video 154-1 – IGAL KAM, THOMAS BAK, AND MICHAEL WACHS

Oncology Snare Polypectomy of a Colon Adenoma Video 193-1 – DOUGLAS O. FAIGEL Laparoscopic-Assisted Double Balloon Enteroscopy with Polypectomy of a Jejunal Adenoma Followed by Surgical Oversew of the Polypectomy Site Video 193-2 – DOUGLAS O. FAIGEL Endoscopic Mucosal Resection Using Saline Lift Polypectomy of a Colon Adenoma Followed by Closure of the Mucosal Defect with Clips Video 193-3 – DOUGLAS O. FAIGEL Endoscopic View of Rectal Cancer Video 193-4 – DOUGLAS O. FAIGEL Endoscopic Ultrasound Video 193-5 – DOUGLAS O. FAIGEL

Nutritional Diseases Laparoscopic Roux-en-Y Gastric Bypass Video 220-1 – JAMES M. SWAIN

Endocrine Diseases Pituitary Surgery Video 224-1 – IVAN CIRIC

Diseases of Allergy and Clinical Immunology Skin Testing Video 251-1 – LARRY BORISH Nasal Endoscopy Video 251-2 – LARRY BORISH

xl

Video Contents

Rheumatic Diseases Hip Arthroscopy Osteochondroplasty Video 276-1 – BRYAN T. KELLY

Neurology Cervical Provocation Video 400-1 – RICHARD L. BARBANO Spurling Maneuver Video 400-2 – RICHARD L. BARBANO Cervical Distraction Test Video 400-3 – RICHARD L. BARBANO Straight Leg Raise Video 400-4 – RICHARD L. BARBANO Contralateral Straight Leg Raise Video 400-5 – RICHARD L. BARBANO Seated Straight Leg Raise Video 400-6 – RICHARD L. BARBANO Discectomy Video 400-7 – JASON H. HUANG Absence Seizure Video 403-1 – SAMUEL WIEBE Left Rolandic Seizure Video 403-2 – SAMUEL WIEBE Left Temporal Complex Partial Seizure Video 403-3 – SAMUEL WIEBE Left Temporal Complex Partial Seizure Postictal Confusion Video 403-4 – SAMUEL WIEBE Left Temporal Complex Partial Seizure Video 403-5 – SAMUEL WIEBE Supplementary Sensory-Motor Seizure Video 403-6 – SAMUEL WIEBE Right Posterior Temporal Seizure-Dramatic Frontal Semiology Video 403-7 – SAMUEL WIEBE Right Mesial Frontal Seizure Video 403-8 – SAMUEL WIEBE Nonconvulsive Status Epilepticus Video 403-9 – SAMUEL WIEBE GTC Seizure Tonic Phase Video 403-10 – SAMUEL WIEBE GTC Seizure Clonic Phase Video 403-11 – SAMUEL WIEBE Myoclonic Facial Seizure Video 403-12 – SAMUEL WIEBE Tonic Seizure Lennox Gastaut Video 403-13 – SAMUEL WIEBE Atonic Seizure Lennox Gastaut Video 403-14 – SAMUEL WIEBE Reflex Auditory Seizure Video 403-15 – SAMUEL WIEBE Four Score Video 404-1 – JAMES L. BERNAT AND EELCO F.M. WIJDICKS Persistent Vegetative State Video 404-2 – JAMES L. BERNAT AND EELCO F.M. WIJDICKS Minimally Conscious State Video 404-3 – JAMES L. BERNAT AND EELCO F.M. WIJDICKS Akinetic Mutism Video 404-4 – JAMES L. BERNAT AND EELCO F.M. WIJDICKS Early Parkinson’s Disease Video 409-1 – ANTHONY E. LANG Freezing of Gait in Parkinson’s Disease Video 409-2 – ANTHONY E. LANG

Gunslinger Gait in Progressive Supranuclear Palsy Video 409-3 – ANTHONY E. LANG Supranuclear Gaze Palsy in Progressive Supranuclear Palsy Video 409-4 – ANTHONY E. LANG Applause Sign in Progressive Supranuclear Palsy Video 409-5 – ANTHONY E. LANG Apraxia of Eyelid Opening in Progressive Supranuclear Palsy Video 409-6 – ANTHONY E. LANG Cranial Dystonia in Multiple System Atrophy Video 409-7 – ANTHONY E. LANG Anterocollis in Multiple System Atrophy Video 409-8 – ANTHONY E. LANG Stridor in Multiple System Atrophy Video 409-9 – ANTHONY E. LANG Alien Limb Phenomenon in Corticobasal Syndrome Video 409-10 – ANTHONY E. LANG Myoclonus in Corticobasal Syndrome Video 409-11 – ANTHONY E. LANG Levodopa-Induced Dyskinesia in Parkinson’s Disease Video 409-12 – ANTHONY E. LANG Essential Tremor Video 410-1 – ANTHONY E. LANG Huntington’s Disease Video 410-2 – ANTHONY E. LANG Hemiballism Video 410-3 – ANTHONY E. LANG Blepharospasm Video 410-4 – ANTHONY E. LANG Oromandibular Dystonia Video 410-5 – ANTHONY E. LANG Cervical Dystonia Video 410-6 – ANTHONY E. LANG Writer’s Cramp Video 410-7 – ANTHONY E. LANG Embouchure Dystonia Video 410-8 – ANTHONY E. LANG Sensory Trick in Cervical Dystonia Video 410-9 – ANTHONY E. LANG Generalized Dystonia Video 410-10 – ANTHONY E. LANG Tics Video 410-11 – ANTHONY E. LANG Tardive Dyskinesia Video 410-12 – ANTHONY E. LANG Hemifacial Spasm Video 410-13 – ANTHONY E. LANG Wernicke Encephalopathy Eye Movements: Before Thiamine Video 416-1 – BARBARA S. KOPPEL Wernicke Encephalopathy Eye Movements: After Thiamine Video 416-2 – BARBARA S. KOPPEL Limb Symptoms and Signs Video 419-1 – PAMELA J. SHAW Bulbar Symptoms and Signs Video 419-2 – PAMELA J. SHAW Normal Swallowing Video 419-3 – PAMELA J. SHAW Charcot-Marie-Tooth Disease Exam and Walk Video 420-1 – MICHAEL E. SHY

XVI

METABOLIC DISEASES 205 APPROACH TO INBORN ERRORS OF METABOLISM

206 DISORDERS OF LIPID METABOLISM 207 GLYCOGEN STORAGE DISEASES

208 LYSOSOMAL STORAGE DISEASES

211 WILSON DISEASE

209 HOMOCYSTINURIA AND

212 IRON OVERLOAD

HYPERHOMOCYSTEINEMIA

210 THE PORPHYRIAS

(HEMOCHROMATOSIS)

1384

CHAPTER 205  Approach to Inborn Errors of Metabolism  

205  APPROACH TO INBORN ERRORS OF METABOLISM OLAF A. BODAMER

DEFINITION

The term metabolism (Greek: metabolé, “change”) refers to the network of chemical reactions that sustain the human organism through the digestion, absorption, transport, and utilization of nutrients. Inborn errors of metabolism are genetic disorders that affect these intrinsic metabolic pathways through deficiencies of enzymes, membrane transporter proteins, signaling peptides, or structural proteins. The resulting clinical phenotype follows a spectrum of different organ manifestations that may be progressive, fluctuating, or stationary in nature and may be manifested at any age. Any inborn error of metabolism can principally be manifested during adolescence or adulthood, although severe presentations are typically recognized during infancy and childhood.

HISTORY

Archibald Garrod pioneered the field of inborn errors of metabolism after recognizing alkaptonuria as one of the first metabolic conditions due to homozygosity of mutant alleles in 1902. He had the foresight to recognize the autosomal recessive inheritance of additional inborn errors of metabolism, including cystinuria, pentosuria, and albinism, and to speculate about “chemical individuality” as one of the driving forces of selection and evolution. However, it was not until the early 1950s that the deficiency of homogentisate 1,2-dioxygenase (HGD) was recognized as the underlying cause of alkaptonuria, and it took many more years to identify pathogenic mutations in the HGD gene. The advent of novel analytical techniques led to the molecular and biochemical characterization of known inborn errors of metabolism and the delineation and recognition of new clinical phenotypes, some of which were previously not presumed to be due to inborn errors of metabolism. The completion of the first human genome in 2001 and the following “genomics” revolution laid the foundation for the successive identification of many additional inborn errors of metabolism through next-generation sequencing, bringing the total number of catalogued inborn errors of metabolism to more than 1500 (March 2014). The initiation of population-based newborn screening in 1964 through Robert Guthrie resulted in its recognition as an important public health measure to prevent morbidity and mortality of inborn errors of metabolism. More than 4 million newborn infants are screened annually in the United States for 31 core conditions, including mostly inborn errors of metabolism. As a consequence, approximately 12,500 newborn infants are diagnosed each year through newborn screening. Rarely, mothers with an inborn error of metabolism are diagnosed through newborn screening of their infants subsequent to placental transfer of pathognomonic metabolites.

EPIDEMIOLOGY

Inborn errors of metabolism occur in all populations, although their incidence and prevalence rates may vary considerably because of differences in carrier rates. These variations are readily explained by the presence of founder mutations, for example, in individuals of Ashkenazi Jewish or Amish ancestry, or by an increased rate of parental consanguinity that leads to a relative increase in mutant allele frequency (Table 205-1). Knowledge of the increased carrier frequencies is instrumental for preconception genetic counseling and targeted carrier screening.

PATHOBIOLOGY

The complexity of human metabolism and its spatial relationship with the human proteome, genome, and methylome are poorly understood. Naturally occurring variants in human nucleotide sequences may or may not result in variation of amino acid sequences in peptides and proteins. It is now well established from whole exome and genome sequencing that individuals may carry in excess of 10,000 nucleotide variants; most variants are silent, singlenucleotide polymorphic variants. Up to 4% of variants may be pathogenic in

either recessive or dominant genes. These variants in particular will lead to functional changes in proteins that may render the affected individual susceptible to disease, increase the risk for undesired side effects on treatment with certain drugs, or increase the risk for genetic conditions in future generations. Variation of human peptides and proteins is not merely explained through genomic sequence variation. Post-transcriptional alternative splicing will generate tissue-specific isoforms of proteins that are adapted to their functional needs through post-translational modification and conformational plasticity.

Genetics

Inborn errors of metabolism are monogenic conditions that follow autosomal recessive or dominant, X-linked recessive or dominant, or mitochondrial inheritance patterns. Of note is the existence of genetic or environmental modifiers that contribute to the interindividual and intrafamilial variability of phenotypic expression, although for most inborn errors of metabolism, these modifiers remain elusive. In case of mitochondrial inheritance, heteroplasmy (the random distribution and expression of mitochondrial mutations in different organs) may explain by itself the striking variability of clinical symptoms in mitochondrial conditions. The concept of synergistic heterozygosity (i.e., heterozygosity for pathogenic mutations affecting different enzymes simultaneously within the same pathway) may explain why some individuals with symptoms reminiscent of inborn errors of metabolism are not formally diagnosed.

Pathophysiology

The severity of any given inborn error of metabolism depends on the degree of enzyme deficiency and the complex interaction of the underlying pathogenic mutations, genetic modifiers, and environment. Hypomorphic mutations may not lead to overt disease until adulthood, whereas severe mutations in the same gene may lead to infantile-onset disease associated with significant morbidity and mortality. The underlying pathophysiologic mechanisms may contribute individually or in combination to the disease state (Table 205-2). Complete blockage of a catabolic pathway may result in accumulation of toxic substrates, activation of secondary minor pathways, or a relative shortage of downstream products. As a consequence, different organs may be affected by the same metabolic defect. An example is homocystinuria due to mutations in the gene for cystathionine β-synthase, which causes lens dislocation and intellectual disabilities and increases the risk for cardiovascular disease. Accumulation of homocysteine contributes to the vascular risk, whereas lack of the downstream product cysteine is an important factor in the dislocation of the lens through loosening of the zonular fibers (Table 205-3).

Clinical Phenotype

Inborn errors of metabolism typically affect multiple organs and, in more than 50% of cases, the central and peripheral nervous systems and muscles. One or more organ manifestations may dominate the clinical phenotype, although oligosymptomatic cases may occur. The clinical phenotype represents a continuous clinical spectrum ranging from the severe end, presenting during infancy, to the mild end of the spectrum, presenting during adolescence or adulthood. Some affected individuals may never come to medical attention because of almost complete absence of symptoms or atypical presentation. Recent data from newborn screening programs suggest much higher incidence rates for some inborn errors of metabolism due to the detection of a high rate of mild cases in which disease-related signs or symptoms may never develop. Some clinical signs are pathognomonic for an inborn error of metabolism, whereas others should raise the suspicion for the presence of an inborn error of metabolism (Table 205-4).

Classification

Inborn errors of metabolism can be classified on the basis of the underlying pathomechanism (see Table 205-2), the nature or localization of the protein involved (see Table 205-3), or the clinical phenotype (see Table 205-4). The most logical classification is based on the nature or localization of the affected protein and pathway.

INBORN ERRORS OF METABOLISM

Disorders of Protein Metabolism

These conditions are due to cytosolic or mitochondrial enzyme deficiencies affecting mostly catabolic pathways (Table 205-5). Disorders of protein

1385

CHAPTER 205  Approach to Inborn Errors of Metabolism  

TABLE 205-1  INCIDENCE OF INBORN ERRORS OF METABOLISM DISORDER

GENE

INCIDENCE*

CARRIER RATE

POPULATION

Familial hypercholesterolemia

LDLR

Phenylketonuria

PAH

1 : 4000 100 mg/day triiodothyronine. These deficiency. Maternal iodine to an individual who was formerly hormones circulate largely bound deficiency leads to fetal deficiency, deficient occasionally induces to thyroxine-binding globulin. which produces spontaneous hyperthyroidism. [1.1 mg] They modulate resting energy abortions, stillbirths, expenditure and, in the hypothyroidism, cretinism, and developing human, growth and dwarfism. Permanent cognitive development. deficits may result from iodine deficiency during the first 2 years of life. In the adult, compensatory hypertrophy of the thyroid (goiter) occurs along with varying degrees of hypothyroidism. [150 µg]

Iodine status of a population can be estimated by the prevalence of goiter. Urinary excretion of iodine is an effective laboratory means of assessment. Thyroid-stimulating hormone blood level is an indirect and therefore not entirely specific means of assessment.

Iron

Conveys the capacity to participate Iron deficiency is the most common in redox reactions to a number of micronutrient deficiency in the metalloproteins, such as world. Women of childbearing age hemoglobin, myoglobin, are the group at highest risk cytochrome enzymes, and many because of menstrual blood losses, oxidases and oxygenases. The pregnancy, and lactation. The primary storage form of iron is classic deficiency syndrome is ferritin and, to a lesser degree, hypochromic, microcytic anemia. hemosiderin. Intestinal Glossitis and koilonychia (“spoon” absorption is 15-20% for “heme” nails) are also observed. Easy iron and 1-8% for iron contained fatigability often is an early in vegetables. Absorption of the symptom, before anemia appears. latter form is enhanced by the In children, mild deficiency of ascorbic acid in foodstuffs; by insufficient severity to cause poultry, fish, or beef; and by an anemia is associated with iron-deficient state. It is behavioral disturbances and decreased by phytate and tannins. poor school performance. [postmenopausal F and M: 8 mg; premenopausal F: 18 mg]

Manganese

Toxicity by oral ingestion is unknown in A component of several Manganese deficiency in the human humans. Toxic inhalation causes metalloenzymes. Most has not been conclusively hallucinations, other alterations in manganese is in mitochondria, demonstrated. It is said to cause mentation, and extrapyramidal where it is a component of hypocholesterolemia, weight loss, movement disorders. [11 mg] manganese superoxide dismutase. hair and nail changes, dermatitis, and impaired synthesis of vitamin K–dependent proteins. [F: 1.8 mg; M: 2.3 mg]

Until the deficiency syndrome is better defined, an appropriate measure of status will be difficult to develop.

A probable case of human deficiency Toxicity not well described in humans, although it may interfere with copper is described as being secondary to metabolism at high doses. [2 mg] parenteral administration of sulfite and resulted in hyperoxypurinemia, hypouricemia, and low sulfate excretion. [45 µg]

Laboratory means of assessment are not meaningful until the deficiency syndrome is better described.

Molybdenum A cofactor in several enzymes, most prominently xanthine oxidase and sulfite oxidase

Iron overload typically occurs when Negative iron balance initially leads habitual dietary intake is extremely to depletion of iron stores in the high, intestinal absorption is excessive, bone marrow; a bone marrow repeated parenteral administration biopsy and the concentration of occurs, or a combination of these serum ferritin are accurate factors exists. Excessive iron stores indicators of early depletion. As usually accumulate in the the severity of deficiency reticuloendothelial tissues and cause proceeds, serum iron (SI) little damage (hemosiderosis). If decreases and total iron-binding overload continues, iron eventually capacity (TIBC) increases; an begins to accumulate in tissues such as iron saturation (SI/TIBC) of the hepatic parenchyma, pancreas, 60% suggest iron recessive trait. Excessive intestinal overload, although systemic absorption of iron is seen in inflammation elevates serum homozygotes. [45 mg] ferritin regardless of iron status.

1452

CHAPTER 218  Vitamins, Trace Minerals, and Other Micronutrients  

TABLE 218-3  NUTRITIONAL TRACE ELEMENTS AND THEIR CLINICAL IMPLICATIONS—cont’d BIOCHEMISTRY AND PHYSIOLOGY

TOXICITY [TUL†]

DEFICIENCY [RDA*]

ASSESSMENT OF STATUS

Selenium

Erythrocyte glutathione peroxidase Toxicity is associated with nausea, Most dietary selenium is in the form Deficiency is rare in North America activity and plasma or whole diarrhea, alterations in mental status, of an amino acid complex. Nearly but has been observed in blood selenium concentrations are peripheral neuropathy, and loss of hair complete absorption of such individuals on long-term TPN the most commonly used and nails; such symptoms were forms occurs. Homeostasis is lacking selenium. Such individuals methods of assessment. They are observed in adults who inadvertently largely performed by the kidney, have myalgias or cardiomyopathies. consumed 27-2400 mg. [400 µg] moderately accurate indicators of which regulates urinary excretion Populations in some regions of the status. as a function of selenium status. world, most notably some parts of Selenium is a component of China, have marginal intake of several enzymes, most notably selenium. In these regions Keshan’s glutathione peroxidase and disease, a condition characterized superoxide dismutase. These by cardiomyopathy, is endemic; it enzymes protect against oxidative can be prevented (but not treated) and free radical damage of by selenium supplementation. [55 µg] various cell structures. The antioxidant protection conveyed by selenium apparently operates in conjunction with vitamin E because deficiency of one seems to potentiate damage induced by a deficiency of the other. Selenium also participates in the enzymatic conversion of thyroxine to its more active metabolite, triiodothyronine.

Zinc

No accurate indicators of zinc status Intestinal absorption occurs by a Zinc deficiency has its most profound Acute zinc toxicity can usually be induced by ingestion of >200 mg of exist for routine clinical use. specific process that is enhanced effect on rapidly proliferating zinc in a single day (in adults). It is Plasma, red blood cell, and hair by pregnancy and corticosteroids tissues. Mild deficiency: growth manifested by epigastric pain, nausea, zinc concentrations are often and diminished by coingestion of retardation in children. More severe vomiting, and diarrhea. Hyperpnea, misleading. Acute illness, in phytates, phosphates, iron, deficiency: growth arrest, diaphoresis, and weakness may follow particular, is known to diminish copper, lead, or calcium. teratogenicity, hypogonadism and inhalation of zinc fumes. Copper and plasma zinc levels, in part by Diminished intake of zinc leads infertility, dysgeusia, poor wound zinc compete for intestinal absorption: inducing a shift of zinc out of the to an increased efficiency of healing, diarrhea, dermatitis on the long-term ingestion of >25 mg/day of plasma compartment and into the absorption and decreased fecal extremities and around orifices, zinc may lead to copper deficiency. liver. Functional tests that excretion, providing a means of glossitis, alopecia, corneal Long-term ingestion of >150 mg/day determine dark adaptation, taste zinc homeostasis. Zinc is a clouding, loss of dark adaptation, has been reported to cause gastric acuity, and rate of wound healing component of more than 100 and behavioral changes. Impaired erosions, low high-density lipoprotein lack specificity. enzymes, among which are DNA cellular immunity is observed. cholesterol levels, and impaired polymerase, RNA polymerase, Excessive loss of gastrointestinal cellular immunity. [40 mg] and transfer RNA synthetase. secretions through chronic diarrhea and fistulas may precipitate deficiency. Acrodermatitis enteropathica is a rare, recessively inherited disease in which intestinal absorption of zinc is impaired. [F: 8 mg; M: 11 mg]

*Recommended daily allowance (RDA) established for female (F) and male (M) adults by the U.S. Food and Nutrition Board, 1999-2001. In some instances, insufficient data exist to establish an RDA, in which case the adequate intake (AI) established by the board is listed. † Tolerable upper limit (TUL) established for adults by the U.S. Food and Nutrition Board, 1999-2001.

A, E, and C to meet the metabolic demands incurred by milk production in addition to the aforementioned needs observed in pregnancy. Aside from its general role in supporting the rapid proliferation of placental and fetal tissues, folate plays a specific role in the prevention of particular birth defects. A 20 to 85% reduction in births complicated by neural tube defects (NTDs, i.e., spina bifida and anencephaly) has been realized by providing women with a daily supplement of folic acid in the form of supplements or fortified foods. The optimal dose is not well defined, but 200 to 400 µg/day clearly affords a substantial degree of protection. Populations with a high background rate of NTD births attain the largest reductions in NTDs from supplemental folate. However, because the nascent neural tube closes about day 20 after conception, the additional folate must be provided before this time to be effective.

Infancy

Infancy carries particular vulnerabilities to specific micronutrient inadequacies. Healthy infants in the United States are typically supplemented with vitamin K at birth and with iron and vitamin D during the course of the first year because of their particular susceptibility to deficiencies of these nutrients.

Women of Childbearing Age

The ability to maintain adequate iron status from menarche through menopause is compromised in women by the additional losses incurred by menstruation, pregnancy, and lactation. Therefore, it is not surprising that the population subset that almost invariably displays the highest rate of iron deficiency is women of childbearing age.

Elderly Persons

Specific dietary recommendations for elderly people have been formally incorporated into the recommended dietary allowances (RDA) because aging has an impact on the need for certain micronutrients. Vitamin B12 status declines significantly with aging, in large part because of the high prevalence of atrophic gastritis and its associated impairment in protein-bound vitamin B12 absorption.2 Estimates suggest that 10 to 20% of the elderly population is at risk for clinically significant vitamin B12 deficiency. Consequently, elderly persons should consume some of their vitamin B12 requirement in the crystalline form rather than solely from the naturally occurring protein-bound forms found in food because absorption of the crystalline form is not impaired by atrophic gastritis. Elderly people also require greater quantities of vitamins B6 and D to maintain health compared with younger adults, as reflected in

CHAPTER 218  Vitamins, Trace Minerals, and Other Micronutrients  

the new RDAs (see Table 218-2). For instance, the RDA of vitamin D in persons older than 70 years is now set at 20 µg/day (800 IU), as opposed to adults who are 70 years of age or younger, whose RDA is 15 µg/day.3,4 This increased need appears to result from diminished cutaneous synthesis of vitamin D by senile skin and from decreased sun exposure, which appears to be particularly important in elders residing in institutional facilities. The need for crystalline vitamin B12 and for a quantity of vitamin D that is difficult to achieve without resorting to a supplement suggests that universal use of a daily supplement pill containing these nutrients would benefit elderly people. Widespread use of a multivitamin that contains a broad spectrum of micronutrients is more controversial, in part because of concerns about subtle toxicity. For example, elders with chronic renal failure appear to have a vulnerability to vitamin A toxicity, suggesting that use of supplements containing this vitamin is contraindicated.

PATHOPHYSIOLOGIC AND PHARMACOLOGIC FACTORS

Diseases of the Gastrointestinal Tract

Malabsorption and maldigestion predispose to multiple micronutrient deficiencies. Both fat- and water-soluble micronutrients (except vitamin B12) are absorbed predominantly in the proximal small intestine. Therefore, diffuse mucosal diseases affecting the proximal portion of the gastrointestinal tract are likely to result in deficiencies. Even in the absence of mucosal disease of the proximal small intestine, extensive ileal disease, small bowel bacterial overgrowth, and chronic cholestasis can each interfere with the maintenance of adequate intraluminal conjugated bile acid concentrations and thereby impair absorption of fat-soluble vitamins. Maldigestion is usually the result of chronic pancreatitis. Untreated, it frequently causes malabsorption and deficiencies of fat-soluble vitamins. Vitamin B12 malabsorption can often be demonstrated in this setting, a result of inadequate R-protein digestion, but clinical vitamin B12 deficiency is rarely reported.

Inborn Errors of Metabolism

Myriad rare inborn errors of metabolism have been described for vitamins and minerals that impair an individual’s ability to assimilate, to use, or to retain a particular micronutrient (Chapter 205). Such defects are usually partial and can often be overcome, to a certain extent, by administering doses of the nutrient that are several orders of magnitude greater than usually required. Suspicion for such defects should be entertained if a known defect exists in the family, a deficiency syndrome arises at birth or during infancy, or the deficiency syndrome is present despite adequate dietary intake and the absence of any disease that would impair the ability to assimilate the nutrient.

Medications

Long-term administration of many drugs may adversely affect micronutrient status. The manner in which drug-nutrient interactions occur varies; some of the more common mechanisms are outlined in Table 218-4. Some drugs exert their therapeutic effects by specifically inhibiting the actions of a micronutrient. Examples include coumarin, which inhibits γ-carboxylation

TABLE 218-4  DRUG-MEDIATED EFFECTS ON MICRONUTRIENT STATUS: EXAMPLES DRUG

NUTRIENT

MECHANISM OF INTERACTION

Dextroamphetamine, Potentially all Induces anorexia fenfluramine, levodopa micronutrients Cholestyramine

Vitamin D, folate

Adsorbs nutrient, decreases absorption

Omeprazole

Vitamin B12

Modest bacterial overgrowth, decreases gastric acid, impairs absorption

Sulfasalazine

Folate

Impairs absorption and inhibits folate-dependent enzymes

Isoniazid

Pyridoxine

Impairs utilization of B6

Nonsteroidal antiinflammatory drugs

Iron

Gastrointestinal blood loss

Penicillamine

Zinc

Increases renal excretion

1453

reactions mediated by vitamin K, and methotrexate, which binds tightly to dihydrofolate reductase, thereby inhibiting folate metabolism.

Toxins

Tobacco smoking alters the metabolism of several vitamins, including folate and vitamins C and E. In large surveys, diminished plasma levels of folate and ascorbic acid have been observed in chronic smokers. Smoking is also associated with diminished levels of folate in cells of the oral mucosa, diminished ascorbic acid levels in leukocytes, and decreased concentrations of vitamin E in the alveolar fluid, findings providing evidence that many tissues can be affected by smoking and that the effect does not simply represent a shift of these micronutrients out of the plasma compartment.

ADVANCES IN NUTRITIONAL SCIENCE

New Frontiers in Marginal Deficiency States of Micronutrients Does Optimal Intake of Micronutrients Optimize Health?

Updating the definition of a micronutrient deficiency and establishing recommended daily intakes that are consistent with the most recent evidence have proved difficult for several reasons. In some instances, a novel biochemical or physiologic role for a nutrient has been identified but the question that arises is whether optimization of such functions translates into optimization of health. For example, providing supplemental vitamin E to elderly individuals whose vitamin E status falls within normative standards enhances T-lymphocyte responsiveness; nevertheless, it is unclear whether this translates into diminished infection rates. Another difficult problem pertains to the use of micronutrients in supraphysiologic quantities that exceed all conventional concepts of what is necessary for health. Some micronutrients, when they are taken in large quantities, have effects on physiologic functions that impart apparent health benefits. The ingestion of gram quantities of niacin to reduce low-density lipoprotein (LDL) cholesterol is an example. Such physiologic effects are not observed at more conventional levels of intake and are therefore usually considered pharmacologic effects of the nutrient. Thus, the determination of optimal nutrient intake is highly dependent on which physiologic effect is sought. Furthermore, if only a segment of the population will benefit from supraphysiologic quantities of a nutrient, should dietary guidelines for the remainder of the population be established according to this effect? Determining an adequate level of intake implies the existence of a means of measuring nutrient status. In seeking an appropriate measure of nutrient status, the diversity of function often makes it difficult to decide which measurement is the most germane. Tobacco smoking, for example, diminishes vitamin E levels in alveolar fluid but not in the serum. Thus, the concepts of localized nutrient deficiencies and tissue-specific requirements add an additional level of complexity to the determination of nutrient status.

Redefinition of Nutritional Requirements Folate

An example of the complexities that have arisen in redefining the criteria for vitamin deficiencies and vitamin requirements is the water-soluble vitamin folate. In the past, guidelines regarding its necessary intake were straightforward because they were based solely on the prevention of megaloblastic anemia. Measurement of serum and erythrocyte folate concentrations was the most common means of assessing status, and maintaining these levels within accepted normative ranges provided assurance that folate status was adequate to prevent anemia. However, degrees of deficiency that are insufficient to cause anemia may still disturb normal biochemical and physiologic homeostasis and, in some instances, cause clinical disease. Clinical trials have demonstrated that women taking folic acid supplements at the time of conception have a markedly lower chance of delivering a baby with an NTD compared with women who are not folate supplemented but whose folate status falls within a conventionally accepted range. This observation compelled the U.S. government to mandate the fortification of flour, beginning in 1998. Present recommendations are that women of childbearing age consume 400 µg/day of folic acid in the form of supplements or fortified foods, although the dose-response curve of this effect is ill-defined. Less than optimal intake of folate is also evidenced by an increase in serum homocysteine, an amino acid that is normally metabolized by a folatedependent pathway. Before the federally mandated fortification of flour, the median intake of folate among adults was half of the present RDA, and a substantial minority of Americans had significantly elevated serum homocysteine levels. Elevated homocysteine is associated with the development of

1454

CHAPTER 218  Vitamins, Trace Minerals, and Other Micronutrients  

occlusive vascular disease and accelerated cognitive decline. In randomized clinical trials, however, supplementation with folate, vitamin B12, and vitamin B6 has shown no benefit against cardiovascular disease despite its ability to lower homocysteine levels. A1  Such supplementation also has no clear benefit for cognitive function, except perhaps in patients with low baseline folate levels. A2  A compelling body of observations in both humans and animals has demonstrated that habitually low consumption of folate substantially increases the risk of colorectal cancer5 and perhaps cancers of other organs, such as those of the breast and pancreas. This inverse relationship is observed even when folate status (or dietary intake) falls within the range of conventionally accepted normative values. This relationship has further complicated the determination of what constitutes an optimal intake of folate because the recent epidemiologic data suggest that about 500 µg constitutes the optimal daily intake for suppressing the risk of colon cancer. The issue is further confounded by observations, albeit controversial, suggesting that exceptionally high doses of supplemental folic acid among those who unknowingly harbor precancerous or cancerous lesions may paradoxically enhance the progression of these neoplasms,6 thereby underscoring the potential for harm produced by taking a nutrient outside of its physiologic window. The most recent update of the U.S. RDA for folate raised the value from 200 to 400 µg/day, citing both the prevention of anemia and optimization of serum homocysteine as criteria, and recommended that women capable of becoming pregnant consume an additional 400 µg/day in the form of supplements or fortified food. The issues surrounding the prevention of cardiovascular disease, cancer, and cognitive decline were not incorporated into that 1998 determination because the existing data at the time were inconclusive. However, future revisions of the RDAs may integrate some of this new knowledge. The potential for toxicity, the criterion for which was primarily linked to its ability to mask vitamin B12 deficiency, was dealt with by setting the TUL at 1000 µg/day of folic acid obtained from supplements and fortified foods in addition to that obtained from natural food sources (see Table 218-2). Table 218-5 lists several examples of biochemical functions of vitamins that were not formerly recognized. As the clinical significance of each of these new roles is defined and as quantities of each vitamin needed to optimize such functions are determined, redefinition of the desirable range of vitamin status is likely to occur. Future efforts to refine appropriate dietary goals for each micronutrient will, however, need to take into consideration an important theme that is underscored by the previous discussion: the level of consumption of a particular micronutrient that conveys health benefits to one segment of the population is not necessarily beneficial, or even appropriate, for all segments of society.

Antioxidant and Free Radical Scavenging Vitamins and Provitamins

Vitamins A, C, and E as well as many of the carotenoids are effective antioxidants. In addition, vitamins C and E and some of the carotenoids can scavenge free radicals when these nutrients are taken in adequate quantities. Oxidation and free radical damage have been implicated as important contributors to common degenerative illnesses, such as atherosclerosis, cancer, cataracts, and retinal degeneration. Clinical trials to test the efficacy of antioxidant supplements have generally shown no benefit and in some instances

TABLE 218-5  NEWLY IDENTIFIED ROLES FOR VITAMINS VITAMIN OR PROVITAMIN

CLASSIC ROLE

NEW ROLE

β-Carotene

Pro-vitamin A

Antioxidant, free radical

Niacin

NAD/NADP coenzyme

Reduction of LDL, elevation of HDL cholesterol

Folate

Hematopoietic factor

Diminishes homocysteinemia

Vitamin A

Transduction of visual input in retina

Induction and maintenance of epithelial differentiation, signal in embryogenesis

Vitamin D

Regulator of calcium

Retards epithelial proliferation; promotes differentiation

Vitamin B6

Coenzyme for transamination

Modulation of steroid activity

HDL = high-density lipoprotein; LDL = low-density lipoprotein; NAD = nicotinamide adenine dinucleotide; NADP = nicotinamide adenine dinucleotide phosphate.

harm,7 although growing evidence indicates that health benefits of such supplements can be realized in populations with marginal antioxidant status. Two large-scale clinical intervention trials with β-carotene supplements conducted in the 1990s reported increased rates of lung cancer among the recipients of the carotenoid. Subsequent mechanistic studies indicated that the large doses administered (20 to 30 mg/day) result in asymmetrical cleavage of the carotenoid into unnatural products that antagonize normal signaling pathways in the lung epithelium, whereas lower supplemental doses undergo symmetrical cleavage into two molecules of vitamin A, thereby protecting against neoplastic transformation. LDL oxidized in vivo is atherogenic. Prevention of LDL oxidation, at least in animal models, retards the process of atherogenesis. Supplementation of human subjects with several times the RDA of α-tocopherol, and perhaps some of the other antioxidant micronutrients, is an effective means of preventing LDL oxidation. Human intervention trials with vitamin E or other antioxidant nutrients, however, have generally been unable to demonstrate clinical benefits in the reduction of cardiovascular events. There nevertheless has been a sizable reduction in cardiovascular events observed with vitamin E supplementation among populations of patients who are under exceptional oxidative stress, such as those with chronic renal failure and certain classes of diabetics, suggesting that it is only among select groups of individuals that a clinical benefit may be realized. Epidemiologic studies indicate that occurrence of cancers of the oral cavity, lung, esophagus, and stomach (and perhaps the colorectum) is inversely related to dietary intake of fresh vegetables and fruits. Careful dissection of dietary data suggests that β-carotene and vitamin E content are strongly predictive components of these foodstuffs. High doses of vitamin A and some of its synthetic analogues (e.g., 13-cis-retinoic acid) can effectively reduce the recurrence of head and neck cancers, although hepatic toxicity is sometimes a limiting factor in such cancer preventive therapy. Similarly, these agents, as well as β-carotene or vitamin E, taken in large doses have been shown significantly to promote the regression of oral leukoplakia, a premalignant lesion. Daily supplementation with one to three times the U.S. RDA of β-carotene, selenium, and vitamin E has been shown to reduce the incidence of adenocarcinoma of the stomach in a region of China where the disease as well as marginal vitamin status is particularly prevalent. However, as mentioned earlier, trials conducted in developed Western countries have observed no diminution of lung cancer among smokers with daily supplementation of β-carotene and vitamin E. Epidemiologic associations also suggest an inverse relationship between lens cataract or macular degeneration and the intake of vitamin C, vitamin E, and β-carotene. These common degenerative conditions of the eye are caused, at least in part, by photo-oxidation. Some evidence in animal models indicates that they can be retarded by supraphysiologic supplementation with vitamin C or E. When tested under the conditions of a rigorously conducted multicenter, controlled trial, daily supplementation with a combination of vitamin C, vitamin E, and β-carotene (with or without zinc) had no effects compared with placebo on the likelihood for development of cataracts. However, the combination that included zinc produced an approximately 30% decline in the progression of early macular degeneration to an advanced stage and the likelihood of moderate visual acuity loss. Further investigation is necessary to define the circumstances more clearly under which antioxidant nutrients can be used to prevent or to treat chronic degenerative diseases.

Vitamin B12 and Neuropsychiatric Disease

Plasma vitamin B12 concentrations are considered to be an accurate indication of vitamin B12 status. Values greater than 150 pg/mL were thought, until recently, to exclude vitamin B12 deficiency as a cause of neurologic or psychiatric syndromes.8 Recent observations now indicate that 7 to 10% of individuals who have plasma vitamin B12 values between 150 and 400 pg/mL may develop neuropsychiatric complications of vitamin B12 deficiency in the absence of any indications of megaloblastic anemia. Such individuals can be identified by the demonstration of an elevated level of methylmalonic acid in the blood that decreases to normal levels with parenteral vitamin B12 administration. An elevation in serum methylmalonic acid is both a sensitive and a specific indication of cellular vitamin B12 deficiency. An alternative approach is to administer several parenteral injections of vitamin B12 to an individual who has an otherwise unexplained neuropsychiatric syndrome and whose plasma vitamin B12 level falls in the range of 150 to 400 pg/mL. Awareness of this phenomenon is particularly important because it has become clear that atrophic gastritis, an asymptomatic condition that affects approximately

CHAPTER 219  Eating Disorders  

30% of the elderly population, frequently produces a modest decrease in vitamin B12 status; similarly, long-term use of proton pump inhibitor drugs inhibits absorption and also increases the risk of clinically significant deficiency.9

Is Routine Multivitamin and Multimineral Supplementation Beneficial?

A common query by patients is whether regular use of a multivitamin or multimineral supplement is safe and efficacious in the maintenance of health. Although there is not a unanimous consensus about the “correct” answer to this question, the weight of available evidence indicates that for the general adult North American population, supplementation offers little or no benefit in regard to the prevention of the common chronic degenerative diseases, such as vascular disease, cancer, and dementia. A3-A5  Although this apparent lack of efficacy has been notably contradicted by two clinical trials conducted in Western industrialized countries in which men taking multivitamins realized modest decreases in the incidence of cancer,10 such benefits have not been substantiated by other investigations. Although daily supplementation at the levels found in most multivitamin preparations probably presents no risk of harm, adverse health effects have been observed in several rigorously performed clinical trials in which longterm supplementation with micronutrients at levels that exceed the RDA (or conventional levels of dietary intake) by several-fold was examined. For example, an increased incidence of prostate cancer was observed in the SELECT trial, in which vitamin E was administered at a dose of 400 IU/day, and β-carotene supplementation resulted in an increased incidence of lung cancer among heavy smokers in the ATBC and CARET trials at doses of 20 to 30 mg/day. This is not to say that health benefits cannot be realized from supplementation in select groups of individuals, although some thought needs to be exercised to determine which segments of the population should be targeted and what specific nutrients should be administered. Certainly, health benefits are likely in individuals whose dietary intake is chronically inadequate or in patients whose medical conditions are often complicated by micronutrient deficiencies, such as those on chronic renal dialysis or among individuals with marginally controlled intestinal malabsorption. The elderly frequently cannot achieve recommended intakes of vitamin D and calcium with diet alone, and therefore targeted supplementation with these nutrients is often indicated. Similarly, the high prevalence of atrophic gastritis among the elderly as well as the frequent use of proton pump inhibitor drugs each conspire to impair adequate vitamin B12 status.11 Moreover, in many regions of the world, there continues to be a high prevalence of marginal micronutrient status among the general adult population, and in such areas widespread supplementation may be indicated; the Linxian trial in China, in which supplementation with a mixture of several antioxidant micronutrients led to a sizable decrease in gastric cancer, is one such example.

1455

219  EATING DISORDERS MARIAN TANOFSKY-KRAFF

DEFINITION

Feeding and eating disorders are defined as syndromes “characterized by a persistent disturbance of eating or eating-related behavior that results in the altered consumption or absorption of food and that significantly impairs physical health or psychosocial functioning.” The Diagnostic and Statistical Manual of Mental Disorders, fifth edition (DSM-5) defines anorexia nervosa (AN), bulimia nervosa (BN), and binge eating disorder (BED) as primary diagnoses in adolescents and adults. All other diagnoses are identified as Unspecified Feeding or Eating Disorder and represent presentations that do not meet the criteria for the primary eating disorders but nonetheless cause significant distress and impairment. The severity of each disorder is also specified as mild, moderate, severe, or extreme. Given the recent publication of the DSM-5, most empirical data available to date involve the criteria of the Diagnostic and Statistical Manual of Mental Disorders, fourth edition, text revision (DSM-IV-TR, published in 2000).

ANOREXIA NERVOSA

AN involves a restriction of “energy intake relative to requirements, leading to a significantly low body weight in the context of age, sex, developmental trajectory, and physical health.”1 Individuals with AN experience an intense fear of gaining weight or becoming fat, are overly concerned with weight or shape, and often may not recognize the seriousness of their low body weight. AN has two subtypes: restricting and binge-eating/purging. DSM-5 criteria for AN are listed in Table 219-1.

BULIMIA NERVOSA

A diagnosis of BN requires recurrent episodes of binge eating (i.e., the consumption of an unambiguously large amount of food given the context, accompanied by a sense of loss of control over eating). Episodes of binge eating co-occur with behaviors intended to compensate for energy consumed and to prevent weight gain, such as self-induced vomiting and fasting. Binge eating and compensatory behaviors must occur, on average, at least once a week for 3 months. The self-esteem of individuals with BN is excessively influenced by their body weight and shape. DSM-5 criteria for BN are outlined in Table 219-2.

BINGE EATING DISORDER

Grade A References A1. Clarke R, Halsey J, Lewington S, et al. Effects of lowering homocysteine levels with B vitamins on cardiovascular disease, cancer, and cause-specific mortality: meta-analysis of 8 randomized trials involving 37,485 individuals. Arch Intern Med. 2010;170:1622-1631. A2. Balk EM, Raman G, Tatsioni A, et al. Vitamin B6, B12, and folic acid supplementation and cognitive function: a systematic review of randomized trials. Arch Intern Med. 2007;167:21-30. A3. Fortmann S, Burda B, Senger C, et al. Vitamin and mineral supplements in the primary prevention of cardiovascular disease and cancer: an updated systematic evidence review for the U.S. Preventive Services Task Force. Ann Intern Med. 2013;159:824-834. A4. Grodstein F, O’Brien J, Kang J, et al. Long-term multivitamin supplementation and cognitive function in men: a randomized trial. Ann Intern Med. 2013;159:806-814. A5. Lamas G, Roineau R, Goertz C, et al. Oral high-dose multivitamins and minerals after myocardial infarction. A randomized trial. Ann Intern Med. 2013;159:797.

GENERAL REFERENCES For the General References and other additional features, please visit Expert Consult at https://expertconsult.inkling.com.

BED is characterized by recurrent episodes of binge eating in the absence of regular compensatory behaviors that are present in BN. The binge

TABLE 219-1  DSM-5 DIAGNOSTIC CRITERIA FOR ANOREXIA NERVOSA A. Restriction of energy intake relative to requirements, leading to a significantly low body weight in the context of age, sex, developmental trajectory, and physical health. Significantly low weight is defined as a weight that is less than minimally normal or, for children and adolescents, less than minimally expected. B. Intense fear of gaining weight or becoming fat, or persistent behavior that interferes with weight gain, even though at a significantly low weight. C. Disturbance in the way in which one’s body weight or shape is experienced, undue influence of body weight or shape on self-evaluation, or persistent lack of recognition of the seriousness of the current low body weight. Specify whether: Restricting type: During the last 3 months, the individual has not engaged in recurrent episodes of binge eating or purging behavior (i.e., self-induced vomiting or the misuse of laxatives, diuretics, or enemas). This subtype describes presentations in which weight loss is accomplished primarily through dieting, fasting, and/or excessive exercise. Binge-eating/purging type: During the last 3 months, the individual has engaged in recurrent episodes of binge eating or purging behavior (i.e., self-induced vomiting or the misuse of laxatives, diuretics, or enemas). From Diagnostic and Statistical Manual of Mental Disorders. 5th ed. Washington, DC: American Psychiatric Association; 2013.

CHAPTER 218  Vitamins, Trace Minerals, and Other Micronutrients  

GENERAL REFERENCES 1. Ribas GS, Vargas CR, Wajner M. l-Carnitine supplementation as a potential antioxidant therapy for inherited neurometabolic disorders. Gene. 2014;533:469-476. 2. Johnson M, Hausman D, Davey A, et al. Vitamin B12 deficiency in African American and white octogenarians and centenarians in Georgia. J Nutr Health Aging. 2010;14:339-345. 3. Dietary reference intakes for calcium and vitamin D. www.iom.edu/Reports/2010/Dietary-Reference-Intakes-for-calcium-and-vitamin-D.aspx; Accessed March 23, 2015. 4. Chung M, Lee J, Terasawa T, et al. Vitamin D with or without calcium supplementation for prevention of cancer and fractures: an updated meta-analysis for the U.S. Preventive Services Task Force. Ann Intern Med. 2011;155:827-838. 5. Gibson T, Weinstein S, Pfeiffer R, et al. Pre- and postfortification intake of folate and risk of colorectal cancer in a large prospective cohort study in the U.S. Am J Clin Nutr. 2011;94:1053-1062.

1455.e1

6. Figueiredo JC, Grau MV, Haile RW, et al. Folic acid and risk of prostate cancer: results from a randomized clinical trial. J Natl Cancer Inst. 2009;101:432-435. 7. Klein E, Thompson I Jr, Tangen C, et al. Vitamin E and the risk of prostate cancer: the Selenium and Vitamin E Cancer Prevention Trial (SELECT). JAMA. 2011;306:1549-1556. 8. Hunt A, Harrington D, Robinson S. Vitamin B12 deficiency. BMJ. 2014;349:g5226. 9. Lam JR, Schneider JL, Zhao W, et al. Proton pump inhibitor and histamine 2 receptor antagonist use and vitamin B12 deficiency. JAMA. 2013;310:2435-2442. 10. Gaziano J, Sesso H, Christen W, et al. Multivitamins in the prevention of cancer in men: the Physicians’ Health Study II randomized controlled trial. JAMA. 2012;308:1871-1880. 11. Stabler SP. Clinical practice. Vitamin B12 deficiency. N Engl J Med. 2013;368:149-160.

1456

CHAPTER 219  Eating Disorders  

TABLE 219-2  DSM-5 DIAGNOSTIC CRITERIA FOR BULIMIA NERVOSA A. Recurrent episodes of binge eating. An episode of binge eating is characterized by both of the following: 1. Eating, in a discrete period of time (e.g., within any 2-hour period), an amount of food that is definitely larger than most people would eat during a similar period of time and under similar circumstances. 2. A sense of lack of control over eating during the episodes (e.g., a feeling that one cannot stop eating or control what or how much one is eating). B. Recurrent inappropriate compensatory behavior in order to prevent weight gain, such as self-induced vomiting; misuse of laxatives, diuretics, enemas, or other medications; fasting; or excessive exercise. C. The binge eating and inappropriate compensatory behaviors both occur, on average, at least once a week for 3 months. D. Self-evaluation is unduly influenced by body shape and weight. E. The disturbance does not occur exclusively during episodes of anorexia nervosa. From Diagnostic and Statistical Manual of Mental Disorders. 5th ed. Washington, DC: American Psychiatric Association; 2013.

TABLE 219-3  DSM-5 DIAGNOSTIC CRITERIA FOR BINGE EATING DISORDER A. Recurrent episodes of binge eating. An episode of binge eating is characterized by both of the following: 1. Eating, in a discrete period of time (e.g., within any 2-hour period), an amount of food that is definitely larger than most people would eat during a similar period of time and under similar circumstances. 2. A sense of lack of control over eating during the episodes (e.g., a feeling that one cannot stop eating or control what or how much one is eating). B. The binge eating episodes are associated with three (or more) of the following: 1. Eating much more rapidly than normal. 2. Eating until feeling uncomfortably full. 3. Eating large amounts of food when not feeling physically hungry. 4. Eating alone because of feeling embarrassed by how much one is eating. 5. Feeling disgusted with oneself, depressed, or very guilty afterward. C. Marked distress regarding binge eating is present. D. The binge eating occurs, on average, at least once a week for 3 months. E. The binge eating is not associated with the recurrent use of inappropriate compensatory behavior as in bulimia nervosa and does not occur exclusively during the course of bulimia nervosa or anorexia nervosa. From Diagnostic and Statistical Manual of Mental Disorders. 5th ed. Washington, DC: American Psychiatric Association; 2013.

episodes are distinguished by at least three associated characteristics, such as eating rapidly, eating until feeling uncomfortably full, and feeling disgust and guilt regarding the episodes. Individuals experience marked distress surrounding the binge episodes, and the binge eating episodes must occur, on average, at least once a week for 3 months. DSM-5 criteria for BED are listed in Table 219-3.

EPIDEMIOLOGY

Data suggest a lifetime prevalence of AN of approximately 0.6%, with higher rates among women (0.9%) compared with men (0.3%).2 The lifetime prevalence of BN appears to be about 1%, with higher rates among women (1.5%) than among men (0.5%). The lifetime prevalence of BED is estimated at 3.5% for women and 2.0% for men. Among adolescents, lifetime prevalence estimates of AN, BN, and BED have been reported at 0.3%, 0.9%, and 1.6%, respectively.3 Contrary to the view that eating disorders afflict only nonHispanic white, affluent women, individuals of all races, ethnicities, and cultures are affected by these diagnoses.4

PATHOBIOLOGY

Research regarding the neuropathology of eating disorders is in nascent stages. Data suggest that several brain regions may be involved in and potentially interact in the manifestation of all eating disorders. Individuals with eating disorders appear to have brain function alterations in emotional/ limbic, reward, and cognitive control circuits.5 Fear circuitry networks involving the amygdala, anterior cingulate cortex, hippocampus, insula, striatum, and prefrontal cortex have demonstrated differential activation among individuals with eating disorders (with the majority of research in AN) compared with controls. Specifically, there tends to be a hyper-responsiveness in the limbic circuitry in response to potentially threatening cues, such as food and body weight/shape. There also appear to be alterations in reward function in patients with AN, but the direction is unclear. By contrast, individuals with BN and BED

consistently demonstrate hyper-responsivity in reward and somatosensory regions on exposure to food images. Data also suggest that individuals with eating disorders may have dysregulated frontal cortical cognitive neural networks acting in concert with regional reward systems.6 Individuals with eating disorders have demonstrated impaired cognitive flexibility. Specific to BN, impulsivity and poor inhibitory control have also been reported. Given the brain regions implicated in eating disorders, current studies have focused on the role of dopamine and serotonin in the manifestation of eating disorders. Individuals with AN appear to have impaired dopaminergic signaling, particularly in striatal circuits, that might contribute to altered reward and affect, decision making, and executive control as well as compulsivity and decreased food ingestion. Moreover, emerging clinical research suggests that striatal dopamine abnormalities exist in individuals with BN and BED. Because serotonin (5-hydroxytryptamine) 1A and 2A receptors and the serotonin transporter may play a part in symptoms of eating disorders, such as impulse control and associated mood symptoms, it is likely that interactions between the serotonin and dopaminergic systems contribute to eating disorders.

Risk Factors

Eating disorders develop as the result of multiple biological, psychological, and sociocultural factors. AN, BN, and BED aggregate in families, with estimates from twin studies suggesting that 40 to 60% of vulnerability for eating disorders is genetic. Studies have reported links between eating disorders and polymorphisms in the serotonin transporter gene (SLC6A4), the dopamine D2 receptor (DRD2) gene, the µ1 opioid receptor (OPRM1) gene, the fat mass and obesity-associated (FTO) gene, and the brain-derived neurotrophic factor (BDNF) gene. Although genetic linkage and association studies have implicated several susceptibility loci for AN, BN, and BED, specific genes that consistently lend vulnerability to eating disorders are less conclusive.7 Female sex, pediatric overweight, elevated shape and weight concerns, sexual abuse, trauma, and mood disorders have been identified as risk factors for all eating disorders. Personality-related variables, such as impulsivity and perfectionism, appear to be linked to eating disorders.8 Importantly, internalization to the “thin ideal” (a sociocultural emphasis on shape and weight and a marked preference for a thin body type) with resulting weight and shape concerns has been proposed to contribute to eating disorder development, particularly among adolescents who are under strong influence from their peer and family environments. For example, parental overconcern about eating, shape, and weight as well as weight-related teasing by family members confers risk for eating disorders. Specific to BED, maltreatment, including teasing and bullying, and perceived stress are risk factors for the disorder.9

CLINICAL MANIFESTATIONS

Symptoms and Signs

For AN, physical symptoms and signs may include amenorrhea, constipation, cold intolerance, anemia, and lanugo hair. Reduced bone density is believed to predict the onset of premature osteopenia and osteoporosis. Health problems associated with malnutrition affect cardiovascular, gastrointestinal, reproductive, and endocrine systems. Individuals with AN frequently present with comorbid psychiatric disorders, including mood and anxiety disorders (e.g., social phobia, specific phobia, post-traumatic stress disorder), and high rates of suicidal ideation and behavior. Individuals with BN present with signs and symptoms most commonly associated with purging behavior. These include dental enamel erosion secondary to vomiting, gastrointestinal symptoms, salivary gland hypertrophy, and electrolyte disturbances. Electrolyte abnormalities can have deleterious effects on the renal and cardiovascular systems. BN patients are at risk for cardiometabolic conditions (e.g., diabetes, stroke) as well as chronic pain. Metabolic acidosis can also occur in patients who are abusing laxatives as a result of the loss of bicarbonate from the bowel. Noninflammatory swelling of the salivary glands is a common clinical manifestation of BN. The most common psychiatric comorbidities in BN are major depressive disorder, anxiety disorders, substance use disorders, and disruptive behavioral disorders. Individuals with BED are frequently overweight or obese. However, adults with BED are likely to report the development of diagnoses of metabolic syndrome components (e.g., dyslipidemia, hypertension, type 2 diabetes) after accounting for the contribution of body weight. The presence of BED may affect bariatric surgery outcome, resulting in less weight loss or more weight regain, but this is not a consistent finding. However, the presence of “loss of control” eating after surgery consistently predicts less weight loss or greater weight regain. Compared with obese adults without BED, those with the disorder experience significant impairment in a number of domains of

CHAPTER 219  Eating Disorders  

psychosocial functioning, including a poorer quality of life and more impaired functioning in their home and social lives. Individuals with BED often have higher levels of disability, health problems, and work productivity impairment compared with obese and healthy controls without binge eating. With regard to comorbid psychiatric diagnoses, adults with BED experience Axis I psychiatric disorders at a rate comparable to (or higher than) that of individuals with AN or BN, including major depressive disorder, anxiety disorders, substance use disorders, and disruptive behavioral disorders.

Natural History

AN is typically manifested during adolescence, although the disorder can develop before puberty. BN frequently develops during later adolescence or early adulthood. BED is often manifested in adulthood, but adolescents also present with the disorder. Several retrospective and prospective studies report that binge and “out of control” eating occur as early as middle childhood. Data on the natural course of eating disorders in the clear absence of treatment are limited. Eating disorders tend to exhibit a remitting and relapsing natural course across the lifespan, and there appear to be high rates of diagnostic crossover.10 Treatment outcome data indicate that AN tends to transition to BN or an Unspecified Eating Disorder, and those with BN and BED tend to migrate from one to the other.

DIAGNOSIS

A number of structured, well-validated assessments for the diagnosis of eating disorders exist. These include but are not limited to the Structured Clinical Interview for the DSM and the Eating Disorder Examination. However, eating disorders are typically diagnosed by review of the patient’s history, symptoms, and behaviors in an interview format. Evaluation of comorbid psychiatric problems, most notably mood, anxiety, substance use disorders, and disruptive behavioral disorders, is also required. Information should be gathered on interpersonal relationships, history of sexual and physical abuse, self-harm, and suicidal ideation or behavior. Family involvement is crucial, particularly for pediatric patients. A complete physical examination to assess body composition, vital signs, cardiovascular function, and hematologic and blood chemistry parameters is recommended for all patients.

TREATMENT  Anorexia Nervosa

There is limited evidence on effective treatments for AN. For severely underweight patients, inpatient medical monitoring and supervised nutrition  rehabilitation are required. The optimal setting (inpatient versus outpatient treatment) remains a subject of debate, and the evaluation of treatment costs in AN plays an important role in determining treatment. However, for pediatric patients, family-based psychotherapy, particularly during the early phases of the disorder, has demonstrated effectiveness.11 Maudsley’s family-based therapy involves both joint family sessions and simultaneous but independent patient/family intervention. Antidepressants (e.g., selective serotonin reuptake inhibitors) are associated with high rates of noncompliance, and compelling evidence of beneficial effects has not been found. The use of antipsychotic drugs has been explored, but results regarding their effectiveness remain nondefinitive.

Bulimia Nervosa

Cognitive-behavioral therapy (CBT) has been recognized as the treatment of choice for BN. A1  Interpersonal psychotherapy (IPT) is also effective for the treatment of BN, particularly for those who are nonresponsive to CBT. There is growing support that pharmacotherapy may be helpful for some patients with BN. Antidepressants, especially selective serotonin reuptake inhibitors, are modestly effective for reducing binge eating in BN over the short and long term. Topiramate has consistently been shown to decrease binge eating in BN, but side effects may limit its usefulness. It is unclear whether combination therapy may be required for optimal outcomes.

Binge Eating Disorder

Psychological treatment for BED aims to reduce binge eating, weight and shape concerns, and prevent excess weight gain and/or induce modest weight loss. The psychotherapies most evaluated in clinical trials include CBT, IPT, behavioral weight loss, and CBT guided self-help (CBTgsh) approaches. CBT and IPT are first-line treatments. Given its cost-effectiveness, CBTgsh may be an optimal treatment option when specialist care is not available. A2  With regard to pharmacologic treatment in BED, three medications or classes of medications have been studied in two or more placebo-controlled trials. Selective serotonin reuptake inhibitors, sibutramine, and topiramate all produce reductions in frequency of binge eating relative to placebo in short-term

1457

trials.12 However, sibutramine has been withdrawn from the market, and topiramate is frequently associated with problematic cognitive effects, thus limiting its clinical utility.

PREVENTION

Whereas an increasing number of macro-level environmental public health initiatives have emerged (i.e., anti-dieting media campaigns and sanctions on advertising practices propagating an ideal of extreme thinness), few empirical data exist evaluating their efficacy. However, there are more data on individual, micro-level interventions aimed at reducing proximal eating disorder risk factors as well as current and distal eating pathology. Selected, interactive, multisession programs with adolescent girls may be more effective than universal, didactic, heterogeneous-sampled and single-session programs in reducing risk factors for eating disorder symptoms. For example, a dissonancebased program aimed at reducing eating disorder risk factors in adolescent girls has demonstrated effectiveness.

  PROGNOSIS

Anorexia Nervosa

Remission rates vary widely for AN. Lower remission rates (29%) have been observed, particularly in studies with the shortest follow-up duration. However, most individuals with AN (approximately 76%) treated in outpatient settings will remit within 5 years after the initiation of treatment. Most individuals who do not achieve remission from AN during follow-up periods transition to a diagnosis of BN or an Unspecified Eating Disorder, which likely captures partial syndrome AN. Among psychiatric diagnoses, AN consistently has one of the highest mortality rates due to suicide, nutritional deficits, cardiac complications, and substance abuse.13 The crude cumulative mortality rate is 2.8%, with longer duration of illness before receiving treatment and the need for inpatient treatment as negative prognostic indicators for AN.14 Predictors of relapse include desiring a lower weight at the end of treatment and receiving treatment in a general (versus specialty) clinic.

Bulimia Nervosa

Similar to AN, most individuals with BN (70% or more) who receive treatment fully remit when assessed 5 to 20 years later, with remission rates being much lower (27 to 28%) at 1-year follow-up. If individuals with BN do not achieve remission within 5 years, however, they are likely to exhibit a chronic course of the illness. Mortality rates for BN range between 0 and 2%. Diagnostic crossover from BN to AN is relatively rare; yet, there is frequent diagnostic crossover between BN and BED, which may suggest a possible common psychological and/or biologic maintaining process. Negative prognostic indicators for BN include endorsement of greater psychiatric comorbidity, multiple impulsive behaviors (e.g., self-harm, substance use disorder), and a family history of alcohol abuse. Individuals who receive inpatient treatment or have a low motivation for engaging in treatment are more likely to relapse.

Binge Eating Disorder

A paucity of data exists on the long-term outcomes for BED patients. There are data to suggest that at 1 year after outpatient treatment, upwards of 80% of patients remit. In one clinical trial that examined 4-year outcomes, between 52 and 76% of individuals receiving psychological treatment for BED demonstrated remission from binge eating.15 These preliminary data suggest that the prognostic trajectory may be similar to that of BN. Diagnostic crossover from BED to BN is high, whereas crossover to AN is relatively rare. Although examination of prognostic indicators for BED is in its early stages, patients reporting an undue influence of their body shape or weight on self-evaluation are less likely to have remission from binge eating at 12-month follow-up.16 Rapid remission of binge eating has also been shown to be a positive prognostic indicator for binge remission.17

Grade A References A1. Lock J, Le Grange D, Agras WS, et al. Randomized clinical trial comparing family-based treatment with adolescent-focused individual therapy for adolescents with anorexia nervosa. Arch Gen Psychiatry. 2010;67:1025-1032. A2. Poulsen S, Lunn S, Daniel SI, et al. A randomized controlled trial of psychoanalytic psychotherapy or cognitive-behavioral therapy for bulimia nervosa. Am J Psychiatry. 2014;171:109-116.

GENERAL REFERENCES For the General References and other additional features, please visit Expert Consult at https://expertconsult.inkling.com.

CHAPTER 219  Eating Disorders  

GENERAL REFERENCES 1. Attia E, Becker AE, Bryant-Waugh R, et al. Feeding and eating disorders in DSM-5. Am J Psychiatry. 2013;170:1237-1239. 2. Smink FR, van Hoeken D, Hoek HW. Epidemiology, course, and outcome of eating disorders. Curr Opin Psychiatry. 2013;26:543-548. 3. Swanson SA, Crow SJ, Le Grange D, et al. Prevalence and correlates of eating disorders in adolescents. Results from the national comorbidity survey replication adolescent supplement. Arch Gen Psychiatry. 2011;68:714-723. 4. Pike KM, Hoek HW, Dunne PE. Cultural trends and eating disorders. Curr Opin Psychiatry. 2014;27:436-442. 5. von Hausswolff-Juhlin Y, Brooks SJ, Larsson M. The neurobiology of eating disorders-a clinical perspective. Acta Psychiatr Scand. 2014;[Epub ahead of print]. 6. Frank GK, Kaye WH. Current status of functional imaging in eating disorders. Int J Eat Disord. 2012;45:723-736. 7. Stefano GB, Ptacek R, Kuzelova H, et al. Convergent dysregulation of frontal cortical cognitive and reward systems in eating disorders. Med Sci Monit. 2013;19:353-358. 8. Kessler RC, Berglund PA, Chiu WT, et al. The prevalence and correlates of binge eating disorder in the World Health Organization World Mental Health Surveys. Biol Psychiatry. 2013;73: 904-914.

1457.e1

9. Tanofsky-Kraff M, Bulik CM, Marcus MD, et al. Binge eating disorder: the next generation of research. Int J Eat Disord. 2013;46:193-207. 10. Allen KL, Byrne SM, Oddy WH, et al. DSM-IV-TR and DSM-5 eating disorders in adolescents: prevalence, stability, and psychosocial correlates in a population-based sample of male and female adolescents. J Abnorm Psychol. 2013;122:720-732. 11. Focker M, Knoll S, Hebebrand J. Anorexia nervosa. Eur Child Adolesc Psychiatry. 2013;22 (suppl 1):S29-S35. 12. McElroy SL, Guerdjikova AI, Mori N, et al. Pharmacological management of binge eating disorder: current and emerging treatment options. Ther Clin Risk Manag. 2012;8:219-241. 13. Campbell K, Peebles R. Eating disorders in children and adolescents: state of the art review. Pediatrics. 2014;134:582-592. 14. Keel PK, Brown TA. Update on course and outcome in eating disorders. Int J Eat Disord. 2010;43:195-204. 15. Franko DL, Keshaviah A, Eddy KT, et al. A longitudinal investigation of mortality in anorexia nervosa and bulimia nervosa. Am J Psychiatry. 2013;170:917-925. 16. Hilbert A, Bishop ME, Stein RI, et al. Long-term efficacy of psychological treatments for binge eating disorder. Br J Psychiatry. 2012;200:232-237. 17. Grilo CM, White MA, Gueorguieva R, et al. Predictive significance of the overvaluation of shape/ weight in obese patients with binge eating disorder: findings from a randomized controlled trial with 12-month follow-up. Psychol Med. 2013;43:1335-1344.

1457.e2

CHAPTER 219  Eating Disorders  

REVIEW QUESTIONS 1. A 21-year-old woman presents with scarred knuckles, faded wrist scarring, electrolyte imbalance, and low weight (body mass index [BMI] = 20). The menstrual cycle is reported as abnormal and infrequent. The patient denies current abnormal or restrictive eating patterns and self-injurious and compensatory behaviors. The patient expresses clear discomfort during collection of weight and avoids eye contact with clinical staff. No binge eating is reported. What is the most likely diagnosis for this individual? A. Anorexia nervosa, general diagnosis: patient exhibits low body weight, potential self-harming behaviors, amenorrhea, and clear discomfort with body weight. B. Bulimia nervosa, general diagnosis: low weight, knuckle scarring, and electrolyte imbalance are consistent with bulimia nervosa. C. Anorexia nervosa, purging subtype: low weight, potential self-harming behaviors, and amenorrhea are consistent with anorexia nervosa; knuckle scarring and electrolyte imbalance are consistent with anorexia nervosa purging subtype. D. No current eating disorder diagnosis: patient’s BMI is not significantly low within the context of age and sex to indicate a clinical diagnosis of anorexia nervosa, and binge eating and compensatory behaviors are not present. E. Bulimia nervosa, restricting subtype: presence of menstrual cycle, discomfort with body weight, knuckle scarring, and electrolyte imbalance are consistent with bulimia nervosa; low body weight is consistent with bulimia nervosa restricting subtype. Answer: D  The patient’s body weight is within a normal range, and denial of abnormal or restrictive eating patterns excludes a diagnosis of anorexia nervosa. Although knuckle scarring and electrolyte imbalance are suggestive of self-induced vomiting, a diagnosis of bulimia nervosa cannot be made without clear indication of the regular use of compensatory behaviors. Although amenorrhea is often a symptom, it is currently neither a necessary nor conclusive criterion for the diagnosis of anorexia nervosa. Although the patient cannot be given a conclusive eating diagnosis, further psychological and physical examination is warranted. 2. A 19-year-old female college student presents with current bulimia nervosa. The patient reports feeling very dissatisfied with her body shape and weight and engaging in binge eating while watching reality television. What is the best initial course of treatment action for you to suggest to the patient? A. Instruct the patient to modify television preferences to reduce episodes of binge eating while opening up a dialogue with friends about healthy eating habits. B. Refer the patient to a therapist for cognitive-behavioral therapy. C. Prescribe topiramate to help the patient reduce episodes of binge eating. D. Instruct the patient to keep a journal documenting emotions surrounding binge episodes for personal reflection. E. Refer the patient to a therapist for dialectical behavioral therapy. Answer: B  Cognitive-behavioral therapy remains the “gold standard” for the treatment of bulimia nervosa. Although other interventions, such as interpersonal psychotherapy and dialectical behavioral therapy, have shown efficacy in treatment of bulimia nervosa, cognitive-behavioral therapy shows the greatest efficacy across studies and subjects. Although discussing or expressing emotions regarding food and body shape and weight can be helpful for individuals with bulimia nervosa, constructive guidance is generally necessary for symptom reduction and cessation. Although topiramate has been shown to help individuals reduce binge eating, it has a number of cognitive side effects that may make its use impractical.

3. Which of the following is not a criterion for the diagnosis of binge eating disorder? A. BMI ≥ 30 (body mass index obesity threshold) B. Marked distress surrounding binge episodes C. Binge eating D. Lack of regular compensatory behaviors E. Weekly episodes of binge eating for at least 3 months Answer: A  There is no weight requirement for a diagnosis of binge eating disorder. Although individuals with binge eating disorder are often overweight or obese, BMI is not a required criterion for the diagnosis. 4. Which of the following statements regarding the genetics of disordered eating is true? A. Identical twins are 100% concordant for anorexia nervosa. B. Several genes have been conclusively linked to increased vulnerability for eating disorders. C. Of the neurotransmitters, dopamine and norepinephrine are the most likely candidates for eating disorder maintenance. D. Media generally play a bigger role than genetics in the onset of eating disorders. E. Children of individuals with binge eating disorder are more likely to experience out of control eating episodes than anorexia nervosa. Answer: E  Studies show that disordered eating behaviors, such as binge or loss of control eating, are often highly heritable. Offspring of mothers with binge eating have a higher genetic propensity to develop binge eating than do children of mothers who do not exhibit the behavior. Although monozygous twins share equal genetic predisposition for the development of disordered eating behaviors, no eating disorder shows 100% concordance between monozygous twins. Furthermore, although several gene loci have been indicated in the development and maintenance of eating disorders, evidence does not conclusively point to specific genes as responsible for disordered eating. Although studies have indicated the role of norepinephrine in the onset and maintenance of eating disorders, dopamine and serotonin are currently generally considered to be the most likely neurotransmitters to influence disordered eating behaviors. Findings regarding the effect of media on increasing risk for disordered eating are mixed; genetics, however, are strongly implicated. 5. Which of the following is not a finding regarding the neuropathology of eating disorders? A. Individuals with bulimia nervosa and binge eating disorders are hyperresponsive to food images in reward and somatosensory regions. B. Individuals with bulimia nervosa exhibit decreased impulsivity and enhanced inhibitory control relative to control subjects. C. Individuals with anorexia nervosa exhibit impaired dopaminergic signaling in striatal circuits. D. Individuals with anorexia nervosa exhibit differential activation of fear circuits relative to control subjects. E. Findings indicate that dysregulation in the limbic system may be influential in the development of eating disorders. Answer: B  Data suggest that individuals with bulimia nervosa exhibit increased impulsivity and decreased inhibitory control relative to control subjects.

1458

CHAPTER 220  Obesity  

220  OBESITY MICHAEL D. JENSEN

Obesity is the most common nutritional disorder in the United States and directly or indirectly accounts for a significant portion of health-related expenses. The safest treatment approaches (lifestyle change and behavior modification) are not those commonly employed by physicians and require considerable time to implement. The recently released Guideline for the Management of Overweight and Obesity in Adults provides direction for clinicians for the treatment of obesity.

DEFINITION

The Guideline for the Management of Overweight and Obesity in Adults produced by the National Institutes of Health and the National Heart, Lung, and Blood Institute (NHLBI) and disseminated by the American College of Cardiology (ACC), the American Heart Association (AHA), and The Obesity Society (TOS) provides evidence-informed, scientifically based recommendations on evaluation and management of overweight and obesity. Body mass index (BMI) continues to be the recommended approach to categorize weight relative to height for adults. BMI is calculated as weight (in kilograms) divided by height squared (in meters): BMI =

weight (kg ) height 2 (m 2 )

To calculate BMI with pounds and inches, the formula is modified as follows: BMI =

weight (lb) height 2 (in 2 )

The guideline suggested that no changes are indicated in the weight classifications by BMI, which are summarized in Table 220-1. Individuals who are overweight (BMI of 25.0 to 29.9) may or may not be overfat. Some adults may be overweight because of increased muscle mass, which is a straightforward clinical observation. Although, in general, the risk for development of adiposity-related health problems increases continuously as the BMI exceeds 25, the new guideline continues to recommend the use of waist circumference measurements to discriminate among patients who may require more testing. Overweight and class I obese patients with a waist circumference in the high-risk category deserve a discussion of lifestyle issues as they relate to health and weight loss. Some individuals with a BMI of 27 to 29.9 develop serious metabolic complications that improve with weight loss and are candidates for more aggressive treatment, including pharmacotherapy if it is needed. Asian populations, in particular, are at risk for the typical metabolic complications of obesity at lower BMI and waist circumferences than those for whites, Hispanics, blacks, and Polynesians; the guideline for at-risk BMI in Asian populations is 23 to 24.

TABLE 220-1  CLASSIFICATION OF OVERWEIGHT AND OBESITY BY BODY MASS INDEX (BMI) BMI (kg/m2) 40) is one of the key features that would prompt consideration of a patient for bariatric surgery when medical treatments have failed. Patients with class II obesity (BMI of 35.0 to 39.9) may be considered for bariatric surgery if medical treatments have failed and if severe, life-threatening complications are present. As noted, the new NHLBI/ACC/AHA/TOS guidelines continue to recommend waist circumference as an office assessment tool to help with the treatment decision-making process. The new guidelines suggest that the previous waist circumference cut points of more than 102 cm (40 inches) for men and more than 88 cm (35 inches) for women are indicators of increased metabolic risk. However, the report stated that the relationships between disease risk and waist circumference are continuous and progressive, with no obvious cut points. The recommendation is to measure waist circumference in overweight and class I obesity adults. Those adults with waist circumferences above the cut points deserve further evaluation to detect other cardiovascular disease risk factors. Adults with class II or class III obesity are at sufficiently high risk that waist circumference information does not appear to add valuable information. These definitions of overweight and obesity and of high-risk waist circumference are generally applicable to those of European and African descent, but lower values are recommended for those of Asian descent. The risks of metabolic abnormalities occur at lower BMI and lower waist circumference in these populations.

I

30.0-34.9

Obesity

II

35.0-39.9

Extreme obesity

III

≥40

Jensen MD, Ryan DH, Apovian CM, et al. 2013 AHA/ACC/TOS guideline for the management of overweight and obesity in adults: a report of the American College of Cardiology/American Heart Association Task Force on Practice Guidelines and The Obesity Society. J Am Coll Cardiol. 2014;63:2985-3023.

Although the number of overweight and obese adults in the United States has increased dramatically during the past 30 years, the increase in the prevalence is now slowing or leveling off.1 In 2009-2010, the prevalence of obesity was 35.5% among adult men and 35.8% among adult women, with no significant change compared with 2003-2008. Approximately 60% of U.S. men and 51% of U.S. women are overweight or obese, although a greater percentage of women than men are obese. There are substantial differences in the prevalence of obesity by age, race, and socioeconomic status. The prevalence of obesity in adults tends to rise steadily from the ages of 20 to 60 years, decreasing in later years. It has been estimated that almost 75% of men aged 60 to 69 years in the United States have a BMI of more than 25. The increase in mean BMI with age is not as much of a threat to population health as is a similar increase in the BMI of younger populations. The lowest mortality rates for young adults are for a BMI in the lower part of the normal range (20.0 to 24.9), whereas the BMI associated with the lowest mortality rates is somewhat above 25 kg/m2 for those in the 60s and 70s. Physicians should base their weight recommendations for individual patients on whether adverse health consequences associated with obesity are present. The differences in overweight and obesity among African Americans, Mexican Americans, and European Americans are not subtle. African American women and Mexican Americans of both sexes have the highest rates of overweight and obesity in the United States. In interpreting these data, however, it is important to keep in mind that there is an inverse relationship between socioeconomic status and obesity, especially among women (Chapter 5). Women in lower socioeconomic classes are much more likely than those in higher socioeconomic classes to be obese. This association reduces but does not eliminate the racial differences in the prevalence of obesity. Whether the remaining racial differences in the prevalence of obesity are due to genetic, constitutional, or social factors is not yet known.

PATHOBIOLOGY

Etiology

Genetic and constitutional susceptibility to obesity are heavily influenced by the environment. Evidence from studies of twins adopted into different families indicates that within a given environment, a significant portion of the variation in weight is genetic.2 That said, the remarkable increase in the prevalence of obesity in the United States during the past 3 decades is unlikely to be due to wholesale changes in the genetic makeup of Americans.

CHAPTER 220  Obesity  

Genetic Aspects of Human Obesity

Although obesity susceptibility is a classic polygenic condition, there are also a number of syndromic and monogenic obesity syndromes. The longrecognized genetic defects resulting in obesity include Prader-Willi and Laurence-Moon-Biedl syndromes. More recently, rare monogenic forms of human obesity due to mutations in the leptin gene, the leptin receptor gene, and the melanocortin signaling system genes have been described. These gene mutations are most often associated with increased appetite rather than with reduced energy expenditure. Genome-wide association studies have reported a number of genes associated with higher BMI. Those that appear to predict the greatest amount of variance in BMI include the fat mass and obesity-associated (FTO) gene and the melanocortin-4 receptor (MC4R) gene. Other genes that have been reliably associated with obesity include TMEM18, KCTD15, GNPDA2, SH2B1, MTCH2, and NEGR1. Together, however, the combined effects of all the identified genetic contributions account for less than 1% of the variance in BMI. This emphasizes both the huge environmental effects and the polygenic nature of susceptibility to obesity.

Constitutional Influences on Obesity

A number of environmental factors can result in long-term, epigenetic effects on body weight regulation and the susceptibility to obesity-related health problems. These epigenetic effects are ascribed to processes that include changes in DNA methylation, acetylation, and chromatin remodeling. The effect of the intrauterine environment and the perinatal period on subsequent weight and health is best studied. Undernutrition in the last trimester of pregnancy and in the early postnatal period decreases the risk of adult obesity, although the low birthweight associated with undernutrition (or smoking) in late pregnancy also increases the risk of adulthood hypertension, abnormal glucose tolerance, and cardiovascular disease. In contrast, undernutrition limited to the first two trimesters of pregnancy is associated with an increased probability of adult obesity. The infants of diabetic mothers tend to be fatter than those of nondiabetic mothers, and children of diabetic mothers have a greater prevalence of obesity when they are 5 to 19 years old, independent of whether their mother is obese. Finally, intrauterine exposure to the diabetic environment results in an increased risk of diabetes mellitus and obesity in the offspring. Thus, the issue of the genes versus the environment in regard to obesity and metabolic complications of obesity is blurred in the intrauterine and perinatal time intervals. One of the striking and worrisome aspects of these metabolic effects is not only the long-term effects on the individual’s weight regulation and health but also the suggestion that these traits can be passed on to future generations.

Environmental Contributors to Human Obesity

Dramatic changes in the environment of Western countries have occurred during the past 50 years, including reduced demands for physical activity and alterations in the food supply. These food supply changes appear to have either increased or prevented the expected decrease in energy intake that would be needed to match the reduced energy expenditure from physical activity.3

Food

A number of environmental factors can influence food intake (Table 220-2). Consuming energy-dense foods results in greater energy intake because

TABLE 220-2  ENVIRONMENTAL FACTORS PROMOTING OBESITY DIETARY

ACTIVITY

↑ Energy density of foods

↑ Sedentary behavior

↑ Portion size

↓ Activities of daily living

↑ Variety

↓ Employment physical activity

*

↑ Palatability ↑ Availability ↓ Cost ↑ Caloric beverages (sugar-sweetened beverages) *

Variety of sweets, snacks, and entrees.

1459

adults tend to respond to food volume rather than to the energy content. This factor likely accounts for the association between high-fat diets and excess body weight; many high-fat foods are also energy dense. When humans consume diets that are high in fat but low in energy density, energy intake is not greater than would be expected on the basis of the energy density of the foods. Larger food portion size has also been shown to increase food intake. Given the trend in the United States to serve larger portions of food and beverage, this could contribute to greater obesity risk. Food variety can also affect energy intake. An increased variety of entrees, sweets, snacks, and carbohydrates in the diet is associated with an increase in body fatness and food intake. In contrast, an increase in the variety of vegetables available does not appear to increase energy intake and is not associated with increased body fatness. Other factors that may have broad population effects in the United States include the reduced costs of food, increased availability, and palatability. Finally, there is evidence that consumption of sugar-sweetened beverages, such as soft drinks and fruit juices, is not accompanied by a decrease in food intake to offset the extra energy intake. The implication is that some types of beverages will add to the energy intake during the day and promote weight gain. A number of psychological factors also influence how the properties of food affect energy intake. Individuals vary with respect to their dietary restraint (the tendency to consciously limit food intake to control weight), their feelings of hunger, or their disinhibition (the tendency to overeat opportunistically). It has been proposed that interindividual differences in these factors modify how food variety and portion size affect the eating profile. The social context in which food is consumed and the emotional state of the individual also modulate food intake.

Physical Activity

Physical activity can be divided into three categories: (1) exercise (fitnessand sports-related activities); (2) work-related physical activity; and (3) nonexercise, nonemployment (spontaneous) activity. Tables are widely available that allow one to calculate energy expenditure on the basis of an individual’s weight as well as the type and duration of exercise. Only about 20 to 30% of Americans engage in exercise at the recommended frequency, intensity, or duration that could be expected to have a protective effect on the development of obesity and other health problems, but this does not seem to have changed in recent decades. Recent data suggest that the amount of time spent in sedentary activities (e.g., watching television, using the computer) is an independent predictor of metabolic abnormalities associated with obesity over and above the effects of exercise. Thus, to the extent that reduced physical activity is contributing to the epidemic of obesity, it is likely that it is reduced employment-related and spontaneous physical activity that is changing. Although it is becoming easier for individuals to measure the energy expended in nonexercise activity with step counters and electronic motion detection devices, there are insufficient longitudinal, population-based data to define the extent to which changes in this activity parameter have occurred. Certainly, employment-related physical activity has decreased with the advent of more automated systems in the workplace. One estimate suggests that between 1982 and 1992, energy expenditure at work decreased by about 50 kcal/day. The additional workplace changes since that time have probably reduced employment physical activity further. The other component of nonexercise physical activity, the activities of daily living, has probably been reduced by the plethora of labor-saving conveniences (e.g., drive-through food and banking, escalators, remote controls, e-mail, online shopping) now available. Again, there are few hard data to assess how much of a change has actually occurred, although a reduction in daily walking trips and an increase in daily automobile trips have been documented. There is a large amount of information on how differences in sedentary activity (television watching, video games, and computer use) relate to obesity and obesity complications. The evidence indicates that more time spent in sedentary pursuits is associated with an increased risk of overweight and obesity. The striking aspect to these studies is that the adverse effect of sedentary activities is independent of participation in traditional exercise activities. Understanding the contributions of decreased work-related physical activity, decreases in activity of daily living, and increases in sedentary behavior can help the physician working with the patient to uncover patterns that may relate to weight gain. Physicians who are aware of these environmental factors are in a better position to help their obese patients identify which of these

1460

CHAPTER 220  Obesity  

environmental factors are contributing to the problem and to develop plans for intervention. In this regard, patients who regularly use step counters or other types of activity-monitoring devices will be better able to self-identify and modify their behavior to obtain sufficient physical activity.

Regulation of Body Weight and Energy Balance

The regulation of adult body weight is a well-balanced process. For example, the typical U.S. adult will take in and expend approximately 2000 to 3000 kcal/day. If there were a consistent error of even 1% in overconsumption of food, this would result in the gain of approximately 25 to 30 pounds of fat every 10 years, assuming no change in energy expenditure. It follows that most adults regulate their average energy balance with greater than 1% precision. There appears to be regulation of both energy intake and energy expenditure through conscious and unconscious processes. The excess energy consumed by adults is generally stored as triglycerides in adipocytes. Humans continuously recruit new adipocytes from a large preadipocyte pool to replace dying adipocytes. Although the primary means by which abdominal adipose tissue mass expands is through increased fat cell size (adipocyte hypertrophy), this process can store only a limited amount of fat. Adults who gain leg fat accumulate more rather than larger adipocytes on average, resulting in a net increase in adipocyte number as more new adipocytes are created than needed to replace dying cells. Some adults recruit new adipocytes more readily than others do and thus gain weight more so from adipocyte hyperplasia (increased fat cell number) than from hypertrophy. Those who gain fat with large adipocytes, especially in association with an adipose tissue inflammatory response (greater numbers of classically activated macrophages and other immune cells), are more likely to be insulin resistant and to have signs of low-grade systemic inflammation (increased C-reactive protein, mildly elevated interleukin-6 and tumor necrosis factor). Leptin, a cytokine family protein that is secreted almost exclusively by adipocytes, was the first identified adipose tissue hormone; it has been shown to have potent central nervous system effects on food intake in humans. Leptin also has other hypothalamic-pituitary functions and is proposed to have diverse peripheral physiologic actions. The leptin-deficient animal model of obesity, the ob/ob mouse, is severely obese, hyperphagic, hypometabolic, and sexually immature and has low levels of spontaneous activity. Administration of leptin to this animal corrects all of these defects. A few leptin-deficient humans (due to mutations in the leptin gene) have been identified. These children had very low plasma leptin concentrations, were hyperphagic and severely obese, and responded to exogenous leptin administration with dramatic weight loss, reduced food intake, and accelerated maturation of the pituitary-gonadal axis. Overwhelmingly, however, obese humans are not leptin deficient and in fact have high plasma leptin concentrations unless they are in a major negative energy balance circumstance. Because leptin is secreted as a function of percentage body fat, and because women have more body fat than men for any given BMI, they also have higher plasma leptin concentrations. Thus, screening for leptin deficiency is not warranted except in severe, hyperphagic obesity that begins in early childhood, is accompanied by sexual immaturity, and exists in the absence of other known causes (e.g., Prader-Willi syndrome). Some animal models of genetic obesity (the db/db mouse and fa/fa rat) have defective leptin receptors, making them unresponsive to leptin. Although rare cases of obese humans with defective leptin receptor genes have been reported, again it appears that leptin resistance due to leptin receptor defects (or genetic post-receptor signaling abnormalities) is extremely uncommon. Clinical screening for leptin receptor mutations is not warranted, given that no treatment exists.

Energy Intake

Much of what has been learned about the biologic regulation of food intake has been from the study of animal models. These signals may affect different aspects of eating behavior. They can affect hunger, the compelling need or desire for food; satiation, the state of being satisfactorily full and unable to take on more; or satiety, the sense of no longer being hungry, a complex set of postprandial events that affect the interval to the next meal or the amount consumed at the next meal. Some of the signals that alter eating behavior affect one aspect and others affect multiple aspects. For example, ghrelin, a peptide produced by the stomach, increases hunger but does not appear to affect satiation or satiety. Cholecystokinin causes satiation but has no effect on satiety. Leptin appears to act on multiple pathways; leptin deficiency is associated with increased hunger and reduced satiation and satiety.

TABLE 220-3  SUGGESTED BIOLOGIC MODULATORS OF FOOD INTAKE PROPOSED EFFECT ON FOOD INTAKE

PERIPHERAL SIGNAL Vagal



Cholecystokinin



Apolipoprotein A-IV



Insulin



Peptide YY3-36



Glucagon-like peptide 1



Other glucagon-related peptides



Leptin

+ when leptin ↓↓

Ghrelin

+

Tumor necrosis factor-α



Obestatin



TABLE 220-4  CENTRAL NERVOUS SYSTEM MODULATORS OF ENERGY BALANCE CENTRAL ANABOLIC (↑ INTAKE)

CENTRAL CATABOLIC (↓ INTAKE)

Neuropeptide Y

α-Melanocyte-stimulating hormone

Agouti-related protein

Corticotropin-releasing hormone

Melanin-concentrating hormone

Thyrotropin-releasing hormone

Hypocretins and orexins

Cocaine- and amphetamine-regulated transcript

Galanin

Interleukin-1β

Norepinephrine

Urocortin

Endogenous endocannabinoids (anandamide and 2-arachidonoylglycerol)

Oxytocin Neurotensin Serotonin

Peripheral satiety signals act to inhibit further food intake at some point during meal consumption. Some of the signals reach the brain through the vagus nerve and some through the systemic circulation. Examples of the proposed factors modulating appetite are listed in Table 220-3. The compounds range from gut-derived (ghrelin, cholecystokinin, glucagon-like peptide 1) and pancreas-derived (insulin) hormones to peptides such as apolipoprotein A-IV, which is secreted with chylomicrons. The signals are thought to be triggered both by mechanical stimuli (e.g., the fullness of the stomach) and by the presence of nutrients in the jejunum and ileum. The central nervous system regulation of food intake is becoming better understood. A number of neuropeptides, lipid derivatives, and monoamines have either anabolic (increased food intake with or without decreased energy expenditure) or catabolic (decreased food intake with or without increased energy expenditure) properties. A list of these molecules is provided in Table 220-4. Many of these compounds serve more than one function, such as regulation of hormone secretion (thyrotropin-releasing hormone and corticotropin-releasing hormone), wakefulness (norepinephrine), and behavior-reinforcing systems (endocannabinoids).

Energy Expenditure

There is a wide range of daily energy expenditure in adults, from less than 1400 kcal/day to more than 5000 kcal/day, with larger, more physically active individuals having the greatest energy needs. Typically, daily energy expenditure is divided into resting (or basal) metabolic rate, the thermic effect of food, and physical activity energy expenditure.

Basal Metabolic Rate

The basal metabolic rate (BMR) is the energy expenditure of lying still at rest, awake, in the overnight postabsorptive state. The resting metabolic rate (RMR) is similarly defined but is not necessarily measured before arising from bed. For most sedentary adult Americans, the RMR represents the

CHAPTER 220  Obesity  

major portion of energy expended during the day and may range from less than 1200 to more than 3000 kcal/day. Most (~80%) of the BMR can be explained by the amount of lean tissue an individual has. There are a number of formulas that can be used to estimate BMR. The Harris-Benedict formula (available through numerous online calculators) predicts BMR on the basis of height, weight, age, and sex and is accurate to within 10% in approximately 90% of adults with BMIs of 18.5 to 45 kg/m2. Not all components of lean tissue consume oxygen at the same relative rates. Visceral or splanchnic bed tissues account for about 25% of RMR but a much smaller proportion of body weight. The brain, which is only a small percentage of body weight, accounts for almost 15% of RMR. Likewise, the heart (~7%) and kidneys (~5 to 10%) account for greater portions of resting energy needs than their relative contribution to body mass. In contrast, resting muscle makes up 40 to 50% of lean tissue mass but accounts for only 25% of RMR. This contribution changes dramatically with exercise, however, at which time muscle can account for 80 to 90% of energy expenditure. Adipose tissue is a minor contributor to daily energy expenditure, consuming only approximately 3 kcal/kg of body fat per day. Brown fat is adipose tissue that expresses large amounts of uncoupling protein-1, a protein that allows a mitochondrial membrane proton leak, resulting in heat release as opposed to chemical work from adenosine triphosphate—“uncoupling” of substrate oxidation from chemical or mechanical work. This thermogenic tissue was thought to be present only in human infants but has recently been shown to exist in adults.4 Methods used to detect brown fat largely rely on 18F-fluorodeoxyglucose positron emission tomography scanning of humans exposed to cold. Lean adults are more likely than obese adults to have brown fat, and brown fat is more readily detectable after obese adults lose weight. Whether brown fat plays any meaningful role in thermogenesis is currently a matter of debate. Although most of the RMR can be accounted for by the mass of lean tissue, there are also other, more subtle influences on RMR. Age, sex (women have slightly lower BMR even corrected for fat-free mass), and fat mass affect RMR. Small changes in BMR occur during the menstrual cycle (luteal phase > follicular phase). There is also evidence that heritable or family factors influence BMR, accounting for as much as 10% of the interindividual differences. There are both obligatory and facultative components to RMR. With an energy-restricted diet, significant reductions in BMR relative to the amount of fat-free mass occur. Reductions in the production of triiodothyronine from thyroxine and the sympathetic nervous system drive are thought to contribute to this phenomenon. Likewise, during brief periods of overfeeding, RMR increases slightly above that which would be expected for the amount of lean tissue present. It has been proposed that individuals with BMRs lower than predicted are at increased risk of future weight gain. Published data suggest that the relative risk is small, and clinical effort to identify such patients is not warranted. Measurement of BMR is sometimes helpful in the evaluation of patients who insist that they are unable to lose weight while following diets containing less than 1000 kcal/day. Almost without fail, if BMR is measured with a reliable instrument, it is substantially greater than the reported food intake. This underscores the fact that most adults are unreliable in assessing their own food intake.

The Thermic Effect of Food

An average of 10% of the energy content of food is expended in the process of digestion, absorption, and metabolism of nutrients. There is a significant interindividual variability in this value, however, ranging from a low of about 3% to a high of about 15% of meal calories that are “wasted” in the postprandial interval. The thermic effect of a meal is related to its carbohydrate and protein calorie content; the fat content has little stimulatory effect. Both obligatory (60 to 70%) and facultative (30 to 40%) components of the thermic effect of food have been identified. The obligatory components no doubt reflect the energy costs of digestion, absorption, and storage of nutrients. The two factors thought to play a role in the facultative component of the thermic effect of food are the postprandial insulin response and activation of the sympathetic nervous system. The thermic effect of food is somewhat lower in insulin-resistant and obese humans, but this has not been linked to future obesity.

Physical Activity Energy Expenditure

The energy expenditure of physical activity is a product of the amount of work done and the work efficiency of the individual. Tracking the total

1461

amount of physical activity that humans perform throughout the day is becoming easier with a variety of relatively inexpensive devices. By doing so, it is also possible to calculate the energy expended with published values for estimating the energy costs of work performed. Work units are expressed as metabolic equivalents (METs), a multiple of the RMR. If an individual’s RMR is 1 kcal/minute (1440 kcal/day), a workload of 5 METs would be 5 kcal/minute. Although most sedentary individuals can work for only a limited amount of time at relatively low workloads, highly trained athletes can work at extremely high METs (>16) for extended periods. This is because athletes have both a greater peak work capacity (or maximal amount of calories or oxygen that can be consumed) and a higher lactate threshold. The lactate threshold is closely related to the level at which exercise begins to become so uncomfortable that it cannot be maintained much longer. The biochemical definition of lactate threshold describes the progressive rise in blood lactate concentrations observed during sufficiently high-intensity exercise. The lactate threshold may range from 50 to 90% of an individual’s peak work capacity. Training raises the lactate threshold closer to the maximal workload and thus allows individuals to work at higher rates for longer periods. Thus, highly fit individuals can expend much greater amounts of energy per minute of exercise with less sense of discomfort than can obese, sedentary individuals who typically have low aerobic fitness and low lactate thresholds (sometimes on the order of 4 to 5 METs). The lactate threshold can be even lower in obese patients with type 2 diabetes, such that walking a mere 3 miles per hour can exceed their lactate threshold. Appreciation of the physical limitations of patients, which can usually be overcome with a carefully designed training program, is necessary to provide realistic activity recommendations. Exercise (fitness- and sports-related activities) is commonly considered the main component of physical activity thermogenesis. Because most adults do not exercise at high levels or for a sufficient duration to expend a large amount of energy, focusing solely on “exercise” as the main component of physical activity will miss significant opportunities for improving energy balance. The benefits of and energy expended in nonexercise activity can be far greater than with exercise, given the limited amount of time and effort that most patients can commit to exercise. Nonexercise activity thermogenesis (NEAT) is the calorie expense of performing all activities other than exercise- or employment-related and spontaneous activity. The range of observed NEAT under controlled (metabolic chamber) conditions has been less than 100 to more than 800 kcal/day. The energy expended from a physically demanding job or volitional exercise may or may not be offset by reductions in spontaneous (nonemployment) activity. For example, young adult men and women respond differently to 1 year of extra exercise; men lose weight and women do not, despite the absence of detectable change in food intake. Women must either reduce spontaneous activity in response to exercise or have subtle increases in food intake. It has also been shown that the variations in unconscious increases in NEAT relate strongly to the amount of fat gained in response to overeating. Low levels of NEAT have been reported to predict future weight gain in some populations, and there may be differences between lean and obese persons in the daily amount of NEAT, which could relate to differential tendencies to regulate weight.

Secondary Causes of Obesity Medications

A number of medications cause weight gain in some or most of the patients for whom they are prescribed. Awareness of the medications that have this potential can facilitate weight loss treatment in some patients. Table 220-5 lists a number of medications that are associated with weight gain as well as alternative treatment approaches, if any, for the underlying condition.

Diseases

Less than 1% of obese patients have an underlying disease that can explain the development of their obesity. Endocrinopathies are the most common secondary cause of obesity. These include Cushing’s syndrome (Chapter 227), hypothalamic damage resulting in overeating (most commonly after pituitary surgery), insulinoma (Chapter 230), and hypothyroidism (Chapter 226). A Cushing syndrome–like fat distribution is common; therefore, other physical or laboratory findings are the best clues to whether to test for this condition. These include the classic purple striae, thinning skin, easy bruising, proximal muscle weakness, and electrolyte abnormalities. Correction of Cushing’s syndrome commonly results in substantial loss of excess body fat. Insulinoma is a rare tumor, and only a small portion of patients with

1462

CHAPTER 220  Obesity  

TABLE 220-5  PHARMACOLOGIC INFLUENCES IN WEIGHT GAIN AND ALTERNATIVE THERAPIES DRUGS THAT MAY PROMOTE WEIGHT GAIN

ALTERNATIVE TREATMENTS: WEIGHT NEUTRAL OR WEIGHT LOSS

PSYCHIATRIC AND NEUROLOGIC MEDICATIONS Antipsychotics: olanzapine, clozapine, risperidone, quetiapine, aripiprazole Antidepressants   Tricyclics: imipramine, amitriptyline   Triazolopyridines: trazodone   Serotonin reuptake inhibitors: paroxetine, fluoxetine, citalopram Tetracyclics: mirtazapine Monamine oxidase inhibitors Antiepileptic drugs: gabapentin (higher doses), valproic acid, carbamazepine, divalproex Mood stabilizers: lithium, carbamazepine, lamotrigine, gabapentin (higher doses)

ALTERNATIVE PSYCHIATRIC AND NEUROLOGIC MEDICATIONS Ziprasidone Nortriptyline, bupropion, nefazodone, fluvoxamine, sertraline, duloxetine Topiramate, zonisamide (weight loss), lamotrigine (less weight gain)

STEROID HORMONES Progestational steroids Corticosteroids Hormonal contraceptives

ALTERNATIVES TO STEROID HORMONES Barrier methods, intrauterine device Nonsteroidal anti-inflammatory drugs

ANTIDIABETES AGENTS Insulin (most forms) Sulfonylureas Thiazolidinediones

ALTERNATIVE ANTIDIABETES AGENTS Metformin Acarbose, miglitol Exenatide Dipeptidyl peptidase 4 inhibitors Liraglutide Sodium-glucose co-transporter 2 inhibitors

ANTIHISTAMINES Commonly reported with older agents; also oxatomide, loratadine, and azelastine

ALTERNATIVE TO ANTIHISTAMINES Decongestants, mast cell stabilizers, antagonists of endogenous mediators of inflammation

ANTIHYPERTENSIVE AGENTS α-Adrenergic and β-adrenergic receptor blockers Calcium channel blockers: nisoldipine

ALTERNATIVE ANTIHYPERTENSIVE AGENTS Angiotensin-converting enzyme inhibitors Calcium channel blockers: most other agents Angiotensin receptor blockers Diuretics

HIGHLY ACTIVE ANTIRETROVIRAL THERAPY

insulinoma develop obesity. The weight gain associated with hypothyroidism is largely due to fluid retention and resolves with thyroid hormone replacement. Unfortunately, successful treatment is not available for hyperphagia due to hypothalamic damage. Adult patients with growth hormone deficiency, most commonly after hypophysectomy, may lose excess body fat with growth hormone replacement therapy.

Psychosocial Aspects of Obesity

Sexual, physical, and emotional abuse, especially in women, can result in long-term adverse consequences, including obesity. The effects of the abuse tend to be most profound if it occurs in childhood and adolescence. These women may be severely obese, suffer from chronic depression, and experience a number of psychosomatic symptoms, particularly chronic gas­ trointestinal distress. Identifying these issues before initiation of weight loss programs is important because successful weight loss may actually aggravate the distress experienced by these women. In addition, appropriate referral for psychiatric help may be needed before initiation of treatment for obesity.

PATHOPHYSIOLOGY

Metabolic Complications of Obesity

A central or upper body fat distribution is more predictive than total fat mass of the metabolic complications of obesity. Adipose tissue release of free fatty acids (FFAs) and glycerol into the circulation through lipolysis provides 50 to 100% of daily energy needs. Adipose tissue lipolysis is regulated primarily by insulin (inhibition) and catecholamines (stimulation), although growth hormone, cortisol, and atrial natriuretic peptide also stimulate lipolysis. Upper body obesity is associated with several abnormalities of adipose tissue lipolysis, most remarkably with higher postprandial FFA release and concentrations; this abnormality is particularly evident in type 2 diabetes mellitus. Abnormally high FFA concentrations can contribute to a number of the metabolic complications of obesity.

Insulin Resistance

The term insulin resistance is typically used in referring to the ability of insulin to promote glucose uptake and to inhibit the release of glucose into the circulation. The primary site of insulin-stimulated glucose uptake, oxidation,

and storage is skeletal muscle. The principal site of glucose production is the liver. Insulin resistance initially leads to hyperinsulinemia and may eventually lead to the development of type 2 diabetes mellitus (Chapter 229). The ability of insulin to promote glucose uptake, oxidation, and storage in muscle and to suppress plasma FFA concentrations is reduced in upper body obesity. High plasma FFA concentrations can induce a state of insulin resistance both in the muscle (glucose uptake) and in the liver (glucose release), independent of obesity. Thus, abnormal regulation of adipose tissue FFA export is a significant component of the development of insulin resistance. It is hypothesized that excess FFAs induce muscle insulin resistance by promoting increased synthesis of diacylglycerols and ceramides, both of which can interfere with the normal insulin signaling pathway. Dysregulated production of a number of adipose-derived hormones, also called adipokines, is hypothesized to contribute to insulin resistance and the metabolic complications of obesity. Adiponectin, an adipocyte-derived hormone that improves insulin action, is secreted at reduced rates in obesity and diabetes. Increased production of resistin, interleukin-6, tumor necrosis factor, and retinol-binding protein-4 by adipose tissue has been linked to insulin resistance in animal models. We currently lack the experimental evidence from human studies to know what role adipokines play in the metabolic complications of obesity.

Islet Cell Failure and Type 2 Diabetes Mellitus

Type 2 diabetes usually results from defects in both insulin secretion and insulin action (Chapter 229). Many obese individuals are insulin resistant, yet only a subset will develop diabetes mellitus. It follows that those who develop type 2 diabetes develop pancreatic β-cell decompensation with subsequent hyperglycemia. Animal (rodent) studies have suggested that a process referred to as lipotoxicity is involved in pancreatic β-cell failure. In this model, increased FFAs are proposed to contribute to the insulin secretory abnormalities seen in obesity and ultimately to lead to β-cell failure. There is some evidence that elevated FFAs have adverse effects on islet β-cell function in humans. Another potential contributor to β-cell failure in obesity is the overproduction of islet amyloid polypeptide. This protein is co-secreted with insulin and, because of its tertiary structure, can form toxic amyloid deposits in β cells. Amyloid deposits have been found in the pancreatic islets obtained at autopsy from patients with type 2 diabetes mellitus.

CHAPTER 220  Obesity  

1463

Blood pressure can be increased by a number of mechanisms (Chapter 67). Increased circulating blood volume, abnormal vasoconstriction, decreased vascular relaxation, and increased cardiac output may all contribute to hypertension in obesity. The effect of hyperinsulinemia to increase renal sodium absorption may contribute to hypertension through increased circulating blood volume. Abnormalities of vascular resistance also contribute to the pathophysiologic process of obesity-related hypertension. Under some experimental conditions, elevated FFAs have been found to cause increased vasoconstriction and reduced nitric oxide–mediated vasorelaxation, similar to that seen in the metabolic syndrome. Some obese adults have increased sympathetic nervous system activity, which could contribute to obesityassociated hypertension. Finally, angiotensinogen (also produced by adipocytes) is a precursor of the vasoconstrictor angiotensin II and is proposed to contribute to elevated blood pressure.

Sleep apnea is most likely explained by enlargement of upper airway soft tissue, resulting in collapse of the upper airways with inspiration during sleep. The obstruction leads to apneas, with hypoxemia, hypercarbia, and high catecholamine and endothelin levels. The frequent arousals to restore breathing result in poor sleep quality. Sleep apnea is associated with an increased risk of hypertension, and if sleep apnea is severe, it can lead to right-sided heart failure and sudden death. A history of daytime hypersomnolence, loud snoring, restless sleep, or morning headaches is suggestive of obstructive sleep apnea. Treatment of sleep apnea is important to improve cardiovascular risk, and the failure to recognize and to treat this complication may make weight loss intervention strategies much less successful. Epidemiologic studies have linked short sleep duration and disruptions of circadian rhythm with increased risk of metabolic syndrome and diabetes. Experimentally induced sleep restriction combined with circadian disruption in humans led to decreased RMR and increased postprandial plasma glucose levels due to inadequate insulin secretion.

Dyslipidemia

Cancer

Hypertension

Upper body obesity and type 2 diabetes mellitus are associated with increased triglycerides, decreased high-density lipoprotein (HDL) cholesterol, and a high proportion of small, dense low-density lipoprotein (LDL) particles (Chapter 206). This dyslipidemia contributes to the increased cardiovascular risk observed in the metabolic syndrome. Fasting hypertriglyceridemia is caused by increased hepatic very low density lipoprotein (VLDL) secretion, which may be driven by increased delivery of FFAs to the liver coming from both visceral fat and upper body subcutaneous fat. The reduced HDL cholesterol concentrations and the increased small, dense LDL particle concentrations associated with upper body obesity are likely an indirect consequence of elevated triglyceride-rich VLDL. Increased cholesterol ester transfer protein activity and hepatic lipase activity can theoretically account for the atherogenic shifts in triglycerides and cholesterol between lipoproteins. Genetic influences play a significant role in the expression of these lipid abnormalities. Polymorphisms in the genes for apolipoprotein E, lipoprotein lipase, apolipoprotein B-100, and apolipoprotein A-II are correlated with increased triglycerides and decreased HDL.

Endocrine Manifestations of Obesity

Obesity is associated with abnormalities of the endocrine system, one of the most common being polycystic ovary syndrome. This syndrome (Chapter 236) is characterized by mild hirsutism and irregular menses or amenorrhea with anovulatory cycles. It is most commonly linked with obesity and often improves with weight loss and other treatments that improve insulin resistance. The insulin resistance associated with obesity may trigger the development of polycystic ovary syndrome in susceptible individuals. Whereas mild to moderate androgen overproduction is a feature of upper body obesity in women, obese men may suffer from mild to severe hypothalamic hypogonadism. This androgen deficiency improves with weight loss, and attempts to treat this condition with testosterone replacement offer little clinical benefit. There has been some concern that testosterone treatment of obese men may increase the risk of obstructive sleep apnea and perhaps even cardiovascular events. Although estrogens are not elevated in obese premenopausal women, they remain somewhat above postmenopausal levels in obese postmenopausal women. Serum growth hormone concentrations are commonly low in obese adults, but insulin-like growth factor-I concentrations are often normal, and growth hormone concentrations increase with weight loss. Treatment of these patients with growth hormone has been reported to worsen insulin resistance and glucose intolerance and cannot be justified, considering the costs and poor risk-to-benefit ratio.

Mechanical Complications of Obesity

The excess body weight associated with obesity is thought to be responsible for the increased prevalence of lower extremity degenerative joint disease. Extreme obesity can result in premature degenerative joint disease, and this may be especially difficult to treat surgically, given the greater stress on joint replacements. Severely obese individuals may also have problems with venous stasis, which is occasionally aggravated by right-sided heart failure (see later).

Obstructive Sleep Apnea and Sleep Restriction

Sleep apnea (Chapter 100) is common in severely obese patients, tending to be more prevalent in men and in women with an upper body/visceral obesity.

The risk of breast cancer and endometrial cancer is increased in obese women (Chapter 180). It is thought that this may be due to the increased estrogen levels associated with obesity in the postmenopausal woman. Obese men also have a higher mortality of cancers of the prostate and colon. The reasons for this association are unknown.

Gastrointestinal Disorders

Gastroesophageal reflux disease and gallstones are more prevalent in obese patients. Likewise, fatty liver and nonalcoholic steatohepatitis (Chapter 152) are more common in obesity. Nonalcoholic steatohepatitis can eventually progress to life-threatening hepatic cirrhosis. Weight loss and interventions that improve insulin sensitivity have been shown to improve fatty liver and nonalcoholic steatohepatitis.

DIAGNOSIS

Evaluation of Obesity

In the office practice, obtaining height and weight allows calculation of BMI. For patients with a BMI above 25 and below 35, a second piece of information—the waist circumference—provides an added indicator as to whether the patient is at greater risk for adverse consequences (see earlier). Measurement of blood pressure (which may require a large blood pressure cuff) then provides a third item of health information at almost no cost. The presence or absence of dyslipidemia (HDL cholesterol < 45 mg/dL for women, HDL cholesterol < 35 mg/dL for men, or triglycerides > 150 mg/ dL), hypertension, glucose intolerance and diabetes, and hyperuricemia should be documented. A history suggestive of sleep apnea should prompt a referral for overnight oximetry or a sleep disorder evaluation. A review of the patient’s lifestyle, including an assessment of physical activity level and eating habits, may help provide information about why the patient is obese. A family history of obesity, or long-standing obesity, provides evidence against a secondary cause of obesity. A careful medication history and social history may help the clinician identify precipitating factors that can be modified. By emphasizing the role of modifiable lifestyle factors that predispose to disease risk, as opposed to focusing solely on the patient’s weight, it may be possible to initiate a conversation about weight/disease management in a less threatening manner from the patient’s perspective. Before a patient enters a weight management program, it is helpful to ensure that the patient is interested and ready to make lifestyle changes and has realistic goals and expectations. Patients who expect to lose large amounts of weight in a short time are virtually doomed to disappointment. Medical treatment programs, even if they include pharmacotherapy, struggle to routinely achieve sustained weight loss of more than 10%. Although this amount of weight loss is sufficient to markedly reduce the medical compli­ cations of obesity, disappointment with “only” 10% weight loss may cause patients to abandon a medically successful program. Helping the patient to understand that lifestyle changes resulting in achievable (10%) weight loss is a reasonable, initial goal can be one of the more challenging aspects for a physician. It is sometimes necessary to delay entry into treatment programs if a patient is not ready to make lifestyle changes. In this case, a reasonable strategy is to remind the patient periodically of the potential health benefits of improved activity and eating habits. Once a willingness to make changes is apparent, treatment is more likely to succeed.

1464

CHAPTER 220  Obesity  

TREATMENT  Obesity represents an individual’s response to the environment based on genetics and learned behavior and is best viewed as a chronic disease. Therefore, treatment must be considered a long-term issue, much like diabetes, hypertension, or dyslipidemia. Substantial weight loss can be induced through severe calorie restriction, but without approaches to ensure behavioral changes, body fat is invariably regained. To the extent that environmental factors contribute to a patient’s overweight status, and to the extent that the macroenvironment is unlikely to change, patients must learn how to make permanent lifestyle changes (eating and activity behavior) to hope for permanent weight loss. Behavior modification approaches,5 which can help patients recognize and circumvent environmental cues for sedentary behavior and overeating, can increase the likelihood that patients will accomplish these lifestyle changes. A1 A randomized study has shown that intensive lifestyle intervention (as compared to only support and education) is associated with fewer hospitalizations, fewer medications, and lower health care costs in overweight or obese adults with type 2 diabetes. A2  Reducing energy intake is the most efficient and effective means to lose weight. For example, creating a 500 kcal/day deficit by reduced food intake will theoretically result in the loss of 1 pound of fat per week. Although possible, it is much more difficult to increase energy expenditure by 500 kcal/ week through exercise. Higher levels of physical activity can prevent weight

Patient encounter

Measure weight, height; calculate BMI

gain (or weight regain after weight loss). Some patients are able to change eating and activity habits on their own, given the proper information, whereas others require formal or informal behavior modification interventions (see later) to help make these changes. In some instances, pharmacotherapy or surgery may be needed for treatment of obesity. A flow diagram on how to evaluate and to manage patients with overweight and obesity is presented in Figure 220-1.

Diet

Changes in eating habits must be permanent if weight loss is to be maintained. An experienced registered dietitian can be helpful in the evaluation of a patient’s eating habits and will be able to provide the needed education. The diet history may identify eating behaviors that result in excess energy intake. Although it is important to address specific adverse eating behaviors, patients need to understand some general principles regarding diet. Reducing the energy density of food (most commonly accomplished by reducing dietary fat) can allow patients to feel satiated while consuming fewer calories. The NHLBI/ AHA/ACC/TOS obesity guideline6 recommends that providers prescribe 1200 to 1500 kcal/day for women and 1500 to 1800 kcal/day for men. Alternatively, diets that produce an energy deficit of 500 to 750 kcal/day can be recommended. Because there appears to be no clear superiority of one diet over another with regard to weight loss, A3  it is recommended that providers prescribe one of the evidence-based diets that restricts selected food types (e.g.,

BMI 25- 20 µg/dL.

ACTH stimulation test: ACTH1-24 (cosyntropin), 0.25 mg IM or IV. Cortisol is measured at 0, 30, and 60 min.

Normal response is GH > 4.1 µg/L. Normal response is GH > 3 µg/L.

In most normal individuals, the basal ACTH increases two- to four-fold and reaches a peak (20-100 pg/mL). ACTH responses may be delayed in cases of hypothalamic dysfunction. Cortisol levels usually reach 20-25 µg/dL. A normal response is cortisol > 18 µg/dL. In suspected hypothalamic-pituitary deficiency, a low-dose (1-µg) test may be more sensitive.

Thyroid-stimulating hormone (TSH)

Basal thyroid function tests: free T4, free T3, TSH

Low free thyroid hormone levels in the setting of TSH levels that are not appropriately increased

Luteinizing hormone (LH), follicle-stimulating hormone (FSH)

Basal levels of LH, FSH, testosterone, estrogen

Basal LH and FSH should be increased in postmenopausal women. Low testosterone levels in conjunction with low or low-normal LH and FSH are consistent with gonadotropin deficiency.

*Values are with polyclonal assays. CRH = corticotropin-releasing hormone; GHRH = growth hormone–releasing hormone; IM = intramuscularly; IV = intravenously; T3 = triiodothyronine; T4 = thyroxine.

TABLE 224-3  HORMONAL REPLACEMENT THERAPY IN HYPOPITUITARISM* PITUITARY AXIS

HORMONAL REPLACEMENTS

Growth hormone (GH)

In children, GH (0.25 mg/kg) SC daily. In adults, GH (0.3-1.2 mg) SC daily. Titrate dose to achieve IGF-I levels in middle to upper part of normal range. Women receiving oral estrogens require higher doses.

Prolactin

None

Adrenocorticotropic hormone–cortisol

Hydrocortisone (10-15 mg PO q am; 5-10 mg PO q pm) or prednisone (2.5 mg PO q am; 2.5 mg PO q pm). Dose adjusted on clinical basis. Stress dosing: 50-75 mg hydrocortisone IV q8h

Thyroid-stimulating hormone–thyroid

l-thyroxine (0.075-0.15 mg) PO daily

Gonadotropins–gonads

FSH and LH (or HCG) can be used to induce ovulation in women. HCG alone or with FSH can be used to induce spermatogenesis in men. In men, testosterone enanthate (100-300 mg) IM q1-3wk or testosterone cyclopentylpropionate (100-300 mg) IM q1-3wk. Testosterone transdermal patches can also be used (5 mg daily). Testosterone gel 5-10 g daily. In women, conjugated estrogens (0.625-1.25 mg) PO days 1-25 each month, cycled with medroxyprogesterone acetate (5-10 mg) PO days 15-25 each month. Low-dose contraceptive pills may also be used. Estrogen-containing transdermal patches are also available.

Posterior pituitary

Desmopressin, 0.05-0.2 mL (5-20 µg) intranasally once or twice daily, or tablets (0.1-0.4 mg q8-12h) or 0.5 mL (2 µg) SC

*Replacement therapy is dictated by the types of hormone deficiencies and by the clinical circumstances. In each case, the recommended preparations and doses are representative but need to be adjusted for individual patients. Other hormonal preparations are also available. FSH = follicle-stimulating hormone; GnRH = gonadotropin-releasing hormone; HCG = human chorionic gonadotropin; IGF-I = insulin-like growth factor-I; IM = intramuscularly; LH = luteinizing hormone; PO, orally; SC = subcutaneously.

  PITUITARY TUMORS PATHOBIOLOGY

Pituitary tumors are classified according to the hormones they produce and their size: microadenomas, less than 10 mm in diameter; macroadenomas, more than 10 mm in diameter; and macroadenomas with extrasellar extension. In general, the levels of hormones produced by the tumors parallel the size of the tumors. The prevalence of the different types of pituitary adenomas, based on surgical data, is summarized in Table 224-4. Immunohistochemical studies using antibodies specific for each of the major pituitary hormones have been used to define tumor phenotype. Pituitary adenomas are rarely malignant but can be locally invasive. Most pituitary tumors are monoclonal. This finding does not exclude a role for hormonal stimulation as a predisposing factor for somatic mutations, and the hormonal environment may also affect the rate of tumor growth (e.g., the growth of ACTH-secreting tumors following bilateral adrenalectomy). Supporting the concept that somatic mutations lead to pituitary tumorigenesis, a subset (35 to 40%) of somatotroph adenomas have activating mutations in the gene for the Gsα-subunit, resulting in two different amino acid (Arg201 and Glu227) substitutions.7 Either mutation causes the Gsα-subunit to

stimulate adenylyl cyclase in a constitutive manner. The elevated intracellular cyclic adenosine monophosphate levels lead to increased cell growth as well as GH production. Mutations in other oncogenes, such as ras, Rb, and p53, are uncommon in pituitary tumors. Thus, the nature of the somatic mutations causing most pituitary tumors remains unknown. At least five types of inherited predispositions to pituitary tumors are recognized. Patients with McCune-Albright syndrome (Chapter 231) occasionally develop pituitary adenomas as well as characteristic abnormalities in other tissues, particularly the ovary, bone, and thyroid. Interestingly, the McCune-Albright syndrome is also caused by mutations in the gene for the Gsα-subunit. However, the somatic mutations in McCune-Albright occur early during development, so that multiple tissues are affected. In multiple endocrine neoplasia type 1 (MEN 1) (Chapter 231), the menin gene is mutated, so that the predisposition to pituitary tumors is inherited in an autosomal dominant manner and occurs in conjunction with tumors of the parathyroid and pancreas.8 Familial isolated pituitary adenoma (FIPA) syndrome is autosomal dominant and has low or variable penetrance. Germline mutations have been found in the gene for the aryl hydrocarbon receptor– interacting protein (AIP), which functions as a tumor suppressor. Such mutations have been found in about one third of FIPA families, most commonly

1484

CHAPTER 224  Anterior Pituitary  

TABLE 224-4  PREVALENCE OF DIFFERENT TYPES OF PITUITARY ADENOMAS TYPE OF PITUITARY ADENOMA

DISORDER

HORMONE PRODUCED

PREVALENCE (%)*

Somatotroph

Acromegaly and gigantism

Growth hormone

10-15

Lactotroph (prolactinoma)

Hypogonadism, galactorrhea

Prolactin

25-40

Corticotroph

Cushing’s disease

Adrenocorticotropic hormone

10-15

Gonadotroph

Mass effects, hypopituitarism

Follicle-stimulating hormone and luteinizing hormone

15-20

Thyrotroph

Hyperthyroidism

Thyroid-stimulating hormone

Nonfunctioning/ null cell

Mass effects, hypopituitarism

None

TREATMENT  Surgery

50% reduction in tumor size), but such size reduction is seen in only about two thirds of patients treated with bromocriptine. Improvements in visual field defects can be seen in about 90% of patients with defects when treated with cabergoline. Thus, it is reasonable to use cabergoline as first-line therapy even in patients with visual field defects, so long as visual acuity is not threatened by rapid progression or recent tumor hemorrhage. Many patients treated with cabergoline whose tumors shrink to the point of nonvisualization on MRI and whose PRL levels are normal can maintain normal PRL levels and not experience tumor reexpansion after therapy has been tapered off. In some cases, prolactinomas appear to be resistant to a dopamine agonist. In these cases, switching from bromocriptine to cabergoline may be successful. Larger-than-standard doses (>2 mg/week) of cabergoline may be effective in normalizing PRL levels. The very high doses of cabergoline used in patients with Parkinson’s disease have been associated with cardiac valvular abnormalities; such abnormalities have not been found with conventional doses of cabergoline used in patients with prolactinomas, but monitoring with echocardiography may be prudent in patients taking larger-than-standard doses. Alternatively, transsphenoidal surgery may be used. Although initial remission rates (80 to 90%) for transsphenoidal surgery of microprolactinomas are good, there is long-term recurrence in about 20% of patients. For macroprolactinomas, the initial remission rates with surgery are closer to 30%, with a similar recurrence rate. Radiation therapy, usually stereotactic, is reserved for patients with macroadenomas not responding to either medical or surgical treatment. Dopamine agonist therapy for infertility, or when there is a possibility of pregnancy, deserves special consideration. These medications can induce ovulation in 80 to 90% of patients with hyperprolactinemia. Although neither bromocriptine nor cabergoline has been associated with congenital malformations, they should be stopped once pregnancy has been achieved. A form of barrier contraception is usually recommended until two to three regular menstrual cycles have occurred. Subsequently, pregnancy can be confirmed if a menstrual period is missed, allowing discontinuation of medication with exposure of the fetus to the drug for only 3 to 5 weeks. At present, the safety data for pregnancy outcome are more limited for cabergoline; therefore, some clinicians prefer bromocriptine when fertility is desired. Less than 3% of patients with microadenomas, but 23% of patients with macroadenomas, develop symptoms of tumor enlargement (headaches, visual field defects) during pregnancy (Fig. 224-6). If symptoms develop, MRI and formal visual field testing should be performed. If there is evidence of visual field compromise or tumor growth, dopamine agonist therapy should be restarted to shrink the tumor. PRL levels are not very useful because they are normally increased in pregnancy and an enlarging tumor may not cause PRL production to increase substantially. Because problems of tumor growth occur most  often in patients with macroadenomas, consideration can also be given to the option of transsphenoidal decompression before pregnancy in women with large tumors, so long as fertility can be preserved. If the patient is far advanced in her gestation at the time tumor growth occurs, consideration could also be given to delivering the baby.

  ADRENOCORTICOTROPIC HORMONE

ACTH is a 39–amino acid peptide that is derived from a precursor polypeptide, proopiomelanocortin (POMC; 241 amino acids), which encodes several peptides, including ACTH and β-lipotropin (Chapter 223). The biologically active portion of ACTH resides within the first 18 of its 39 amino acids. However, because a synthetic peptide (cosyntropin) that includes the first 24 amino acids has a longer half-life, it is used clinically to assess adrenocortical function. In cases with neoplastic ectopic production of ACTH, the levels of precursor peptides or their processed products may be elevated.

CHAPTER 224  Anterior Pituitary  

A

B

C

D

1489

FIGURE 224-6.  Magnetic resonance image showing enlargement of prolactinoma during pregnancy. Above, Coronal (A) and sagittal (B) views show intrasellar prolactin-secreting macroadenoma prior to conception. Below, Coronal (C) and sagittal (D) views show enlargement of the prolactinoma at 7 months’ gestation. (From Molitch ME. Medical treatment of prolactinomas. Endocrinol Metab Clin North Am. 1999;28:143.)

The primary effect of ACTH is to stimulate the adrenal gland to produce cortisol. It also stimulates secretion of adrenal androgens and mineralocorticoids, although production of mineralocorticoids is controlled primarily through non–ACTH-dependent mechanisms (i.e., the renin-angiotensin system) (Chapter 227). Consequently, mineralocorticoid function is preserved in ACTH deficiency, in contrast to primary adrenal insufficiency, which is characterized by loss of glucocorticoid and mineralocorticoid function. Long-term stimulation by ACTH causes adrenal hyperplasia and enlargement. On the other hand, ACTH deficiency leads to adrenal atrophy. Hypothalamic corticotropin-releasing hormone (CRH) is the most important stimulator of ACTH secretion. Chronic stimulation by CRH causes corticotroph cell hyperplasia, which can be seen in cases of ectopic CRH production. Cortisol inhibits ACTH secretion, blunts the ACTH response to CRH, and inhibits CRH production. After prolonged glucocorticoid suppression of the hypothalamic-pituitary-adrenal axis, the amount of endogenous CRH secretion appears to be rate limiting and can require several months to recover. Plasma ACTH is secreted in discrete pulses (10 to 80 pg/mL), so random measurements are of little value. Most clinical tests are therefore based on levels of cortisol or its metabolites, which tend to integrate the effects of ACTH. ACTH and cortisol secretion exhibit marked diurnal rhythms, being greatest at night several hours after the initiation of sleep. Cortisol levels are highest in the early morning and reach a nadir in the late afternoon and evening. Patients with Cushing’s disease lose or exhibit a blunted diurnal rhythm of ACTH and cortisol secretion. ACTH secretion can be stimulated by a variety of different forms of stress, including psychological stimuli such as fright, anticipation of athletic competition, or surgery. Depression is associated with activation of the hypothalamic-pituitary-adrenal axis and impaired dexamethasone suppressibility. Hypoglycemia induces ACTH

secretion through a central mechanism. The resulting increase in cortisol secretion represents one of several counter-regulatory mechanisms that increase glucose production. Insulin-induced hypoglycemia provides a mechanism for testing the integrity of the hypothalamic-pituitary-adrenal axis (see Table 224-2). Serious trauma and infection activate an array of cytokines that stimulate CRH and ACTH secretion. Because cortisol levels are often increased substantially in these circumstances, similar adjustments in cortisol replacement doses may be required in seriously ill patients with adrenal insufficiency.

Adrenocorticotropic Hormone Deficiency: Secondary Hypocortisolism Secondary hypocortisolism causes symptoms of glucocorticoid deficiency, including nausea, vomiting, weakness, fatigue, fever, and hypotension. In addition to reduced levels of cortisol, abnormal laboratory test findings can include hyponatremia, hypoglycemia, and eosinophilia. Depending on its cause, the severity of cortisol deficiency in cases of secondary adrenal insufficiency is often not as marked as in primary adrenal insufficiency (Chapter 227). In addition, mineralocorticoid function is preserved in secondary adrenal deficiency. Consequently, the clinical manifestations of volume depletion are less pronounced, and hyperkalemia is not a feature of ACTH deficiency. Because ACTH levels are low, hyperpigmentation is not seen as in primary adrenal insufficiency. In women, reduced adrenal androgens can decrease libido and cause loss of axillary and pubic hair. The most common cause of ACTH deficiency is treatment with exogenous glucocorticoids, which causes suppression of the hypothalamic-pituitaryadrenal axis. Sudden withdrawal of glucocorticoids or an increased requirement induced by the superimposition of severe illness can elicit symptoms of glucocorticoid deficiency. Congenital forms of ACTH deficiency are rare. When present, ACTH deficiency usually occurs in combination with the loss

1490

CHAPTER 224  Anterior Pituitary  

of other pituitary hormones, although acquired, isolated ACTH deficiency does occur, particularly in women with lymphocytic hypophysitis. ACTH reserve is most often evaluated using the insulin tolerance test. Caution should be exercised before inducing hypoglycemia in patients with suspected adrenal insufficiency. Insulin-induced hypoglycemia stimulates central responses to neuroglycopenia (Chapter 230) and mimics some stresses that activate ACTH secretion. ACTH stimulation tests using ACTH1-24 (cosyntropin) can accurately evaluate primary adrenocortical insufficiency but may less accurately assess secondary adrenal insufficiency. A variation of the ACTH stimulation test using the low dose of 1 µg has been found to be useful for diagnosing secondary adrenal insufficiency in some studies. Deficiency of ACTH is treated by replacement with glucocorticoids. Doses need to be individualized and are based largely on clinical criteria in which symptoms of glucocorticoid deficiency are balanced against features of glucocorticoid excess. Typical amounts of hydrocortisone are in the range of 15 to 20 mg/day in divided doses. Such doses are usually doubled in the event of mild to moderate illness. Patients should wear MedicAlert tags and be instructed in the warning signs of cortisol deficiency: nausea, vomiting, abdominal pain, low-grade fever, fatigue, and postural dizziness. Emergency injection kits of hydrocortisone are frequently provided for home use in the event that vomiting precludes taking oral steroids, or for severe sudden stress (e.g., a fracture). Stress doses of steroids should be used during times of illness. Current recommendations call for doses in the range of 50 to 75 mg every 8 hours for severe stress. Mineralocorticoid replacement is not required in patients with ACTH deficiency.

Cushing’s Disease

PATHOBIOLOGY

Cushing’s disease results from a pituitary adenoma that causes excess production of ACTH. It should be distinguished from a variety of other causes of Cushing’s syndrome (glucocorticoid excess), which include adrenal causes (adenomas, carcinomas) of cortisol excess, ectopic production of ACTH and CRH, and physiologic states that result in overproduction of cortisol. Cushing’s disease accounts for 60 to 70% of cases of Cushing’s syndrome. Ten to 15% of pituitary tumors secrete ACTH. Cushing’s disease occurs about eight times more often in women than in men. Most ACTH-producing pituitary neoplasms, like other pituitary tumors, are monoclonal, implying a primary defect in corticotroph cells. In addition, there are rare cases of corticotroph hyperplasia causing Cushing’s syndrome that are secondary to CRH production by either adjacent CRH-producing intrasellar gangliocytomas or ectopic CRH-producing cancers. Most (80 to 90%) of the ACTH-secreting tumors are microadenomas at the time of diagnosis. The clinical features of cortisol excess may allow detection of corticotroph adenomas before they have grown to a larger size. High levels of cortisol may also restrain tumor growth. ACTH-secreting macroadenomas may be locally invasive.

CLINICAL MANIFESTATIONS

The clinical features of Cushing’s disease are caused by the effects of excess glucocorticoids and by the hypersecretion of ACTH and other POMC peptide products. The severity of the features of Cushing’s disease varies greatly and appears to reflect not only the level of free cortisol but also the duration of the disease and perhaps the sensitivity to glucocorticoid action. In florid cases of Cushing’s disease (Fig. 224-7), the constellation of symptoms and physical features is readily recognized. Early in the disease or in mild cases, it can be challenging to distinguish the clinical features of Cushing’s disease from similar traits that are seen in the normal population. Clinical suspicion is of paramount importance. On the other hand, one must be discriminating and not formally evaluate everyone with obesity, hypertension, and glucose intolerance. Of the many features listed in Table 224-6, some are relatively specific for Cushing’s disease. For example, the centripetal distribution of fat with the characteristic “buffalo hump,” “moon facies,” and deposition of fat in the supraclavicular area but not in the extremities is much more specific than generalized obesity. Striae that are wide (>1 cm) and purple reflect steroid-induced thinning of the dermis and can be distinguished from the more common “stretch marks.” Numerous spontaneous ecchymoses also occur because of thinning of the skin and capillary fragility. Proximal muscle weakness represents another manifestation of glucocorticoid excess. Osteopenia and hypokalemia, when present, provide objective evidence consistent with ACTH excess. Hypokalemia results from the effects of ACTH on mineralocorticoid production but also from the ability of high levels of cortisol to saturate 11β-dehydrogenase, an enzyme in the kidney that

A

B

FIGURE 224-7.  (A) This 30 year old woman initially presented with a three year history of increasing facial hair, facial rounding, abdominal obesity, hypertension, diabetes, and oligomenorrhea. She had no muscle weakness or pigmented striae. (B) Following successful transsphenoidal resection of her ACTH-secreting microadenoma, she had a dramatic improvement in her clinical appearance with resolution of her diabetes and hypertension.

TABLE 224-6  CLINICAL FEATURES OF CUSHING’S DISEASE GENERAL Obesity (centripetal distribution) “Moon facies” and mild proptosis Increased supraclavicular fat and “buffalo hump” Hypertension SKIN Hyperpigmentation Facial plethora Hirsutism Violaceous striae and thin skin Capillary fragility and easy bruising Acne Edema MUSCULOSKELETAL Muscle weakness (proximal) Osteoporosis and back pain REPRODUCTIVE Decreased libido Oligomenorrhea and amenorrhea NEUROPSYCHIATRIC Depression Irritability and emotional lability Psychosis METABOLIC Hypokalemia and alkalosis Hypercalciuria and renal stones Glucose intolerance or diabetes mellitus Impaired wound healing Impaired resistance to infection Granulocytosis and lymphopenia TUMOR MASS EFFECTS Headache Visual field loss Hypopituitarism

inactivates cortisol. As a result, cortisol can “spill over” and act on mineralocorticoid receptors in the distal tubule. The hyperpigmentation associated with Cushing’s disease is not as striking as that seen in Addison’s disease or in ectopic ACTH syndrome, but in association with other findings, it should raise the suspicion of Cushing’s disease and help distinguish it from adrenal causes of hypercortisolemia. Hirsutism and acne are caused by increased production of adrenal androgens and are more prominent in patients with Cushing’s disease than in those with adrenal adenomas, in whom glucocorticoids tend to be the predominant product. Oligomenorrhea and amenorrhea probably have several causes, including androgen effects on the reproductive axis and glucocorticoid inhibition of GnRH, which may also account for diminished libido. Hypertension and glucose intolerance are caused by glucocorticoid excess. Immunosuppression, venous thrombo-

1491

CHAPTER 224  Anterior Pituitary  

TABLE 224-7  TESTS USED IN THE DIFFERENTIAL DIAGNOSIS OF CUSHING’S SYNDROME* ETIOLOGY

OVERNIGHT DEXAMETHASONE SUPPRESSION TEST

PLASMA ACTH

CORTICOTROPIN-RELEASING HORMONE STIMULATION OF ACTH

PETROSAL-TO-PERIPHERAL ACTH RATIO

Normal

Suppression

Normal

Normal

Pituitary

No suppression

Normal or high

Normal or increased

>2

Ectopic

No suppression

High or normal

No response

200 pg/ mL) and can be associated with hyperpigmentation. Clinical features of Cushing’s syndrome may be altered by the rapid onset of extreme hyper­ cortisolemia coincident with elements of tumor cachexia. Pronounced weakness, fluid retention, glucose intolerance, hypokalemia, and poor skin integrity are often seen. Ectopic ACTH syndrome is readily recognized in its classic form. However, a subset of tumors, particularly carcinoids (Chapter 232), exhibit dexamethasone suppression that is similar to that seen with pituitary adenomas. When suspected, carcinoids can sometimes be detected by CT or MRI, but many are too small to be seen even with these techniques.

Because of these exceptions to the high-dose dexamethasone test, a variety of procedures have been devised in an attempt to further distinguish ectopic and pituitary dependent sources of ACTH. CRH testing may also prove useful, with pituitary tumors exhibiting an increase in ACTH and tumors making ACTH ectopically having little or no response. In recent years, inferior petrosal sinus sampling has been used to distinguish pituitary and ectopic sources of ACTH when the source of ACTH is not obvious based on the clinical circumstances, biochemical evaluation, and imaging studies. This test requires an experienced radiologist for safe and effective catheterization of the petrosal sinuses. Blood samples are taken simultaneously from the left and right petrosal sinuses and from the periphery before and after CRH stimulation. In the case of ACTH-producing pituitary adenomas, there is a gradient in ACTH levels between the central and peripheral blood specimens. When clinical and biochemical studies suggest the presence of a pituitary adenoma, pituitary imaging should be performed using CT or MRI. Most ACTH-secreting pituitary adenomas are small, and scans are normal in more than half of patients.

TREATMENT  The efficacy of transsphenoidal surgery for Cushing’s disease is greatly aided by making the correct diagnosis preoperatively. In experienced hands, surgical cures of ACTH-producing microadenomas occur in 75 to 90% of patients undergoing a first operation. As in other pituitary tumors, complete remissions with macroadenomas are much less common. In the event of surgical remission or cure, postoperative hypocortisolism is to be expected because of suppression of the hypothalamic-pituitary axis. After coverage for steroid withdrawal in the postoperative period, cortisol replacement should gradually be decreased to allow recovery of the hypothalamic-pituitary-adrenal axis; recovery may take up to 1 year. If transsphenoidal surgery is unsuccessful, reoperation may be indicated and can result in remission in up to 50% of patients18; in this circumstance, consideration should be given to performing a total hypophysectomy at reoperation. If transsphenoidal surgery cannot be performed or has failed, alternative forms of therapy should be used to prevent the long-term consequences of hypercortisolism.19 Pituitary irradiation is often the second line of treatment for Cushing’s disease. It is more efficacious in children and in younger patients, but even in older adults, remissions can be achieved in about 50% within 2 years. To prevent the continued ravages of hypercortisolism during this period, however, concomitant medical therapy (see later) is usually given. Bilateral adrenalectomy represents another alternative for patients with severe hypercortisolism after transsphenoidal surgery. It rapidly and effectively lowers cortisol levels but is associated with relatively high morbidity and mortality rates (as high as 5%) because of the associated metabolic and immune system alterations caused by hypercortisolism. The morbidity has been reduced in recent years by introduction of the laparoscopic approach. After adrenalectomy, patients must be maintained on glucocorticoids and mineralocorticoids and are at risk for the development of Nelson syndrome (see later). Medical therapy for Cushing’s disease has its primary role in preparation for surgery or control of hypercortisolism following unsuccessful surgery. It may also be used during the interval when radiation therapy is taking effect (Fig. 224-8). The antifungal agent ketoconazole is effective in decreasing glucocorticoid biosynthesis and also inhibits ACTH secretion, so it has been the most common medical therapy used; it has hepatotoxicity and is not actually approved for use for Cushing’s syndrome. Other drugs that interfere with cortisol synthesis (e.g., mitotane, etomidate, metyrapone) have been used less commonly. In a few small studies, cabergoline has also been shown to cause a normalization of cortisol levels in about one third of patients with Cushing’s disease, although it has not been approved for this indication. Recently,  two new drugs have been approved for the treatment of Cushing’s disease. Mifepristone is a progesterone receptor blocker that also is a glucocorticoid receptor blocker and is highly effective in improving clinical signs and symptoms of Cushing’s syndrome; because of its mechanism of action, with

1492

CHAPTER 224  Anterior Pituitary  

GONADOTROPINS (FOLLICLE-STIMULATING HORMONE AND LUTEINIZING HORMONE)

CRH Pituitary

Cabergoline Pasireotide

ACTH

Adrenals

Ketoconazole Mitotane Metyrapone Etomidate

Cortisol

Mifepristone GR Tissues

FIGURE 224-8.  Medical therapies for Cushing’s disease. Cabergoline and pasireotide act to decrease ACTH secretion from the corticotroph tumor. Several drugs (ketoconazole, metyrapone, mitotane, etomidate) work at the adrenal level to decrease cortisol synthesis. Mitotane works at the adrenal level by blocking cortisol action at the glucocorticoid receptor. ACTH = adrenocorticotropic hormone; CRH = corticotropin-releasing hormone; GR = glucocorticoid receptor. (Adapted from Petersenn S. Medical management of Cushing’s disease. In: Swearingen B, Biller BMK (eds) Endocrine updates vol. 31: Cushing’s disease. 2011, Springer, New York, pp 167–182, Figure 1. As adapted from Petersenn S, Endocrine Updates, 2011, Springer Science+Business Media B.V.)

The pituitary glycoprotein hormones include FSH, LH, and TSH. Chorionic gonadotropin, which is structurally very similar to LH, is made in the placenta. Each of the glycoprotein hormones has a specific β-subunit that forms a noncovalently bound dimer with the common α-subunit. The αand β-subunits each undergo glycosylation, which is important for correct hormone folding, intracellular transport, and secretion. Glycosylation is also required for biologic activity, presumably because of effects on the tertiary structure of the hormones. The gonadotropins are involved in sexual differentiation, sex steroid production, and gametogenesis. The regulation and physiologic roles of gonadotropins are quite different in males and females. In males, receptors for FSH are located on Sertoli cells and seminiferous tubules, whereas LH receptors are located on Leydig cells in the testis. LH stimulates androgen production by the Leydig cells. FSH is involved primarily in sperm maturation in the seminiferous tubules. Thus, FSH and LH act together to induce spermatogenesis (Chapter 234). In females, ovarian FSH receptors are located on granulosa cells, where they induce enzymes involved in estrogen biosynthesis. LH receptors are located predominantly on thecal cells in the ovary and stimulate the production of ovarian androgens and steroid precursors that are transported to granulosa cells for aromatization to estrogens. The pattern of FSH and LH secretion during the menstrual cycle results in follicular recruitment and maturation (largely FSH mediated), followed by ovulation (largely LH mediated) and steroid production by the corpus luteum (Chapter 235). Gonadotropin secretion is regulated primarily by the hypothalamic decapeptide GnRH. The gonadotroph cell is exquisitely sensitive to the pattern of GnRH stimulation. Continuous, rather than pulsatile, exposure to GnRH causes gonadotroph desensitization and suppression of LH and FSH. Gonadotroph sensitivity to GnRH is modulated by sex steroids and probably other hypothalamic peptides such as neuropeptide Y. Increased GnRH secretion, in combination with a higher density of GnRH receptors and rising estradiol concentrations, accounts in part for the dramatic release of gonadotropins that induces ovulation.

Hypogonadotropic Hypogonadism

CLINICAL MANIFESTATIONS AND DIAGNOSIS

treatment, cortisol and ACTH levels actually rise while the clinical signs improve. The higher levels of cortisol may “spill over” to the mineralocorticoid receptor, causing a blood pressure rise and hypokalemia. Symptoms of glucocorticoid withdrawal and even adrenal insufficiency may occur. Blockade of the progesterone receptor by mifepristone may cause menorrhagia. Pasireotide is a somatostatin receptor analog that has much greater activity on corticotroph adenomas than other somatostatin analogs because it has additional activity at the somatostatin-5 receptor. A large multicenter study has recently shown clinical and biochemical efficacy in patients with Cushing’s disease A3 ; the major adverse effect is hyperglycemia and a worsening of preexisting diabetes mellitus. This worsening of glucose levels is likely due to a reduction in insulin and glucagon-like peptide (GLP)-1 levels.

Nelson Syndrome Nelson syndrome was initially described as the appearance of a pituitary adenoma after bilateral adrenalectomy. In addition to an enlarging pituitary mass, the syndrome is characterized by very high ACTH levels and hyperpigmentation. It is caused by a preexisting ACTH-producing tumor that grows in the absence of feedback inhibition by high levels of glucocorticoids. The incidence of clinically significant Nelson syndrome after adrenalectomy for Cushing’s disease varies from 10 to 50% in different series.20 Patients with Cushing’s disease who have undergone adrenalectomy should be followed with imaging studies and plasma ACTH levels, because tumors that cause Nelson syndrome can be very aggressive. When there is evidence of mass effects or rapid growth, transsphenoidal surgery should be performed. Postoperative irradiation may provide additional benefit, although it appears to be less efficacious than in other ACTH-producing adenomas. In theory, Nelson syndrome could also occur in patients being treated with mifepristone, and such patients should also be monitored with periodic pituitary MRI scans, although short-term follow-up data are reassuring at this point.

Clinical features of hypogonadotropic hypogonadism in women are primarily due to estrogen deficiency and include breast atrophy, vaginal dryness, and diminished libido. Hot flashes are uncommon, in contrast to postmenopausal estrogen deficiency. In premenopausal women, normal menstrual cycles provide evidence for an intact hypothalamic-pituitary-gonadal axis. LH and FSH levels should be increased in postmenopausal women, and normal levels may indicate deficiency. Hypogonadism in men causes decreased libido and sexual function. In men, low testosterone without elevation of LH and FSH is consistent with impaired hypothalamic-pituitary reserve. GnRH stimulation can distinguish hypothalamic and pituitary deficiency but may require multiple injections to prime the pituitary. A congenital form of hypogonadotropic hypogonadism is caused by deficiency of GnRH, which in turn causes deficiencies of LH and FSH. When associated with anosmia (absent sense of smell), the condition is referred to as Kallmann’s syndrome (Chapter 223). Secondary hypogonadotropic hypogonadism is relatively common. In most cases, it is reversible and is caused by weight loss, anorexia nervosa, stress, heavy exercise, or severe illness. Reversible forms of secondary hypogonadotropic hypogonadism are caused by GnRH deficiency and are more common in women than men. A variety of pathologic conditions can cause secondary hypogonadotropic hypogonadism, often in association with deficiencies of other pituitary hormones (see Table 224-1). These include hypothalamic lesions and central nervous system irradiation. Pituitary tumors can suppress gonadotropins because of stalk compression and disruption of pulsatile GnRH input, as well as by direct destruction of normal pituitary tissue. Hyperprolactinemia can suppress GnRH and lead to reduced gonadotropin levels. In contrast to the aforementioned causes of hypogonadotropic hypogonadism that result from GnRH deficiency, primary deficiencies of LH and FSH are uncommon. An acquired form of isolated gonadotropin deficiency is rarely encountered and may have an autoimmune basis. Mutations in the LHβ or FSHβ genes have been described in case reports and cause selective loss of individual gonadotropins. Inactivating mutations in the GnRH

CHAPTER 224  Anterior Pituitary  

In women, the pattern of GnRH pulse frequency varies across the menstrual cycle (Chapter 235). The combination of GnRH stimulation in conjunction with ovarian feedback regulation results in a complex orchestration of positive and negative hormonal signals that converge at the gonadotroph to regulate LH and FSH secretion. The typical 28-day menstrual cycle is divided into follicular and luteal phases that are separated by ovulation on day 14. Unlike chronic exposure to low concentrations of estrogens, which exert negative feedback regulation and inhibit GnRH, the increasing concentration of estrogen before the LH surge exerts positive feedback regulation that results in increased GnRH pulse frequency. Increased GnRH in combination with increased gonadotroph sensitivity to GnRH results in the LH-FSH surge. During the luteal phase, the gonadotropin pulse frequency is reduced. In addition to feedback regulation by steroids, ovarian peptides such as inhibin also play a role in control of the reproductive axis. Inhibin causes selective suppression of FSH without affecting LH secretion. A homodimer of inhibin β-subunits, referred to as activin, has opposite actions and

1492.e1

selectively stimulates FSH, but its predominant physiologic action is to increase ovarian granulosa cell responsivity to FSH. Circulating inhibin provides one of the negative feedback inputs that leads to FSH suppression as the follicle develops. The perimenopause is characterized by a gradual cessation of ovarian function. After several years of menstrual cycles that are sometimes anovulatory or irregular, menses cease, thereby defining the menopause (Chapter 240). At menopause, the decline in estrogen and progesterone causes loss of feedback inhibition and a marked increase in LH and FSH levels. In males, the regulation of the hypothalamic-pituitary-gonadal axis is relatively constant. Testosterone inhibits the hypothalamic-pituitary axis, although its actions are mediated in part by aromatization to estrogens. Most of the inhibition by gonadal steroids occurs at the hypothalamic level. In contrast to menopause in women, there is no analogous abrupt change in hormone levels in men. There is, however, a gradual decline in testosterone levels associated with an increase in LH and FSH with aging.

CHAPTER 224  Anterior Pituitary  

receptor and the LH and FSH receptors causing hypogonadotropic hypogonadism have also been reported.

TREATMENT  In premenopausal women, preparations of estrogen and progestins should be used for hormonal replacement and to allow cyclical growth of the endometrium. Pulsatile GnRH (for GnRH-deficient patients) has been given to induce ovulation and fertility but is not commonly used at present. Gonadotropin injections are more commonly used when fertility is desired. Testosterone can be replaced in men, using intramuscular injections that are given at 2- to 4-week intervals. Doses and the intervals between injections should be adjusted on an individual basis using libido and testosterone levels before the next injection as a guide. Oral preparations of androgens should be avoided because of hepatotoxicity. Transdermal patch and gel preparations are also available and maintain more stable testosterone levels but are more expensive. With gels, care has to be used to prevent exposure of the partner or children. Although induction of spermatogenesis can be achieved using pulsatile GnRH (for GnRH-deficient patients), injections of gonadotropins are more commonly used. Pulsatile GnRH has been used to induce puberty and fertility in both males and females with Kallmann’s syndrome and other forms of GnRH deficiency, but more commonly, injections of gonadotropins are used. Secondary hypogonadotropic hypogonadism is ideally treated by correcting the underlying cause.21 Many women have a discrete threshold for weight or exercise level that will cause loss of menstrual periods. When it is not possible to correct the underlying abnormality, hormonal replacement can be used in women for protection against osteopenia and to cycle the endometrium. Permanent idiopathic hypogonadotropic hypogonadism can also occur in both sexes and will require hormone replacement.

Follicle-Stimulating Hormone– and Luteinizing Hormone–Producing Tumors

PATHOBIOLOGY

The majority (70 to 80%) of pituitary tumors classified previously as nonfunctioning adenomas can be shown to produce low levels of intact glycoprotein hormones or their uncombined α- or β-subunits. Biosynthetic defects in the tumor cells account for relatively inefficient hormone secretion as well as the propensity to produce uncombined subunits. FSH is produced more commonly than LH. Elevated levels of free α-subunits are noted more often than increased free β-subunits.

CLINICAL MANIFESTATIONS

Gonadotropin-producing tumors are somewhat more common in men than women and increase in prevalence with age. FSH- and LH-producing tumors do not usually cause a characteristic hormone excess syndrome. The tumors, typically large macroadenomas, present as clinically nonfunctioning tumors with symptoms and signs related to local mass effects. Visual field loss is found in more than 70% of patients. Many are detected incidentally by CT and MRI performed for unrelated indications. Symptoms of hypogonadism with loss of libido are also common. Men with predominantly FSH-secreting tumors may present with testicular enlargement from hypertrophy of the seminiferous tubules but may also be hypogonadal due to low levels of testosterone. These patients must be distinguished from those with primary hypogonadism due to testicular dysfunction. Tumors that primarily secrete LH are rare but can cause increased testosterone levels. Premenopausal women with gonadotropin-producing tumors may experience menstrual irregularity or secondary hypogonadism. Postmenopausal women often show reduced gonadotropin levels because the mass effects of the gonadotropin-producing tumors cause stalk compression, impairing GnRH stimulation of gonadotropins from normal pituitary cells.

DIAGNOSIS

Because of the absence of a clinical syndrome in most patients, almost all gonadotropin-producing pituitary tumors are diagnosed by postoperative immunohistochemistry, because they had presented with mass effects. There is no particular clinical benefit to distinguish whether a nonfunctioning adenoma is truly a gonadotroph adenoma. Some patients can have moderately elevated PRL levels that are caused by tumor mass effects. It is important to distinguish this group from patients with true prolactinomas. As noted earlier, many women, including those in the postmenopausal group, have

1493

paradoxically low gonadotropin levels. Thus, the absence of elevated gonadotropins does not exclude the diagnosis of a gonadotropin-producing tumor.

TREATMENT  Because the major symptoms of the gonadotropin-producing tumors are due to extrasellar extension and local mass effects, the main aim of treatment is reduction in tumor size. Complete or partial reversal of visual field defects and hypopituitarism can be accomplished by surgery unless these conditions have been of long standing. Transsphenoidal surgery is rarely curative, however, because of the large size of the tumors. Patients with significant residual tumor may benefit from radiation therapy. Because most tumors are slow growing, when no tumor is visible postoperatively by MRI, the patient may be followed with yearly monitoring for tumor recurrence, using visual fields and CT or MRI. If tumor markers such as free α- or β-subunit levels are available, they can also be used to monitor tumor function. When follow-up studies show tumor regrowth, repeat surgery, radiation therapy, or both are indicated. Medical therapy with dopamine agonists and somatostatin analogues has been successful in only a minority of patients.

  THYROID-STIMULATING HORMONE

Like the other glycoprotein hormones, TSH is a heterodimer composed of the common α-subunit and the unique TSH β-subunit. Normal levels of TSH range from 0.4 to 4.0 µU/mL. The detection limit for current TSH assays is less than 0.01 µU/mL, allowing measurement of suppressed TSH levels in patients with hyperthyroidism. TSH controls thyroid hormone (T4 and T3) synthesis and secretion from the thyroid gland. Hypothalamic TRH stimulates TSH synthesis and secretion. Somatostatin and dopamine can inhibit TSH secretion, but their roles in normal physiology have not been clearly elucidated. Thyroid hormones have an inhibitory effect on the production of TRH and TSH and constitute a powerful negative feedback loop acting at both the hypothalamic and pituitary levels. Secretion of TSH is pulsatile, but the amplitude of the pulses is relatively small and does not create the difficulties in measurement of TSH that are encountered with measurements of other pituitary hormones. Because of the integrated nature of the hypothalamic-pituitary-thyroid axis, thyroid function tests are best interpreted when concentrations of TSH, free T4, and free T3 levels are known. Except in conditions of secondary hypothyroidism or TSH-secreting pituitary tumors (see later), TSH levels provide an excellent screening test for thyroid dysfunction.

Central Hypothyroidism Central forms of hypothyroidism are due to loss of either TSH or TRH.22 Three different types of congenital TSH deficiency are caused by genetic mutations. One type involves mutations in the TSHβ gene. A second involves mutations in PIT1, which causes combined deficiencies of GH, PRL, and TSH (see earlier). A third involves a mutation in the gene for TRH. Acquired central forms of hypothyroidism are usually associated with other pituitary hormone deficiencies, and usually there is no goiter because of low TSH levels. Tests for TSH deficiency are best performed by analyzing free T4 levels in combination with TSH. Low free T4 without elevated TSH is consistent with central hypothyroidism. In some patients with hypothalamic disease, the TSH level is partially elevated in the presence of low levels of free T4, but the bioactivity of the TSH is reduced. Central forms of hypothyroidism must be distinguished from the sick-euthyroid condition (Chapter 226). Laboratory tests in the sick-euthyroid syndrome progress through several phases but can include prolonged periods when both TSH and free thyroid hormone levels are low. It can be very difficult in these patients to exclude central hypothyroidism unequivocally. In addition to the clinical setting in which thyroid function tests are measured, the presence of normal thyroid function tests before the illness and the absence of known hypothalamic or pituitary disease make true central hypothyroidism unlikely. Increased levels of reverse T3 are suggestive of sick-euthyroidism, and free T4 and T3 may be in the normal or low-normal range in sick-euthyroid patients. When TSH deficiency is documented, thyroid hormone is replaced using daily doses of l-thyroxine (0.05 to 0.15 mg/day). Because TSH cannot be used as an end point, one monitors the patient clinically and with serum levels of free T4 and T3.

Thyroid-Stimulating Hormone–Secreting Tumors TSH-secreting tumors are rare and account for between 1 and 3% of pituitary tumors. A recent analysis from Sweden showed that the prevalence was only

1494

CHAPTER 225  Posterior Pituitary  

23

2.8 per 1 million. As many as 45% of TSH-producing tumors are plurihormonal. GH and PRL are co-secreted most often, perhaps reflecting the common cellular lineage for thyrotrophs, somatotrophs, and lactotrophs. Long-standing severe hypothyroidism can cause thyrotroph hyperplasia and pituitary enlargement. These hyperplastic masses regress with thyroid hormone replacement therapy, however. Most true TSH-producing tumors are relatively autonomous and respond weakly, if at all, to TRH stimulation or thyroid hormone suppression. TSH-secreting tumors are usually macroadenomas by the time a diagnosis has been made. Consequently, many patients exhibit mass effects of the tumor, as well as hyperthyroidism. The clinical features of TSH-secreting tumors resemble those of Graves’ disease, except that features of autoimmunity (e.g., ophthalmopathy) are absent. Circulating levels of T4 and T3 range widely but can be elevated as much as two- to three-fold. Diffuse goiter is present in most patients with TSH-producing tumors, and the 24-hour uptake of radioiodine is elevated. Because feedback inhibition of TSH is impaired in TSH-producing tumors, TSH levels are inappropriately elevated in the presence of high levels of T4 and T3. TSH levels produced by tumors range from the low-normal range to as high as 500 µU/mL, but most levels are minimally elevated. Most TSHproducing tumors (>80%) secrete excess free α-subunit, and its assessment can be very useful in confirming the diagnosis. Thus, the diagnosis can usually be made by demonstrating that a hyperthyroid patient has a detectable serum TSH level associated with excess secretion of the free α-subunit. The finding of a mass lesion on CT or MRI confirms the diagnosis. Several other causes of inappropriate TSH secretion should be considered, including resistance to thyroid hormone and familial dysalbuminemic hyperthyroxinemia and other disorders that alter serum thyroid hormone binding proteins.

TREATMENT  The goals of therapy are to treat the underlying TSH-secreting tumor and to correct the hyperthyroidism. Transsphenoidal surgery alone is rarely curative because of the large size of most tumors, but it can alleviate mass effects and lower TSH levels. As in other large pituitary tumors, adjunctive irradiation may be required to control tumor growth. Somatostatin analogues have been used as adjunctive medical therapy, and they decrease TSH and α-subunit levels in about 80% of patients with TSH-secreting tumors, but consistent effects on tumor growth have not been demonstrated. Hyperthyroidism caused by TSH-secreting tumors can also be treated using antithyroid drugs or radioiodine.

  CLINICALLY NONFUNCTIONING PITUITARY TUMORS

Most clinically nonfunctioning adenomas can be shown to produce low levels of the free α-subunit, free β-subunits of FSH and LH, and intact FSH and LH when analyzed by immunocytochemistry or messenger RNA expression. A smaller fraction can be shown to produce low levels of other pituitary hormones, particularly ACTH or GH, that escaped detection based on routine endocrine testing.24 Even with detailed analyses of hormone production, a subset (10 to 20%) of nonfunctioning adenomas does not appear to produce any of the known pituitary hormones. The clinical features and management of nonfunctioning tumors are similar to those for gonadotropin-producing tumors. The major signs and symptoms result from tumor mass effects that cause visual field defects, headache and other neurologic symptoms, and hypopituitarism. Transsphenoidal surgery is the primary mode of treatment, with a goal of debulking the tumor to relieve mass effects. Because there are no serum tumor markers, patients must be followed by CT or MRI in conjunction with visual field tests. Some pituitary tumors are discovered as incidental findings on CT or MRI scans that were done for other reasons.25 Such tumors should be screened for hormone oversecretion with measurement of PRL, IGF-I, and a midnight salivary cortisol or an overnight dexamethasone suppression test, but most will be found to be nonfunctioning. If the tumor abuts the optic chiasm, a formal visual field examination should be performed. Over several years, about 10% of incidental microadenomas and 20% of macroadenomas enlarge. Indications for surgery include compression of the optic chiasm, with or without visual field defects and significant tumor enlargement. Hypopituitarism is also a relative indication for surgery (Fig. e224-1). In the absence of these indications for surgery, it is reasonable to follow such patients with MRI scans to look for size change at yearly intervals initially and then at less frequent intervals.

Grade A References A1. Barake M, Klibanski A, Tritos NA. Effects of recombinant human growth hormone therapy on bone mineral density in adults with growth hormone deficiency: a meta-analysis. J Clin Endocrinol Metab. 2014;99:852-860. A2. Gadelha MR, Bronstein MD, Brue T, et al. Pasireotide versus continued treatment with octreotide or lanreotide in patients with inadequately controlled acromegaly (PAOLA): a randomised, phase 3 trial. Lancet Diabetes Endocrinol. 2014;2:875-884. A3. Colao A, Petersenn S, Newell-Price J, et al. A 12-month phase 3 study of pasireotide in Cushing’s disease. N Engl J Med. 2012;366:914-924.

GENERAL REFERENCES For the General References and other additional features, please visit Expert Consult at https://expertconsult.inkling.com.

225  POSTERIOR PITUITARY JOSEPH G. VERBALIS

  ANATOMY AND HORMONE SYNTHESIS

The hormones of the posterior pituitary, vasopressin and oxytocin, are synthesized in specialized neurons in the hypothalamus, the neurohypophysial neurons. These neurons, notable for their large size, are termed magnocellular neurons. In the hypothalamus, the magnocellular neurons are clustered in the paired paraventricular and supraoptic nuclei (Fig. 225-1). Vasopressin and oxytocin are also synthesized in parvicellular (i.e., small cell) neurons of the paraventricular nuclei, and vasopressin (but not oxytocin) is also synthesized in the suprachiasmatic nucleus. Transcription of vasopressin and oxytocin messenger RNA and translation of the vasopressin and oxytocin prohormones occur entirely in the cell bodies of the neurohypophysial neurons. The prohormones provasopressin and prooxytocin are packaged along with processing enzymes into neurosecretory granules that are transported out of the perikaryon of the neurohypophysial neurons via microtubules and down the long axons that form the supraopticohypophysial tract, which terminates in the posterior pituitary. During transport, the processing enzymes cleave provasopressin into vasopressin (9 amino acids), vasopressin-neurophysin (95 amino acids), and vasopressin glycopeptide, or copeptin (39 amino acids). Pro-oxytocin is similarly cleaved to oxytocin (which differs from vasopressin by only two of nine amino acids) and oxytocin-neurophysin. The neurophysins form neurophysin-hormone complexes that stabilize the hormones. Stimulatory (e.g., glutamatergic, cholinergic, and angiotensin) neurotransmitter terminals and inhibitory (e.g., γ-aminobutyric acid and noradrenergic) neurotransmitter terminals control the release of vasopressin through the activity of synaptic contacts on the neurohypophyseal cell bodies. Physiologic release of vasopressin or oxytocin into the general circulation occurs at the level of the posterior pituitary, where, in response to an action potential, intracellular calcium is increased and causes the neurosecretory granules to fuse with the axon membrane, thereby releasing each hormone into the general circulation. Although each of the other prohormone fragments are released into the circulation, vasopressin and oxytocin are the only biologically active components of the prohormones. Factors that stimulate the release of neurohypophysial hormones also stimulate their synthesis. Because synthesis is delayed, maintenance of a large store of hormone in the posterior pituitary is essential to enable the instantaneous release of each hormone that is necessary following acute hemorrhage (vasopressin) or during parturition (oxytocin). In most species, sufficient vasopressin is stored in the posterior pituitary to support maximal antidiuresis for several days and to maintain baseline levels of antidiuresis for weeks.

Vasopressin Vasopressin and Regulation of Osmolality

The primary physiologic action of vasopressin is its function as a waterretaining hormone. The central sensing system (osmostat) for controlling the release of vasopressin is anatomically discrete, located in a small area of the hypothalamus just anterior to the third ventricle (see Fig. 225-1). The

CHAPTER 224  Anterior Pituitary  

1494.e1

Evaluation of pituitary function

Hyperfunctioning

Clinically nonfunctioning

Prolactinoma

Other

Dopamine agonist

Surgery

1 cm

Visual fields R/O pituitary hypofunction

Repeat MRI at 1, 2, 5 yrs

Repeat MRI at 0.5, 1, 2, 5 yrs

No change

Tumor growth Abnormal fields

No further studies (?)

Surgery

E-FIGURE 224-1.  Flow diagram indicating the approach to the patient found to have a pituitary incidentaloma. The first step is to evaluate patients for pituitary hyperfunction and then treat those found to be hyperfunctioning. Of patients with tumors that are clinically nonfunctioning, those with macroadenomas are evaluated further for evidence of chiasmal compression and hypopituitarism. Scans are then repeated at progressively longer intervals to assess for enlargement of the tumors. (Reproduced from Molitch ME. Nonsecreting tumors and pituitary incidentalomas. Endocrinol Metab Clin North Am. 2008;37:151-171.)

1494.e2

CHAPTER 224  Anterior Pituitary  

GENERAL REFERENCES 1. Cohen LE. Genetic disorders of the pituitary. Curr Opin Endocrinol Diabetes Obes. 2012;19:33-39. 2. Mete O, Asa SL. Clinicopathological correlations in pituitary adenomas. Brain Pathol. 2012;22: 443-453. 3. Hess CP, Dillon WP. Imaging the pituitary and parasellar region. Neurosurg Clin North Am. 2012;23:529-542. 4. Caturegli P, Iwama S. From Japan with love: another tessera in the hypophysitis mosaic. J Clin Endocrinol Metab. 2013;98:1865-1868. 5. Burman P, Mattsson AF, Johannsson G, et al. Deaths among adult patients with hypopituitarism: hypocortisolism during acute stress, and de novo malignant brain tumors contribute to an increased mortality. J Clin Endocrinol Metab. 2013;98:1466-1473. 6. Molitch ME, Clemmons DR, Malozowski S, et al. Evaluation and treatment of adult growth hormone deficiency: an Endocrine Society clinical practice guideline. J Clin Endocrinol Metab. 2011;96:1587-1609. 7. Melmed S. Pathogenesis of pituitary tumors. Nat Rev Endocrinol. 2011;7:257-266. 8. Vasilev V, Daly AF, Petrossians P, et al. Familial pituitary tumor syndromes. Endocr Pract. 2011;17(suppl 3):41-46. 9. Beckers A. Means, motives and opportunity: SDH mutations are suspects in pituitary tumors. J Clin Endocrinol Metab. 2013;98:2274-2276. 10. Fraser CL, Biousse V, Newman NJ. Visual outcomes after treatment of pituitary adenomas. Neurosurg Clin North Am. 2012;23:607-620. 11. Melmed S, Casaneuva FF, Hoffman AR, et al. Diagnosis and treatment of hyperprolactinemia. An Endocrine Society clinical practice guideline. J Clin Endocrinol Metab. 2011;96:273-288. 12. Devin JK. Hypopituitarism and central diabetes insipidus. Perioperative diagnosis and management. Neurosurg Clin North Am. 2012;23:679-689.

13. Swearingen B. Update on pituitary surgery. J Clin Endocrinol Metab. 2012;97:1071-1081. 14. Sheehan JP, Xu Z, Lobo MJ. External beam radiation therapy and stereotactic radiosurgery for pituitary adenomas. Neurosurg Clin North Am. 2012;23:571-586. 15. Katznelson L, Laws E, Molitch M, et al. Diagnosis and treatment of acromegaly. An Endocrine Society clinical practice guideline. J Clin Endocrinol Metab. 2014;99:3933-3951. 16. Rogers A, Karavitaki N, Wass JA. Diagnosis and management of prolactinomas and non-functioning pituitary adenomas. BMJ. 2014;349:g5390. 17. Dekkers OM, Horváth-Puhó E, Jørgensen JOL, et al. Multisystem morbidity and mortality in Cushing’s syndrome: a cohort study. J Clin Endocrinol Metab. 2013;98:2277-2284. 18. Bertagna X, Guignat L. Approach to the Cushing’s disease patient with persistent/recurrent hypercortisolism after pituitary surgery. J Clin Endocrinol Metab. 2013;98:1307-1318. 19. Feelders RA, Hofland LJ. Medical treatment of Cushing’s disease. J Clin Endocrinol Metab. 2013;98:425-438. 20. Barber TM, Adams E, Wass JA. Nelson syndrome: definition and management. Handb Clin Neurol. 2014;124:327-337. 21. Silveira LFG, Latronico AC. Approach to the patient with hypogonadotropic hypogonadism. J Clin Endocrinol Metab. 2013;98:1781-1788. 22. Persani L. Clinical Review: central hypothyroidism: pathogenic, diagnostic, and therapeutic challenges. J Clin Endocrinol Metab. 2012;97:3068-3078. 23. Önnestam L, Berinder K, Burman P, et al. National incidence and prevalence of TSH-secreting pituitary adenomas in Sweden. J Clin Endocrinol Metab. 2013;98:626-635. 24. Cooper O, Melmed S. Subclinical hyperfunctioning pituitary adenomas: the silent tumors. Best Pract Clin Endocrinol Metab. 2012;26:447-460. 25. Molitch ME. Management of incidentally found nonfunctional pituitary tumors. Neurosurg Clin North Am. 2012;23:53-554.

CHAPTER 224  Anterior Pituitary  

REVIEW QUESTIONS 1. A 25-year-old woman has had amenorrhea and galactorrhea for 2 years and is found to have hyperprolactinemia. Her PRL level is 513 ng/mL (normal 2.5 to 23.6 ng/mL) and magnetic resonance imaging showed a 1.3-cm macroadenoma. The rest of her evaluation is normal, with the exception of a serum calcium of 11.4 mg/dL. She recalls that one of her cousins also has a pituitary tumor. Which of the following genes should be analyzed for a mutation? A. PROP1 B. Pit1 C. Menin D. Ret proto-oncogene E. Ras Answer: C  Menin is encoded by the gene mutated in multiple endocrine neoplasia (MEN1), characterized by multiple endocrine tumors of the parathyroid glands, pancreatic islets, and anterior pituitary, especially prolactinomas. For incorrect answers, see introductory section on “Anatomy and Embryology.” 2. Which of the following medications used to treat pituitary tumor syndromes acts by blocking a hormone receptor? A. Pasireotide B. Cabergoline C. Ketoconazole D. Mifepristone E. Bromocriptine Answer: D  See Figure 224-8 and its legend.

1494.e3

3. A 67-year-old man is found to have enlarging hands and feet and has been referred by his dentist because of prognathism. To determine whether he has acromegaly, which of the following tests should be carried out? A. Pituitary magnetic resonance imaging B. Overnight 1-mg dexamethasone suppression test C. Insulin-induced hypoglycemia stimulation test D. Measurement of insulin-like growth factor (IGF)-I E. Inferior petrosal sinus sampling with GHRH stimulation Answer: D  See section on “Diagnosis” under “Growth Hormone Excess: Acromegaly and Gigantism.” 4. Magnetic resonance imaging (MRI) on a 73-year-old woman experiencing dizziness reveals an incidental 3-mm lesion in her pituitary that is compatible with a pituitary adenoma. She has otherwise been well. Testing for hormone oversecretion is negative. What should be the next step in management? A. Refer her to an experienced neurosurgeon. B. Repeat the MRI in 1 year to look for size change. C. Refer her to ophthalmology for a visual field examination. D. Begin bromocriptine treatment. E. Refer her for a preoperative cardiac stress test. Answer: B  See section on “Clinically Nonfunctioning Pituitary Tumors” at the end of the chapter and also e-Figure 224-1.

1495

CHAPTER 225  Posterior Pituitary  

Osmoreceptor

PVN

Thirst Osm

Plasma Plasma osmolality AVP (mOsm/kg) (pg/mL) 304

SON

302 Ant. Pit.

300

8

298

7

296

6

294 Thirst

FIGURE 225-1.  Sagittal view of the head, demonstrating the position of the neuro-

hypophysis. The magnocellular neurons are clustered in two paraventricular nuclei (PVN) and two supraoptic nuclei (SON). Only one nucleus of each pair is illustrated. The supraoptic nuclei are lateral to the edge of the optic chiasm, whereas the paraventricular nuclei are central along the wall of the third ventricle. The axons of the four nuclei combine to form the supraopticohypophysial tract as they course through the pituitary stalk to their storage terminals in the posterior pituitary. The osmostat (Osm) is in the hypothalamus anterior to the third ventricle; the thirst center (Thirst) is distributed across different brain areas. Ant. Pit. = anterior pituitary. (From Buonocore CM, Robinson AG. Diagnosis and management of diabetes insipidus during medical emergencies. Endocrinol Metab Clin North Am. 1993;22:411-423.)

osmostat controls the release of vasopressin to cause water retention and also stimulates thirst to cause water repletion. Osmotic regulation of vasopressin release and osmotic regulation of thirst are usually tightly coupled, but they can be dissociated under pathologic conditions. The primary extracellular osmolyte to which the osmoreceptor responds is sodium. Under normal physiologic conditions, glucose and urea cross neuron cell membranes and do not stimulate the release of vasopressin. Although basal osmolality in normal subjects ranges between 280 and 295 mOsm/kg H2O, extracellular fluid osmolality for each individual is maintained within narrow ranges. Increases in plasma osmolality as small as 1 to 2% are sufficient to stimulate vasopressin release. Basal plasma levels of vasopressin are generally 0.5 to 2 pg/mL, which maintains urine osmolality above plasma osmolality and urine volume in the range of 2 to 3 L/day. When vasopressin levels are suppressed below 0.5 pg/mL, maximal urine osmolality decreases to below 100 mOsm/kg H2O, and a free water diuresis (or “aquaresis”) ensues at levels that approach 800 to 1000 mL/hour (18 to 24 L/day). Increases in plasma osmolality cause a linear increase in plasma vasopressin and a corresponding linear increase in urine osmolality. At a plasma osmolality of approximately 295 mOsm/kg H2O, urine osmolality is maximally concentrated to 1000 to 1200 mOsm/kg H2O. Thus, the entire physiologic range of urine osmolality is accomplished by relatively small changes in plasma vasopressin levels of 0 to 5 pg/mL (Fig. 225-2). To maintain fluid balance, water must be not only conserved but also consumed to replace insensible water losses and obligate urine output. Thirst is not stimulated until a somewhat higher plasma osmolality (5 to 10 mOsm/ kg H2O) than the threshold for release of vasopressin. Most humans derive sufficient water from habitual fluid intake and catabolism of food to maintain plasma osmolality below the threshold that activates thirst. Therefore, under normal physiologic conditions, water balance (and hence plasma osmolality) is regulated more by secretion of vasopressin than by thirst. However, with severe degrees of dehydration, thirst is essential to restore body water deficits. Vasopressin acts on the V2 subtype of vasopressin receptors in the collecting duct principal cells of the kidney to cause water retention, or antidiuresis. Vasopressin V2 receptors are G protein–coupled receptors that activate adenylate cyclase, with subsequent increased intracellular cyclic adenosine monophosphate (cAMP) levels upon ligand activation of the receptor. The increased cAMP initiates the movement of aquaporin-2 (AQP2) water channels to the apical (luminal) membrane of the collecting duct cells. These channels allow facilitated rapid transport of water from the collecting duct lumen into the principal cell along osmotic gradients. The water then exits the cell through the basolateral membrane into the kidney medullary circulation through constitutively expressed aquaporin-3 and aquaporin-4 water channels.1 This entire process is termed antidiuresis. In the absence of vasopressin, the AQP2 channels are reinternalized from the apical membrane into subapical vesicles. This prevents active reabsorption of water from

Osmotic threshold

9

292

5

290

4

288

3

286

2

284 282

1

Urine osmolality (mOsm/kg) 1000 800 600 400 200

280

2 4 6 8 10 12 14 16 18 20

278

Urine volume (L/day)

276 274 272 270 FIGURE 225-2.  Idealized schematic of the normal physiologic relationships among plasma osmolality, plasma vasopressin (AVP), urine osmolality, and urine volume. The entire physiologic range of urine osmolality occurs with plasma vasopressin levels from 0 to 5 pg/mL. Increases in plasma osmolality above approximately 290 to 295 mOsm/kg H2O result in increases in plasma vasopressin but no further concentration of the urine, which is limited by the maximal osmolality in the inner medulla. The relation of volume (calculated on the basis of a constant osmolar load) is inversely exponential to the other parameters. Because of this relationship, urine volume does not change substantially until there is nearly absent vasopressin secretion, after which urine volume increases dramatically. (Calculated from formulas presented in Robertson GL, Shelton RL, Athar S. The osmoregulation of vasopressin. Kidney Int. 1976;10:25-37. Figure drawn by J.G. Verbalis, Georgetown University, Washington, DC.)

the collecting duct lumen, resulting in diuresis. In addition to this rapid “shuttling” of the AQP2 channels to regulate water reabsorption on a minute-to-minute basis, vasopressin also acts through the V2 receptors to regulate long-term stores of AQP2—that is, increased vasopressin stimulates AQP2 synthesis, and the absence of vasopressin results in decreased AQP2 synthesis. The hypertonic medullary interstitium is the determinant of the maximal concentration of the urine, which is in equilibrium with the osmolality of the inner medulla of the kidney under conditions of maximal antidiuresis (Chapter 115).

Vasopressin and Pressure and Volume Regulation

High-pressure baroreceptors are located in the aorta and carotid sinus, and low-pressure baroreceptors are located in the right and left atria. Decreases in blood pressure or intravascular volume stimulate vasopressin release, whereas situations that increase blood volume or left atrial pressure (e.g., negative-pressure breathing) decrease the secretion of vasopressin. The release of vasopressin in response to changes in volume or pressure is much less sensitive than the release in response to osmoreceptors; generally a 10 to 15% reduction in blood volume or pressure is needed to stimulate the release of vasopressin. However, once arterial pressure falls below this threshold, the stimulated response is exponential resulting in plasma levels of vasopressin that are markedly greater than those resulting from osmotic stimulation. The pressor effects of vasopressin are mediated through a separate vasopressin receptor subtype, the V1a receptors, located on vascular smooth muscle. The relatively insensitive regulation of vasopressin secretion by changes in volume and pressure and the modest role of vasopressin to regulate blood pressure are consistent with the notion that regulation of sodium homeostasis by the renin-angiotensin-aldosterone system (Chapter 227) is more important for controlling extracellular and blood volume than is the regulation of water homeostasis. However, the pressor effects of vasopressin to increase blood pressure can become prominent when other blood pressure

1496

CHAPTER 225  Posterior Pituitary  

regulatory systems are deficient (e.g., autonomic neuropathy or reninangiotensin-aldosterone system blockade) or in states of pathologic vasodilation (e.g., liver cirrhosis, septic shock).

Vasopressin and Adrenocorticotropic Hormone

Vasopressin stimulates adrenocorticotropic hormone (ACTH) secretion via stimulation of the vasopressin V1b receptor subtypes that are located on anterior pituitary corticotroph cells. Although the major regulator of ACTH secretion is corticotropin-releasing hormone (Chapter 224), vasopressin activates a different signal transduction system in the corticotrophs, so these hormones have synergistic effects on ACTH secretion.

Interaction of Osmotic and Volume Regulation

The vasopressin system has evolved to optimize mammalian water homeostasis. Water is consumed as available in the absence of stimulated thirst, and vasopressin secretion then regulates water excretion to maintain plasma osmolality. Thirst serves as a backup mechanism if dehydration becomes excessive. Because pressure-volume regulation of vasopressin is less sensitive, modest changes in pressure or volume, which are exacerbated by upright posture, do not interfere with the regulation of osmolality. Yet the pressor effect of high vasopressin levels serves to maintain blood pressure if volume depletion or hypotension becomes excessive. Usually, the physiologic regulation of osmolality and pressure-volume are synergistic. Dehydration causes an increase in plasma osmolality and a decrease in blood volume, both of which stimulate the release of vasopressin. Conversely, excess fluid administration causes a decrease in plasma osmolality and an expansion of blood volume, both of which inhibit vasopressin secretion. Other factors can also modulate osmotic release and action of vasopressin. With volume expansion, natriuretic factors such as atrial natriuretic peptide and brain natriuretic peptide are released from atrial myocytes and act at the kidney to induce a natriuresis. Brain natriuretic peptide is also synthesized in the hypothalamus, where it may act to decrease vasopressin secretion. During pregnancy, there is a decrease of plasma osmolality by approximately 10 mOsm/kg H2O as a result of a resetting of the osmostat for vasopressin secretion, and the osmostat for thirst is reset downward in parallel. These effects appear to be mediated by the placental hormone relaxin. Abnormalities in water and electrolyte balance are common in the elderly. This is due in part to age-related changes in body volume (as much as a 50% decrease in total body water occurs in those older than 75 years) and renal function. The elderly also have a decreased sense of thirst. Although there is a normal or even increased ability to secrete vasopressin with age, there is a decreased ability to achieve either maximal urine concentration to retain water or maximal dilution of urine to excrete water. Consequently, the elderly are particularly prone to both hypernatremia or hyponatremia with diseases that affect water balance or from the drugs used to treat various diseases.2

Oxytocin

Prolactin is the main hormone necessary for milk production, but oxytocin is essential for milk secretion. Suckling stimulates tactile receptors, producing an afferent signal to the hypothalamus that causes a synchronized release of oxytocin from the posterior pituitary. Oxytocin binds to oxytocin receptors in the breast and induces contraction of myoepithelial cells around the alveoli and ductules to eject milk. In addition, upregulation of uterine oxytocin receptors dramatically increases uterine smooth muscle contractions in response to oxytocin secretion at the end of pregnancy. The greatest release of oxytocin occurs with, not before, delivery of the infant, probably secondary to stretching of the vaginal wall. Because transgenic mice lacking either oxytocin or oxytocin receptors have normal parturition, oxytocin release may be more important to induce uterine contraction to inhibit blood loss after delivery than to initiate parturition. No pathologic syndromes of either increased or decreased secretion of oxytocin have yet been defined, but experimental studies have implicated oxytocin in maternal and affiliative behavior as well as bone formation.3 However, because of the structural similarity between vasopressin and oxytocin, at high plasma levels oxytocin can activate vasopressin receptors, and vasopressin can activate oxytocin receptors, both of which can have pathologic consequences.

  SYNDROME OF INAPPROPRIATE ANTIDIURETIC HORMONE SECRETION

Excess secretion of vasopressin can be caused by abnormally regulated secretion from the posterior pituitary, or by ectopic synthesis and secretion of vasopressin by tumors. Osmotically inappropriate secretion of vasopressin

causes renal water retention and volume expansion of body fluids, with consequent dilutional hyponatremia. This disorder is called the syndrome of inappropriate antidiuretic hormone secretion (SIADH) and is discussed in Chapter 116.

  DIABETES INSIPIDUS DEFINITION

Diabetes insipidus is the excretion of a large volume of hypotonic insipid (tasteless) urine, usually manifested by polyuria (increased urination) and polydipsia (increased thirst).4 The large urine volume, usually in excess of 50 to 60 mL/kg/day, must be distinguished from an increased frequency of small urine volumes and from large volumes of isotonic or hypertonic urine, both of which have different clinical significance.

PATHOBIOLOGY

Five pathophysiologic mechanisms must be considered in the differential diagnosis of diabetes insipidus. 1. Central diabetes insipidus is caused by the inability of the hypothalamus– posterior pituitary to secrete (and usually to synthesize) vasopressin in response to increased osmolality. No concentration of the dilute glomerular filtrate takes place in the renal collecting duct, and consequently, a large volume of hypotonic (i.e., dilute) urine is excreted. This produces a secondary increase in serum osmolality, with stimulation of thirst and secondary polydipsia. Levels of vasopressin in plasma are unmeasurable or inappropriately low for the plasma osmolality. 2. Nephrogenic diabetes insipidus is caused by the inability of an otherwise normal kidney to respond to vasopressin. As in hypothalamic (central) diabetes insipidus, the dilute glomerular filtrate entering the collecting duct is excreted as a large volume of hypotonic urine. The rise in plasma osmolality that occurs stimulates thirst and produces polydipsia. Unlike central diabetes insipidus, however, measured levels of vasopressin in plasma are high or appropriate for plasma osmolality. 3. Gestational diabetes insipidus is a rare condition produced by elevated levels or activity of placental cystine aminopeptidase (oxytocinase or vasopressinase) during pregnancy. The rapid destruction of vasopressin produces diabetes insipidus with polyuria and secondary stimulation of thirst with polydipsia. Because of the circulating vasopressinase, plasma vasopressin levels usually cannot be measured. 4. Primary polydipsia is a disorder of excess fluid ingestion rather than of vasopressin secretion or activity. Excessive ingested water produces a mild decrease in plasma osmolality that shuts off the secretion of vasopressin. In the absence of vasopressin action on the kidney, urine does not become concentrated, and a large volume of hypotonic urine is excreted. The amount of vasopressin in plasma is unmeasurable or low but is appropriate for the low plasma osmolality. 5. Osmoreceptor dysfunction is a variant of central diabetes insipidus in which the neurohypophysis is intact, but the osmoreceptive cells in the anterior hypothalamus have been damaged (see Fig. 225-1). Because the osmoreceptor cells are necessary for osmotically stimulated vasopressin secretion, the patient manifests polyuria. However, because the osmoreceptor cells also control thirst, these patients do not have polydipsia. As a result, they are characterized by elevated serum sodium levels and plasma osmolalities. For this reason, this disorder has also been called essential hypernatremia and adipsic hypernatremia, in recognition of the profound thirst deficits found in most of the affected patients. Although the pathophysiologic mechanisms for each of these five disorders are distinct, patients in the first four categories usually manifest polyuria and polydipsia, and the serum sodium level is usually normal because an intact thirst mechanism is sufficiently sensitive to maintain water homeostasis in the first three disorders, and the normal kidney has sufficient capacity to excrete the excess water load in the fourth. The fifth category of osmoreceptor dysfunction is the exception, owing to a defective thirst mechanism leading to hypernatremia.

CLINICAL MANIFESTATIONS

Central Diabetes Insipidus

The sudden appearance of hypotonic polyuria5 after transcranial surgery in the area of the hypothalamus or after head trauma with basal skull fracture and hypothalamic damage obviously suggests the diagnosis of central diabetes insipidus.6 In these situations, if the patient is unconscious and unable to recognize thirst, hypernatremia is a common accompaniment. However, even in patients with more insidious progression of a specific disease or in patients

CHAPTER 225  Posterior Pituitary  

with idiopathic central diabetes insipidus, the onset of polyuria is often relatively abrupt and occurs over several days or weeks. Most patients do not notice polyuria until urine volume exceeds 4 L/day, and as illustrated in Figure 225-2, urine volume does not exceed 4 L/day until the ability to concentrate the urine is severely limited and plasma vasopressin is nearly absent. As few as 10 to 15% of the normal number of vasopressinergic neurons in the hypothalamus is sufficient to maintain an asymptomatic urine volume, but the further loss of just a small number of these neurons produces a rapid increase in urine volume and symptomatic polyuria. Urine volume seldom exceeds the amount of dilute fluid delivered to the collecting duct (≈18 to 24 L in humans); in many cases, urine volume is significantly less because patients voluntarily restrict fluid intake, which causes some mild volume contraction and increased proximal tubular reabsorption of fluid. Patients often express a preference for cold liquids, which are more effective in assuaging thirst. Both thirst and increased urine output persist through the night, impairing sleep. Patients with partial central diabetes insipidus have some ability to secrete vasopressin, but this secretion is markedly attenuated at normal levels of plasma osmolality. Therefore, these patients often have symptoms and urine volume similar to those of patients with complete central diabetes insipidus. Because most patients with central diabetes insipidus have sufficient thirst to drink fluid to match urine output, few laboratory abnormalities are present at the time of initial evaluation. The serum sodium level can be in the high-normal range, whereas the blood urea nitrogen level can be low secondary to the large urine volume. Uric acid is relatively high because of the modest volume contraction and lack of action of vasopressin on V1a receptors in the kidney, which stimulate the clearance of uric acid. Uric acid levels greater than 5 mg/dL can distinguish diabetes insipidus from primary polydipsia.

Osmoreceptor Dysfunction

A variant of central diabetes insipidus is the syndrome of osmoreceptor dysfunction. Physiologic maneuvers demonstrate that when patients are euvolemic, an increase in plasma osmolality produces neither secretion of vasopressin nor a sensation of thirst. However, vasopressin is still synthesized by the hypothalamus and stored in the posterior pituitary, because stimulation of baroreceptors by hypovolemia or hypotension results in the prompt secretion of vasopressin; the kidney is responsive because vasopressin release by volume receptor stimulation causes urinary concentration. Because patients lack thirst, they are chronically dehydrated, often with markedly increased serum sodium levels. However, it is the dehydration-induced volume depletion, not the increased osmolality, that eventually stimulates the secretion of vasopressin. The volume of urine output depends on the degree of dehydration-induced secretion of vasopressin. If sufficient fluid replacement is given to return extracellular fluid volume to normal, these patients are unable to regulate vasopressin by osmolality and again become polyuric, thereby manifesting their underlying central diabetes insipidus. Lesions that cause osmoreceptor dysfunction are similar to lesions that can cause central diabetes insipidus, but in contrast to central diabetes insipidus these lesions usually occur more rostrally in the hypothalamus, consistent with the anterior hypothalamic location of the primary osmoreceptor cells (see Fig. 15-2). One lesion that is unique to this disorder is an anterior communicating cerebral artery aneurysm, particularly following resection of the aneurysm. Central diabetes insipidus can be inherited as an autosomal dominant disease that is typically characterized by an asymptomatic infancy and an onset later in childhood. Most genetic defects are either in the signal peptide of the pre-prohormone or in the neurophysin portion of the prohormone.7 Mutations involving the vasopressin sequence itself are few. Most cases are believed to result from disruption of cleavage from the signal peptide or abnormal folding of the neurophysin, which slows trafficking of the mutant prohormone through the endoplasmic reticulum, leading to neuronal cell dysfunction and/or death. Because this is a cumulative process, this explains the later onset of central diabetes insipidus with these types of mutations. Myxedema and adrenal insufficiency both impair the ability to excrete free water by renal mechanisms. The simultaneous occurrence of either of these diseases with central diabetes insipidus (as can occur with a tumor of the hypothalamus or pituitary) can decrease an otherwise large urine output, thereby masking the symptoms of diabetes insipidus. Replacement treatment for the anterior pituitary deficiency, especially glucocorticoids, can then cause a sudden and massive excretion of dilute urine. Similarly, the onset of either hypothyroidism or adrenal insufficiency during the course of diabetes insipidus can decrease the need for vasopressin replacement and in some cases can even cause hyponatremia. Central diabetes insipidus occurs commonly in patients with severe brain ischemia, and is often indicative of

1497

brain death. Treatment of the diabetes insipidus along with any coexistent anterior pituitary hormone deficiencies can be used to preserve donor organs in such cases.

Nephrogenic Diabetes Insipidus

Nephrogenic diabetes insipidus is caused by mutations of the vasopressin V2 receptor or the vasopressin-induced water channel AQP2, or by impairments in the signal transduction system linking V2 receptor activation and AQP2 membrane insertion. Familial nephrogenic diabetes insipidus is a rare disease, most cases of which (>90%) are due to mutations of the V2 receptor. More than 100 different V2 receptor mutations have been described and can be classified into several different general categories based on differences in transport of the mutant receptor to the cell surface and vasopressin binding or stimulation of adenylate cyclase. Because the gene for the V2 receptor is located on the X chromosome, this is an X-linked recessive disease. Symptoms are noted only in affected males, who often present with vomiting, constipation, failure to thrive, fever, and polyuria during the first week of life. Hypernatremia with a hypotonic urine is typically present. The phenotype is similar in the less than 10% of patients with mutations of the AQP2 water channel, but because the AQP2 gene is located on chromosome 12, mutations cause autosomal recessive disease; consequently, consanguinity and a family history of the disease in men and women is common, and this disorder should be suspected when the proband is female.8 Nephrogenic diabetes insipidus can also be acquired during treatment with certain drugs such as demeclocycline (which can be used to treat inappropriate secretion of vasopressin), lithium carbonate (used to treat bipolar disorders), and fluoride (previously used in fluorocarbon anesthetics), and from electrolyte abnormalities such as severe hypokalemia and hypercalcemia. All causes of acquired nephrogenic diabetes insipidus have in common the decreased synthesis and function of AQP2 due to impaired vasopressin signaling from V2 receptor binding and activation. Other diseases of the kidney can produce polyuria and inability to concentrate the urine secondary to altered renal medullary blood flow or to other disorders that inhibit maintenance of the hyperosmolar concentrating gradient in the inner medulla. Renal manifestations of such disorders (e.g., sickle cell disease, sarcoidosis, pyelonephritis, multiple myeloma, analgesic nephropathy) are discussed in Chapter 121.

Gestational Diabetes Insipidus

In pregnancy, there is an increased metabolism of vasopressin due to cystine aminopeptidase (oxytocinase or vasopressinase), an enzyme that degrades oxytocin and prevents premature uterine contractions. Normally, this is compensated for by increased synthesis and secretion of vasopressin. Rarely, women with normal regulation of vasopressin develop diabetes insipidus because of markedly elevated levels of vasopressinase. Some of these patients have accompanying preeclampsia, acute fatty liver, and coagulopathies, but causal relations between diabetes insipidus and these abnormalities have not been identified. In general, diabetes insipidus does not persist after the pregnancy ends and does not recur in subsequent normal pregnancies.9 Polyuria can also manifest in patients who have limited vasopressin reserve (partial central diabetes insipidus) or who respond poorly to vasopressin action (compensated nephrogenic diabetes insipidus). Treatment may be required only during the pregnancy, and the patient often returns to her previous baseline function without the need for therapy when the pregnancy ends. Less commonly, central diabetes insipidus of another cause first becomes symptomatic during pregnancy and then persists afterward, following the usual course of diabetes insipidus.

Primary Polydipsia

Excessive fluid intake also causes hypotonic polyuria and, by definition, polydipsia. This disorder must be differentiated from the various causes of diabetes insipidus. Despite normal pituitary and kidney function, patients with this disorder share many characteristics of both central diabetes insipidus (vasopressin secretion is suppressed as a result of decreased plasma osmolality) and nephrogenic diabetes insipidus (kidney AQP2 expression is decreased as a result of suppressed plasma vasopressin levels). Many different names have been used for this excessive fluid intake, including dipsogenic diabetes insipidus, but primary polydipsia remains the best descriptor to avoid confusing this order with diabetes insipidus as classically defined. Primary polydipsia is sometimes due to a severe mental illness such as schizophrenia, mania, or obsessive-compulsive disorder, in which case it is called psychogenic polydipsia. These patients usually deny true thirst and attribute their polydipsia to bizarre motives, such as a need to cleanse the

CHAPTER 225  Posterior Pituitary  

body of poisons. The incidence in psychiatric hospitals can be as high as 40%, and there is no obvious explanation for the polydipsia. Primary polydipsia can also be caused by an abnormality in the osmoregulatory control of thirst, in which case it is called dipsogenic diabetes insipidus. These patients have no overt psychiatric illness and invariably attribute their polydipsia to a nearly constant thirst. Dipsogenic diabetes insipidus is usually idiopathic, but it can also be secondary to organic structural lesions in the hypothalamus identical to those causing central diabetes insipidus, such as neurosarcoidosis of the hypothalamus, tuberculous meningitis, multiple sclerosis, or trauma. Consequently, all polydipsic patients should be evaluated with magnetic resonance imaging (MRI) of the brain before it is concluded that excessive water intake is due to an idiopathic or psychiatric cause. Primary polydipsia can also be produced by diseases or drugs that cause a dry mouth, or by any peripheral disorder causing marked elevations of renin or angiotensin. Finally, primary polydipsia is sometimes caused by physicians, nurses, lay practitioners, or health writers who recommend a high fluid intake for valid (e.g., recurrent nephrolithiasis) or unsubstantiated health reasons. These patients lack overt signs of mental illness, but they also deny thirst and usually attribute their polydipsia to habits acquired from years of adherence to a drinking regimen. Laboratory studies in these patients are generally normal, although the serum sodium concentration is sometimes at the low end of the normal range, and the level of uric acid is lower than in patients with other forms of diabetes insipidus.

Normal Primary polydipsia Partial central DI

Complete central DI Nephrogenic DI

1000 Urine osmolality (mOsm)

1498

desmopressin

800 600 400 200 0 0

2

4 6 8 10 Hours of water deprivation

12

14

FIGURE 225-3.  Responses to the water deprivation test to differentiate various types of diabetes insipidus (DI) and primary polydipsia (as described by Miller M, Dalakos T, Moses AM, et al. Recognition of partial defects in antidiuretic hormone secretion. Ann Intern Med. 1970;73:721-729). The response to dehydration reaches a plateau, and the subsequent change in urine osmolality in response to administered desmopressin is illustrated. See the discussion in the text.

DIAGNOSIS

Physiologic Diagnosis

Diabetes insipidus should be considered in all patients presenting with significant polyuria, defined as urine output greater than 50 mL/kg/day. Although osmotic diuresis secondary to hyperglycemia, intravenous contrast agents, or renal injury is a more common clinical cause of polyuria, the medical history, an isotonic urine osmolality, and routine clinical laboratory tests generally distinguish these disorders from diabetes insipidus. A diagnosis of diabetes insipidus can be made when urine osmolality is inappropriately low in the presence of an elevated plasma osmolality as a result of increased serum sodium concentration. These criteria are sometimes met at the initial examination, especially in cases of acute diabetes insipidus occurring after trauma or surgery with inadequate fluid replacement. In such patients with hypernatremia and hypotonic urine osmolality with normal renal function, one need only administer a vasopressin agonist to differentiate central diabetes insipidus, in which a renal response with decreased urine volume and increased urine osmolality occurs, from nephrogenic diabetes insipidus, in which a subnormal renal response is seen. Sometimes in the postoperative state, a water diuresis occurs as a result of water retention during the surgical procedure. Vasopressin is normally secreted in response to surgical stress, causing fluid administered intravenously during the procedure to be retained. During recovery, vasopressin levels fall, and a diuresis of the retained fluid occurs. In this case, the serum sodium level is almost always normal; however, if additional fluid is administered to match the urine output, persistent polyuria can be mistaken for diabetes insipidus. In this situation, the physician should decrease the rate of fluid administered and follow the urine output and serum sodium level. If the urine output decreases and the serum sodium level remains normal, no treatment is necessary; if serum sodium rises above the normal range and the urine remains hypotonic, diabetes insipidus is likely, and the response to a vasopressin agonist can ascertain the type (central versus nephrogenic). Most outpatients with diabetes insipidus are not hypernatremic, because the polydipsia produced by a normal thirst response is generally sufficient to maintain water homeostasis. Instead, they present with polyuria, polydipsia, and a normal sodium level. In these patients, further testing is necessary to increase serum osmolality and then measure the plasma vasopressin level or the urinary response to an administered vasopressin agonist. The best described test is the water deprivation test (Fig. 225-3),10 which should be carried out under controlled observation in the hospital or an appropriately equipped outpatient area. The exact timing of the test depends on the patient’s symptoms. If the patient has marked polyuria during the night, it is best to begin the test during the day because the patient may become overly dehydrated overnight. However, if the patient has only two or three episodes of nocturia per night, it is best to begin the test in the evening so that the major part of the dehydration takes place when the patient is asleep. In either case, the patient is weighed at the beginning of the test, and all subsequent fluids are withheld. The volume and osmolality of all excreted urine are measured, and the patient is reweighed after each liter of urine output. When three

consecutive urine samples have an osmolality differing by no more than 10% and the patient has lost at least 2% of body weight, a blood sample is obtained for the measurement of serum osmolality, sodium, and plasma vasopressin. The patient is then given 2 µg of desmopressin intravenously or intramuscularly and observed for an additional 2 hours. Adults with normal vasopressin secretion concentrate their urine to greater than 800 mOsm/kg H2O and have less than a 10% increase in urine osmolality in response to administered desmopressin. Patients with complete central diabetes insipidus have minimal concentration of the urine with dehydration, and a marked increase in urine osmolality (usually >50%) in response to administered desmopressin. Patients with nephrogenic diabetes insipidus usually have no increase in urine concentration in response to administered desmopressin, although in some cases of acquired nephrogenic diabetes insipidus, some increased urinary concentration (but generally < 10%) can occur. Nephrogenic diabetes insipidus is best distinguished from central diabetes insipidus by the measurement of vasopressin in plasma; plasma vasopressin levels are elevated in cases of nephrogenic diabetes insipidus, especially after dehydration. In patients with partial central diabetes insipidus and patients with primary polydipsia, the urine is often somewhat concentrated in response to dehydration, but not to the maximum of a normal person. The chronically reduced level of vasopressin downregulates the synthesis of AQP2 water channels, and the large urine volume, regardless of cause, washes out the medullary osmotic gradient that is the determinant of maximal urine concentration. When desmopressin is administered, patients with partial central diabetes insipidus have a further increase (usually > 10% but < 50%) in urine osmolality, whereas most patients with primary polydipsia have no further increase (i.e., 145 mmol/L). Consequently, some investigators recommend a limited infusion of hypertonic (3%) sodium chloride solution to achieve these elevated levels if they are not achieved by the water deprivation itself. Measurement of the C-terminal fragment of the vasopressin prohormone copeptin may be a better surrogate measure of vasopressin secretion.11 In some difficult cases, the response to treatment with a vasopressin agonist can be a useful aid to diagnosis. If a decrease in polyuria and thirst with maintenance of normal serum sodium concentration occurs, a diagnosis

CHAPTER 225  Posterior Pituitary  

of partial central diabetes insipidus is likely; however, if polydipsia persists and hyponatremia develops, a diagnosis of primary polydipsia is confirmed.

Etiologic Diagnosis

If the water deprivation test confirms that inadequate vasopressin secretion is responsible for the polyuria, the underlying cause must be determined. MRI of the hypothalamic-pituitary area is the most important diagnostic tool in these cases. The three areas of interest are the immediate suprasellar region of the hypothalamus, the pituitary stalk, and the posterior pituitary within the sella turcica (see the earlier discussion of anatomy). Most slow-growing tumors confined to the sella do not cause diabetes insipidus. To cause central diabetes insipidus, tumors in the hypothalamic area immediately above the sella must be either sufficiently large to destroy 80 to 90% of the vasopressin cells or located where the paths of the four nuclear groups converge at the origin of the pituitary stalk, just above the diaphragma sellae. Primary tumors, especially craniopharyngioma and suprasellar germinoma, metastatic tumors, and infiltrative diseases can also cause diabetes insipidus by infiltration of the pituitary stalk, which is then thickened (i.e., >2 mm) on MRI. On T1-weighted MRI, the vasopressin and oxytocin stored in neurosecretory granules in the posterior pituitary are visualized as a bright spot in the sella turcica. Most but not all normal subjects have this bright spot (it is absent more frequently in elderly and dehydrated patients); in most but not all patients with central diabetes insipidus, the bright spot is absent. Thickening of the stalk and absence of the bright spot are therefore especially suggestive of a hypothalamic disease process.12 Tumors that cause central diabetes insipidus are most often benign primary intracranial tumors such as craniopharyngioma, ependymoma (suprasellar germinoma), and pinealoma, which arise in the third ventricle. Primary tumors of the anterior pituitary (Chapter 224) cause diabetes insipidus only when substantial suprasellar extension is present. However, rapidly growing intrasellar lesions, such as metastases from carcinomas of the lung, breast, and melanoma or hemorrhage into pituitary adenomas, can cause diabetes insipidus because there is insufficient time for the vasopressin axons to adapt by releasing vasopressin from the hypothalamus. Metastases to the hypothalamus can also destroy the supraopticohypophysial tract and produce diabetes insipidus. Granulomatous diseases, such as Langerhans cell histiocytosis, sarcoidosis, tuberculosis, and leukemic infiltrates and lymphomas of the hypothalamus, can cause diabetes insipidus by destroying vasopressin cells. In such patients, the diagnosis is usually suspected on the basis of peripheral manifestations of the respective diseases. Lymphocytic infundibuloneurohypophysitis is an autoimmune disease similar to lymphocytic hypophysitis of the anterior pituitary (Chapter 224) in which lymphocytes infiltrate the neurohypophysis to produce diabetes insipidus. The hallmarks of this process are a thickened pituitary stalk and an absence of the pituitary bright spot in a patient with the abrupt onset of polyuria and polydipsia, particularly a postpartum female. The diagnosis was originally demonstrated by pituitary biopsy, but now is more commonly made by regression of the thickened stalk with continued MRI follow-up. When no specific cause is identified, the diagnosis of exclusion is idiopathic diabetes insipidus; but most such cases are probably caused by an autoimmune disease, and other autoimmune diseases, including anterior pituitary hypophysitis,13 are often recognized in affected patients. When central nervous system disease is suspected but not diagnosed by MRI or general physical examination, cerebrospinal fluid obtained by lumbar puncture may be helpful in identifying tumor cells or markers of tumors or inflammatory processes (e.g., elevated angiotensin-converting enzyme levels with neurosarcoidosis, elevated β-HCG levels with germinomas). A family history suggestive of diabetes insipidus should be investigated with genetic testing for inherited mutations in the vasopressin or vasopressin receptor genes depending on the site of the defect.

TREATMENT  Because excess excretion of water is the primary manifestation of diabetes insipidus, water replacement in adequate quantities avoids the metabolic complications of all forms of this disease. However, oral or intravenous administration of the volume of fluid required to replace the often large urinary losses in diabetes insipidus is difficult and inconvenient. The goal of therapy is therefore to reduce the amount of polyuria and polydipsia to a tolerable level while avoiding overtreatment, which can produce water retention and hyponatremia.

1499

Central Diabetes Insipidus

The best therapeutic agent for the treatment of central diabetes insipidus is the vasopressin agonist desmopressin.14 Desmopressin is different from vasopressin in that the amino group of the N-terminal cystine residue has been removed to prolong the duration of action, and d-arginine has been substituted for l-arginine in position 8 to decrease the vasopressor effects. At therapeutic dosages, this agent acts primarily on V2 or antidiuretic receptors, with minimal activity at V1a or pressor receptors. Desmopressin is available as tablets of 0.1 or 0.2 mg for oral administration and in either a spray bottle that delivers a fixed dose of 10 µg in 100 µL or a bottle with a rhinal catheter that can deliver 50 to 200 µL (5 to 20 µg) for intranasal administration. When therapy is initiated, it is generally best to begin with a low dose (e.g., half of a 0.1-mg tablet, 5 µg by the rhinal tube, or a single 100 µL spray of 10 µg) at bedtime to allow the patient to sleep through the night, and then determine the duration of action by quantifying the polyuria the next day. The duration of action of a single dose varies from 6 to 24 hours, but in most patients, a good therapeutic response can be achieved on an every-12-hour schedule for the nasal spray or an 8- or 12-hour schedule for the tablets. Desmopressin is also available for parenteral use in 1-mL vials of 4 µg/mL. Parenteral administration is especially useful postoperatively or when a patient is unable to take the nasal preparation. In hospitalized patients, some physicians add vasopressin directly to a crystalloid solution to infuse doses in the range of 0.25 to 2.7 mIU/kg/hour to cause modest but persistent urinary concentration as a treatment of diabetes insipidus. With any form of desmopressin administration, serum sodium levels should be monitored regularly to prevent the development of hyponatremia.15

Osmoreceptor Dysfunction

Because the diabetes insipidus of patients with osmoreceptor dysfunction is central, they respond to desmopressin as do patients with central diabetes insipidus. However, because of their thirst defect, this is usually not sufficient to maintain normal plasma osmolality. Consequently, they must be given a “prescription” for amounts of fluids to be consumed each 24 hours in order to maintain normal serum sodium levels and plasma osmolalities. This must be individualized to each patient because overconsumption of fluid coupled with desmopressin administration can produce severe hyponatremia. Body weights using an accurate scale is useful as a guide to preventing under- or overhydration, but frequent monitoring of serum sodium levels is usually necessary as well.

Nephrogenic Diabetes Insipidus

Although most patients with nephrogenic diabetes insipidus do not respond to desmopressin, a small number have a partial response to higher doses (e.g., 10 to 20 µg subcutaneously or intranasally). For the majority of patients who have no response to desmopressin, some orally administered pharmacologic agents are also useful in treating nephrogenic diabetes insipidus. Thiazide diuretics cause sodium depletion and volume contraction and decrease urine volume by increasing proximal tubular reabsorption of glomerular filtrate. Prostaglandin synthase inhibitors (e.g., indomethacin) block the action of prostaglandin E to inhibit the action of vasopressin on the kidney. Chlorothiazide, amiloride, and prostaglandin synthase inhibitors are useful to reduce polyuria in nephrogenic diabetes insipidus. However, none of these agents has been approved by the U.S. Food and Drug Administration for the treatment of diabetes insipidus; therefore, the prescribing physician should be aware of potential toxicities and side effects. In cases of drug-induced nephrogenic diabetes insipidus, the most direct therapy is discontinuation of the offending agent, if possible. Symptomatic nephrogenic diabetes insipidus is usually treated with a thiazide diuretic, which is enhanced by coadministration of the potassium-sparing diuretic amiloride. Amiloride can be especially beneficial in cases of nephrogenic diabetes insipidus induced by lithium, because the drug decreases the entrance of lithium into cells in the distal tubule. When diuretics are used to treat nephrogenic diabetes insipidus, special attention should be paid to the possibility that the induced dehydration may increase the concentration of other drugs.

Gestational Diabetes Insipidus

During pregnancy, vasopressinase increases the metabolism of vasopressin but not of desmopressin, so desmopressin is the drug of choice for these patients. The vasopressinase activity subsides by a few weeks after delivery, and patients with the onset of partial diabetes insipidus during pregnancy may become asymptomatic after delivery. An additional advantage of des­ mopressin is that it has little action on the oxytocin receptors of the uterus. During pregnancy, normal plasma osmolality decreases by approximately 10 mOsm/kg H2O because of changes in serum sodium, so pregnant patients with diabetes insipidus require only enough desmopressin to maintain the serum sodium at this lower level.

Correction of Hyperosmolality

Some situations require special attention during therapy. Rarely, if patients with diabetes insipidus are unable to drink or are given a hypertonic solution, severe hypernatremia can develop acutely. Osmotic equilibrium with the

1500

CHAPTER 226  Thyroid  

intracellular water of neurons and glia produces shrinking of the brain. The brain is in a closed vault (i.e., the skull), and when the brain shrinks, traction on the vasculature of the central nervous system can cause the rupture of blood vessels and subarachnoid or intracerebral hemorrhage. If the hypernatremia persists for a longer time, the neurons accommodate by producing organic osmolytes (previously called idiogenic osmoles), which limit the amount of brain shrinkage. Once this adaptation has occurred, a too-rapid lowering of osmolality in the extracellular fluid will produce a shift of water into the brain and cause cerebral edema. In this situation, desmopressin can be administered to produce constant antidiuresis, and the amount of water given can be regulated to decrease osmolality by no more than approximately 12 mEq/L every 24 hours. Postoperatively or after head trauma, diabetes insipidus can be transient (see Prognosis), and the need for long-term maintenance therapy cannot be immediately established.

PROGNOSIS

The prognosis of properly treated diabetes insipidus is excellent. If nephrogenic diabetes insipidus is diagnosed and treated early, intracranial calcification and mental retardation do not occur. When the diabetes insipidus is secondary to a recognized disease process, that disease generally determines the ultimate prognosis. In some specific clinical situations, the course is different and characteristic. The development of diabetes insipidus after surgical or traumatic injury to the neurohypophysis can follow any of several welldefined patterns (Fig. 225-4). In some patients, polyuria develops 1 to 4 days after injury and resolves spontaneously. Less often, the diabetes insipidus is permanent and continues indefinitely. Most interestingly, one can see a “triphasic” response that has been well described after pituitary stalk transection. The first phase of diabetes insipidus is due to axon shock and lack of function of the damaged neurons. This phase lasts several hours to several days and is followed by a second, antidiuretic phase that is due to the uncontrolled release of vasopressin from the disconnected and degenerating posterior pituitary or from the remaining severed neurons. Overly aggressive administration of fluids during this second phase does not suppress the

UV

Transient DI

A

T

UV

Permanent DI

B

T Triphasic Response I

UV

III II

uncontrolled vasopressin release from the damaged neurohypophysis and can lead to hyponatremia. The antidiuresis can last 2 to 14 days, after which diabetes insipidus recurs after depletion of vasopressin from the degenerating posterior pituitary gland (third phase). Transient hyponatremia without preceding or subsequent diabetes insipidus has been reported after transsphenoidal surgery for pituitary microadenomas. Once a deficiency of vasopressin secretion has been present for more than a few weeks, it rarely improves, even if the underlying cause of the neurohypophysial destruction is eliminated. The major exception to this is postoperative diabetes insipidus, in which spontaneous resolution is the rule. Although recovery from diabetes insipidus that persists more than several weeks postoperatively is less common, and is uncommon after 1 year of continued diabetes insipidus, well-documented cases of recovery as long as 10 years after the initiating event have been reported. Potential return of function is a reason to occasionally withhold therapy during long-term treatment. Diabetes insipidus should not be considered idiopathic until at least 4 years of follow-up. During this interval, annual computed tomography or MRI is indicated to search for a tumor or infiltrative process that may not have been detected at the initial examination. GENERAL REFERENCES For the General References and other additional features, please visit Expert Consult at https://expertconsult.inkling.com.

226  THYROID MATTHEW KIM AND PAUL W. LADENSON The adult thyroid gland contains two lobes that wrap along the anterolateral aspects of the trachea, midway between the thyroid cartilage and the suprasternal notch. Each lobe is demarcated into upper, middle, and lower poles. The right and left lobes are connected by an isthmus on the anterior aspect of the trachea just below the cricoid cartilage. The normal terminus of the thyroglossal duct can persist as a pyramidal lobe, which is often palpably enlarged in diffuse thyroid disorders such as autoimmune thyroiditis and Graves disease. With thyroid enlargement, the attachment of the sternothyroid muscle to the trachea limits upward expansion of the lobes, but further lateral, posterior, and downward growth may lead the gland to extend into the superior mediastinum, compressing the trachea and veins at the thoracic outlet. The parathyroid glands usually lie behind the superior and inferior poles of the thyroid lobes. The recurrent laryngeal nerves course upward along the tracheoesophageal groove, from which branches pass behind each thyroid lobe to innervate the larynx. Thyroid tissue is composed of clustered spherical follicles, each containing a single layer of follicular epithelial cells known as thyrocytes that surround a lumen containing colloid. The principal component of colloid is thyroglobulin, a thyrocyte-specific protein. Parafollicular C cells, which are derived from neural crest tissue and produce calcitonin, are widely dispersed between follicles.

  PHYSIOLOGY

Thyroid Hormone Synthesis and Secretion C

T

FIGURE 225-4.  A to C, Diagrammatic summary of the major patterns of postoperative and post-traumatic diabetes insipidus (DI). The abscissa represents time (T) after the initial injury (arrow); the ordinate represents urinary volume (Uv) relative to a hypothetical “normal” urine output of 2 to 3 L/24 hours (solid line). See the discussion in the text. During the triphasic response (C), uncontrolled release of vasopressin from the disconnected or damaged posterior pituitary gland causes an antidiuresis that can lead to water retention and a dilutional hyponatremia. Diabetes insipidus returns as the third phase after the stored hormone in the posterior pituitary has been depleted. (From Verbalis JG, Robinson AG, Moses AM. Postoperative and post-traumatic diabetes insipidus. In: Czernichow AP, Robinson A, eds. Diabetes Insipidus in Man: Frontiers of Hormone Research. Basel: S Karger; 1985:247.)

Dietary iodine in the form of iodide (I−) or iodate (IO3−) is absorbed by the gastrointestinal tract and distributed in the extracellular fluid. Daily iodine intake in the United States equals or exceeds the recommended daily intake of 150 µg because of the widespread use of iodized salt and iodate preservatives in baked goods. Circulating iodide is actively transported into the thyrocyte by the sodium-iodide symporter. Within the thyrocyte, iodide is rapidly oxidized by H2O2 in a reaction catalyzed by thyroid peroxidase. The reactive intermediate formed is covalently bound to tyrosyl residues present in thyroglobulin to generate monoiodotyrosine and diiodotyrosine residues through a process known as organification. Thyroid peroxidase also catalyzes the coupling of the monoiodotyrosine and diiodotyrosine residues to generate thyroxine (T4) and triiodothyronine (T3) residues in thyroglobulin,

CHAPTER 225  Posterior Pituitary  

GENERAL REFERENCES 1. Kortenoeven ML, Fenton RA. Renal aquaporins and water balance disorders. Biochim Biophys Acta. 2014;1840:1533-1549. 2. Cowen LE, Hodak SP, Verbalis JG. Age-associated abnormalities of water homeostasis. Endocrinol Metab Clin North Am. 2013;42:349-370. 3. Colaianni G, Tamma R, Di BA, et al. The oxytocin-bone axis. J Neuroendocrinol. 2014;26:53-57. 4. Leroy C, Karrouz W, Douillard C, et al. Diabetes insipidus. Ann Endocrinol (Paris). 2013;74: 496-507. 5. Jakes AD, Bhandari S. Investigating polyuria. BMJ. 2013;347:f6772. 6. Schreckinger M, Szerlip N, Mittal S. Diabetes insipidus following resection of pituitary tumors. Clin Neurol Neurosurg. 2013;115:121-126. 7. Babey M, Kopp P, Robertson GL. Familial forms of diabetes insipidus: clinical and molecular characteristics. Nat Rev Endocrinol. 2011;7:701-714. 8. Bichet DG. Physiopathology of hereditary polyuric states: a molecular view of renal function. Swiss Med Wkly. 2012;142:w13613. 9. Aleksandrov N, Audibert F, Bedard MJ, et al. Gestational diabetes insipidus: a review of an underdiagnosed condition. J Obstet Gynaecol Can. 2010;32:225-231.

1500.e1

10. Fenske W, Allolio B. Current state and future perspectives in the diagnosis of diabetes insipidus: a clinical review. J Clin Endocrinol Metab. 2012;97:3426-3437. 11. Fenske W, Quinkler M, Lorenz D, et al. Copeptin in the differential diagnosis of the polydipsiapolyuria syndrome–revisiting the direct and indirect water deprivation tests. J Clin Endocrinol Metab. 2011;96:1506-1515. 12. Di IN, Napoli F, Allegri AE, et al. Diabetes insipidus–diagnosis and management. Horm Res Paediatr. 2012;77:69-84. 13. Bando H, Iguchi G, Fukuoka H, et al. The prevalence of IgG4-related hypophysitis in 170 consecutive patients with hypopituitarism and/or central diabetes insipidus and review of the literature. Eur J Endocrinol. 2014;170:161-172. 14. Oiso Y, Robertson GL, Norgaard JP, et al. Clinical review: treatment of neurohypophyseal diabetes insipidus. J Clin Endocrinol Metab. 2013;98:3958-3967. 15. Behan LA, Sherlock M, Moyles P, et al. Abnormal plasma sodium concentrations in patients treated with desmopressin for cranial diabetes insipidus: results of a long term retrospective study. Eur J Endocrinol. 2015;172:243-250.

1500.e2

CHAPTER 225  Posterior Pituitary  

REVIEW QUESTIONS 1. The hormone vasopressin (AVP) that regulates body water balance is synthesized where? A. The anterior pituitary gland B. The posterior pituitary gland C. The supraoptic and paraventricular nuclei of the hypothalamus D. The anterior hypothalamus near the osmoreceptor cells E. The principal collecting duct cells of the kidney Answer: C  The supraoptic and paraventricular nuclei of the hypothalamus. The hormones of the posterior pituitary, vasopressin and oxytocin, are synthesized in specialized neurons in the hypothalamus, the neurohypophysial neurons. These neurons, notable for their large size, are termed magnocellular neurons. In the hypothalamus, magnocellular neurons are clustered in the paired paraventricular and supraoptic nuclei (see Fig. 232-1). The synthesized vasopressin prohormone is transported down the axons of the magnocellular neurons to the posterior pituitary gland, where vasopressin is released in response to specific stimuli. 2. Vasopressin secretion is stimulated at what levels of plasma osmolality? A. Plasma osmolality above the thirst threshold of 295 mOsm/kg H2O B. Increases of plasma osmolality of 4 to 5% C. Increases of plasma osmolality of 2 to 3% D. Increases of plasma osmolality of 1 to 2% E. Decreases in plasma osmolality of 1 to 2% Answer: D  Increases of plasma osmolality of 1 to 2%. Increases in plasma osmolality as small as 1 to 2% stimulate vasopressin release. Basal plasma levels of vasopressin are generally 0.5 to 2 pg/mL, which is sufficient to maintain urine osmolality above plasma osmolality and urine volume in the range of 2 to 3 L/day. When vasopressin levels are suppressed below 0.5 pg/mL, maximal urine osmolality decreases to less than 100 mOsm/kg H2O, and a free water diuresis (or “aquaresis”) ensues at levels approaching 800 to 1000 mL/hour (18 to 24 L/day). Increases in plasma osmolality cause a linear increase in plasma vasopressin and a corresponding linear increase in urine osmolality. At a plasma osmolality of approximately 295 mOsm/kg H2O, urine osmolality is maximally concentrated to 1000 to 1200 mOsm/kg H2O. Thus, the entire physiologic range of urine osmolality is accomplished by relatively small changes in plasma vasopressin of 0 to 5 pg/mL (see Fig. 232-2). 3. Patients with diabetes insipidus and inability to concentrate their urine usually present with what manifestations? A. Polyuria, polydipsia, and hyperosmolality B. Polyuria, polydipsia, and dehydration C. Polyuria, polydipsia, and hypernatremia D. Polyuria, polydipsia, and elevated BUN and creatinine E. Polyuria, polydipsia, and normal serum sodium, osmolality, BUN, and creatinine Answer: E  Polyuria, polydipsia, and normal serum sodium, osmolality, BUN, and creatinine. Most patients with diabetes insipidus are not hypernatremic, because the polydipsia produced by a normal thirst response is generally sufficient to maintain water homeostasis. Instead, they present with polyuria, polydipsia, and a normal sodium level and osmolality. Because their fluid intake is sufficient to maintain homeostasis, they are not dehydrated and do not have an elevated BUN or creatinine. In these patients, further testing is necessary to increase serum osmolality and then measure the plasma vasopressin level or the urinary response to an administered vasopressin agonist.

4. After a water deprivation test, a patient increases their urine osmolality from 350 to 375 mOsm/kg H2O following administration of desmopressin (7% increase). What is the most likely diagnosis? A. Primary polydipsia B. Nephrogenic diabetes insipidus C. Partial hypothalamic diabetes insipidus D. Gestational diabetes insipidus E. Osmoreceptor dysfunction Answer: A  Primary polydipsia. Adults with normal vasopressin secretion can concentrate their urine to greater than 800 mOsm/kg H2O and have less than a 10% increase in urine osmolality in response to administered desmopressin. Patients with complete central diabetes insipidus have minimal concentration of the urine with dehydration, and a marked increase in urine osmolality (usually > 50%) in response to administered desmopressin. Patients with nephrogenic diabetes insipidus usually have no increase in urine concentration in response to administered desmopressin, although in some cases of acquired nephrogenic diabetes insipidus, some increased urinary concentration (generally < 10%) can occur. In patients with partial central diabetes insipidus and patients with primary polydipsia, the urine is often somewhat concentrated in response to dehydration, but not to the maximum of a normal person. The chronically reduced level of vasopressin downregulates the synthesis of aquaporin-2 water channels, and the large urine volume, regardless of cause, washes out the medullary osmotic gradient that determines the maximal urine concentration. When desmopressin is administered, patients with partial central diabetes insipidus have a further increase (usually > 10% but < 50%) in urine osmolality, whereas most patients with primary polydipsia have no further increase (i.e., 10 mIU/L) in conjunction with a free T4 level below the lower limit of the reference range. In subclinical hypothyroidism, the TSH level is only modestly elevated; the free T4 level remains in the low-normal to normal range. Secondary or central hypothyroidism refers to deficient thyroid gland function that is the result of inadequate stimulation by TSH. This is due in turn to production of either insufficient or inactive TSH from a number of congenital or acquired pituitary and hypothalamic disorders (Chapter 224).

EPIDEMIOLOGY

Primary hypothyroidism is common, occurring in 5% of individuals. Mild hypothyroidism is present in as many as 15% of older adults. Hypothyroidism is more common in women. It is more prevalent among whites and Latin Americans. Secondary hypothyroidism is rare, representing less than 1% of cases.

PATHOBIOLOGY

Dietary iodine deficiency is a cause of primary hypothyroidism in regions where this micronutrient deficiency exists and is uncorrected by iodine supplementation. The most common cause of primary hypothyroidism in developed countries is autoimmune (or Hashimoto’s) thyroiditis, a condition in which defective immune tolerance leads to inflammatory destruction of thyroid tissue and impaired gland function.2 The condition is characterized by a lymphocytic infiltrate and fibrosis. Circulating antithyroid

CHAPTER 226  Thyroid  

antibodies directed against thyroid peroxidase and thyroglobulin are markers of the disease, but glandular inflammation is principally the result of altered T-cell-mediated immunity. There is a genetic predisposition to the condition, with linkage studies suggesting a polygenic basis. Patients with autoimmune thyroiditis may have other endocrine and nonendocrine autoimmune disorders. It may be a component of the type 2 polyglandular autoimmune syndrome associated with autoimmune adrenal insufficiency and type 1 diabetes mellitus. It is less commonly a component of the type 1 syndrome, which includes adrenal insufficiency, hypoparathyroidism, and chronic mucocutaneous candidiasis (Chapter 231). Other nonendocrine autoimmune conditions associated with autoimmune thyroiditis include atrophic gastritis, pernicious anemia, systemic sclerosis, Sjögren’s syndrome, celiac disease, and vitiligo. Individuals treated with the immunomodulatory agent interferon-α may develop autoimmune thyroiditis with transient or permanent hypothyroidism. Surgical resection of the thyroid gland predictably leads to hypothyroidism. Radioactive iodine therapy for treatment of hyperthyroidism commonly destroys sufficient thyroid tissue to cause postablative hypothyroidism. External beam radiation therapy for head and neck cancer can also cause thyroid gland failure. Exposure to pharmacologic and radiocontrast agents that contain large amounts of iodine (e.g., amiodarone, radiocontrast dyes, some expectorants, topical disinfectants) can disrupt thyroid hormone production. Lithium inhibits secretion of T4 and T3, leading to hypothyroidism in 10% of treated patients. Other pharmacologic agents reported to cause hypothyroidism include stavudine, thalidomide, lenalidomide, imatinib, sunitinib, sorafenib, motesanib, bexarotene, ipilimumab, and aminoglutethimide. There are a number of other rare causes of primary hypothyroidism (see Table 226-3). Congenital hypothyroidism can be due to agenesis or dysgenesis of the thyroid gland or to mutations in genes encoding the enzymes catalyzing thyroid hormone synthesis. Infiltrative disorders that can disrupt thyroid function include hemochromatosis, amyloidosis, systemic sclerosis, and invasive fibrous thyroiditis (also known as Riedel’s thyroiditis). The thyroid gland inflammation that occurs with subacute thyroiditis and painless (postpartum) thyroiditis causes transient hypothyroidism from which most patients recover. Consumptive hypothyroidism can occur in individuals with hemangiomas expressing the type 3 deiodinase, which converts T4 to biologically inactive reverse T3. Secondary or central hypothyroidism may be caused by a number of disorders that impair normal hypothalamic or pituitary control of the thyroid gland (Chapter 224). Infiltrative disorders affecting the hypothalamus that can interfere with TRH secretion include sarcoidosis, hemochromatosis, and histiocytosis. Masses that impinge on the pituitary stalk can impede TRH delivery through the hypophyseal portal system. Compression of thyrotrophic cells by pituitary adenomas and other masses in the sella turcica can inhibit synthesis and secretion of TSH. Surgery and radiation therapy to treat pituitary adenomas can destroy thyrotrophic cells, leading to secondary hypothyroidism that develops as a component of panhypopituitarism. Other disorders associated with secondary hypothyroidism include lymphocytic hypophysitis, pituitary metastases from primary malignant neoplasms,

apoplexy, infarction caused by hemorrhage at the time of delivery in women (also known as Sheehan syndrome [Chapter 224]), and head trauma.

CLINICAL MANIFESTATIONS

Symptoms and Signs

Symptoms of hypothyroidism include fatigue, lethargy, weight gain despite poor appetite, cold intolerance, hoarseness, constipation, weakness, myalgias, arthralgias, paresthesias, dry skin, and hair loss. Females may develop precocious puberty, menorrhagia, amenorrhea, and galactorrhea. Affected individuals may experience depressed mood with limited initiative and sociability. Cognitive deficits can range from mild lapses in memory to delirium, dementia, seizures, and coma. The nonspecific nature of most of these symptoms makes it difficult to determine which patients presenting with them have hypothyroidism rather than other causes. Furthermore, in most cases, hypothyroidism is insidious in onset, making its recognition difficult. Symptoms that are new, progressive, or present in combination are more likely to be due to hypothyroidism. The physical findings associated with hypothyroidism vary according to the age at onset and disease severity. Children may present with delayed linear growth despite weight gain, precocious or delayed puberty, and pseudohypertrophy of muscle. Adults can present with bradycardia, diastolic hypertension, and mild hypothermia. The skin may be coarse, dry, yellow, and cool to the touch as a result of peripheral vasoconstriction. Diffuse thinning of scalp hair accompanied by thinning of the lateral eyebrows may occur. The nails may become brittle. Examination of the chest may reveal distant heart sounds. The extremities may reveal diffuse nonpitting edema caused by the deposition of glycosaminoglycans. Neurologic examination may reveal slow, dysarthric speech and diffuse slowing of deep tendon reflexes with a marked delay in the terminal relaxation phase. Examination of the neck may reveal a range of findings. Healed cervical incisional scars in this region may indicate a history of surgical resection of thyroid tissue. In autoimmune thyroiditis, the thyroid gland may be normal in size, diffusely enlarged, or atrophic to the degree it may be difficult to palpate at all. It may be soft and smooth with a lobular texture, or firm and irregular with a variegated nodular texture.

Other Routine Test Abnormalities

Routine blood tests may reveal anemia (which is typically macrocytic), hyponatremia, hypoglycemia, and elevated creatine phosphokinase, prolactin, homocysteine, triglyceride, and total and LDL cholesterol levels. Electrocardiography may show sinus bradycardia with low voltage in the limb leads. Chest radiography may show a widened cardiac silhouette, and echocardiography may confirm a pericardial effusion.

DIAGNOSIS

Suspected primary hypothyroidism is confirmed by an elevated TSH level (Fig. 226-1). Established reference ranges for TSH levels typically extend from 0.5 to 4.5 mIU/L. However, the distribution of values within this range is skewed toward the lower half, such that the mean TSH level in adults is

Suspected Hypothyroidism

Check TSH and free T4

TSH

TSH

Normal / TSH

Normal TSH

Free T4

Normal free T4

Free T4

Normal free T4

Mild or subclinical hypothyroidism

• Central hypothyroidism • Nonthyroidal illness • Drug effect

Primary hypothyroidism • Thyroid hormone replacement targeted to maintain a normal TSH level

• Consider thyroid hormone replacement

1503

FIGURE 226-1.  Laboratory assessment of suspected hypothyroidism. TSH = thyroid-stimulating hormone.

Normal

1504

CHAPTER 226  Thyroid  

1.5 mIU/L. Measurement of the free T4 level confirms the diagnosis of primary hypothyroidism and characterizes its severity. A low free T4 level in conjunction with a persistently elevated TSH level represents overt primary hypothyroidism, whereas a low-normal free T4 level with an elevated TSH level is termed mild or subclinical primary hypothyroidism. Other uncommon causes of isolated TSH elevation should be considered in appropriate settings, including recovery from severe systemic illness, renal failure, and adrenal insufficiency. The underlying cause of primary hypothyroidism is usually clinically obvious, and laboratory testing is unnecessary in most cases. When confirmation is required (e.g., to convince a patient the condition is permanent), serum antithyroid antibodies may be assessed. Measurement of thyroid peroxidase antibody is a more sensitive test than thyroglobulin antibody for this purpose. However, 10% of patients with histologically documented autoimmune thyroiditis have no circulating antithyroid antibodies. When clinical findings such as the presence of a sellar mass, previous pituitary surgery or irradiation, or other pituitary axis hormone deficiencies suggest the possibility of secondary hypothyroidism, the TSH level cannot be relied on to provide an accurate index of thyroid function. In these settings, the serum free T4 level must be assessed, and a low or even low-normal free T4 level can confirm the diagnosis. The TSH level in patients with secondary hypothyroidism can be low, normal, or even modestly elevated.

TREATMENT  The goals of thyroid hormone replacement therapy are straightforward: to replace endogenous thyroid hormone production, to avoid iatrogenic thyrotoxicosis, and (rarely) to treat systemic complications of severe hypothyroidism. Levothyroxine sodium (hereafter thyroxine) is the hormonal preparation of choice.3 Thyroxine has a number of favorable pharmacokinetic characteristics. It is well absorbed, and its plasma protein binding gives it a 7-day half-life, permitting daily dosing. Thyroxine is physiologically deiodinated to the more biologically active T3 in peripheral tissues. However, thyroxine has a narrow therapeutic index, and doses differing by as little as 12% can have clinical consequences. Tablets of multiple dose strengths ranging from 25 to 300 µg are available. Regulatory standards ensure pharmaceutical equivalence in terms of mass of thyroxine, but bioavailability may differ by as much as 12% among different preparations. Consequently, adherence to a single thyroxine formulation is advisable. The optimal dose of thyroxine for replacement therapy is related to lean body weight, with most adults requiring a daily dose of approximately 1.8 µg/ kg. The dose requirement for elderly adults is typically lower (e.g., 1 µg/kg/day) because of slower metabolic clearance. Patients with postsurgical or postablative hypothyroidism usually require a higher daily dose than patients with autoimmune thyroiditis, who may have some residual gland function. Patients with coexisting malabsorptive disorders may require higher and variable doses. Certain medications, mineral supplements, and foods can interfere with thyroxine absorption, including ferrous sulfate, calcium carbonate, aluminum hydroxide, sucralfate, cholestyramine, and soy-containing foods (Table 226-4). Thyroxine doses should be separated from these substances by 8 hours or longer. Thyroxine dose requirements may increase as a result of accelerated metabolic clearance in several circumstances. Patients with nephrotic syndrome and other systemic illnesses that lead to rapid clearance of thyroid hormone require higher daily doses. Dose requirements increase by an average of 75% in most pregnant women as a result of placental deiodinative metabolism of thyroxine.4 Simultaneous treatment with phenytoin, phenobarbital, carbamazepine, or rifampin also typically accelerates thyroxine metabolism. Most adults without known or suspected coronary artery disease can be started on a full replacement dose of thyroxine. The initial dose can be calculated on the basis of the patient’s weight and age, rounding down to the nearest available dose strength. For patients with primary hypothyroidism, adequacy of thyroxine therapy can be assessed by TSH measurement 4 to 6 weeks after therapy is started. The target TSH level for most treated individuals should be the lower half of the reference range (i.e., 0.5 to 2.0 mIU/L). Once an adequate dose has been established, the TSH level should be checked annually. In patients with secondary hypothyroidism, the serum free T4 level should be monitored 2 to 4 weeks after the thyroxine dose is started or adjusted, with a target free T4 level in the upper half of the reference range.

Management of Complications

Complications of thyroxine therapy are limited to iatrogenic thyrotoxicosis and, rarely, adverse effects of restoring euthyroidism. Typical symptoms and signs of thyrotoxicosis usually accompany significant degrees of overtreatment. However, even a modestly excessive thyroxine dose can induce bone mineral loss, especially in postmenopausal women, and it can increase the risk of atrial fibrillation in older individuals. In patients with underlying coronary

TABLE 226-4  INTERFERENCE WITH THYROXINE REPLACEMENT THERAPY FACTORS CONTRIBUTING TO UNDERREPLACEMENT Inadequate prescribed dose Limited compliance Decreased absorption due to ingestion of agents that bind thyroxine Ferrous sulfate Calcium carbonate Aluminum hydroxide Sucralfate Cholestyramine Soy protein Increased metabolism of thyroxine Pregnancy Drugs Phenytoin Phenobarbital Carbamazepine Rifampin Diminishing residual thyroid function Changing formulations FACTORS CONTRIBUTING TO OVERREPLACEMENT Excessive prescribed dose Factitious ingestion of additional doses Decreased metabolism of thyroxine due to aging Increasing residual thyroid function Changing formulations

artery disease, the positive chronotropic and inotropic effects of thyroxine may exacerbate myocardial ischemia.5 Consequently, adults with known or suspected ischemic heart disease should be started on a low dose that is titrated upward in small increments once tolerance is ensured (e.g., starting with 25 µg daily, then increasing the dose by 12.5 to 25 µg every 4 to 6 weeks). In some cases, β-blocker therapy may need to be intensified to counter the induction of myocardial ischemia. However, deliberate suboptimal dosing of thyroxine should be avoided. If necessary, coronary revascularization may be required before euthyroidism can be fully restored. Coexisting adrenal insufficiency associated with hypopituitarism or the type 2 polyglandular autoimmune syndrome may be unmasked when cortisol clearance is accelerated by a return to the euthyroid state. Other adverse effects that infrequently occur with thyroxine therapy include transient hair loss, acute sympathomimetic symptoms that resolve with dose reduction and slow advancement, and pseudotumor cerebri in children. A minority of patients with thyroxine-treated hypothyroidism continue to report bothersome symptoms despite biochemical evidence of adequate thyroid hormone replacement. Several randomized clinical trials have shown that combinations of T3 and T4—in the form of desiccated thyroid or synthetic thyroid hormone preparations—are not superior to T4 alone. A1 

Subclinical and Mild Hypothyroidism

Whether individuals diagnosed with subclinical hypothyroidism (i.e., an elevated or high-normal TSH level with a free T4 level within the reference range) benefit from thyroxine therapy remains controversial.6 In practice, many providers opt for a trial of therapy in mildly hypothyroid patients who are symptomatic, have underlying hypercholesterolemia, or have a high likelihood of progressing to overt hypothyroidism. Predictors of progressive thyroid failure include age older than 65 years, TSH level higher than 10 mIU/L, and the presence of circulating thyroid autoantibodies, indicating underlying autoimmune thyroiditis.

Myxedema Coma

Severe hypothyroidism can culminate in myxedema coma, a life-threatening condition characterized by hypothermia, bradycardia, hypotension, altered mental status, and multisystem organ failure. Risk factors include advanced age, poor access to health care, and other underlying major organ system diseases. Most patients have severe and long-standing thyroid hormone deficiency. Treatment should include thyroxine (1.8 µg/kg/day, with or without a 500-µg loading dose). Some experts advocate coadministration of triiodothyronine in divided doses to compensate for impaired conversion of T4 to T3. No controlled trials have been performed to evaluate the relative benefits and risks of these different approaches. Glucocorticoids should be administered in stress doses after a cosyntropin stimulation test has been performed to check for evidence of concomitant adrenal insufficiency (Chapter 227). Care should be taken to avoid exposure to potent sedative or analgesic agents that may exacerbate altered mental status. Hypothermia should be treated with external warming to reduce the risk of circulatory collapse.

CHAPTER 226  Thyroid  

Nonthyroidal Illness

In patients with severe nonthyroidal illness, a characteristic constellation of thyroid function test changes occurs that often appears to be consistent with hypothyroidism (see Fig. 226-1).7 The T3 level usually declines as a result of decreased extrathyroidal T4-to-T3 conversion. With increasingly severe disease, total T4 and free T4 levels also decline. TSH levels are usually low to low-normal. During the course of recovery, the TSH level can rise above the upper limit of the normal range, producing a profile that can be mistaken for primary hypothyroidism. Clinical correlation is essential to assess thyroid function in severely ill patients (e.g., a history of preexisting thyroid or pituitary disease, the presence of a goiter, or features suggesting other elements of hypopituitarism). Because no benefit of thyroid hormone treatment has been shown for these patients, observation with retesting 6 to 8 weeks after recovery is the preferred approach.

  THYROTOXICOSIS DEFINITION AND EPIDEMIOLOGY

Thyrotoxicosis is a systemic syndrome caused by exposure to excessive thyroid hormone (Table 226-5). Its prevalence is 1 in 2000 adults, affecting 1% of all individuals during the course of their lifetime.

PATHOBIOLOGY

Thyrotoxicosis is the result of excessive circulating and tissue effects of thyroid hormone. Strictly speaking, hyperthyroidism refers to those forms of thyrotoxicosis that are caused by excessive production of thyroid hormone by the thyroid gland due to a thyrotropic stimulus or autonomous thyroid tissue function (see Table 226-5). In Graves disease, the most common cause of hyperthyroidism, the thyroid gland is stimulated by autoantibodies that bind to and activate the TSH receptor. Excessive secretion of TSH causes hyperthyroidism in patients with rare TSH-secreting pituitary adenomas (Chapter 224). CG, a glycoprotein with high TSH homology, can cause transient gestational hyperthyroidism during pregnancy, when a choriocarcinoma or a germ cell tumor produces variant forms of HCG that are more active or when mutant TSH receptors bind HCG more avidly, as occurs in familial gestational thyrotoxicosis. Autonomous production of thyroid hormone occurs when thyrocytes function independently of TSH receptor activation. This can occur as a result of growth of a benign functioning thyroid adenoma or growth of multiple autonomously functioning nodules forming a toxic multinodular goiter. In rare cases, it can occur when patients with well-differentiated thyroid cancer present with functioning metastases. In some toxic adenomas, somatic

TABLE 226-5  ETIOLOGIES OF THYROTOXICOSIS HYPERTHYROIDISM Antibody-mediated stimulation of thyroid tissue Graves disease Autonomously functioning thyroid tissue Toxic multinodular goiter Toxic adenoma Iodine exposure Autonomously functioning heterotopic thyroid tissue Struma ovarii Metastatic differentiated thyroid cancer Excessive secretion of TSH TSH-secreting pituitary adenoma NONHYPERTHYROID THYROTOXICOSIS Ingestion of exogenous thyroid hormone Pharmacologic Levothyroxine Liothyronine Combination preparations Nonpharmacologic Dietary supplements Improperly processed meat products Inflammation causing release of endogenous thyroid hormone Subacute thyroiditis Autoimmune thyroiditis TSH = thyroid-stimulating hormone.

1505

mutations in the TSH receptor gene lead to constitutive activation. In patients whose thyroid glands have the potential for autonomous function, exposure to excessive amounts of iodine in the form of amiodarone or iodinated contrast agents can provoke hyperthyroidism. Transient thyrotoxicosis can also be caused by inflammatory conditions that release an excessive amount of thyroid hormone stored in the gland (see the section on thyroiditis). These include subacute thyroiditis, which is believed to be caused by a viral infection; acute or suppurative thyroiditis, caused by bacterial infection; radiation-induced thyroiditis; and pharmacologic thyroiditis (e.g., due to amiodarone). Autoimmunity can also provoke an inflammatory thyroiditis that causes transient thyrotoxicosis. This commonly occurs in the setting of lymphocytic thyroiditis (also known as silent, painless, or postpartum thyroiditis). It rarely occurs in the setting of autoimmune thyroiditis (also known as Hashimoto’s thyroiditis). In rare cases, excess thyroid hormone can be secreted by ectopic thyroid tissue located anywhere from the base of the tongue to the mediastinum, or by heterotopic thyroid tissue that develops as part of an ovarian teratoma (a condition known as struma ovarii). Thyrotoxicosis can also be caused by ingestion of excessive amounts of thyroid hormone. This is most often the result of the prescription of excessive doses of pharmacologic preparations of thyroid hormone, but it can rarely be due to surreptitious or accidental ingestion.

CLINICAL MANIFESTATIONS

Symptoms and Signs

The classic symptoms of thyrotoxicosis include weight loss despite a hearty appetite, heat intolerance, palpitations, tremor, and hyperdefecation (increased frequency of formed bowel movements). Thyrotoxicosis can escape early detection because of its presentation with common nonspecific symptoms such as fatigue, insomnia, anxiety, irritability, weakness, atypical chest pain, or dyspnea on exertion. Delayed recognition may also occur when atypical symptoms such as headache, weight loss, periodic paralysis, or nausea and vomiting dominate the clinical picture. Elderly patients may present with apathetic thyrotoxicosis typified by weight loss and the absence of sympathomimetic symptoms and signs. Signs of thyrotoxicosis include resting tachycardia, systolic hypertension with a widened pulse pressure, warm moist skin with a velvety texture, onycholysis, and a staring gaze with lid lag (noted to be present when a rim of sclera is visible between the upper eyelid and the superior margin of the iris on downward gaze). Cardiac examination may reveal a prominent apical impulse and a systolic flow murmur. Neurologic findings may include a restless, impatient demeanor, pressured speech, proximal muscle weakness, distal hand tremor, and brisk deep-tendon reflexes. Clinical findings often provide clues to the underlying cause.8 In Graves disease, the gland is diffusely enlarged with a smooth or slightly lobulated contour, and may manifest an audible bruit or palpable thrill. Thyroid ophthalmopathy and dermopathy are also unique to Graves disease. In patients with toxic nodular goiter, one or more discrete nodules may be appreciated. In subacute thyroiditis, the gland is modestly enlarged, extremely tender, and firm. A history of recent pregnancy suggests possible painless thyroiditis. Recent exposure to amiodarone, other iodine-containing compounds, interferon-α, or pharmacologic preparations of thyroid hormone may suggest the characteristic forms of thyrotoxicosis associated with these agents.

Graves Disease

DEFINITION

Graves disease is an autoimmune disorder characterized by a variable combination of hyperthyroidism, ophthalmopathy (also known as thyroid eye disease), and dermopathy.

EPIDEMIOLOGY

Graves disease is more common among women, but it also affects men. It can develop at any time during life, but the onset most often occurs between 30 and 60 years of age.

PATHOBIOLOGY

The proximate cause of hyperthyroidism in Graves disease is the production of thyroid-stimulating immunoglobulins that bind to and activate the TSH receptor, promoting thyroid hormone secretion and gland growth. Thyrotropin (TSH) receptor antibodies of the stimulating variety are the hallmark of hyperthyroidism in Graves disease. Other thyroid autoantibodies commonly identified in the setting of Graves disease include thyroid peroxidase

1506

CHAPTER 226  Thyroid  

antibodies, thyroglobulin antibodies, and TSH receptor antibodies. Although the fundamental cause of Graves disease remains unknown, a genetic predisposition is implicated by a higher incidence in monozygotic twins and firstdegree relatives of affected individuals. Environmental factors implicated in triggering the onset of Graves disease include exposure to cigarette smoke, high dietary iodine intake, and perhaps stressful life events and certain antecedent infections.

CLINICAL MANIFESTATIONS

Affected individuals usually present with thyrotoxicosis and a thyroid gland that is diffusely enlarged with a rubbery consistency, smooth contour, definable pyramidal lobe, and audible bruit or palpable thrill due to increased blood flow. When it is clinically evident, thyroid eye disease usually presents within a few months of onset. In rare cases, it may develop long before, long after, or without any biochemical confirmation of hyperthyroidism.

PROGNOSIS

The hyperthyroidism associated with this condition often follows a persistent and progressive course, but one fourth of patients with Graves disease demonstrate spontaneous disease remission.

OPHTHALMOPATHY DEFINITION

Thyroid eye disease is a distinctive disorder characterized by inflammation and swelling of the extraocular muscles and orbital fat, eyelid retraction, periorbital edema, episcleral vascular injection, conjunctival swelling (chemosis), and proptosis (also called exophthalmos).9 Swelling of soft tissues within the confines of the orbits precipitated by fibroblast growth and inflammatory cell infiltrate can cause proptosis, entrapment of extraocular muscles, and compression of the optic nerve.

CLINICAL MANIFESTATIONS

Affected individuals typically complain of a change in eye appearance, ocular irritation, foreign body sensation, dryness, and ironically, excessive tearing. More severe involvement may cause exposure keratitis with corneal ulceration, diplopia, and blurred vision. On examination, patients may have a staring gaze, a rim of sclera visible between the upper eyelid and the superior margin of the iris during downward gaze (lid lag), signs of conjunctival inflammation, periorbital edema, and abnormalities of conjugate gaze, color vision, and visual acuity (Fig. 226-2). The precise degree of proptosis can be

measured with an exophthalmometer. Orbital imaging with CT scanning or ultrasonography can confirm the diagnosis, which must sometimes be distinguished from other causes of bilateral and unilateral proptosis.

TREATMENT  Treatment of mild thyroid eye disease focuses on protecting the cornea from exposure and desiccation with moisturizing drops and ointment, glasses, and sometimes taping the eyelids closed at bedtime. Selenium supplementation may help to relieve some of the symptoms associated with active inflammation in mild to moderate cases. A2  High-dose systemic glucocorticoid therapy can attenuate orbital inflammation in more severe cases. Orbital irradiation may be helpful in controlling inflammatory symptoms in some patients. Persistent corneal exposure, diplopia, altered vision due to optic nerve compression, and cosmetic issues may require surgery to decompress the orbits and readjust the extraocular muscles. Immunosuppressive agents and plasmapheresis have been used in severely affected patients, with anecdotal success.

DERMOPATHY

Infiltrative dermopathy, the least common aspect of Graves disease, is precipitated by the deposition of glycosaminoglycans in the dermis of the skin. Affected individuals usually present with mildly pruritic, orange peel–like thickening of the skin along the anterior aspects of the shins, known as pretibial myxedema. The dorsal aspects of the feet and fingers, the extensor surface of the elbows, and the face are more rarely affected. The diagnosis can be confirmed by skin biopsy. Treatment of early infiltrative dermopathy with topical glucocorticoids under an occlusive wrap may limit its progression. Treatments involving the use of intradermal or systemic glucocorticoids, long-acting somatostatin analogues, and even surgical resection of soft tissue have demonstrated limited success.

Toxic Adenoma A toxic adenoma is a solitary, autonomously functioning thyroid neoplasm that synthesizes and secretes excessive amounts of thyroid hormone independent of TSH stimulation. These neoplasms are almost always benign. Most grow large enough to be palpated by the time they present with thyrotoxicosis. Somatic gene mutations causing constitutive activation of the TSH receptor and the α-subunit of the stimulatory guanine nucleotide binding protein (Gsα) have been identified in a subset of toxic adenomas. Hyperthyroidism caused by a toxic adenoma does not remit spontaneously, except in unusual cases complicated by hemorrhagic infarction of the neoplasm.

Toxic Multinodular Goiter A toxic multinodular goiter is composed of multiple autonomously functioning thyroid nodules that synthesize and secrete excessive amounts of thyroid hormone. In some patients with nontoxic multinodular goiters, hyperthyroidism can be precipitated by exposure to excessive amounts of iodine. Most affected individuals have a goiter with multiple palpable thyroid nodules. Progressive enlargement may go undetected when there is substernal extension of nodular tissue. Toxic multinodular goiters are more common among older individuals.

A

B FIGURE 226-2.  Graves ophthalmopathy. A, A 59-year-old woman with excess proptosis, moderate eyelid edema, and erythema with moderate eyelid retraction affecting all four eyelids. Conjunctival chemosis (edema) and erythema with bilateral edema of the caruncles, with prolapse of the right caruncle, are evident. B, A 40-year-old woman with excess proptosis, minimal bilateral injection, and chemosis with slight erythema of the eyelids. On slit lamp examination, she also had evidence of moderate superior limbic keratoconjunctivitis. (From Bahn RS. Graves’ ophthalmopathy. N Engl J Med. 2010;362:726738. Copyright 2010, Massachusetts Medical Society. All rights reserved.)

TSH-Secreting Pituitary Adenoma TSH-secreting pituitary adenomas represent less than 1% of all functioning pituitary tumors (Chapter 224). Patients may present with typical clinical manifestations of thyrotoxicosis, a diffuse goiter, symptoms and signs precipitated by an expanding sellar mass, syndromes associated with co-secretion of other anterior pituitary hormones (growth hormone, prolactin, or adrenocorticotropic hormone), or symptoms and signs of hypopituitarism. The key to suspecting the condition is usually recognition of an inappropriately nonsuppressed TSH level in a patient with thyrotoxicosis. The diagnosis is confirmed in most cases when laboratory testing reveals an elevated circulating level of the pituitary glycoprotein α-subunit in conjunction with a radiographically definable sellar mass.

DIAGNOSIS

Laboratory Findings

Abnormalities detected in routinely ordered laboratory tests are often the first clues to the presence of thyrotoxicosis. Thyrotoxic patients may have hypercalcemia or hypercalciuria, increased alkaline phosphatase levels, modestly elevated transaminase levels, and low or declining total and LDL

CHAPTER 226  Thyroid  

1507

Suspected Thyrotoxicosis

Check TSH and free T4

TSH

TSH

Normal / TSH

Normal TSH

Free T4

Normal free T4

Free T4

Normal free T4

Thyrotoxicosis

• T3 thyrotoxicosis • Mild or subclinical thyrotoxicosis • Nonthyroidal illness

• TSH-secreting pituitary adenoma • Thyroid hormone resistance syndrome • Familial dysalbuminemic hyperthyroxinemia

Normal

FIGURE 226-3.  Laboratory assessment of suspected thyrotoxicosis. TSH = thyroid-stimulating hormone.

cholesterol levels. When they are measured, ferritin and angiotensinconverting enzyme levels are often increased. Electrocardiography typically reveals resting sinus tachycardia or atrial tachyarrhythmias, particularly atrial fibrillation with a rapid ventricular response. In severe cases, chest radiography may reveal cardiomegaly. In most patients with suspected thyrotoxicosis, the diagnosis can be confirmed by measurement of a TSH level (Fig. 226-3). Sensitive TSH immunoassays with a detection limit of less than 0.02 mIU/L can accurately discriminate between clearly suppressed TSH levels, characteristic of all common forms of thyrotoxicosis, and mildly suppressed levels that fall just beneath the reference range, as may occur in otherwise sick individuals. Only the rare conditions associated with TSH-mediated hyperthyroidism (TSH-secreting pituitary tumors and isolated pituitary resistance to thyroid hormone) lack TSH suppression when testing for thyrotoxicosis. Measurement of serum free T4 and T3 levels confirms the diagnosis of thyrotoxicosis, defines its severity, and occasionally provides a clue to its underlying cause. Overt thyrotoxicosis is characterized by free T4 or T3 levels above the upper limit of the reference range, whereas mild or subclinical thyrotoxicosis is characterized by a suppressed TSH level with free T4 and T3 levels within the normal reference range. When only the free T4 or T3 concentrations are elevated, the terms T4 toxicosis and T3 toxicosis are applied, respectively.

Differential Diagnosis

Once thyrotoxicosis is confirmed, it is important to define its underlying cause to determine the most appropriate course of treatment. The relative degrees of T4 and T3 elevation sometimes can be helpful. Predominantly T3 toxicosis is typical of Graves disease and can also occur with toxic nodular goiter. In contrast, predominantly T4 toxicosis is more typical of subacute or painless thyroiditis. T4 toxicosis is also more common in patients with iodineinduced hyperthyroidism. Other laboratory tests are sometimes helpful in differential diagnosis. Antithyrotropin-receptor antibodies are pathognomonic of Graves disease. Levels of antithyrotropin-receptor antibodies are especially high in thyroid dermopathy and correlate positively with the clinical features and prognosis of Graves ophthalmopathy. An elevated ESR is typically seen in subacute thyroiditis. Imaging studies can be helpful for the differential diagnosis. The fractional thyroidal uptake of radiotracer by the thyroid and its distribution in the gland on scintigraphic scanning often helps establish a definitive diagnosis (Table 226-6). Thyroid ultrasonography can confirm the presence of solitary or multiple thyroid nodules. Chest radiography and CT scanning may help delineate a substernal goiter.

TREATMENT  Selection of the most effective treatment for a specific condition causing thyrotoxicosis requires an understanding of the underlying pathophysiologic process and natural history. A3  For example, toxic multinodular goiter does not

TABLE 226-6  RADIOGRAPHIC EVALUATION OF SUSPECTED THYROTOXICOSIS

ETIOLOGY

FRACTIONAL 24-HOUR RADIOIODINE UPTAKE (%)

THYROID SCAN APPEARANCE

Graves disease

35-95

Diffuse increased homogeneous uptake; visible pyramidal lobe extending from isthmus

Toxic adenoma

20-60

Solitary focus of intense uptake; suppression of uptake in remainder of thyroid

Toxic multinodular goiter

20-60

Patchy heterogeneous foci of increased uptake interspersed with regions of diminished uptake

Subacute thyroiditis

0-2

Minimal to absent uptake

Autoimmune thyroiditis

0-2

Minimal to absent uptake; patchy heterogeneous uptake during recovery

Iodine-induced hyperthyroidism

0-2

Minimal to absent uptake

Exogenous thyroid hormone intoxication

0-2

Minimal to absent uptake

Metastatic differentiated thyroid cancer

0-5

Focal uptake in metastases

TSH-secreting pituitary adenoma

30-80

Diffuse increased homogeneous uptake

TSH = thyroid-stimulating hormone.

remit and requires definitive radioiodine treatment or surgery; subacute thyroiditis subsides spontaneously and requires only temporizing symptomatic therapy.

β-Blockers

β-Blockers help alleviate the sympathomimetic manifestations of thyrotoxicosis, regardless of the underlying cause. Palpitations, tremor, and anxiety can often be promptly controlled. However, other clinical features of thyrotoxicosis, including weight loss, heat intolerance, and fatigue, are not ameliorated by these agents. In thyrotoxic patients with marked sinus tachycardia or atrial fibrillation with a rapid ventricular response rate, β-blockers can be used as rate-controlling agents. Propranolol also partially inhibits extrathyroidal conversion of T4 to T3, which may be of added benefit in patients with severe thyrotoxicosis. Propranolol can be started at a dose of 20 to 40 mg every 8 hours and titrated upward to a maximal daily dose of 240 mg on the basis of symptom control. Sustained-release propranolol or longer-acting β-blockers, such as

1508

CHAPTER 226  Thyroid  

metoprolol and atenolol, can also be used. β-Blockers should be used with caution in thyrotoxic patients with a history of obstructive pulmonary disease, Raynaud’s phenomenon, or heart failure. Esmolol can be used when a shortacting parenteral agent is required for heart rate control in patients with thyrotoxic heart failure. For patients with transient forms of thyrotoxicosis (subacute thyroiditis, autoimmune thyroiditis, or exogenous thyroid hormone intoxication), a β-blocker may be the only treatment required. In patients with more sustained conditions, such as Graves disease or toxic nodular goiter, β-blockers provide prompt initial relief of symptoms while definitive treatment with antithyroid drugs, radioiodine, or surgery is implemented.

Antithyroid Drugs

The thionamides inhibit thyroid hormone biosynthesis by competitively inhibiting iodine organification and iodotyrosine coupling. These agents are used for the treatment of thyrotoxicosis caused by overproduction of thyroid hormones. Because the thionamides block only new thyroid hormone synthesis, glandular stores of preexisting thyroid hormone must be exhausted before they are fully effective. This may require 3 to 8 weeks in patients with Graves disease or toxic multinodular goiter. Although antithyroid drugs can provide long-term control of hyperthyroidism, they are most appropriately used when there is a possibility that the underlying condition will remit, as in Graves disease, or when thyrotoxicosis must be attenuated before radioiodine treatment or surgery. Two thionamide agents are currently available: methimazole and propylthiouracil. Methimazole can be taken as a single daily dose because of its longer half-life and higher effective intrathyroidal concentration. This can bolster patients’ adherence and drug effectiveness. Propylthiouracil also inhibits extrathyroidal conversion of T4 to T3, an effect that may be beneficial in patients with severe complicated thyrotoxicosis. Propylthiouracil is preferred for pregnant hyperthyroid women in the first trimester because methimazole has been rarely associated with the congenital anomalies of choanal atresia and cutis aplasia.10 However, the shorter half-life of propylthiouracil necessitates its administration three or four times daily. Furthermore, risk of severe hepatotoxicity associated with the use of propylthiouracil has prompted the recommendation that methimazole be the first-line antithyroid drug to treat hyperthyroidism in children and adults, including women after the first trimester of pregnancy. A4  For patients with mild to moderate hyperthyroidism, methimazole is usually started at a dose of 10 to 30 mg once daily and increased to as much as 90 mg daily. For patients with more severe hyperthyroidism, thyrotoxicosis complicated by cardiac disease, or concomitant pregnancy, propylthiouracil can be started at a dose of 50 to 200 mg every 6 to 8 hours. Methimazole can be given rectally if necessary. The anticipated duration of treatment depends on the underlying cause. In patients with toxic multinodular goiter, antithyroid drugs are generally used only to restore euthyroidism in anticipation of definitive therapy. An effective dose can be continued for 6 to 24 months in a patient with Graves disease, before it is tapered off to determine whether there has been a remission of the patient’s autoimmune thyroid disease. Patients most likely to respond are those who present with mild clinical and biochemical hyperthyroidism, a small thyroid gland, and no active ophthalmopathy. Patients treated with antithyroid drugs should have thyroid function tests checked every 3 to 12 weeks during dose titration to monitor for iatrogenic hypothyroidism. Common side effects include rash, pruritus, fever, and arthralgias, which affect 5% of thionamide-treated patients. Agranulocytosis and hepatitis are rare but potentially fatal adverse reactions to thionamide medications. Their presentations are relatively sudden in onset and unpredictable. Monitoring of leukocyte counts and liver function test results is not useful as a preventive measure. Patients who are prescribed antithyroid drugs should be cautioned about manifestations of these adverse reactions and should be instructed to discontinue treatment and seek medical attention if they develop a high fever, pharyngitis, jaundice, or abdominal pain.

Radioactive Iodine

The selective uptake and concentration of iodide in thyrocytes permits the use of radioactive iodine to treat hyperthyroidism.11 Once it is concentrated in the gland after oral administration, 131I destroys thyroid tissue and controls hyperthyroidism, usually within 1 to 2 months. The dose of 131I can be calculated on the basis of the fractional uptake of radioiodine, but the outcome of dosimetry is not superior to that achieved with the administration of empirical doses. Patients can be treated on an outpatient basis, with precautions taken to prevent exposure of others. Approximately three quarters of patients are cured with a single dose of radioiodine. The principal side effect of radioactive iodine therapy is postablative hypothyroidism, which develops in most individuals receiving treatment for Graves disease and in a lesser proportion of patients treated for toxic nodular goiter. Lifelong monitoring of thyroid function is required because patients develop postablative hypothyroidism at a rate of 3% per year. Another less common complication is a transient exacerbation of thyrotoxicosis, which occurs in one quarter of patients during the first month after treatment as a result of radiation thyroiditis. Long-term follow-up studies have shown that radioiodine-

treated patients with Graves disease do not have any greater risk of thyroid cancer or other malignant neoplasms. However, hyperthyroid children and adolescents treated with radioactive iodine are more likely to develop benign nodules. Among hyperthyroid women treated with radioiodine, the incidences of infertility, spontaneous abortion, and children with birth defects are not increased. Diagnostic or therapeutic radioactive iodine is contraindicated in women during pregnancy, and treated women should avoid pregnancy until euthyroidism has been confirmed 3 to 6 months after administration of a dose.

Other Drugs

Saturated solution of potassium iodide (SSKI) or Lugol’s solution transiently inhibit the synthesis and release of thyroid hormone from the gland. They may be used to accelerate recovery after radioactive iodine treatment, to prepare patients for thyroidectomy, and to augment other treatments used to control severe thyrotoxicosis (see later). Iodinated radiocontrast agents inhibit the release of thyroid hormone while blocking peripheral conversion of T4 to T3. Lithium carbonate also inhibits the release of thyroid hormone. Rarely, these agents are used in combination with thionamides to treat patients with severe thyrotoxicosis. They may also help provide temporary control of hyperthyroidism when severe allergies preclude the continued use of thionamides. Cholestyramine can be used to bind thyroid hormone in the gut to interrupt enterohepatic circulation in cases of suspected exogenous thyroid hormone intoxication.

Surgery

Surgery has a limited role because of its potential to injure the adjacent recurrent laryngeal nerves and parathyroid glands. Resection of a toxic adenoma by lobectomy is curative and often preserves sufficient normal thyroid tissue for euthyroidism to be maintained. Consequently, it is often recommended in younger individuals. Toxic multinodular goiters causing compressive symptoms or cosmetic disfigurement may be appropriately managed with surgical resection. Although surgery is seldom recommended in the United States for the treatment of hyperthyroid Graves disease, it may be appropriate when other modalities are contraindicated, such as when there has been an adverse reaction to an antithyroid drug in pregnancy, when a thyroid nodule is thought to be malignant, or when hyperparathyroidism also requires surgical intervention.

Specific Treatment Scenarios Pregnancy

Pregnant patients with hyperthyroidism present special challenges. Diagnosis requires a careful assessment of symptoms, especially heat intolerance, palpitations, and vomiting, which also occur during normal pregnancy. The serum total T4 level is elevated because of increased TBG, and the TSH level can be suppressed in the first trimester as a result of HCG-mediated thyroid stimulation. Diagnostic radionuclide imaging studies are contraindicated. After diagnostic confirmation, hyperthyroidism must be treated because it is associated with an increased risk of spontaneous abortion, premature labor, low birth weight, and toxemia. β-Blockers should be used only transiently to control severe symptoms. Propylthiouracil is the preferred thionamide for treatment of Graves disease during the first trimester of pregnancy, because it crosses the placenta less readily than methimazole and because methimazole has been rarely linked to congenital malformations (i.e., choanal atresia and cutis aplasia). However, owing to the risk of very rare but potentially fatal propylthiouracil-related hepatitis, methimazole is preferred after the first trimester. Because Graves disease often remits later in pregnancy, antithyroid drug dose requirements often decline as gestation progresses. Measurement of maternal thyroid-stimulating immunoglobulin levels can help predict the risk of an infant developing neonatal Graves disease.

Subclinical and Mild Hyperthyroidism

Patients with subclinical or mild hyperthyroidism (i.e., a suppressed serum TSH with normal free T4 and T3 levels) may have symptoms that justify treatment. In patients with a serum TSH level suppressed to less than 0.1 mIU/L, bone mineral loss can lead to osteoporosis, particularly in postmenopausal women. Atrial fibrillation occurs more commonly in mildly hyperthyroid patients aged 60 years and older with TSH suppression below normal. It is less clear, however, whether younger asymptomatic patients with modestly suppressed TSH levels (e.g., 0.1 to 0.5 mIU/L) require anything more than periodic monitoring.

Thyrotoxic Crisis

Thyrotoxic crisis, also known as thyroid storm, is a potentially life-threatening syndrome that is usually the end result of severe and sustained thyrotoxicosis. It can affect patients with other medical conditions that render them vulnerable to the cardiovascular, neuropsychiatric, and gastrointestinal effects of exposure to excessive amounts of thyroid hormone. Thyrotoxic crisis typically develops in the setting of inadequately treated Graves disease and may be precipitated by intercurrent illness, surgery, or treatment with radioactive iodine. Affected individuals present with fever, atrial tachyarrhythmias,

CHAPTER 226  Thyroid  

congestive heart failure, nausea and vomiting, diarrhea, and seizures. Mental status changes can include agitation, delirium, psychosis, and coma. Prompt recognition and treatment in a monitored setting are crucial. A multifaceted treatment regimen should incorporate antipyretics, β-blockers, thionamides, iodinated contrast agents, and glucocorticoids, as well as aggressive evaluation and management of underlying medical problems.

1509

PATHOBIOLOGY

This painless inflammation of the thyroid gland can cause transient thyrotoxicosis followed by transient or persistent hypothyroidism. Each of these phases of thyroid dysfunction typically lasts 2 to 8 weeks. This condition is believed to reflect transient autoimmunity.

DIAGNOSIS

  THYROIDITIS

Subacute (de Quervain’s) Thyroiditis

PATHOBIOLOGY

Transient thyrotoxicosis results from the uncontrolled release of thyroid hormone from the inflamed gland. After 2 to 8 weeks, when the supply of stored hormone is exhausted, thyrotoxicosis resolves spontaneously. Hypothyroidism ensues because the gland’s biosynthetic capabilities remain impaired. This is also transient (lasting ≈ 1 month), with subsequent restoration of normal thyroid function in most patients.

CLINICAL MANIFESTATIONS

Subacute thyroiditis is characterized by painful enlargement of the thyroid, systemic inflammatory symptoms, and transient thyrotoxicosis that is often followed by transient hypothyroidism. The histologic pattern shows inflammatory cell infiltrates that are believed to be the result of a viral infection. Many patients with subacute thyroiditis report antecedent upper respiratory infections. Patients usually present with pain localized to the thyroid or radiating to the throat, ears, or jaw. Constitutional symptoms, including fever, chills, sweats, and malaise, are often present. On occasion, these inflammatory features may dominate the presentation. Examination of the thyroid typically reveals an exquisitely tender, modestly enlarged, and woody, hard gland.

DIAGNOSIS

Differential Diagnosis

The differential diagnosis of thyroid pain must be considered in the evaluation of patients presenting with pain and tenderness localized to the lower anterior neck. In addition to subacute thyroiditis, potential causes of thyroid pain include acute (suppurative) thyroiditis, hemorrhage into an existing thyroid nodule, and rapid growth of anaplastic thyroid cancer, diffusely infiltrating thyroid cancer, or thyroid lymphoma.

Laboratory Findings

Laboratory testing in patients with subacute thyroiditis reveals a profile of overt thyrotoxicosis. Elevated T4 levels are usually proportionately higher than T3 levels. Patients typically have an elevated ESR during the acute phase. The fractional uptake of radioiodine is typically less than 2% at 24 hours (see Table 226-6).

TREATMENT  High-dose aspirin or naproxen sodium can be used to treat thyroid pain and systemic inflammatory symptoms. Patients who fail to respond may require glucocorticoid therapy, but it must be tapered over several weeks to prevent a relapse, prolonging the overall course of the illness. Symptoms ascribed to transient thyrotoxicosis may respond to treatment with a β-blocker continued for a limited course of 1 to 3 weeks. Patients who progress to symptomatic hypothyroidism may need short-term thyroxine replacement therapy, but most do not require long-term thyroid hormone replacement.

Lymphocytic (Postpartum, Painless, Silent) Thyroiditis

EPIDEMIOLOGY

Lymphocytic thyroiditis occurs most commonly in postpartum women, affecting as many as 6% of women 2 to 12 months after delivery or termination. Rarely, this condition occurs in non-postpartum women or in men. Predisposing factors include a history of previous episodes of postpartum thyroiditis, type 1 diabetes mellitus, and circulating antithyroid autoantibodies.

The diagnosis of lymphocytic thyroiditis is often overlooked when nonspecific symptoms of thyrotoxicosis (e.g., weight loss, insomnia, anxiety) or hypothyroidism (e.g., fatigue, depression) are misinterpreted as common postpartum complaints. The thyroid gland is nontender and either normal in size or modestly enlarged. Once it is considered, a diagnosis of lymphocytic thyroiditis can be readily confirmed or excluded by laboratory testing, which reveals a suppressed TSH level during phases of thyrotoxicosis and an elevated TSH level during phases of hypothyroidism. This condition must be distinguished from Graves disease, which can also present in the same time frame after delivery. Relative degrees of T4 and T3 elevation can sometimes provide a clue to which condition is present; lymphocytic thyroiditis is typically characterized by predominant increases in T4 levels. Fractional uptake of radioiodine is either absent or very low in the setting of lymphocytic thyroiditis, whereas it is increased in active Graves disease (see Table 226-6).

TREATMENT  Lymphocytic thyroiditis can often be managed with reassurance and observation alone. Symptomatic thyrotoxicosis can be treated with a course of β-blocker therapy. Overt hypothyroidism may require short-term thyroxine replacement.

PROGNOSIS

Most patients with lymphocytic thyroiditis eventually return to a euthyroid state, but 25% develop persistent hypothyroidism due to classic autoimmune thyroiditis.

Acute (Suppurative) Thyroiditis Infection of the thyroid gland is a rare condition that typically presents with severe thyroid pain, fever, and other systemic manifestations of infection. Bacterial infection of thyroid tissue can be the result of direct spread of grampositive or gram-negative pathogens through fistulas communicating with the piriform sinus or the skin. Hematogenous spread of bacterial, mycobacterial, fungal, or parasitic organisms, especially Pneumocystis carinii, can occur in immunocompromised individuals. On examination, affected patients are typically febrile, with asymmetrical swelling of a thyroid that is tender, warm, and fluctuant to firm in consistency beneath erythematous skin. Ultrasonography may reveal an abscess that can be aspirated to identify a pathogen. Patients with suppurative thyroiditis require prompt treatment with appropriate antibiotics. Surgical drainage of abscesses may be required.

Other Forms of Thyroiditis Certain drugs can cause thyroid gland inflammation. Amiodarone can produce a painless thyroiditis associated with thyrotoxicosis. Whenever possible, this should be distinguished from the iodine-induced form of thyrotoxicosis that can also be associated with amiodarone therapy. The former is optimally treated with glucocorticoid therapy, whereas the latter is managed with antithyroid drugs.12 Interferon-α can provoke a painless thyroiditis associated with transient thyrotoxicosis. This must be differentiated from interferon-α–induced Graves disease; the former is managed with β-blockers and the latter with antithyroid drugs. Riedel’s thyroiditis or struma is characterized by fibrotic replacement of the thyroid, with adherence and infiltration of adjacent structures that causes local compressive symptoms. In this idiopathic condition, the thyroid is substantially enlarged, hardened, and fixed. Affected patients may also develop mediastinal and retroperitoneal fibrosis, sclerosing cholangitis, or orbital pseudotumor. Diagnosis requires open biopsy. Surgical excision is difficult or impossible. Glucocorticoid therapy and tamoxifen therapy have been anecdotally reported to be effective.

1510

CHAPTER 226  Thyroid  

  GOITER DEFINITION

Goiters can be classified as diffuse or nodular, nontoxic or toxic (i.e., associated with thyroid hormone overproduction), and benign or malignant. Thyroid enlargement can be the result of thyrocyte proliferation stimulated by circulating factors (e.g., TSH and thyroid-stimulating autoantibodies), infiltration of the gland by inflammatory or malignant cells, or benign or malignant neoplastic changes within the gland itself. In a patient with a goiter, three clinical issues must be considered: enlargement causing local compressive or cosmetic concern, gland hyperfunction or hypofunction, and potential malignancy.

EPIDEMIOLOGY

Dietary iodine deficiency represents the most common cause of goiter worldwide. It is encountered in the United States only among immigrants from iodine-deficient regions. Younger patients present with diffuse or simple goiters that shrink in response to adequate iodine supplementation. In older individuals, iodine-deficient goiters become multinodular and do not decrease in size with iodine repletion. Excessive iodine exposure can provoke thyrotoxicosis in these patients.

PATHOBIOLOGY

Benign multinodular goiter or adenoma can be the result of genetic defects that lead to dyshormonogenesis, including mutations in the thyroglobulin, thyroid peroxidase, dual oxidase, and pendrin genes. Similarly, exposure to goitrogenic substances in foodstuffs, water, or drugs (e.g., lithium carbonate) that inhibit the normal steps in thyroid hormone synthesis can lead to goiter. In most patients, the underlying cause is unknown. Autoimmune thyroiditis typically produces a modest goiter as a result of glandular infiltration with lymphocytes, inflammatory changes in thyrocytes, and fibrosis. The hypothyroid state caused by autoimmune thyroiditis results in increased TSH, which further stimulates thyroid enlargement. Graves disease is also characterized by diffuse thyroid enlargement due to the action of thyroid-stimulating immunoglobulins. Other forms of thyroiditis can present with goitrous enlargement of the thyroid gland, including subacute, lymphocytic, and acute (suppurative) thyroiditis (see earlier sections). Malignant neoplasms that involve the gland diffusely, including thyroid lymphoma and infiltrative papillary, medullary, and anaplastic thyroid cancers, may present as rapidly enlarging goiters (see later sections). Affected patients often experience local pain and symptoms related to tumor expansion.

DIAGNOSIS

Laboratory Findings

A TSH level determines whether there is primary hypothyroidism or thyrotoxicosis. Elevated antithyroid peroxidase antibody titers can confirm suspected autoimmune thyroiditis. In asymptomatic patients with a modest diffuse goiter, no further evaluation may be indicated. Other blood tests (e.g., ESR for subacute thyroiditis or calcitonin for medullary thyroid cancer) can be useful when clinical clues suggest specific diagnoses.

Imaging

Cervical ultrasonography is the best imaging technique to define the character and extent of a goiter limited to the neck. It can help determine whether a goiter is diffuse or nodular, whether the thyroid is impinging on other cervical structures, and whether lymphadenopathy is present. Ultrasonography is also essential for guidance of fine-needle aspiration for cytologic differential diagnosis (see later). When a goiter extends posteriorly or beneath the sternal notch into the thorax, CT or MRI may be required. The administration of iodine-containing radiocontrast dye should generally be avoided in the evaluation of patients with goiters, because the stable iodide load may interfere with subsequent radioiodine imaging or therapy. Thyroid radionuclide uptake studies with 99mTc pertechnetate or 123I can help characterize the functional status of the gland. Radionuclide scanning can help determine the cause of a goiter and whether a superior mediastinal mass is thyroid tissue. Barium swallow radiographs with fixed-diameter markers and pulmonary function testing with flow-volume loops can help determine whether symptoms are directly related to compression of the esophagus or trachea, respectively. Laryngoscopy is useful to evaluate vocal cord function in patients with potential recurrent laryngeal nerve involvement.

TREATMENT  Once thyroid dysfunction and malignant disease have been excluded, asymptomatic patients with goiters can be observed with periodic clinical assessment. Ultrasonography can be relied on as a reproducible technique for monitoring the size of an enlarged thyroid gland. Thyroxine therapy to suppress TSH levels is effective in shrinking goiters in only a minority of patients. Furthermore, chronic thyroid hormone treatment carries the risks of symptomatic thyrotoxicosis, atrial fibrillation, and bone mineral loss. Patients with benign multinodular goiters causing local compressive symptoms or cosmetic concerns can be treated with surgery or radioactive iodine therapy. Surgery is often preferred when a patient has substantial gland enlargement causing compressive complications, especially when there is substernal extension of the goiter or acute obstructive symptoms. When surgery is contraindicated by the patient’s health status, radioactive iodine therapy has been shown to reduce goiter size by an average of 50% over 12 to 24 months.

Clinical Examination

The first step in evaluating a suspected goiter is to confirm whether neck swelling represents enlargement of the thyroid. Redundant skin and subcutaneous fat in the lower anterior neck can be mistaken for an enlarged thyroid. These findings can usually be distinguished from true thyroid enlargement by palpating a normal thyroid beneath the misleading soft tissue and by observing that the fullness does not rise and fall with deglutition. Ultrasonography may help resolve uncertainty. A patient’s history can provide important clues to the underlying cause. A childhood social history may confirm previous iodine deficiency. Symptoms of hypothyroidism may suggest autoimmune thyroiditis, whereas clinical evidence of thyrotoxicosis may suggest Graves disease or toxic multinodular goiter. Clinical findings may lead to recognition of one of the various forms of thyroiditis (e.g., pain in subacute thyroiditis or postpartum status in lymphocytic thyroiditis). Symptoms suggesting the invasion of adjacent structures may raise concerns about malignant disease or Riedel’s thyroiditis. On examination, diffuse enlargement favors one of the forms of thyroiditis, Graves disease, or a diffusely infiltrating malignant neoplasm. Nodular enlargement is more likely to reflect a benign multinodular goiter or malignant neoplasm. The precise size of the gland should be documented. Dysphonia, tracheal deviation, cervical lymphadenopathy, and venous engorgement in the neck should be noted. Subtotal obstruction of the thoracic outlet may be revealed by having the patient touch his or her hands together above the head (Pemberton’s maneuver) while checking for signs of facial plethora and cervical venous distention.

  THYROID NODULES EPIDEMIOLOGY

Thyroid nodules are common, being detected by palpation in 6% of women and 2% of men. Contemporary high-resolution ultrasonography identifies thyroid nodules in as many as 50% of all adults. Although the majority of these represent small, benign adenomatoid nodules or cysts, 5 to 10% of thyroid nodules are malignant. Less commonly, thyroid nodules are clinical problems by virtue of being hyperfunctioning or causing local compressive symptoms or cosmetic dissatisfaction.

DIAGNOSIS

Thyroid nodules can be noted by the patient or their physician in the absence of any other complaints.13 It is also common for thyroid nodules to be detected incidentally on imaging procedures, such as carotid ultrasonography and cervical spine CT or MRI. Symptoms of compression or invasion of adjacent tissues suggest that a nodule may be malignant. These include pain in the lower anterior neck, cough or dyspnea due to tracheal compression, hemoptysis due to tracheal invasion, dysphonia due to recurrent laryngeal nerve encasement, and dysphagia or odynophagia due to esophageal compression. Certain other symptoms and signs lead to the consideration of specific underlying conditions. A toxic adenoma should be suspected in a patient with a thyroid nodule and the classic clinical manifestations of thyrotoxicosis. Hypothyroid symptoms and signs suggest autoimmune thyroiditis

CHAPTER 226  Thyroid  

with asymmetrical thyroid enlargement. Hypercalcitoninemia associated with the metastatic spread of medullary thyroid cancer can cause pruritus, flushing, and diarrhea. The clinical assessment should also include symptoms and signs related to common sites of thyroid cancer metastasis, such as chest pain, dyspnea, bone pain, and neurologic findings. Thyroid nodules rarely can be due to metastasis from other primary malignant neoplasms including kidney, colon, and breast cancers.

History

A special predisposition to thyroid cancer is suggested by a personal history of therapeutic neck irradiation in childhood. Family history can be informative if relatives have had medullary or papillary thyroid cancers, which are familial in 50% and 10% of cases, respectively. The possibility of medullary thyroid cancer should also be considered when there is a personal or family history of clinical problems associated with multiple endocrine neoplasia type 2 (MEN 2) syndromes, including hyperparathyroidism and pheochromocytoma (Chapter 231).

Physical Examination

Physical examination of a thyroid nodule should seek to define its size, consistency, surface texture, mobility, and tenderness. The presence of malignant disease is suggested by fixation and ipsilateral regional adenopathy or vocal cord paresis. Multinodularity of the gland may reflect benign nodular goiter, but it is not sufficiently reassuring to dispense with further diagnostic testing.14 This is particularly true for a so-called dominant nodule that is larger, enlarging faster, or more symptomatic than others present in the thyroid.

Laboratory Findings

Routine laboratory testing includes measurement of TSH levels to identify patients with hyperthyroidism or hypothyroidism. When the TSH level is low or undetectable, the possibility of a benign autonomously functioning toxic adenoma can be pursued with radionuclide thyroid scanning (see Table 226-6). If an elevated TSH level indicates primary hypothyroidism, antithyroid peroxidase antibody titers can confirm whether the patient has autoimmune thyroiditis. Ultrasonography can distinguish asymmetrical enlargement caused by autoimmune thyroiditis from a discrete nodule. Calcitonin levels should be measured in patients with a known or suspected family history of MEN 2 or familial medullary thyroid cancer. Serum thyroglobulin measurement is not helpful in distinguishing benign from malignant thyroid abnormalities.

Imaging

Cervical ultrasonography helps confirm that a mass is within the thyroid, accurately defines its size, classifies it as cystic or solid, and determines whether additional nodules are present. Ultrasonography occasionally reveals other suspicious findings in nodules, such as fine calcifications, irregular nodule borders, and cervical adenopathy. Radionuclide scanning with radioiodine or technetium pertechnetate is helpful only in selected cases. In patients with a thyroid nodule and a suppressed TSH level, scanning can confirm that the nodule is hyperfunctioning or “hot,” in which case biopsy is usually not required.

Invasive Evaluation Fine-Needle Aspiration Biopsy

Fine-needle aspiration biopsy is the most accurate test to exclude or confirm malignant disease in patients with a nodule and a normal TSH level (Fig. 226-4). Most solid nodules and complex cysts larger than 1.0 to 1.5 cm in diameter should be sampled. Although aspiration can be directed by palpation alone when a nodule is readily definable, ultrasonography provides more certain guidance for the sampling of poorly localized lesions, often revealing additional nodules that should be assessed. The cytologic assessment of aspirated material must first confirm that there is adequate material for assessment (e.g., 6 clumps of 10 cells on 2 slides). Biopsies with inadequate specimens, which are more common in cystic lesions, must be repeated. Ultrasonographic guidance and on-site preliminary cytologic assessment can improve the yield of biopsy. In accordance with the Bethesda System for Reporting Thyroid Cytopathology, a sampled nodule can be categorized as benign, atypical, suspicious for a follicular neoplasm, suspicious for malignancy, or malignant (Table 226-7).15 Benign nodules typically yield samples containing clusters of normalappearing follicular epithelial cells with colloid. Pure colloid cysts may have

1511

scant epithelium. This classification is highly accurate, with a false-negative rate of less than 3% in sonographically directed biopsy specimens, and surgical resection is not required. In most cases, conservative observation based on yearly clinical or sonographic reassessment can be recommended. Further enlargement during observation (i.e., >20% increase in two of three dimensions) should prompt a repeat biopsy. Surgical resection should be considered if a cytologically benign nodule continues to grow, causing compressive symptoms or cosmetic disfigurement. Cytologic material classified as malignant typically contains abundant epithelial cells with atypical nuclear features, overlapping, and scant or absent colloid. This is also a highly reliable finding, with 98% of such lesions found to be thyroid cancers on subsequent resection. Consequently, bilateral thyroidectomy is indicated in patients without contraindications to operation. Samples that contain sparser quantities of epithelial cells with similar atypical nuclear features may be classified as suspicious for malignancy. Approximately 75% of nodules in this category represent thyroid cancers. One in five biopsies yields adequate but diagnostically indeterminate cytologic material.16 Specific findings that classify an indeterminate nodule as suspicious for a follicular neoplasm include abundant follicular or Hürthle cells in microfollicles with little or no colloid and minor degrees of nuclear atypia, potentially indicative of papillary cancer. Although the majority of such indeterminate nodules are benign follicular adenomas, 15 to 30% are thyroid carcinomas. Biopsy samples that reveal nuclear or architectural features considered to be abnormal but not clearly suspicious for malignancy or a follicular neoplasm are classified as demonstrating atypia of undetermined significance. Nodules that initially fall into this category have been estimated to harbor malignancy at rates ranging from 5 to 25%. Repeat sampling may provide a more specific diagnosis to guide further management in 75% of cases. Definitive determination of whether a suspicious or atypical nodule represents a focus of malignancy requires surgery targeted to remove either the lobe of the thyroid containing the nodule or the entire gland for surgical pathologic examination. Unilateral thyroid lobectomy has the advantage of a lower incidence of surgical complications and postoperative hypothyroidism when the lesion is benign, but it necessitates a subsequent completion thyroidectomy for most patients who prove to have cancer. Molecular diagnostic testing is available to reduce the number of surgeries performed in patients with cytologically indeterminate nodules, approximately 75% of which prove to be histopathologically benign. There are two general strategies: (1) testing aspirated material for oncogenic mutations associated with thyroid malignancies and (2) gene expression classifier microarrays designed to identify benign nodules. For a typical population of cytologically indeterminate nodule patients with a prevalence of thyroid cancer of 20 to 35%, the negative predictive value of oncogenic testing and gene expression classification have been shown to be approximately 85% and 95%, respectively. For patients with no clinical features of malignancy, particularly middle-aged or older women with multinodular glands in whom the prevalence of malignancy is 5% or less, vigilant observation with serial sonography is an alternative.

  THYROID CANCER

Cancers of the thyroid gland have a spectrum of behavior that ranges from incidentally detected and clinically inconsequential microcarcinomas to aggressive and virtually untreatable anaplastic malignant neoplasms. When thyroid cancer is diagnosed early, treatment is effective for most types. Most thyroid cancers present as thyroid nodules that are either asymptomatic or associated with local cervical symptoms or adenopathy. Less often, thyroid cancers first present with manifestations of metastatic disease, such as a pulmonary mass or bone pain.

Papillary and Follicular (Epithelial) Thyroid Carcinomas Papillary and follicular thyroid cancers arise from follicular epithelium and often retain responsiveness to TSH, produce thyroglobulin, and concentrate iodide. They are distinguished by their histopathologic appearances and characteristic patterns of progression. Hürthle cell carcinoma of the thyroid is composed of thyrocytes with abundant mitochondria-laden cytoplasm, and behaves like a follicular thyroid cancer, although it typically does not have iodine-concentrating ability.

EPIDEMIOLOGY

Approximately 60,000 new cases of thyroid cancer are diagnosed annually in the United States. Thyroid cancer is three times more common in women, in

1512

CHAPTER 226  Thyroid  

Thyroid nodule

Check TSH

TSH

Normal TSH

TSH

Check thyroid scan

Greater than 1.0–1.5 cm in maximal diameter

Toxic adenoma

Fine-needle aspiration biopsy

Check antithyroid antibody titers

Autoimmune thyroiditis Consider ultrasonography to distinguish between unilateral enlargement and a discrete nodule

Radioiodine Surgery

Thyroid hormone replacement

Benign No specific therapy Consider surgery if enlargement leads to compressive symptoms or cosmetic concerns

Papillary thyroid carcinoma Surgery

Malignant Suspicious for malignancy

Medullary thyroid carcinoma

Atypia of undetermined significance or follicular neoplasm Consider surgery for definitive diagnosis Molecular genetic testing Monitor size

Thyroid lymphoma External beam radiation Chemotherapy

Surgery

Nondiagnostic Unsatisfactory Atypia of undetermined significance Ultrasound-guided fine needle aspiration biopsy

Anaplastic thyroid carcinoma External beam radiation Chemotherapy

FIGURE 226-4.  Evaluation of a thyroid nodule. TSH = thyroid-stimulating hormone.

TABLE 226-7  BETHESDA SYSTEM FOR REPORTING CYTOPATHOLOGY CYTOLOGIC DIAGNOSIS Benign

RISK OF MALIGNANCY 0-3%

Atypia of undetermined significance

20-25%

Suspicious for a follicular neoplasm

15-30%

Suspicious for malignancy

60-77%

Malignant

97-99%

Diagnostic categories associated with risk of malignancy. Adapted from Bongiovanny M, Spitale A, Faquin WC, et al. The Bethesda System for Reporting Thyroid Cytopathology: a meta-analysis. Acta Cytol. 2012;56:333-339.

whom its incidence is currently rising faster than that of any other malignancy. There are estimated to be 450,000 U.S. thyroid cancer survivors who require lifelong follow-up for recurrence. Papillary thyroid carcinoma is the most common form of thyroid cancer, representing 90% of cases. The mean age at diagnosis is 45 years, but papillary thyroid carcinoma does occur in children and increases in incidence with age.

PATHOBIOLOGY

Irradiation of the thyroid gland in childhood is a risk factor, as evidenced by the epidemics of thyroid cancer that have followed both external beam radiation therapy for benign childhood conditions (e.g., tonsillitis and acne) and

radioiodine exposure after nuclear incidents. A substantial body of evidence now implicates RET/PTC and BRAF gene mutations that activate the MAP kinase signaling pathway in the pathogenesis and progression of papillary thyroid cancer. Most papillary thyroid carcinomas are slow growing and either remain confined to the gland or metastasize to cervical lymph nodes. Papillary microcarcinomas are a common incidental pathologic finding in 5% of thyroid glands excised for other reasons. However, papillary thyroid carcinomas can be more aggressive, with extension into adjacent tissues, extensive nodal involvement, and distant metastatic spread, most commonly to the lungs. Such aggressive behavior is, in general, more common in older patients. Follicular and Hürthle cell thyroid carcinomas account for 9% of all thyroid cancers. When these tumors show histologic evidence of invading only the tumor capsule, they are termed minimally invasive and generally behave like papillary thyroid carcinomas. However, follicular and Hürthle cell carcinomas with vascular invasion are more likely to be associated with distant metastatic disease, which most commonly involves the lungs and skeleton.

TREATMENT  Treatment of epithelial thyroid cancer entails surgery, often followed by radioiodine ablation of remnant thyroid tissue. A5  Total or near-total thyroidectomy with selective central compartment lymph node resection is usually the appropriate initial surgical procedure. Thyroid surgery can be complicated by hypoparathyroidism or recurrent laryngeal nerve injury, which causes hoarseness if it is unilateral and airway obstruction if it is bilateral. The rationale for bilateral surgery is the frequent presence of bilateral disease in papillary

CHAPTER 226  Thyroid  

thyroid cancer and the lower risk of recurrence after bilateral gland removal. In addition, there is greater accuracy in detecting residual disease after the eradication of all remaining normal thyroid tissue. A prospective cohort study of a national database of 30-day follow-up of patients undergoing thyroidectomy (for cancer or other indications) documented the increased risk of major pulmonary, cardiac, and infectious complications in the elderly. Elderly patients (65 to 79 years old) are twice as likely, and the most elderly (80 years or older) are 5 times as likely as young patients (16 to 64 years old) to have major systematic complications.17

Follow-up

131

Postoperatively, I administration after TSH stimulation can be employed to ablate the small amount of normal thyroid tissue that usually remains after surgery. This tissue, if it is not destroyed, leaves patients with circulating thyroglobulin and iodine-concentrating tissue on whole body scanning, decreasing the accuracy of these tests to identify residual disease. In controlled but nonrandomized trials, radioiodine has been associated with a lower rate of tumor recurrence in patients with advanced disease (stages 3 and 4) at presentation (see later), but there is no demonstrated clinical benefit of adjunctive radioiodine therapy for patients with lower stages of disease. TSH stimulation of residual thyroid tissue, which is essential for effective radioiodine therapy, can be accomplished either by the temporary withdrawal of thyroid hormone therapy to promote endogenous TSH production or by the administration of recombinant thyrotropin, which avoids the morbidity of hypothyroidism. Thyroxine therapy is appropriate for all patients with treated thyroid cancer, regardless of the extent of surgery and whether they received radioiodine ablative therapy. In addition to providing thyroid hormone replacement, thyroxine can be adjusted to suppress the patient’s circulating TSH level to the low or low-normal range to reduce the likelihood of tumor recurrence. In determining the extent to which the TSH level should be suppressed, the patient’s risk of cancer recurrence must be balanced against potential thyrotoxic complications such as bone mineral loss in postmenopausal women and atrial fibrillation in older patients. Long-term monitoring of patients entails periodic clinical assessment, measurement of serum thyroglobulin levels, radioiodine imaging in the early postoperative phase, and occasional use of ultrasonography. Clinically, patients should be assessed for local neck symptoms or recurrent cervical masses, as well as for optimization of thyroid hormone therapy. For patients with treated epithelial thyroid cancers, thyroglobulin is a more specific tumor marker if all remaining normal thyroid tissue has been ablated. For patients with undetectable thyroglobulin levels on TSH-suppressive thyroid hormone therapy, thyroglobulin measurement after recombinant TSH stimulation can sometimes reveal residual disease. Radioiodine scanning after TSH stimulation can be helpful in patients who have previously undergone radioiodine ablation, but once radioiodine imaging is negative, it offers little or no advantage over measurement of stimulated thyroglobulin levels. This is particularly true in recurrent papillary thyroid cancers, which often lose the ability to concentrate iodine. Unfortunately, thyroglobulin testing is impossible in the 20% of patients who have circulating thyroglobulin autoantibodies that interfere with thyroglobulin immunoassays. Because most epithelial thyroid cancer recurrences are in cervical nodes or soft tissues, ultrasonography is useful for postoperative monitoring, particularly in patients who presented with extensive cervical disease or who have persistently detectable serum thyroglobulin. CT scanning of the chest should be employed to detect intrathoracic disease in patients whose findings suggest recurrence outside the neck. In patients with substantial detectable thyroglobulin levels (>10 ng/mL) and negative findings on standard imaging studies, PET scanning can identify sites of residual disease in more than 50% of patients. Localization of recurrent cervical disease is usually an indication for comprehensive compartmental neck dissection. Distant and nonresectable metastases that are iodine avid, which occur more commonly in patients with invasive follicular thyroid cancer, can be treated with repeated doses of 131I. Symptomatic hilar node and bone metastases can be treated palliatively with external beam radiation therapy. Surgery can be employed for isolated metastatic disease sites. Conventional chemotherapy has limited efficacy in the treatment of differentiated thyroid cancer, but newer biologic agents targeting the molecular pathways involved in the pathogenesis of thyroid cancer hold promise.18 For example, the multikinase inhibitor sorafenib was shown in a phase 3 trial to double progression free survival to almost 11 months in patients with metastatic non–iodine avid epithelial thyroid cancers and shrink disease sites in 12% of patients. A6  Another multikinase inhibitor, vandetanib, is also effective against locally advanced or metastatic differentiated thyroid cancer. A7  However, multikinase inhibitors commonly have adverse effects, and because they are only tumoristatic must be used continuously.

PROGNOSIS

The TNM (tumor, node, metastasis) staging system is commonly used to stage epithelial thyroid cancers. In addition to tumor size, extent of node

1513

involvement, and presence of distant metastatic disease, the age of the patient at presentation is an important predictor of outcome. Patients younger than 45 years have a better prognosis than older individuals. The overall age-adjusted 10-year survival rates for patients with papillary and follicular thyroid cancer are 98% and 92%, respectively. However, disease recurrence is relatively common, occurring in approximately one third of patients with papillary thyroid cancer. Consequently, patients with treated thyroid cancer must be monitored for recurrent disease.

Medullary Thyroid Carcinoma Patients with medullary thyroid cancer (Chapter 246) typically present with a thyroid nodule, cervical adenopathy, distant disease, or symptoms of flushing, diarrhea, and pruritus when the circulating calcitonin level is markedly elevated. Features of the other elements of MEN 2a (e.g., hypertension) or MEN 2b (e.g., marfanoid habitus, submucosal neuromas) should be sought.

Anaplastic Thyroid Carcinoma Anaplastic thyroid carcinoma is a rare, histologically undifferentiated, clinically aggressive malignant neoplasm that typically arises in older patients, one fourth of whom present with evidence of a preceding differentiated thyroid cancer. Affected patients present with a rapidly enlarging mass in the anterior or lateral neck associated with pain, tenderness, and compressive symptoms including dysphagia, dysphonia, and stridorous dyspnea. Fineneedle aspiration biopsy of the mass usually yields large, pleomorphic, undifferentiated cells, but open surgical biopsy is sometimes required to confirm the diagnosis. Most cases are unresectable at presentation because of invasion of cervical structures. Surgery is not curative and should aim to secure the patient’s airway. A percutaneous gastrostomy tube is often placed to ensure adequate nutrition in the face of esophageal impingement. Conventional therapy consisting of combined external beam radiation therapy and chemotherapy with doxorubicin with or without cisplatin produces an initial response in 25% of patients. Rare patients with disease limited to the neck may have extended survival, but almost all patients relapse within a few months and succumb to their disease, with median survival ranging from 3 to 7 months. Current research is focused on the use of targeted antiangiogenic agents to treat unresponsive disease.

Thyroid Lymphoma Lymphoma rarely arises in the thyroid gland, typically presenting in older persons as a rapidly enlarging and painful diffuse goiter. Patients often have a preceding history of autoimmune thyroiditis. The diagnosis is further suspected when fine-needle aspiration biopsy yields abundant lymphocytes without other cellular features of autoimmune thyroiditis. Immunohistochemical staining and flow cytometry of sampled material can characterize a monoclonal lymphocyte population. Surgical biopsy is sometimes required to establish the diagnosis. In 50% of cases, lymphoma is primary to the thyroid gland, and it is usually an intermediate-grade non-Hodgkin’s–type lymphoma (Chapter 185). Surgical resection of the thyroid is usually not indicated, but elective tracheostomy may be required if tracheal compression is imminent. Most patients respond to treatment with combined external beam radiation therapy and chemotherapy. Disease-free survival rates vary with the disease stage at diagnosis and the initial response to combination therapy.

Grade A References A1. McDermott MT. Does combination T4 and T3 therapy make sense? Endocr Pract. 2012;18: 750-757. A2. Marcocci C, Kahaly GJ, Krassas GE, et al. Selenium and the course of mild Graves’ orbitopathy. N Engl J Med. 2011;364:1920-1931. A3. Bahn Chair RS, Burch HB, Cooper DS, et al. Hyperthyroidism and other causes of thyrotoxicosis: management guidelines of the American Thyroid Association and American Association of Clinical Endocrinologists. Thyroid. 2011;21:593-646. A4. Stagnaro-Green A, Abalovich M, Alexander E, et al. Guidelines of the American Thyroid Association for the diagnosis and management of thyroid disease during pregnancy and postpartum. Thyroid. 2011;21:1081-1125. A5. Smallridge RC, Ain KB, Asa SL, et al. American Thyroid Association guidelines for management of patients with anaplastic thyroid cancer. Thyroid. 2012;22:1104-1139. A6. Brose MS, Nutting CM, Jarzab B, et al. Sorafenib in radioactive iodine-refractory, locally advanced or metastatic differentiated thyroid cancer: a randomised, double-blind, phase 3 trial. Lancet. 2014; 384:319-328.

1514

CHAPTER 227  Adrenal Cortex  

A7. Leboulleux S, Bastholt L, Krause T, et  al. Vandetanib in locally advanced or metastatic differentiated thyroid cancer: a randomised, double-blind, phase 2 trial. Lancet Oncol. 2012;13: 897-905.

GENERAL REFERENCES For the General References and other additional features, please visit Expert Consult at https://expertconsult.inkling.com.

227  ADRENAL CORTEX LYNNETTE K. NIEMAN The adrenal glands weigh 6 to 8 g in adults (Fig. 227-1). Each contains a cortex, which makes steroid hormones, and a medulla, which produces catecholamines. Diseases of the adrenal medulla are discussed in Chapter 228. In the adrenal cortex, production of the three major classes of steroids occurs in specific zones: the outermost layer, the glomerulosa, produces mineralocorticoids, primarily aldosterone; the middle layer, the fasciculata, produces glucocorticoids, primarily cortisol; the innermost layer, the reticularis, produces adrenal “androgens,” primarily dehydroepiandrosterone (DHEA) and its sulfated conjugate (DHEA-S) (Fig. 227-2). This division reflects the fact that certain critical enzymes are restricted to specific zones, resulting in the ability or inability to synthesize specific end products.

  FUNCTION

The actions and regulation of these steroid classes differ. Mineralocorticoids act through the renal mineralocorticoid receptor to promote the reabsorption of sodium and the secretion of potassium. In addition to this classic action, mineralocorticoids have important action on the vasculature and may exacerbate the metabolic syndrome.1 Aldosterone secretion is stimulated primarily by hyperkalemia and angiotensin II (which itself is stimulated by hypovolemia and excess renin). These agents increase the production of aldosterone synthase to restore homeostasis through this feedback loop. Aldosterone production is stimulated to a much smaller degree by adrenocorticotropic hormone (ACTH). Cortisol and other glucocorticoids act through the glucocorticoid receptor type 2 and its isoforms. The actions of this class of steroids are much broader, including effects on carbohydrate handling, lipid and calcium metabolism, and the immune and nervous systems. Cortisol production is regulated primarily by ACTH, which is secreted in a circadian rhythm in response to corticotropin-releasing hormone (CRH) so that cortisol levels are highest in the morning and fall to a nadir around midnight. Cortisol coordinates ACTH production through negative feedback at the pituitary (ACTH) and hypothalamus (CRH). Vasopressin secretion also plays a role in stimulating ACTH release. DHEA and DHEA-S are the most abundant products of the adrenal gland. They exert their estrogenic and androgenic effects as prohormones, being converted to estrogens and testosterone in the peripheral tissues and activating the androgen and estrogen receptors. There is no known regulator of DHEA synthesis, but its production declines with age.

  DISORDERS OF ADRENAL FUNCTION

Most disorders of the adrenal cortex reflect overproduction or underproduction of the products of a single synthetic zone—cortisol, aldosterone, or testosterone or estrogen (Fig. 227-3). The congenital adrenal hyperplasias are an exception and are manifested with both overproduction and underproduction. Abnormal secretion is suggested by clinical features of each disorder and is reflected in plasma or urine levels of the relevant hormones or by the consequent increases or decreases in feedback systems, which form the basis of the biochemical diagnostic tests.

Glucocorticoid Excess: Cushing Syndrome

CLINICAL MANIFESTATIONS

FIGURE 227-1.  Magnetic resonance images of the abdomen showing the position and relative size of the normal adrenal glands.

Capsule Zona glomerulosa

Zona fasciculata

Zona reticularis Medulla

Cushing syndrome is a symptom complex that reflects excessive tissue exposure to cortisol. Classic features of Cushing syndrome include weight gain, plethora, hypertension, and striae (Table 227-1). Not all patients have all features; the number and severity of features correlate roughly with the duration and severity of hypercortisolism. Because many of the signs and symptoms are nonspecific, the diagnosis may be confused with psychiatric disorders, polycystic ovary syndrome, the metabolic syndrome, simple obesity, fibromyalgia, or acute illness. However, because worsening hyper­ cortisolism may precipitate hypertension, glucose intolerance, infections, psychiatric disturbances, impaired cognition, and hypercoagulability, it is important to identify this treatable disorder to prevent its associated morbidity and mortality.2 Changes in mood and cognition are useful markers of hypercortisolism. These include irritability, crying, and restlessness; depressed mood; decreased libido; insomnia; anxiety; and decreased concentration and impaired memory.

DIAGNOSIS

Clinical Examination

Cushing syndrome screening is most likely to be positive in the presence of signs that are typical of glucocorticoid excess, such as abnormal fat distribution in the supraclavicular and temporal fossae, proximal muscle weakness, wide (>1 cm) purple striae, and new irritability, decreased cognition, and decreased short-term memory. Testing is indicated when clinical features have progressed over time. For example, oligomenorrhea is more suggestive of Cushing syndrome if a woman previously had regular menses. Serial seven subtractions and recall of three cities (or objects) are useful bedside strategies to identify deficits in cognition and memory.

Laboratory Findings FIGURE 227-2.  Histologic section through a normal adult adrenal gland showing

the progression (from outside to inside) of the zona glomerulosa, zona fasciculata, zona reticularis, and medulla.

Exogenous administration of glucocorticoid should be excluded before screening for endogenous Cushing syndrome. In the absence of pseudoCushing states (see later), at least two different screening test results should

CHAPTER 226  Thyroid  

GENERAL REFERENCES 1. Bremner AP, Feddema P, Leedman PJ, et al. Age-related changes in thyroid function: a longitudinal study of a community-based cohort. J Clin Endocrinol Metab. 2012;97:1554-1562. 2. Caturegli P, De Remigis A, Rose NR. Hashimoto thyroiditis: clinical and diagnostic criteria. Autoimmun Rev. 2014;13:391-397. 3. Garber JR, Cobin RH, Gharib H, et al. Clinical practice guidelines for hypothyroidism in adults: cosponsored by the American Association of Clinical Endocrinologists and the American Thyroid Association. Thyroid. 2012;22:1200-1235. 4. Stagnaro-Green A, Pearce E. Thyroid disorders in pregnancy. Nat Rev Endocrinol. 2012;8:650-658. 5. Grais IM, Sowers JR. Thyroid and the heart. Am J Med. 2014;127:691-698. 6. Rugge JB, Bougatsos C, Chou R. Screening and treatment of thyroid dysfunction: an evidence review for the U.S. Preventive Services Task Force. Ann Intern Med. 2015;162:35-45. 7. Pappa TA, Vagenakis AG, Alevizaki M. The nonthyroidal illness syndrome in the non-critically ill patient. Eur J Clin Invest. 2011;41:212-220. 8. Vaidya B, Pearce SH. Diagnosis and management of thyrotoxicosis. BMJ. 2014;349:g5128. 9. Bahn RS. Graves’ ophthalmopathy. N Engl J Med. 2010;362:726-738.

1514.e1

10. Cooper DS, Laurberg P. Hyperthyroidism in pregnancy. Lancet Diabetes Endocrinol. 2013;1: 238-249. 11. Lee SL. Radioactive iodine therapy. Curr Opin Endocrinol Diabetes Obes. 2012;19:420-428. 12. Bogazzi F, Tomisti L, Bartalena L, et al. Amiodarone and the thyroid: a 2012 update. J Endocrinol Invest. 2012;35:340-348. 13. Niedziela M. Thyroid nodules. Best Pract Res Clin Endocrinol Metab. 2014;28:245-277. 14. Brito JP, Yarur AJ, Prokop LJ, et al. Prevalence of thyroid cancer in multinodular goiter versus single nodule: a systematic review and meta-analysis. Thyroid. 2013;23:449-455. 15. Bongiovanni M, Spitale A, Faquin WC, et al. The Bethesda System for Reporting Thyroid Cytopathology: a meta-analysis. Acta Cytol. 2012;56:333-339. 16. Alexander EK, Kennedy GC, Baloch ZW, et al. Preoperative diagnosis of benign thyroid nodules with indeterminate cytology. N Engl J Med. 2012;367:705-715. 17. Grogan RH, Mitmaker EJ, Hwang J, et al. A population-based prospective cohort study of complications after thyroidectomy in the elderly. J Clin Endocrinol Metab. 2012;97:1645-1653. 18. Marotta V, Sciammarella C, Vitale M, et al. The evolving field of kinase inhibitors in thyroid cancer. Crit Rev Oncol Hematol. 2015;93:60-73.

1514.e2

CHAPTER 226  Thyroid  

REVIEW QUESTIONS 1. A 74-year-old man with a history of hypertension and hypercholesterolemia is hospitalized after presenting with a 3-month history of progressive fatigue and dyspnea on exertion. His weight on admission is 164 lbs, his pulse is 44 bpm, and lab tests show CPK 528 U/L, TSH 65 mU/L, free T4 0.2 ng/dL, and T4 2.3 µg/dL. A pharmacologic nuclear stress test reveals findings consistent with diffuse myocardial ischemia. Subsequent coronary angiography reveals diffuse three-vessel disease that is not amenable to percutaneous stenting. A consulting cardiologist has recommended that he undergo coronary artery bypass surgery. What would you recommend? A. Start levothyroxine at a dose of 125 µg daily. B. Administer a 60-µg intravenous dose of levothyroxine daily. C. Check antithyroid peroxidase and antithyroglobulin antibodies. D. Start levothyroxine at a dose of 12.5 µg daily. Answer: D  Start levothyroxine at a dose of 12.5 µg daily. This patient requires treatment with levothyroxine, but it should be started at the lowest possible dose, with provisions to gradually increase it as tolerated in light of his coronary artery disease. A full replacement dose given orally or an adjusted dose given intravenously could exacerbate cardiac ischemia to the point of causing an infarction. Checking antithyroid peroxidase and antithyroglobulin antibodies would not provide any additional information. In the absence of any known history of thyroid surgery or external radiation treatment to the head and neck, it can be presumed that his severe hypothyroidism is due to autoimmune thyroiditis. 2. A 28-year-old woman with a 6-year history of hypothyroidism has been treated with levothyroxine at a dose of 125 µg daily. Lab tests checked 5 months ago right before she stopped taking cyclic oral contraceptives showed TSH 1.3 mU/L. Three weeks ago she checked a home pregnancy test that was positive. She had lab tests checked by her obstetrician that showed hemoglobin 10.2 g/dL, T4 12.5 µg/dL, and T3 185 ng/dL. She is estimated to be at 8 weeks’ gestation and has started taking prenatal vitamins and iron sulfate at a dose of 325 mg twice daily. What should you do? A. Have her continue levothyroxine at a dose of 125 µg daily, with instructions to take it regularly with other medications. B. Increase her dose of levothyroxine to 150 µg daily. C. Decrease her dose of levothyroxine to 112 µg daily. D. Check a thyroid uptake and scan. E. Check a TSH level. Answer: E  Check a TSH level. The only way to determine whether her current dose of levothyroxine is providing an adequate level of replacement is to check a TSH level. Total T4 and T3 levels measured while taking oral contraceptives or during pregnancy may be elevated or high-normal owing to increased production of thyroxine-binding globulin stimulated by increased estrogen levels. As such, they may not correlate with free thyroid hormone levels. Decreasing her dose of levothyroxine would be inappropriate because she may require a moderate to substantial increase in her dose during the course of a pregnancy. Empirically increasing a dose of levothyroxine may cause iatrogenic thyrotoxicosis characterized by symptoms that may be difficult to distinguish from normal physiologic changes of pregnancy. Exposure to radionuclide tracer is contraindicated during pregnancy, and in any event a thyroid uptake and scan would be unnecessary in a patient with known hypothyroidism. Doses of levothyroxine should always be separated from doses of iron sulfate by at least 4 hours to avoid interactions that can block absorption of both agents.

3. A 76-year-old woman presenting with a 1-week history of a cough, fever, and audible stridor is diagnosed with community-acquired pneumonia. A chest x-ray does not show evidence of an infiltrate but does reveal marked rightward tracheal deviation with a visible mediastinal soft tissue mass. A non-contrast chest computed tomography scan reveals multiple bilateral thyroid nodules, with an 8.5-cm left lower-pole nodule extending below the clavicle and sternum, with compression and narrowing of the trachea. Lab tests show TSH less than 0.001 mU/L, free T4 2.8 ng/dL, and T3 245 ng/dL. A thyroid uptake and scan reveals 24-hour uptake of 37%, with tracer accumulation localized to two right-sided thyroid nodules and the substernal left-sided thyroid nodule. What should you do next? A. Check pulmonary function tests with flow-volume loops. B. Prescribe a 12-month course of methimazole 10 mg daily. C. Refer the patient to a thyroid surgeon. D. Administer a 30-mCi dose of I-131. E. Start levothyroxine at a dose of 137 µg daily. Answer: C  Refer the patient to a thyroid surgeon. A multinodular goiter that has extended substernally to the point of causing tracheal compression should be resected by an experienced thyroid surgeon, irrespective of its functional status. The presence of audible stridor and evidence of tracheal narrowing on radiographic images obviates the need for pulmonary function testing with flow-volume loops. Treatment with methimazole might help control hyperthyroidism caused by autonomously functioning thyroid nodules but will not be a permanent solution and would not shrink the dominant nodule to any extent. Treatment with I-131 might help shrink the dominant nodule over time but will require 12 to 24 months to be effective. Treatment with levothyroxine to try to suppress further enlargement of thyroid tissue may be marginally effective at best in euthyroid patients and would be completely ineffective—and potentially dangerous—in an elderly woman presenting with hyperthyroidism. 4. A 33-year-old man is noted to have palpable enlargement of the right side of his thyroid on a routine physical exam. A thyroid ultrasound reveals a solitary 3.1-cm right-sided nodule with smooth borders. Lab tests show TSH 0.1 mU/L and T4 11.5 µg/dL. He reports a history of occasional symptomatic palpitations and weight loss of 5 lbs over the course of 3 months despite an increase in his appetite. He is not taking any medications and has not noted any problems with dysphagia or dysphonia. What should you do next? A. Perform a fine-needle aspiration biopsy of the right-sided nodule. B. Administer a 15-mCi dose of I-131. C. Refer the patient to a thyroid surgeon. D. Start methimazole at a dose of 5 mg daily. E. Perform a 123-I thyroid scan. Answer: E  Perform a 123-I thyroid scan. When a patient presenting with a thyroid nodule who is not taking levothyroxine is noted to have a suppressed TSH level, a radionuclide thyroid scan should be checked to determine if the nodule is an autonomously functioning toxic adenoma. If a nodule is “hot” on the scan, it does not need to be biopsied. If it is “cold” on the scan, with evidence of increased tracer uptake in surrounding tissue consistent with Graves disease, fine-needle aspiration biopsy should be performed to confirm it is benign. Treatment of a toxic adenoma or Graves disease with methimazole or radioactive iodine may be indicated but should only be considered after it has been determined whether a biopsy is necessary. Referral for thyroid surgery would only be indicated if a biopsy of a cold nodule revealed suspicious or malignant cytopathology, or if there were contraindications to treatment of a toxic adenoma with radioactive iodine or methimazole.

CHAPTER 226  Thyroid  

5. A 55-year-old woman is noted to be tachycardic with a resting pulse of 104 during a routine physical exam. Lab tests checked to evaluate this show TSH 0.002 mU/L, with follow-up lab tests showing free T4 2.3 ng/ dL and T3 289 ng/dL. She denies any history of anterior neck discomfort, weight loss, palpitations, anxiety, tremor, heat intolerance, or insomnia. Physical examination reveals tachycardia with a regular rhythm, a slightly enlarged thyroid without any discrete nodularity, and no evidence of proptosis or ocular irritation. She does have a history of osteopenia, with a lumbar spine T-score of −2.3 identified on a DEXA scan checked soon after the onset of menopause. What should you do next? A. Check antithyroid peroxidase and antithyroglobulin antibodies. B. Check a radionuclide thyroid uptake and scan. C. Start methimazole at a dose of 10 mg daily. D. Refer the patient to a thyroid surgeon. E. Check an ESR.

1514.e3

Answer: B  Check a radionuclide thyroid uptake and scan. This patient is presenting with thyrotoxicosis without any referable symptoms or clinical findings suggestive of a specific cause. Checking a thyroid uptake would help distinguish between a high-uptake state caused by hyperthyroidism driven by increased production of thyroid hormone, and a low-uptake state caused by inflammation with leakage of stored thyroid hormone. If a high-uptake state is identified, a scan will help distinguish whether hyperthyroidism is caused by Graves disease, a toxic adenoma, or a toxic multinodular goiter. Elevated antithyroid peroxidase and antithyroglobulin antibodies may identify underlying autoimmune thyroiditis but will not definitively determine the proximate cause of thyrotoxicosis. Treatment with methimazole or referral for thyroid surgery would only be considered after confirmation of a diagnosis. Checking an erythrocyte sedimentation rate (ESR) would not be informative, because subacute thyroiditis would be unlikely in the absence of localized discomfort.

1515

CHAPTER 227  Adrenal Cortex  



CRH

CRH

CRH

CRH

CRH

ACTH

Cortisol ACTH

Cortisol ACTH

+ –

ACTH

Cortisol ACTH

Cortisol

Cortisol

+

A

B

C

D

E

FIGURE 227-3.  Physiology of the adrenal axis in health, Cushing syndrome, and pseudo-Cushing states. A, In healthy individuals, cortisol production is stimulated by the increased hypothalamic release of corticotropin-releasing hormone (CRH), which then travels down the pituitary stalk to stimulate adrenocorticotropic hormone (ACTH) secretion and release by corticotropes. Circulating ACTH stimulates adrenal gland production and secretion of cortisol. Cortisol then functions in a negative feedback mechanism to inhibit both CRH and ACTH. B, In Cushing disease, a pituitary tumor releases excessive amounts of ACTH, which results in increased cortisol secretion by the adrenal glands. C, In ectopic ACTH secretion, a nonpituitary ACTH-secreting tumor releases excessive amounts of ACTH, which results in increased cortisol secretion by the adrenal glands. D, In ACTH-independent adrenal forms of Cushing syndrome, the adrenal tumor autonomously releases excess amounts of cortisol. In all forms of Cushing syndrome, the negative feedback effects of excessive cortisol inhibit endogenous CRH and ACTH secretion, so that circulating ACTH levels reflect the underlying tumor (levels are normal or increased) or independent cortisol production (levels are suppressed). E, In pseudo-Cushing states, central stimulation increases CRH secretion, which in turn increases ACTH and hence cortisol production. In this setting, the negative feedback effects of excessive cortisol inhibit endogenous CRH and ACTH secretion, so that cortisol levels are ultimately constrained, albeit at an increased level.

TABLE 227-1  THE FREQUENCY OF CLINICAL SIGNS AND SYMPTOMS OF CUSHING SYNDROME SIGN OR SYMPTOM Decreased libido in men and women

PERCENTAGE 100

Obesity or weight gain

97

Plethora

94

Round face

88

Menstrual changes

84

Hirsutism

81

Hypertension

74

Ecchymoses

62

Lethargy, depression

62

Striae

56

Weakness

56

Electrocardiographic changes or atherosclerosis

55

Dorsal fat pad

54

Edema

50

Abnormal glucose tolerance

50

Osteopenia or fracture

50

Headache

47

Backache

43

Recurrent infections

25

Abdominal pain

21

Acne

21

Female balding

13

be abnormal to establish the diagnosis. Tests for the differential diagnosis of Cushing syndrome should not be used to make the diagnosis. Figure 227-4 is the Endocrine Society’s recommended algorithm for testing of patients suspected of having Cushing syndrome.3

Urine, Saliva, and Serum Cortisol Measurements

Urine free cortisol (UFC) excretion during 24 hours is a good screening test. Specific, structurally based assay techniques, such as high-performance liquid chromatography and tandem mass spectrometry, are the “gold standard.” The upper-normal limit of these tests is much lower and more specific than that of antibody-based assays, in which other steroids may cross-react. This crossreactivity may be an advantage in screening for hypercortisolism. UFC excretion also may be increased in the so-called pseudo-Cushing states, including psychiatric disorders (depression, anxiety disorder, obsessivecompulsive disorder), chronic pain, severe exercise, alcoholism, uncontrolled

diabetes, and morbid obesity. Here, it is hypothesized that higher brain pathways stimulate CRH release and activation of the entire hypothalamicpituitary-adrenal axis (see Fig. 227-3E). Cortisol negative feedback inhibition on CRH and pituitary ACTH release restrains the resulting hypercortisoluria to less than four-fold greater than normal. Thus, Cushing syndrome cannot be diagnosed with certainty unless values reach this threshold. Conversely, patients with Cushing syndrome may have normal UFC excretion because of mild or intermittent hypercortisolism or altered renal metabolism of cortisol. If UFC is only mildly elevated and clinical features are minimal, it is best to treat any pseudo-Cushing state and to remeasure UFC excretion with the expectation that it will normalize. Alternatively, if UFC values are normal but clinical suspicion is high, repeated measurement might disclose intermittent hypercortisolism. Measurement of plasma cortisol at midnight distinguishes pseudo-Cushing states from Cushing syndrome with 95% diagnostic accuracy; a level greater than 7.5 µg/dL is required for the diagnosis of Cushing syndrome. Measurement of salivary cortisol at bedtime or at midnight works as well, is more convenient, and may be the best screening test in patients with mild or intermittent hypercortisolism.4,5 However, the criteria for its interpretation differ, so each assay must be validated before it is used for this purpose.

Dexamethasone Suppression Tests

The dexamethasone suppression test is a simple screening test that takes advantage of the negative feedback effect of glucocorticoids to reduce ACTH (and hence serum cortisol). Dexamethasone 1 mg is given orally between 11:00 pm and midnight, and plasma cortisol is measured between 8:00 and 9:00 the next morning. The test has an 8% false-negative rate in patients with Cushing disease and a 30% false-positive rate in chronic illness, obesity, psychiatric disorders, and normal individuals. As a result, Cushing syndrome cannot be diagnosed by this test alone unless the result is extremely abnormal. The 2-day, 2-mg dexamethasone suppression test discriminates patients with a pseudo-Cushing state if plasma cortisol end points of less than 1.4 or 2.2 µg/dL are used. Dexamethasone 500 µg is given orally every 6 hours for eight doses, and plasma cortisol is measured 2 hours after the last dose. The test has excellent sensitivity (90 to 100%) and specificity (97 to 100%) for discriminating Cushing syndrome, but it is costly and requires excellent compliance of the patient. The immediate subsequent administration of CRH (1 µg per kilogram of body weight intravenously) and the measurement of cortisol 15 minutes later increased the sensitivity and specificity to 100% in a small study of patients, with values above 1.4 µg/dL indicating Cushing syndrome. Although this combined dexamethasone-CRH test has high diagnostic accuracy, it has the same disadvantages as the 2-day dexamethasone suppression test and the added cost of CRH testing. Because of these drawbacks, these tests are usually reserved for patients with ambiguous or confusing results on other screening tests. CRH is available commercially (Acthrel), with Food and Drug Administration–approved labeling for the differential diagnosis of Cushing syndrome. Its use in the dexamethasone-CRH test is an off-label use.

1516

CHAPTER 227  Adrenal Cortex  

Cushing syndrome suspected (consider endocinologist consultation)

TABLE 227-2  ETIOLOGY OF CUSHING SYNDROME EXOGENOUS Most common cause of Cushing syndrome:    Glucocorticoid or ACTH driven    May be factitious or iatrogenic

Exclude exogenous glucocorticoid exposure Perform one of the following tests

24-h UFC (≥ 2 tests)

Overnight 1-mg DST

Late night salivary cortisol (≥ 2 tests)

Use 48-h, 2-mg DST in certain populations (see text)

ENDOGENOUS ACTH independent—autonomous adrenal activation (20% of all cases)    Adrenal adenoma (40-50%)    Adrenal carcinoma (40-50%)    Primary pigmented nodular adrenal disease    McCune-Albright syndrome     Massive macronodular adrenal disease     Gastric inhibitory polypeptide or food induced ACTH dependent—adrenal activation by excessive ACTH (80% of all cases)    Corticotrope adenoma (80%)    Ectopic ACTH secretion (20%)    Ectopic CRH secretion (rare)

ACTH = adrenocorticotropic hormone; CRH = corticotropin-releasing hormone.

Any abnormal result

Normal (CS unlikely)

Exclude physiologic causes of hypercortisolism Consult endocrinologist Perform 1 or 2 other studies shown above Suggest consider repeating the abnormal study Suggest Dex-CRH or midnight serum cortisol in certain populations

Discrepant (Suggest additional evaluation)

Abnormal

Normal (CS unlikely)

Cushing syndrome

FIGURE 227-4.  Algorithm for testing of patients suspected of having Cushing syndrome (CS). All statements are recommendations except for those prefaced by “suggest”. Diagnostic criteria that suggest Cushing syndrome are urine free cortisol (UFC) greater than the normal range for the assay, serum cortisol greater than 1.8 µg/dL (50 nmol/liter) after 1 mg dexamethasone (1-mg DST), and late-night salivary cortisol greater than 145 ng/dL (4 nmol/liter). Dex-CRH = dexamethasone–corticotropin-releasing hormone test; DST = dexamethasone suppression test. (Reprinted with permission from Nieman LK, Biller BM, Findling JW, et al. The diagnosis of Cushing syndrome: an Endocrine Society clinical practice guideline. J Clin Endocrinol Metab. 2008;93:1526-1540.)

Any dexamethasone test may give false results in patients with abnormal metabolic clearance of the drug. Agents that induce the cytochrome P-450 CYP3A4 enzymes (alcohol, rifampin, phenytoin, phenobarbital) increase dexamethasone clearance, whereas renal or hepatic failure decreases it. Measurement of a dexamethasone level can determine whether its clearance has been altered.

Differential Diagnosis

The causes of endogenous Cushing syndrome can be divided broadly into ACTH-dependent (80%) and ACTH-independent (20%) forms (Table 2272). Hypercortisolism from autonomously functioning adrenal tumors suppresses ACTH, whereas in primary disorders of ACTH excess, the adrenal glands respond to tumor-derived ACTH. Plasma ACTH concentration distinguishes between these causes. ACTH is usually less than 10 pg/mL in primary adrenal disorders but is also suppressed by exogenous steroids, whether they are prescribed intentionally (iatrogenic Cushing syndrome) or taken factitiously. Patients in the latter group often have had multiple surgical procedures and do not reveal that they are self-administering steroids. As a

result, patients must be queried closely about exogenous steroid administration, recognizing that parenteral, inhaled, and topical steroids can all cause glucocorticoid excess. Patients with endogenous Cushing syndrome and low ACTH concentrations should undergo adrenal imaging to identify the site of adrenal abnormality. Nonautonomous adrenal tissue atrophies when ACTH support is subnormal. Because of this, the common ACTH-independent forms of Cushing syndrome—adrenal adenoma and carcinoma—are manifested as a unilateral adrenal mass, with atrophy of the adjacent and contralateral tissue on magnetic resonance imaging or computed tomography. Bilateral forms of primary adrenal disease are rare and may be manifested with small or large adrenal nodules.6 Primary pigmented nodular adrenal disease occurs primarily in children and young adults and is characterized by small to normal-sized adrenal glands containing small (15 pg/mL; 3.3 pmol/L) is consistent with an ACTH-producing tumor.7 Intermediate ACTH concentrations between 5 and 15 pg/mL (1.1 to 3.3 pmol/L) in a two-site sandwich assay are not diagnostic. In these patients, suboptimal cortisol responses to CRH stimulation may identify the minority of cases of ACTH-independent Cushing syndrome with borderline basal ACTH values. In addition, a suppressed plasma DHEA-S value supports the diagnosis of an ACTHindependent disorder. Cushing disease,8 an ACTH-secreting pituitary adenoma, is the most common cause of Cushing syndrome. It is more common in women than in men (6 : 1 ratio), with a mean age at onset in the fourth decade. ACTH also may be secreted ectopically by a variety of neuroendocrine tumors, as shown in Table 227-3. Pituitary magnetic resonance imaging shows a tumor in only about 40 to 50% of patients with Cushing disease, but it is obtained routinely in patients with ACTH-dependent disease to exclude a macroadenoma or abnormal anatomy before petrosal sinus sampling or surgery. A pituitary lesion less than 6 mm is seen in up to 10% of healthy individuals and so does not always indicate Cushing disease. Biochemical tests must be used to distinguish among the ACTH-dependent causes of Cushing syndrome, and they must be performed after a 6- to 8-week period of sustained hypercortisolism sufficient to suppress normal corticotrope function. Inferior petrosal sinus sampling is the best test to distinguish between a pituitary and an ectopic source of excess ACTH; worldwide, the overall

CHAPTER 227  Adrenal Cortex  

TABLE 227-3  THE INCIDENCE AND TYPES OF TUMORS CAUSING THE SYNDROME OF ECTOPIC ACTH SECRETION TUMOR TYPE Carcinoma of lung (small cell or oat cell)

PERCENTAGE 19-50

Carcinoid of bronchus

2-37

Carcinoid of thymus

8-12

Pancreatic tumors, carcinoid and islet cell

4-12

Pheochromocytoma, neuroblastoma, ganglioma, paraganglioma

5-12

Medullary carcinoma of the thyroid

0-5

Miscellaneous*

20) of morning aldosterone to plasma renin activity (Fig. 227-5). One of four tests (usually salt loading) is used to confirm primary hyperaldosteronism by demonstrating a lack of aldosterone suppression.10

Differential Diagnosis

Having made the diagnosis of aldosterone-dependent mineralocorticoid excess, one must differentiate between the two most common adrenal causes—hyperplasia and adenoma—after excluding potential rare causes of hyperaldosteronism. Two rare autosomal dominant forms of familial hyperaldosteronism are type 1, a glucocorticoid-suppressible hyperaldosteronism, and type 2. Familial hyperaldosteronism type 1 is caused by a genetic swap of the promoter for CYP11B1 (11β-hydroxylase) with that of CYP11B2 (aldosterone synthase), forming a chimeric gene in which ACTH stimulates aldosterone synthase. It should be suspected in the setting of familial disease, particularly if there is a history of early-onset cardiovascular events, and is confirmed by gene testing (see http://www.brighamandwomens .org/Departments_and_Services/medicine/services/endocrine/Services/ gra/default.aspx). The genetic abnormality in familial hyperaldosteronism

1518

CHAPTER 227  Adrenal Cortex  

SUSPECTED PRIMARY ALDOSTERONISM (PA) (Hypertension at early age with hypokalemia or difficult to control)

TABLE 227-4  CAUSES OF MINERALOCORTICOID EXCESS PRIMARY HYPERALDOSTERONISM: HIGH ALDOSTERONE, LOW RENIN Aldosterone-producing adenomas (30-50%) Bilateral zona glomerulosa hyperplasia Familial hyperaldosteronism   Type 1: glucocorticoid-remediable hyperaldosteronism—this results from formation of a chimeric gene containing the regulator portion of 11β-hydroxylase (normally regulated by ACTH) and the synthetic region of aldosterone synthase; as a result, ACTH stimulates aldosterone synthase and hence aldosterone production   Type 2: adrenal adenomas or hyperplasia expressed in a familial pattern   Type 3: caused by mutant KCNJ5, often younger and more severe than Type 2 Aldosterone-producing adrenal carcinoma Ectopic aldosterone secretion (rare): kidney, ovary

Morning plasma aldosterone : renin ratio > 20 : 1

No

Yes

PA unlikely

Any of 4 confirmatory tests: • oral Na+ loading or • saline infusion or • fludrocortisone suppression or • captopril challenge

SECONDARY HYPERALDOSTERONISM: HIGH ALDOSTERONE, HIGH RENIN

Positive

Renovascular hypertension and aortic stenosis Diuretic use Renin-secreting tumors Severe cardiac failure APPARENT MINERALOCORTICOID EXCESS: LOW ALDOSTERONE, LOW RENIN Licorice ingestion: licorice (candy or flavored tobacco) containing glycyrrhetinic acid (or similar compounds such as carbenoxolone) inhibits renal 11β-hydroxysteroid dehydrogenase type 2, reducing cortisol conversion to cortisone and enabling cortisol to act as an endogenous mineralocorticoid Severe hypercortisolism: similar in mechanism to licorice ingestion; very high cortisol levels are thought to overwhelm the ability of 11β-hydroxysteroid dehydrogenase type 2 to convert cortisol to cortisone in the kidney; cortisol itself then acts as a potent mineralocorticoid Liddle’s syndrome: mutation of the β or γ subunit of the collecting tubule sodium channel leads to a constitutive increase in sodium reabsorption and potassium excretion 11β-Hydroxylase deficiency form of congenital adrenal hyperplasia: 11-deoxycortisol accumulates because of an inability to convert it to cortisol 17-Hydroxylase deficiency form of congenital adrenal hyperplasia: deoxycorticosterone and corticosterone are increased

type 2 is not known; its clinical presentation is similar to sporadic hyperaldosteronism. Recent studies demonstrate rare germline mutations of a potassium channel in familial hyperalsosteronism, and somatic adrenal mutations in about 40% of patients. For the more common conditions, adrenal computed tomography scans may show nonfunctioning nodules and falsely suggest an adenoma.11 The responses to physiologic maneuvers, such as upright posture, and salt loading with oral or intravenous sodium tend to be preserved in patients with hyperplasia, but there is significant overlap among groups of patients. The best diagnostic test involves the measurement of cortisol and aldosterone in bilateral adrenal venous effluent and a peripheral vein before and during an ACTH infusion. Cortisol is used to evaluate catheter placement in the adrenal veins, as levels from the two sides should be similar. When an adenoma is present, the aldosterone-to-cortisol ratio on one side is usually at least five-fold greater than the other, which may be similar to the periphery, indicating suppression. Bilateral hyperplasia tends to produce similar values on each side.

Negative

Adrenal CT

Surgical treatment possible

Medical treatment

Adrenal venous sampling

Mineralocorticoid receptor antagonist

Unilateral

Bilateral

Laparoscopic adrenalectomy FIGURE 227-5.  Algorithm for the detection, confirmation, subtype testing, and treatment of primary aldosteronism (PA). We recommend the case detection of PA in patient groups with relatively high prevalence of PA. These include patients with moderate, severe, or resistant hypertension; spontaneous or diuretic-induced hypokalemia; hypertension with adrenal incidentaloma; or a family history of early-onset hypertension or cerebrovascular accident at a young age (20 HU) E. Rapid contrast washout with more than 50% washout at 10 minutes Answer: D  See section on diagnosis. Imaging characteristics of an incidentally discovered adrenal mass consistent with pheochromocytoma include increased baseline CT Hounsfield unit density (e.g., >20 HU); marked enhancement with intravenous contrast medium on CT; high signal intensity on T2-weighted MRI; cystic and hemorrhagic changes; bilaterality; and large size (>4 cm). In addition, pheochromocytomas are characterized by slow contrast washout (e.g., 50% from untreated baseline); all others would be considered for moderateintensity treatment (lowering of LDL cholesterol 30 to 1000 mg/dL) is indicated to prevent acute pancreatitis.

ANTIPLATELET THERAPY

Prophylactic aspirin therapy is widely used for prevention of cardiovascular events in high-risk patients (i.e., those with prior myocardial infarction or stroke), with reported risk reductions of about 12% (Chapter 38). Results from clinical trials in patients with diabetes suggest that aspirin may be somewhat less effective for CVD prevention than in patients without diabetes, although this has not been a consistent finding. Current guidelines recommend aspirin therapy for diabetic patients with a prior CVD event (secondary prevention) or with increased CVD risk (10-year risk of >10%). This includes most men older than 50 years or women older than 60 years who also have one or more additional CVD risk factors: smoking, hypertension, albuminuria, dyslipidemia, or family history of CVD. For patients at lower CVD risk, the potential adverse effects from bleeding may outweigh the potential benefits, and routine use is not recommended. The optimal dose (balancing thrombosis prevention with the risk of bleeding) of aspirin has not been established and may differ according to patient characteristics, but 75 to 162 mg/day is commonly recommended. For high-risk patients who are unable to tolerate aspirin, clopidogrel is an effective alternative.

TREATMENT OF ESTABLISHED CARDIOVASCULAR DISEASE IN DIABETES  In general, treatment of clinically established CVD, including acute coronary syndromes and stable angina, is similar in diabetic and nondiabetic patients. There is some evidence that ischemic symptoms may be less intense, atypical, or absent in diabetic patients, leading to higher rates of “silent” myocardial infarction. However, a strategy of screening for ischemic heart disease, by exercise stress testing, in asymptomatic patients did not result in lower event rates or improved outcomes. Therefore, current recommendations are for coronary artery disease screening in patients with symptoms suggestive of ischemia. The role of intravenous insulin (with or without potassium and glucose infusion) in the setting of acute myocardial infarction has been considered in a few studies. In the Diabetes and Insulin-Glucose Infusion in Acute Myocardial Infarction (DIGAMI) study, acute myocardial infarction patients with diabetes were treated with standard therapy or with insulin infusion during the first 48 hours, followed by continued insulin use after hospital discharge. Mortality after 1 year was reduced by 30% in the insulin-treated group. However, the implications of these results have been debated because factors other than insulin treatment differed between the two groups (i.e., sulfonylureas were routinely used in the standard therapy group but withdrawn from the insulin group). These findings subsequently were not confirmed in a follow-up study, and this approach has largely been abandoned.

1548

CHAPTER 230  Hypoglycemia and Pancreatic Islet Cell Disorders  

Several studies have addressed the roles of medical therapy and revascularization in diabetic patients with coronary artery disease. Among them, the Bypass Angioplasty Revascularization Investigation 2 Diabetes (BARI 2D) study demonstrated that a policy of medical management (including aggressive risk factor modification) was as effective as early revascularization in diabetic patients with stable angina. In the Future Revascularization Evaluation in Patients with Diabetes Mellitus: Optimal Management of Multivessel Disease (FREEDOM) trial, diabetic patients with multivessel coronary disease had better outcome (reduced rates of death from any cause or nonfatal myocardial infarction) with coronary bypass surgery compared with percutaneous intervention with drug-eluting stents, although strokes were more frequent in the surgical group.

Grade A References A1. Misso ML, Egberts KJ, Page M, et al. Continuous subcutaneous insulin infusion (CSII) versus multiple insulin injections for type 1 diabetes mellitus. Cochrane Databse Syst Rev. 2010;1: CD005103. A2. Hemmingsen B, Lunc S, Gluud C, et al. Targeting intensive glycaemic control versus targeting conventional glycaemic control for type 2 diabetes mellitus. Cochrane Database Syst Rev. 2013;11:CD008143. A3. Schauer PR, Bhatt DL, Kirwan JP, et al. Bariatric surgery versus intensive medical therapy for diabetes—3 year outcomes. N Engl J Med. 2014;370:2002-2013. A4. Reznik Y, Cohen O, Aronson R, et al. Insulin pump treatment compared with multiple daily injections for treatment of type 2 diabetes (OpT2mise): a randomised open-label controlled trial. Lancet. 2014;384:1265-1272. A5. Finfer S, Liu B, Chittock DR, et al. The NICE-SUGAR Study Investigators. Hypoglycemia and risk of death in critically ill patients. N Engl J Med. 2012;367:1108-1118. A6. Diabetes Prevention Program Research Group. 10-year follow-up of diabetes incidence and weight loss in the Diabetes Prevention Program Outcomes Study. Lancet. 2009;374:1677-1686. A7. Gaede P, Lund-Andersen H, Parving HH, et al. Effect of a multifactorial intervention on mortality in type 2 diabetes. N Engl J Med. 2008;358:580-591. A8. Nguyen Q, Brown D, Marcus D, et al. Ranibizumab for diabetic macular edema: results from 2 phase III randomized trials: RISE and RIDE. Ophthalmology. 2012;119:789-801. A9. Keech AC, Mitchell P, Summanen PA, et al. Effect of fenofibrate on the need for laser treatment for diabetic retinopathy (FIELD study): a randomized controlled trial. Lancet. 2007;370: 1687-1697. A10. Fried L, Emanuele N, Zhang J, et al. Combined angiotensin inhibition for the treatment of diabetic nephropathy. N Engl J Med. 2013;369:1892-1903. A11. Wing RR, Bolin P, Brancati FL, et al. Cardiovascular effects of intensive lifestyle intervention in type 2 diabetes. N Engl J Med. 2013;369:145-154. A12. Zoungas S, Chalmers J, Neal B, et al. Follow-up of blood-pressure lowering and glucose control in type 2 diabetes. N Engl J Med. 2014;371:1392-1406.

GENERAL REFERENCES For the General References and other additional features, please visit Expert Consult at https://expertconsult.inkling.com.

230  HYPOGLYCEMIA AND PANCREATIC ISLET CELL DISORDERS KHALID HUSSAIN

to 3 days after birth, low plasma glucose concentrations are common in normal neonates, which makes it difficult to identify the minority who have a persistent hypoglycemia disorder or a genetic hypoglycemia disorder. Third, the importance of early recognition and treatment of such persistent hypoglycemia disorders in neonates is emphasized by reports that developmental handicap, which might have been avoidable by early recognition and treatment, occurs in 25 to 50% of cases with congenital hyperinsulinism.1 There is no absolute number that defines hypoglycemia in adults and in children. The current adult recommendations define clinical hypoglycemia as a plasma (or serum) glucose concentration low enough to cause symptoms and/or signs, including impairment of brain function. Because the clinical manifestations and symptoms of hypoglycemia are nonspecific, it is therefore not possible to state a single plasma glucose concentration that categorically defines hypoglycemia. The measured plasma or serum glucose concentration may be low owing to an artifact (e.g., when the blood sample is collected in a tube that does not contain an inhibitor of glycolysis and when separation of the plasma or serum from the formed elements is delayed). For these reasons, guidelines in adults emphasize the value of Whipple triad for confirming hypoglycemia: (1) symptoms and/or signs compatible with hypoglycemia, (2) a low measured plasma glucose concentration, and (3) resolution of symptoms and signs when glucose concentrations are raised. Because circulating fuels such as ketone bodies can be used by the brain, lower plasma glucose concentrations can occur in healthy individuals, particularly in women and children, without symptoms or signs during extended fasting. Therefore, for all of these reasons, it is not possible to state a single plasma glucose concentration that categorically defines hypoglycemia. The aim of this chapter is to outline the physiologic and biochemical changes associated with maintenance of a normal blood glucose level, describe the role of the counter-regulatory hormones, review the different hypoglycemia disorders observed in adults and children, and then finally discuss the various management strategies.

PATHOBIOLOGY

Physiologic and Biochemical Changes During Fasting and Feeding Overview

Plasma glucose concentration is tightly controlled by a balance between glucose production and utilization. Glucose is derived from three sources: (1) intestinal absorption that follows digestion of dietary carbohydrates; (2) glycogenolysis, the breakdown of glycogen, which is the polymerized storage form of glucose; and (3) gluconeogenesis, the formation of glucose from precursors including lactate (and pyruvate), amino acids (especially alanine and glutamine), and to a lesser extent, glycerol. Normally, there is tight coordination between rates of endogenous glucose influx into the circulation and glucose efflux out of the circulation into insulin-dependent tissues (skeletal muscle, adipose tissue, and liver). This coordination, despite periods of feeding and fasting, maintains the plasma glucose concentration in a relatively narrow range between 70 and 110 mg/dL (3.8 to 6 mmol/L). Figure 230-1 shows an outline of glucose physiology. Glucose is an obligate metabolic fuel for the brain under physiologic conditions. Unlike other body tissues, the brain cannot oxidize fatty acids, and neither can it synthesize/store glucose for later use. It is dependent on a continuous supply of glucose from the circulation. Given the vital importance of brain function and the above circumstances, it is not surprising that physiologic mechanisms have evolved for the maintenance of plasma glucose concentrations.

Changes During Fasting

DEFINITIONS

Hypoglycemia is one of the most common biochemical abnormalities observed in clinical practice. It is a biochemical finding and not a diagnosis. Hypoglycemic disorders are more common in neonates, infants and children as compared to adults. Inappropriately treated hypoglycemia can have severe consequences, including seizures, permanent brain injury, or death. This is especially the case in neonates with persistent forms of hypoglycemia, who are at high risk of brain injury from delays in diagnosis and effective therapy. Hypoglycemic disorders in neonates, infants, and children differ from adults in important aspects. First, they are most often due to congenital or genetic disorders, such as disorders of insulin secretion, as well as a range of metabolic and endocrine diseases. Second, during a transitional period of 1

During fasting, the basal rate of glucose output by the liver is precisely matched to glucose uptake by various body tissues. They average 2.2 mg/kg/ minute in healthy adults after an overnight fast. In infants, these rates are much higher (≈6 mg/kg/minute) because of their greater brain mass relative to their body weight. The brain is responsible for nearly two thirds of basal glucose utilization. The remaining one third is used by red blood cells, renal medulla, and to some extent muscle and fat. Hepatic glucose production results from a combination of glycogenolysis and gluconeogenesis. Endogenous glucose production is also contributed by gluconeogenesis in the kidneys. Breakdown of stored hepatic glycogen is a readily available source of free glucose. However, in an average adult, this process can only provide less than an 8-hour supply of free glucose. (In infants, this may provide only 4 hours of free glucose.) Considering this

CHAPTER 229  Diabetes Mellitus  

GENERAL REFERENCES 1. Atkinson MA, Eisenbarth GS, Michels AW. Type 1 diabetes. Lancet. 2014;383:69-82. 2. Keenan H, Sun JK, Levine J, et al. Residual insulin production and pancreatic B-cell turnover after 50 years of diabetes: Joslin Medalist Study. Diabetes. 2010;59:2846-2853. 3. Evert A, Boucher J, Cypress M, et al. Nutrition therapy recommendations for the management of adults with diabetes. Diabetes Care. 2014;37(suppl 1):S120-S143. 4. Skyler JS. Primary and secondary prevention of type 1 diabetes. Diabet Med. 2013;30:161-169. 5. Miller RG, Secrest AM, Sharma RK. Improvements in the life expectancy of type 1 diabetes: the Pittsburgh Epidemiology of Diabetes Complications study cohort. Diabetes. 2012;61:2987-2992. 6. Lind M, Svensson AM, Kosiborod M, et al. Glycemic control and excess mortality in type 1 diabetes. N Engl J Med. 2014;371:1972-1982. 7. Gregg EW, Li Y, Wang J, et al. Changes in diabetes-related complications in the United States, 19902010. N Engl J Med. 2014;370:1514-1523. 8. Kahn SE, Cooper ME, Del Prato S. Pathophysiology and treatment of type 2 diabetes: perspectives on the past, present, and future. Lancet. 2014;383:1068-1083. 9. Ng HJ, Gloyn AL. Bridging the gap between genetic associations and molecular mechanisms for type 2 diabetes. Curr Diab Rep. 2013;13:778-785. 10. Hales CN, Barker DJ. Type 2 (non-insulin-dependent) diabetes mellitus: the thrifty phenotype hypothesis. Int J Epidemiol. 2013;42:1215-1222. 11. Bray G, Edelstein S, Crandall JP, Diabetes Prevention Program Research Group, et al. Long-term safety, tolerability, and weight loss associated with metformin in the Diabetes Prevention Program Outcomes Study. Diabetes Care. 2012;35:731-737. 12. Qaseem A, Humphrey LL, Chou R, et al. Use of intensive insulin therapy for the management of glycemic control in hospitalized patients: a clinical practice guideline from the American College of Physicians. Ann Intern Med. 2011;154:260-267.

1548.e1

13. Seaquist ER, Anderson J, Childs B, et al. Hypoglycemia and diabetes: a report of a workgroup of the American Diabetes Association and the Endocrine Society. Diabetes Care. 2013;36: 1384-1395. 14. Nathan DM, Bayless M, Cleary P, et al. Diabetes control and complications trial/epidemiology of diabetes interventions and complications study at 30 years: advances and contributions. Diabetes. 2013;62:3976-3986. 15. Wheeler ML, Dunbar SA, Jaacks LM, et al. Macronutrients, food groups and eating patterns in the management of diabetes: a systematic review of the literature. Diabetes Care. 2012;35:434-445. 16. Griebeler ML, Morey-Vargas OL, Brito JP, et al. Pharmacologic interventions for painful diabetic neuropathy: An umbrella systematic review and comparative effectiveness network meta-analysis. Ann Intern Med. 2014;161:639-649. 17. Kishore P, Kim S, Crandall JP. Glycemic control and cardiovascular disease: what’s a doctor to do? Curr Diab Rep. 2012;12:255-264. 18. James PA, Oparil S, Carter BL, et al. 2014 evidence-based guideline for the management of high blood pressure in adults: report from the panel members appointed to the Eighth Joint National Committee ( JNC 8). JAMA. 2014;311:507-520. 19. Stone NJ, Robinson JG, Lichtenstein AH, et al. 2013 ACC/AHA guideline on the treatment of blood cholesterol to reduce atherosclerotic cardiovascular risk in adults: a report of the American College of Cardiology/American Heart Association Task Force on Practice Guidelines. J Am Coll Cardiol. 2014;63:2889-2934. 20. Preiss D, Seshasai SR, Welsh P. Risk of incident diabetes with intensive-dose compared with moderate-dose statin therapy: a meta-analysis. JAMA. 2011;305:2556-2564.

1548.e2

CHAPTER 229  Diabetes Mellitus  

REVIEW QUESTIONS 1. A 51-year-old man has a history of type 2 diabetes mellitus for 6 years. Past medical history is significant for chronic hepatitis C infection, chronic kidney disease stage 3, and a recent hospitalization for an upper gastrointestinal bleed. He takes a sulfonylurea for blood glucose control and rarely checks his blood glucose level. Fasting plasma glucose concentration in the office is 195 mg/dL, and his HbA1c is 6.8%. What do you conclude about his glucose control? A. His average blood glucose concentration during the past 3 months is approximately 140 mg/dL. B. HbA1c may be falsely high because of chronic kidney disease. C. HbA1c may be falsely low because of liver disease. D. HbA1c levels are increased after acute blood loss. E. HbA1c levels are more reflective of postprandial than of fasting glucose concentration. Answer: C  This patient has several reasons that his HbA1c may not accurately reflect his mean plasma glucose concentration. HbA1c results may be influenced by a number of factors, including conditions that alter red cell survival or cause interference with a specific assay. The HbA1c may be falsely low in this patient because of cirrhosis (increased red cell turnover), recovery from recent acute blood loss (greater percentage of younger erythrocytes with shorter exposure to glucose), or transfusion (dilution of patient’s blood with nondiabetic donor blood). In these instances, measurement of glycated serum proteins (fructosamine) or direct measurement of plasma glucose concentration will more accurately reflect glycemic control. Additional information about HbA1c assay methodology and interpretation of results can be obtained from the National Glycohemoglobin Standardization Program: http://www.ngsp.org. 2. A 38-year-old woman has had type 1 diabetes mellitus since the age of 12 years. She has maintained excellent control (HbA1c 6.0%) with a basal/ bolus injection regimen. She tests her glucose level four or five times a day, and review of her meter download shows many glucose levels in the 30s and 40s. However, the patient is unconcerned because she has no symptoms at these times. On questioning, she admits to recently “spacing out” while driving, which led to a minor traffic accident. Regarding the etiology and treatment of hypoglycemia in this patient: A. She has adapted to low blood glucose concentration and no change in treatment is required. B. She has developed hypoglycemia unawareness and her target HbA1c should be increased. C. Strict avoidance of hypoglycemia is of little benefit in reversing hypoglycemia-associated autonomic failure. D. An excessive counter-regulatory hormone response to hypoglycemia may contribute to her lack of symptoms. E. Treatment with β-blocker should be considered. Answer: B  In patients with long-standing diabetes, the counter-regulatory systems that normally would counteract the decline of glucose to dangerous levels may be impaired. This is especially true for patients with type 1 diabetes, who often have defects in glucagon and epinephrine response during hypoglycemia. This decrease in epinephrine response during hypoglycemia is accompanied by an attenuated autonomic neural response, which results in the clinical syndrome of impaired awareness of hypoglycemia. Without autonomic symptoms, mild hypoglycemia may proceed unnoticed to more advanced and dangerous phases. There is, however, evidence that hypoglycemia-associated autonomic failure can be reversed by strict avoidance of hypoglycemia, which can be facilitated by increasing target glucose levels. Reno CM, Litvin M, Clark AL, Fisher SJ. Defective counterregulation and hypoglycemia unawareness in diabetes: mechanisms and emerging treatments. Endocrinol Metab Clin North Am. 2013;42:15-38.

3. A mother brings her 19-year-old son, who has type 1 diabetes and uses an insulin pump, to see you and to ask for referral to a dietitian. She complains that her son refuses to follow his “diabetic diet” and frequently eats junk food, including fast food (burgers, fries, pizza) and ice cream. She also worries that he sometimes skips meals, saying he is not hungry. His body mass index is 22, and his recent laboratory results show an HbA1c of 7.8%, low-density lipoprotein cholesterol of 95, and normal triglycerides. Which of the following dietary recommendations is most appropriate for this patient? A. An 1800-calorie/day American Diabetes Association diet B. A low-carbohydrate diet C. A low-protein diet D. A low-fat, high-fiber diet E. A flexible “heart healthy” meal plan that limits concentrated sweets and emphasizes fruits and vegetables Answer: E  Dietary recommendations for patients with diabetes have changed substantially over time, from the extremely low-carbohydrate, highfat diets used before the discovery of insulin as a therapy, to “exchange diets,” to more flexible meal plans. For patients with type 1 diabetes, the key element is for the patient to learn to match mealtime insulin doses to the carbohydrate content of the meal. Severely restricted diets (very low carbohydrate or low calorie) are neither required nor advisable, although avoidance of large carbohydrate loads will help minimize post-meal glycemic excursions. For most patients with type 2 diabetes who are typically overweight or obese, moderate carbohydrate intake and reduction in total calories are advised. Current recommendations allow a variety of eating styles and ethnic food preferences, with emphasis on fruits and vegetables, low-fat protein sources, and use of monounsaturated or polyunsaturated fats. Evert A, Boucher J, Cypress M, et al. Nutrition therapy recommendations for the management of adults with diabetes. Diabetes Care. 2013;36: 3821-3842. Lasa A, Miranda J, Bullo M, et al. Comparative effect of two Mediterranean diets versus a low-fat diet on glycaemic control in individuals with type 2 diabetes. Eur J Clin Nutr. 2014;68:767-772. 4. A 54-year-old woman presents to her physician for treatment of hypertension. She had gestational diabetes during her last pregnancy 15 years ago, and there is a family history of type 2 diabetes (mother and older brother). Her body mass index is 36. Fasting glucose concentration is 110 mg/dL, and HbA1c is 6.2%. Which of the following have been shown to reduce the progression to diabetes in high-risk patients? A. Weight loss by reduced calorie diet B. Treatment with metformin C. Treatment with acarbose D. Bariatric surgery E. All of the above Answer: E  This patient has multiple risk factors for the development of type 2 diabetes, including family history, prior history of gestational diabetes, obesity, and hypertension. In addition, her glucose and HbA1c levels are already elevated above normal, in the defined “pre-diabetes” range. All of the treatments listed have been shown to prevent or to delay the onset of diabetes in randomized clinical trials. The most consistent evidence comes from weight loss trials. Both the Finnish Diabetes Prevention Study and the U.S. Diabetes Prevention Program reported 58% reduction in diabetes with a hypocalorie, reduced fat diet combined with moderate-intensity physical activity. Weight loss achieved with bariatric surgery is also highly effective in preventing (or even reversing) diabetes. Medications, including metformin, the α-glucosidase inhibitor acarbose, and troglitazone (a thiazolidinedione), have also been shown to reduce diabetes in high-risk patients, although somewhat less effectively than by lifestyle modification. Lifestyle changes and metformin are reported to be cost-effective interventions, but whether delay or prevention of type 2 diabetes will result in lower rates of cardiovascular disease and diabetes microvascular complications will require longer-term follow-up studies. Schwarz PE, Greaves CJ, Lindstrom J, et al. Nonpharmacologic interventions for the prevention of type 2 diabetes mellitus. Nat Rev Endocrinol. 2012;8:363-373.

CHAPTER 229  Diabetes Mellitus  

5. A 28-year-old woman with type 1 diabetes since the age of 12 years is considering having a child. Currently, her blood glucose is reasonably well controlled, although she admits this was not the case during her teens and early 20s, when her HbA1c was in the 9 to 11% range. She has mild background diabetic retinopathy, normal blood pressure, urine albumin-tocreatinine ratio of 25 mg/g, and normal findings on foot examination. Which of the following is true? A. She should delay pregnancy until she has achieved optimal glucose control (HbA1c ~6.5%). B. She should be treated with an angiotensin-converting enzyme inhibitor to prevent progression of renal disease during pregnancy. C. Progression of her retinopathy during pregnancy is likely to result in vision loss. D. The risk of her child’s developing type 1 diabetes is 25 to 50%. E. She should be advised to avoid pregnancy because of the risk of both maternal and fetal complications.

1548.e3

Answer: A  Although pregnancies in women with type 1 diabetes are generally considered “high risk,” the outlook for patients with minimal complications and good metabolic control is good. Women with advanced renal disease (proteinuria, reduced glomerular filtration rate) or proliferative retinopathy may experience rapid progression during pregnancy because of the influence of hormonal and hemodynamic changes and should be monitored closely by specialists. Use of angiotensin-converting enzyme inhibitors is contraindicated during pregnancy because of the risk of fetal renal damage. The key to successful pregnancy outcomes is achieving optimal glucose control before conception because the developing fetus is most susceptible to the teratogenic effects of hyperglycemia in the first 6 to 8 weeks of pregnancy, before the time that most women are aware of being pregnant. Maintaining strict glucose control during the pregnancy will reduce the risk of fetal complications, such as macrosomia and hyperbilirubinemia. Many patients benefit from use of an insulin pump and continuous glucose monitoring during this time. Although a child of a mother with type 1 diabetes is at increased risk of diabetes compared with the general population, the risk is less than 10%.

CHAPTER 230  Hypoglycemia and Pancreatic Islet Cell Disorders  

Phosphorylase

1549

Glycogen Liver

Glucose Glucokinase

Glycogen synthase

Glucose-6-phosphatase

Glucose-6-phosphate Phosphomutase

Glucose-1-phosphate

Fructose-6-phosphate Phosphofructokinase

Fructose 1,6-bisphosphatase

Fructose 1,6-bisphosphate

Glyceraldehyde 3-phosphate

Phosphoenolpyruvate Lactate dehydrogenase Pyruvate Phosphoenolpyruvate carboxykinase

Lactate

Pyruvate dehydrogenase Acetyl CoA

Citrate Isocitrate

Oxaloacetate Malate

Citric acid cycle

Fumarate

α-ketoglutarate Succinyl-CoA

Mitochondria Succinate

FIGURE 230-1.  Outline of the biochemical pathways involved in glucose physiology.

limited capacity of glycogenolysis, gluconeogenesis is very important in supporting hepatic glycogen stores during an overnight fast. Gluconeogenesis uses a number of key enzymes: pyruvate carboxylase, phosphoenolpyruvate carboxykinase (PEPCK), and fructose-1,6bisphosphatase and its precursors, including lactate, alanine, glutamine, glycerol, and pyruvate. Muscle and adipose tissue, which utilize glucose in the fed state, respond to prolonged fasting by reducing their glucose uptake virtually to zero and satisfying their energy requirements by the β-oxidation of fatty acids. Additionally, through the process of proteolysis, muscle tissue provides amino acids to the liver to serve as gluconeogenic precursors for net glucose formation. Changes in the hormonal milieu during fasting (suppressed insulin and elevated counter-regulatory hormones) stimulate ketogenesis. Ketones become a major source of fuel for the brain when glucose utilization by the brain declines. This results in a decrease in the rate of gluconeogenesis required to maintain the plasma glucose concentration and hence in diminished protein wasting.

Changes During Feeding

After a meal, glucose absorption into the circulation increases glucose concentrations, which stimulates secretion of insulin from the pancreatic β-cells and suppresses secretion of glucagon from the pancreatic α-cells. This change

in the hormonal milieu switches off endogenous hepatic glucose production and accelerates glucose utilization by liver, muscle, and adipose tissue. Glucose concentration then returns gradually to the postabsorptive level, at which endogenous glucose production is equal to the glucose uptake by peripheral tissues.

Counter-Regulatory Hormonal Responses to Hypoglycemia The counter-regulatory hormones play a key role in the maintenance of normal blood glucose concentration.2 If counter-regulation is intact, hypoglycemia (irrespective of the cause) will result in a decrease in insulin secretion and an increase in glucagon, epinephrine, norepinephrine, cortisol, and growth hormone (GH) secretion. Glucagon secretion increases rapidly in response to hypoglycemia, and studies have shown that the glucagon response is the primary essential defense mechanism against acute hypoglycemia. GH and cortisol have numerous effects on glucose metabolism, including increasing the rate of gluconeogenesis and antagonizing the effects of insulin. In adults, the glycemic thresholds for the activation of glucose counterregulatory hormones such as GH and cortisol lie within or just below the physiologic blood glucose concentration and slightly higher than the threshold for symptoms. This suggests that GH and cortisol secretion increase in response to blood glucose concentrations within the normoglycemic range,

1550

CHAPTER 230  Hypoglycemia and Pancreatic Islet Cell Disorders  

Glucose and ketone bodies

• Glycogenolysis • Gluconeogenesis • Ketogenesis

Lactate Alanine

Proteolysis activation Muscle tissue

Liver

Fatty acids and glycerol Lipolysis

Cortisol, growth hormone, glucagon and adrenaline act as counter regulatory hormones and increase glucose output from the liver by increasing the processes of glycogenolysis and gluconeogenesis

Adipose tissue FIGURE 230-2.  The role of the counter-regulatory hormones, glycogenolysis, gluconeogenesis, and lipolysis in glucose physiology.

and these increases are inversely proportional to the nadir in blood glucose. Figure 230-2 outlines the role of the counter-regulatory hormones. Insulin secretion from β-cells of the pancreas in healthy individuals is inhibited as blood glucose concentration falls below 72 mg/dL (4.0 mmol/L). As insulin secretion is reduced, the repressive effect of insulin on pancreatic α-cell function is removed, thereby rapidly increasing glucagon secretion. Glucagon acts on the liver to increase hepatic glycogenolysis and gluconeogenesis. When the blood glucose concentration falls further (≈68 mg/dL [3.8 mmol/L]), epinephrine and norepinephrine are released both from the adrenals and directly into interstitial fluid from nerve terminals, further suppressing insulin secretion, increasing glucagon secretion, and decreasing peripheral glucose utilization in the muscle and increasing lipolysis in the adipose tissues. Additional responses include GH and cortisol secretion, which occur below a blood glucose concentration of around 66 mg/dL (≈3.7 mmol/L) and are initiators of the adaptive response to hypoglycemia (e.g., during prolonged starvation); glucose-raising actions are much slower in onset (several hours). These hormone responses stimulate lipolysis, ketogenesis, and gluconeogenesis. Permissive amounts of cortisol and GH are required for a normal hepatic response to glucagon and epinephrine. In healthy individuals, this system ensures that hypoglycemia is rarely experienced and would only occur during starvation or ultra-endurance sports. Drugs or diseases that inhibit counter-regulatory secretion or action predispose patients to hypoglycemia. Activation of counter-regulation depends on effective detection of falling blood glucose levels. This is achieved by the complex integration of various glucose-sensing systems in both the periphery and central nervous system.3 Fluctuations in peripheral glucose levels are detected by glucose-sensing neurons in the oral cavity, gut, portal/mesenteric vein (PMV), and carotid body. PMV neurons detect changes in blood glucose prior to entry into the liver from the gut. This information is then relayed through the vagus nerve and spinal cord to the hindbrain and then to the hypothalamus. In addition, the hypothalamus, because of its location adjacent to the third ventricle and median eminence, may sample factors from peripheral circulation, including glucose, as well as hormones such as insulin and leptin. Although a complex network of glucose sensors has been described in the central nervous system and peripherally, the brain appears to have the dominant role during hypoglycemia and, specifically, the ventromedial region of the hypothalamus (VMH). VMH neurons contain the same glucose-sensing mechanisms (e.g., glucokinase, ATP-sensitive K+ channels) as found in pancreatic β-cells.

CLINICAL MANIFESTATIONS

Symptoms of Hypoglycemia

The symptoms of hypoglycemia reflect the responses of the brain to a decrease in the blood glucose level; such symptoms may be nonspecific and

vague, especially in the childhood period. Children may not be able to communicate their hypoglycemic symptoms. The symptoms of hypoglycemia may be categorized into two main groups: (1) those that arise as a result of the central nervous system being deprived of glucose (neuroglycopenic) and (2) symptoms arising from the perception of physiologic changes caused by the central nervous system–mediated sympatho-adrenal discharge triggered by hypoglycemia (neurogenic or autonomic).4 The neurogenic symptoms of hypoglycemia are largely the result of sympathetic neural, rather than adrenomedullary, activation. Neuroglycopenic symptoms (e.g., dizziness, confusion, tiredness, difficulty with speaking, headache, inability to concentrate, coma, and seizures) arise from the failure of brain function itself and are caused by deficient supply of glucose to the brain.5 Neurogenic symptoms include both adrenergic responses (catecholamine-mediated symptoms such as palpitations, tremor, and anxiety) and cholinergic responses (acetylcholine-mediated symptoms such as sweating, hunger, paresthesias). Awareness of hypoglycemia chiefly depends on perception of the central and peripheral effects of neurogenic (as opposed to neuroglycopenic) responses to hypoglycemia. In nondiabetic adults during acute insulin-induced hypoglycemia, autonomic symptoms become apparent at a threshold of approximately 60 mg/ dL (3.3 mmol/L), and impairment of brain function manifested by neuroglycopenic symptoms occurs at a threshold of approximately 50 mg/dL (2.8 mmol/L) in arterialized venous blood (venous levels would be ≈ 3 mg/dL [0.16 mmol/L] less). However, in patients with recurrent hypoglycemia, the glycemic thresholds for responses to hypoglycemia are reset at a lower plasma glucose concentration. The glucose thresholds for the activation of neuroglycopenic and autonomic symptoms in children are not as clearly defined as in adults. The symptoms and signs of hypoglycemia are not influenced by the rate of blood glucose decline in nondiabetic individuals.

Clinical Approach to the Patient with Hypoglycemia

A careful clinical history, description of symptoms, physical examination, and a systematic step-by-step approach are the cornerstones of establishing a diagnosis. The symptoms of hypoglycemia may be very nonspecific, hence any symptomatic child or adult must have the blood glucose level measured and documented. The relationship of a hypoglycemic episode to the most recent meal can be important diagnostically. Hypoglycemia occurring after a short fast (2 to 3 hours) may be suggestive of hyperinsulinism or glycogen storage disease. Hypoglycemia occurring after a long fast (12 to 14 hours) may suggest a disorder of gluconeogenesis. Postprandial hypoglycemia may indicate galactosemia, hereditary fructose intolerance, dumping syndrome, insulinoma, insulin autoimmune syndrome, and noninsulinoma pancreatogenous hypoglycemia syndrome. In both children and adults, a clear documentation of the medication history is important.

DIAGNOSIS

After the clinical history has been taken and the examination completed, a diagnostic cascade of appropriate tests is necessary. These may be guided in the context of the most common causes of hypoglycemia as listed in Table 230-1. The current adult recommendations6 state that evaluation and management of hypoglycemia should only be undertaken in patients in whom Whipple’s triad—symptoms, signs, or both consistent with hypoglycemia, a low plasma glucose concentration, and resolution of those symptoms or signs after the plasma glucose concentration is raised—is documented. However, this does not apply to children for the reasons discussed earlier.

Causes of Hypoglycemia

Hypoglycemia is more common in the childhood period than in adults and can be due to a large number of causes. Table 230-1 summarizes the differential diagnosis of hypoglycemia.

Hypoglycemia Due to Excess Production of Hormones

Inappropriate and excess production of certain hormones can lead to hypoglycemia. The two most common conditions associated with excess production of a hormone are hyperinsulinemic hypoglycemia (HH) and non–islet cell tumor hypoglycemia (NICTH) or IGF-2-oma (insulin-like growth factor–secreting tumor). Inappropriate production of insulin can either lead to fasting hypoglycemia or postprandial hypoglycemia.

CHAPTER 230  Hypoglycemia and Pancreatic Islet Cell Disorders  

1551

Hyperinsulinemic Hypoglycemia TABLE 230-1  DIFFERENTIAL DIAGNOSIS OF HYPOGLYCEMIA* HYPERINSULINEMIC HYPOGLYCEMIA (INCLUDING POSTPRANDIAL) Transient: infant of diabetic mother, perinatal asphyxia, Rhesus disease, intrauterine growth retardation, Beckwith-Wiedemann syndrome Congenital: ABCC8, KCNJ11, GCK, GDH, HADH, HNF4A, SLC16A1 Dumping syndrome Insulin receptor mutations and antibodies Insulinoma Noninsulinoma pancreatogenous hypoglycemia (adults) Gastric bypass surgery for morbid obesity Non–islet cell tumor hypoglycemia (NICTH) or IGF-2-oma Insulin autoimmune syndrome Insulin factitious hypoglycemia HORMONAL DEFICIENCY/RESISTANCE Adrenocorticotropic hormone Cortisol Growth hormone Glucagon† Adrenaline†

HH is a heterogeneous group of disorders characterized by unregulated insulin secretion from pancreatic β-cells. In the face of hypoglycemia, patients have inappropriately detectable serum insulin levels, low ketone bodies, and low fatty acids and show a glycemic response to glucagon.7 Congenital Forms of Hyperinsulinemic Hypoglycemia

In patients with congenital forms of HH, mutations in the key genes (ABCC8, KCNJ11, GLUD1, GCK, HADH, SLC16A1, HNF4A, HNF1A, and UCP2) regulating insulin secretion have been identified.8 Children with inactivating mutations in the genes ABCC8 and KCNJ11 present with the most severe forms of congenital HH, typically in the newborn period. Hyperinsulinismhyperammonemia syndrome due to activating mutations in the GLUD1 gene and activating mutations in the GCK gene, leading to HH, have both been described in adults as well as children. Exercise-induced HH due to activating mutations in the SLC16A1 gene has also been recognized in adults. Insulinoma

Carnitine deficiency (primary and secondary) Carnitine palmitoyltransferase deficiency (CPT 1 and 2) Carnitine transporter defects

An insulinoma is the commonest cause of endogenous HH in adults. Insulinomas have the highest incidence in the fifth and sixth decades.9 Insulinomas are insulin-secreting tumors of pancreatic origin, with an incidence of 1 to 4 per million. The majority (90%) of them are benign, solitary, intrapancreatic and less than 2 cm in diameter. Classically, symptoms become evident in the fasting state or following exercise. However, it is now known that insulinoma can also present with postprandial symptoms. Diagnosis is based on findings of abnormal serum levels of insulin and C-peptide (also proinsulin) at the time of fasting hypoglycemia. An insulinoma can occur either in isolation or in association with multiple endocrine neoplasia type 1 (MEN 1), with a lifetime prevalence of 10% among adults carrying mutations in MEN1 (Chapter 231). Around 6% of insulinomas occur in patients with MEN 1, and most insulinomas are benign, but 5 to 10% are malignant.

FATTY ACID OXIDATION

Postprandial Hyperinsulinemic Hypoglycemia

DEFECTS IN HEPATIC GLYCOGEN RELEASE/STORAGE Glycogen storage diseases: glucose-6-phosphatase, amylo-1,6-glucosidase deficiency, liver phosphorylase deficiency, glycogen storage disease type 0 DEFECTS IN GLUCONEOGENESIS Fructose-1,6-bisphosphatase deficiency, phosphoenolpyruvate carboxykinase deficiency, pyruvate carboxylase deficiency CARNITINE METABOLISM

Medium-chain acyl-CoA dehydrogenase (MCAD) deficiency Very long-chain acyl-CoA dehydrogenase (VLCAD) deficiency Short-chain acyl-CoA dehydrogenase (SCAD) deficiency Long/short-chain L-3-hydroxyacyl-CoA (L/SCHAD) deficiency DEFECTS IN KETONE BODY SYNTHESIS/UTILIZATION HMG-CoA synthase deficiency, HMG-CoA lyase deficiency Succinyl-CoA: 3-oxoacid-CoA transferase (SCOT) deficiency METABOLIC CONDITIONS (COMMON ONES) Organic acidemias (propionic, methylmalonic) Maple syrup urine disease, galactosemia, fructosemia, tyrosinemia Hereditary fructose intolerance Mitochondrial respiratory chain complex deficiencies Congenital disorders of glycosylation (CGD) DRUG INDUCED Sulfonylureas Insulin β-Blockers Salicylates Alcohol Quinine Haloperidol Pentamidine Levofloxacin Methadone Disopyramide Indomethacin Cibenzoline Gatifloxacin MISCELLANEOUS CAUSES (MECHANISM[S] NOT CLEAR) Idiopathic ketotic hypoglycemia (diagnosis of exclusion) Infections (sepsis, malaria), congenital heart disease *Boldface indicates more common in adults. † No human case yet reported with glucagon or adrenaline deficiency. HMG = 3-hydroxy-3-methylglutaryl; IGF = insulin-like growth factor.

Postprandial hyperinsulinemic hypoglycemia (PPHH) refers to hypoglycemia within a few hours of meal ingestion, secondary to inappropriate insulin secretion in response to a meal. If PPHH is clinically suspected, then an oral glucose tolerance test (OGTT) or a mixed-meal provocation test is performed. (See later in section “Investigations for Hypoglycemia.”) A physiologic dip in the blood glucose level seen in OGTT might lead to misdiagnosis. However, corresponding biochemical evidence of endogenous HH and symptoms of neuroglycopenia during a hypoglycemic episode would help distinguish between pathologic PPHH and reactive hypoglycemia. A decrease of more than 108 mg/dL (6 mmol/L) between peak and nadir blood glucose during OGTT has been used as a diagnostic criterion for dumping syndrome in adults. dumping syndrome.  Dumping syndrome seen in infants after Nissen fundoplication is a classic example of PPHH. Precipitous emptying of hyperosmolar carbohydrate-containing solutions into the small bowel results in rapid glucose absorption, hyperglycemia, and reactive hypoglycemia. These children also tend to have abnormally exaggerated secretion of glucagon-like peptide-1 (GLP-1), which may contribute to the exaggerated insulin surge and resultant hypoglycemia.10 insulin autoimmune syndrome.  Insulin autoimmune syndrome, or Hirata’s disease, is a rare condition characterized by HH associated with high titers of antibodies to endogenous insulin in the absence of pathologic abnormalities of pancreatic islets and prior exposure to exogenous insulin. The disease is extremely uncommon in Western countries. Insulin autoimmune syndrome affects men and women equally and is seen more frequently in patients older than 40 years. The binding kinetics of endogenous insulin by the antibodies are thought to lead to physiologically inappropriate levels of bioavailable insulin, causing either hyper- or hypoglycemia. In this syndrome, the insulin levels are markedly elevated, usually above 100 mU/L. After a meal or glucose load, these patients often demonstrate initial hyperglycemia, followed by hypoglycemia a few hours later. The hyperglycemia is caused by the anti-insulin antibodies that bind the insulin secreted in response to rising blood glucose levels after a meal. This binding reduces the bioavailability of the secreted insulin to the receptors in the liver and peripheral tissues, resulting in hyperglycemia and further insulin secretion. As blood glucose concentrations begin to decrease and insulin secretion declines, the insulin bound to the antibodies is released, resulting in inappropriately high free insulin concentrations for the blood glucose, causing hypoglycemia.

1552

CHAPTER 230  Hypoglycemia and Pancreatic Islet Cell Disorders  

pphh in patients with insulin-receptor mutations.  PPHH has been described in patients who carried a heterozygote mutation (Arg1174Gln) in the insulin-receptor gene. Hyperinsulinism seems to be associated with decreased degradation rather than increased secretion of insulin, as evidenced by increased fasting levels of serum insulin despite normal levels of serum C-peptide and reduced clearance of exogenous insulin during clamp studies. pphh after gastric bypass surgery.  A consequence of the obesity epidemic is the increasing use of gastric bypass surgery for patients with severe, medically complicated obesity (Chapter 220), which has led to a number of reports of postprandial HH.11 In a review of the Swedish Bariatric Surgery registry, the incidence of hospitalization for hypoglycemia in post–gastric bypass patients was reported as less than 1%. A number of different explanations have been suggested to explain hypoglycemia post gastric bypass surgery. This can either be a manifestation of dumping syndrome or improved insulin sensitivity following weight loss unmasking an underlying hyperinsulinemia syndrome. The hypoglycemia could also be due to an effect on the enteroinsular axis induced by the diversion of nutrients into the small intestine. The principal reason seems to be enhanced postprandial insulin secretion, thought to be due primarily to increased secretion of glucose-dependent insulinotropic polypeptide (GIP) and especially GLP-1. GLP-1 levels are now well documented to be increased two- to five-fold after gastric bypass. The elevations of incretins tend to be seen early, even as early as 2 days after gastric bypass, and levels may decline as substantial weight loss and normalization of insulin sensitivity occurs. In patients with PPHH, elevated levels of GIP and GLP-1 persist for years after surgery. Increased postprandial insulin secretion by incretins is mediated by islet cell hypertrophy and hyperplasia. Both GIP and GLP-1 have been implicated in increasing pancreatic β-cell mass in rodent models. GLP-1 regulates islet growth by inducing the expression of the transcription factor pancreaticduodenum homeobox-1 (PDX-1). Overexpression of IGF-2 and IGF-1 receptor alpha (IGF1Rα) have been found in pancreatic tissue removed from patients with persistent PPHH after gastric bypass surgery as compared to controls. These findings are suggestive of the role of growth factors in islet hyperfunction seen in post–gastric bypass patients. noninsulinoma pancreatogenous hypoglycemia syndrome.  Noninsulinoma pancreatogenous hypoglycemia syndrome (NIPHS) is characterized by postprandial neuroglycopenia in the presence of negative prolonged fasting tests and negative perioperative localization studies for insulinoma.12 However, in some patients the selective arterial calcium stimulation test is positive, with the histology of the resected pancreas showing nesidioblastosis. The underlying genetic basis of NIPHS is not known. These patients are negative for ABCC8/KCNJ11 mutations and show islet hypertrophy histologically (as observed in diffuse congenital HH). Immunohistologic studies of the resected pancreatic tissue have failed to show an increased rate of proliferation of β-cells or abnormal synthesis and/ or processing of either proinsulin or amylin. Neither has there been any evidence of overexpression of pancreatic differentiation factors, PDX-1, and Nkx-6.1, nor the calcium-sensing receptor (CaSR). Insulin Factitious Hypoglycemia

Hypoglycemia can also be induced pharmacologically, either intentionally as a diagnostic tool, accidentally as a complication of the treatment of diabetes mellitus, or as a consequence of poisoning either with insulin itself or with drugs (e.g., sulfonylureas) that stimulate insulin release. Whenever severe hypoglycemia occurs with documented hyperinsulinism, the possibility of Munchausen’s syndrome by proxy should be considered in children. The possibility of malicious administration of insulin or an oral sulfonylurea should always be suspected in cases of sudden onset of hypoglycemia in a previously healthy individual. In the case of insulin administration, the clue in the biochemistry will be a raised insulin level accompanied by normal C-peptide.

Non–Islet Cell Tumor Hypoglycemia, or IGF-2-oma

NICTH, or IGF-2-oma, denotes the syndrome of hypoglycemia produced by or associated with any neoplasm other than an insulinoma. These are usually tumors of mesenchymal and epithelial origin (including hepatomas, fibromas, and fibrosarcomas). The underlying mechanism of hypoglycemia in nearly all patients with this syndrome is overproduction of IGF-2 by the

tumor, which includes mature IGF-2 and incompletely processed forms of IGF-2, referred to collectively as “big” IGF-2.13 The elevated IGF-2–related peptides mimic the fasting hypoglycemia characteristic of patients with insulin-producing islet cell tumors. Rarely, markedly elevated IGF-2 levels produce somatic changes suggestive of acromegaly. Typically, the elevated IGF-2 levels are associated with suppressed plasma levels of insulin, IGF-1, and GH.

Hypoglycemia Due to Hormone Deficiency

Deficiency of glucagon, adrenaline, GH, and cortisol can cause hypoglycemia. Glucagon and adrenaline deficiency is extremely rare, and so far no true human, genetically proven defects in glucagon and adrenaline deficiency have been described. Children and adults can present with hypoglycemia due to deficiency of various hormones. This might be either in isolation (e.g., isolated GH, adrenocorticotropic hormone [ACTH], or cortisol deficiency) or in combination with other hormones, such as in patients with hypopituitarism. The etiology of the hypoglycemia resulting from cortisol and GH deficiency is due to a combination of factors, including reduced gluconeogenic substrate availability (decreased mobilization of fats and proteins) and increased glucose utilization due to increased insulin sensitivity of tissues in the absence of these two hormones. Acquired hypopituitarism may result from tumors (most commonly craniopharyngioma), radiation, infection, hydrocephalus, vascular anomalies, and trauma. Addison disease (AD) results from adrenal cortex hypofunction/ dysfunction, with deficient production of glucocorticoids, mineralocorticoids, and androgens, and with high levels of both ACTH and plasma renin activity (Chapter 227). Autoimmune AD is the most frequent etiologic form in adult patients, accounting for about 80% of cases, followed by posttuberculosis AD in 10 to 15%; the remaining 5% of cases are due to vascular, neoplastic, or rare genetic forms. The markers of autoimmune AD are adrenal cortex (ACA) or 21-hydroxylase autoantibodies (21-OHAbs), and they are present at diagnosis in more than 90% of cases. In autoimmune AD, the adrenal cortex is infiltrated by lymphocytes and plasma cells, and the glands are sclerotic and reduced in volume. Autoimmune AD occurs mainly in middle-aged women, alone or associated with other (clinical, subclinical, or potential) autoimmune diseases, giving rise to various forms of autoimmune polyglandular syndrome. Replacement therapy with gluco- and mineralocorticoids is life-saving for patients with chronic adrenal insufficiency.

Hypoglycemia Due to Defects in Hepatic Glycogen Release/Storage

Glucose-6-phosphatase deficiency (glycogen storage disease [GSD] type I, Von Gierke disease) is the commonest of the glycogen storage diseases causing hypoglycemia (Chapter 207). Deficiency of this enzyme results in the inability to release free glucose from glucose-6-phosphate, with resultant hepatomegaly due to stored glycogen. These children and adults present with recurrent hypoglycemia associated with lactic acidosis, hyperuricemia, and hyperlipidemia.14 The two other glycogen storage diseases causing hypoglycemia are due to deficiencies of the enzymes amylo-1,6-glucosidase (GSD type III) and liver phosphorylase (GSD type VI). The clinical and biochemical features of GSD-III subjects are quite heterogeneous.

Hypoglycemia Due to Defects in Gluconeogenesis

Gluconeogenesis, or the formation of glucose from mainly lactate/pyruvate, glycerol, glutamine, and alanine, plays an essential role in the maintenance of normoglycemia during fasting. Inborn deficiencies are known in each of the four enzymes of the glycolytic-gluconeogenic pathway that ensure a unidirectional flux from pyruvate to glucose: pyruvate carboxylase, phosphoenolpyruvate carboxykinase (PEPCK), fructose-1,6-bisphosphatase, and glucose-6-phosphatase. Gluconeogenesis can essentially be viewed as a reversal of glycolysis but with a few important differences. Patients with defects in gluconeogenesis present with fasting hypoglycemia and lactic acidosis. Pyruvate carboxylase deficiency may lead to a more widespread clinical presentation, with lactic acidosis, severe mental and developmental retardation, and proximal renal tubular acidosis.

Hypoglycemia Due to Disorders of Carnitine Metabolism and Defects of Fatty Acid Oxidation

Serious clinical consequences may occur if fatty acid oxidation (FAO) is impaired, including hypoglycemic seizures, muscle damage, cardiomyopathy, metabolic acidosis, and liver dysfunction. Fatty acids are taken up by

1553

CHAPTER 230  Hypoglycemia and Pancreatic Islet Cell Disorders  

hepatocytes and muscle, where they are subsequently activated to their coenzyme A (CoA) esters. FAO disorders are individually rare, but they are collectively common because of the number of different enzymes affected. When defects occur in fatty acid degradation, excess acylcarnitine intermediates accumulate in the tissues, including heart, liver, and skeletal muscle, which can lead to organ dysfunction. The diversion of acyl-CoA intermediates into β-oxidation results in accumulation of toxic dicarboxylic acids. Acylcarnitines that spill into the blood provide a marker for diagnosis. Primary carnitine deficiency is an autosomal recessive disorder of fatty acid oxidation that can present at different ages with hypoketotic hypoglycemia and cardiomyopathy and/or skeletal myopathy (Chapter 205). This disease is suspected based on reduced levels of carnitine in plasma and confirmed by measurement of carnitine transport in the patient’s fibroblasts. Carnitine transport is markedly reduced (usually < 5% of normal) in fibroblasts from patients with primary carnitine deficiency. Patients with the hepatic isoform of carnitine palmityltransferase (CPT)-1 deficiency present with hypoketotic hypoglycemia in the neonatal period. The commonest disorder of fatty acid β-oxidation is medium-chain acylCoA dehydrogenase (MCAD) deficiency, an autosomal recessive disease presenting in children who are typically asymptomatic except during times of fasting and metabolic stress, usually associated with a viral illness, when they present with fasting nonketotic hypoglycemia; if undiagnosed, 20 to 25% of affected patients will die during the first episode.

Metabolic Diseases

Hypoglycemia can also be due to a number of metabolic conditions (Chapter 205), including galactosemia, fructosemia, tyrosinemia, organic acidemias, maple syrup urine disease, glutaric aciduria type II, and in mitochondrial respiratory chain defects. Hereditary fructose intolerance, caused by catalytic deficiency of aldolase B (fructose-1,6-phosphate aldolase), is a recessively inherited condition in which affected homozygotes develop hypoglycemia and severe abdominal symptoms after taking foods containing fructose and cognate sugars. Continued ingestion of noxious sugars leads to hepatic and renal injury and growth retardation.

Noninsulinoma Islet Cell Tumors

Islet cell tumors present an important challenge to the clinician because of their protean manifestations and potential lethality. These tumors can be clinically silent or active (functioning). Early diagnosis is essential and depends on recognition of the classic and variant clinical syndromes followed by confirmation of elevated peptide levels by radioimmunoassay.15 Glucagonoma, gastrinoma, VIPoma (VIP = vasoactive intestinal peptide), somatostatinoma, and ACTHoma are functioning tumors that may occur in isolation but can also be part of MEN 1 syndrome (Chapter 231) and von Hippel-Lindau disease (Chapter 417). Tumor marker measurement gives useful information for the follow-up and management of patients with noninsulinoma islet cell tumors (neuroendocrine tumors). The currently used tumor markers are neuron-specific enolase (NSE) and chromogranin A (CgA). The clinical accuracy of these biomarkers depends on histotype and disease extent. CgA is thought to be the optimal marker for most neuroendocrine tumors, because it is independent of the biological characteristics of the tumor.

Glucagonoma

Glucagonomas are α-cell tumors that, when they are active, produce a syndrome characterized by necrolytic migratory erythema, diabetes mellitus, weight loss, anemia, glossitis, thromboembolism, neuropsychiatric disturbances, and hyperglucagonemia. Tumor characterization is made by computed tomography (CT) and/or pancreatic endoscopic ultrasonic and indium-labeled octreoscan. The diagnosis is established by documenting the presence of hyperglucagonemia, with diagnostic levels being generally above 500 pg/mL (normal, 1.5 mL

  Morphology   Vitality (live)

>15 million/mL >39 million/ejaculate >40% motile >32% progressively motile >4% normal† >58%

Leukocytes

30-40 mIU/mL) Gonadal dysgenesis With stigmata of Turner syndrome Pure (46,XX or 46,XY) Mixed Ovarian failure with normal ovarian development Genetic disorders Autoimmune disorders Gonadotropin receptor or postreceptor defects (resistant ovary or Savage’s syndrome?) Enzymatic defects (17α-hydroxylase deficiency, galactosemia) Physical causes Irradiation Chemotherapeutic agents Viral agents Idiopathic

Hypogonadotropic or normogonadotropic hypogonadism (LH and FSH 30 mIU/mL) imply ovarian failure and require further evaluation. Incipient ovarian failure should be considered in any woman with basal FSH levels of 15 mIU/mL or higher other than during the midcycle LH surge. Many clinicians believe that chromosomal evaluation is indicated in all individuals with elevated FSH levels before age 40 years, and it is certainly indicated if hypergonadotropic amenorrhea begins before age 30 years. If FSH levels are low or normal, the measurement of total testosterone levels may be helpful whether or not there is any evidence of hirsutism or virilization. Hyperandrogenic women need not be hirsute because some have relative insensitivity of the hair follicles to androgens. Mildly increased levels of testosterone (and perhaps dehydroepiandrosterone sulfate as well) suggest polycystic ovary syndrome (PCOS). However, total circulating androgen levels need not be elevated because of the alterations in metabolic clearance rate and sex hormone–binding globulin that are present in PCOS. Consequently, some clinicians prefer to measure circulating free testosterone levels. Circulating levels of LH and FSH may aid in differentiation of PCOS from hypothalamic-pituitary dysfunction. LH levels are often elevated in PCOS so that the ratio of LH to FSH is increased; however, LH levels may be identical to those observed in normal women in the follicular phase. In contrast, levels of LH and FSH are normal or slightly reduced in hypothalamicpituitary dysfunction. There is some overlap between women with “polycystic ovarian–like” disorders and those with hypothalamic-pituitary dysfunction. Radiographic assessment of the sella turcica is indicated in all amenorrheic women in whom both LH and FSH levels are consistently low (both 200 ng/dL or DS > 7.0 µg/mL

Normal or mildly T DS 5.0-7.0 µg/mL

Evaluate for neoplasm

Evaluate for “adult-onset” CAH

Normal or mildly T and/or DS (normal or LH)

Mildly T (normal or LH)

PCOlike

Normal T (normal or LH)

HCA

Evaluate for “adult-onset” CAH FIGURE 236-3.  Biochemical evaluation of amenorrhea. This schema must be considered an adjunct to the clinical evaluation of the patient. See text for details. CAH = congenital adrenal hyperplasia; DS = dehydroepiandrosterone sulfate; FSH = follicle-stimulating hormone; HCA = hypothalamic chronic anovulation; LH = luteinizing hormone; PCO-like = polycystic ovarian syndrome–like; PRL = prolactin; T = testosterone; TSH = thyroid-stimulating hormone.

mutations, and complex multifactorial polygenic inheritance), physical and environmental causes, and autoimmune disturbances. In addition, there may be families in which menopause begins earlier than the expected age without any further pathologic cause.

Genetic Abnormalities

Several pathologic conditions with dysgenetic gonads involve elevated gonadotropin levels and amenorrhea as well as abnormalities of the X chromosome. The term gonadal dysgenesis refers to individuals with undifferentiated streak gonads without any association with either extragonadal stigmata or sex chromosome aberrations. Because individuals with gonadal dysgenesis have the normal complement of oocytes at 20 weeks of fetal age but virtually none by birth, this disorder is a form of premature ovarian failure.

Turner’s Syndrome

Turner’s syndrome (also see Chapter 233) describes patients with streak gonads composed of fibrous stroma and four cardinal features: a female phenotype; sexual infantilism; short stature; and several physical abnormalities, sometimes including webbed neck, low-set ears, multiple pigmented nevi, double eyelashes, micrognathia, epicanthal folds, shieldlike chest with microthelia, short fourth metacarpals, increased carrying angle of the arms, and certain renal and cardiovascular defects (most commonly coarctation of the aorta and aortic stenosis) (Fig. 236-4).11 The diagnosis can sometimes be made at birth because of unexplained lymphedema of the hands and feet. The syndrome is associated with an abnormality of sex chromosome number, morphology, or both. Most commonly, the second sex chromosome is absent (45,X). Turner’s syndrome is the single most common chromosome disorder in humans, but more than 95% of such fetuses are aborted, and the incidence in newborns is approximately 1 in 3000 to 5000. Chromosome breakage and mosaicism occur as well. In mosaic individuals with a normal 46,XX cell line, sufficient follicles may persist postnatally to initiate pubertal changes and to cause ovulation so that pregnancy is possible. Deletions of the X-chromosomelinked SHOX gene explain many of the dysmorphic skeletal features that are present, including the short stature. It is believed that the number of phenotypic findings may be related to the percentage of cells that are 45,X. There also may be an effect of imprinting with the variation in phenotype partly explained by the parental origin of the one remaining X chromosome.

Pure Gonadal Dysgenesis

Pure gonadal dysgenesis is the term given to phenotypically female individuals with streak gonads who are of normal stature and have none of the physical stigmata associated with Turner’s syndrome. Such individuals have either a 46,XX or 46,XY karyotype. The 46,XX defect may be inherited as an autosomal recessive, with 10% having associated nerve deafness. The 46,XY defect may be inherited as an X-linked recessive, with clitorimegaly occurring in 10 to 15% and gonadal tumors developing in 25% if the gonads are not removed.

Mutations in the X Chromosome Associated with Premature Ovarian Failure

Several regions of the X chromosome are now recognized to contain mutations in genes that may result in premature ovarian failure. Of particular note is the fragile X mental retardation (FMR1) gene. More than 5% of women with 46,XX spontaneous premature ovarian failure have mutations of the FMR1 gene. This risk is increased if there is a family history of premature ovarian failure. A family history of fragile X syndrome, unexplained mental retardation, dementia, developmental delay of a child, or tremor-ataxia syndrome is reason for genetic counseling. Mutations in the FMR1 gene are known to be associated with a neurodegenerative disorder. Women with mutations in the FMR1 gene are at risk for having a child with mental retardation, should they be one of the 6 to 8% of women with premature ovarian failure who conceive spontaneously. For FMR1, a CGG repeat sequence occurs, with up to 60 repeats being normal. Expansion to more than 200 repeats leads to the fragile X syndrome, with the high level of repeats causing hypermethylation of the gene promoter and silencing of the gene. Female carriers of the permutation have an unstable intermediate number of repeats (i.e., 60 to 199) and the predisposition for premature ovarian failure.

Trisomy X

Trisomy X (46,XXX karyotype) is also associated with premature menopause, although many such individuals have normal reproductive lives. Premature menopause can also occur in mosaic individuals with cell lines with excess X chromosomes. When gonadal abnormalities occur in women with excess X chromosomes, they seem to occur after ovarian differentiation so that some ovarian function is possible. Only later in life do such women develop secondary amenorrhea and premature ovarian failure.

CHAPTER 236  Reproductive Endocrinology and Infertility  

1591

(p450arom) enzymes may also lead to ovarian failure. Women with galactosemia also experience ovarian failure early in life, even when a galactoserestricted diet is introduced early in infancy. Mutations of several autosomal genes result in premature ovarian failure. Included in this growing list are mutations involving FSHR (the FSH receptor gene), FOXL2 (a forkhead transcription factor associated with the blepharophimosis-ptosis-epicanthus inversus syndrome), INHA (the inhibin-α gene), E1F2B (a family of genes associated with central nervous system leukodystrophy and ovarian failure), PMM2 (the gene for phos­phomannomutase), GALT (the gene for galactose-1-phosphate uridyltransferase), and AIRE (leading to the autoimmune polyendocrinopathycandidiasis-ectodermal dystrophy syndrome). Myotonic dystrophy (Chapter 421) is caused by an autosomal triple repeat mutation, like the fragile X syndrome, that is similarly associated with premature loss of germ cells from the ovary. The list of mutations associated with early ovarian failure continues to increase as the function of more genes is determined.

Mutations Involving Reproductive Hormones, Their Receptors, and Action

The resistant ovary (Savage’s) syndrome occurs in young amenorrheic women who have elevated peripheral gonadotropin concentrations, normal (although immature) follicles present on ovarian biopsy, 46,XX karyotype with no evidence of mosaicism, fully developed secondary sex characteristics, and ovarian resistance to stimulation with human menopausal or pituitary gonadotropins. At least some of these women have mutations in the FSH receptor. It is probably inappropriate to use the term “resistant ovary syndrome” because it is likely that this is a heterogeneous disorder due to various genetic mutations.

A

Other Causes Physical and Environmental

Irradiation and chemotherapeutic agents used to treat various malignant diseases may also cause premature ovarian failure. Ovulation and cyclic menses return in some of these patients even after prolonged intervals of hypergonadotropic amenorrhea associated with signs and symptoms of profound hypoestrogenism. In general, the younger the individual at the time of treatment, the less likely is she to have permanent ovarian failure after the completion of therapy. Rarely, mumps affects the ovaries and causes ovarian failure.

Autoimmune Disorders

Premature ovarian failure may occur in conjunction with a variety of autoimmune disorders. The most well-known syndrome (autoimmune polyglandular syndrome type 1) involves hypoadrenalism, hypoparathyroidism, and mucocutaneous candidiasis together with ovarian failure (Chapter 231). Testing for adrenal antibodies by indirect immunofluorescence will identify the 4% of women with spontaneous premature ovarian failure who have steroidogenic cell autoimmunity and are at risk for adrenal insufficiency. Thyroiditis is the most commonly associated abnormality. Antibodies to the FSH receptor have been identified in a few cases. These associations make it mandatory to rule out other potentially life-threatening endocrinopathies in young women with hypergonadotropic amenorrhea.

TREATMENT 

B FIGURE 236-4.  Adult with Turner’s syndrome. This woman was seen at age 56 years by the author of the chapter (Dr. Rebar), and was case number 2, an adolescent at that time, in the original publication of Dr. Henry Turner describing the syndrome.

Known Genetic Alterations of Specific Genes

In girls with the rare syndrome of 17α-hydroxylase deficiency involving p450c17 who survive until the expected age of puberty, sexual infantilism and primary amenorrhea occur together with elevated levels of gonadotropins (also see Chapter 233). Defects in the 20,22-lyase (p450scc) or aromatase

Women with hypergonadotropic amenorrhea and ovarian failure should be treated identically whether or not they have signs of hypoestrogenism or desire pregnancy. Counseling and psychological support are indicated in women in whom the diagnosis of premature ovarian failure is made. Ovarian biopsy is not indicated to document the existence of follicles because only a small portion of each ovary can be sampled and because pregnancies have resulted in patients who had biopsy samples devoid of follicles. Estrogen replacement is warranted to prevent the accelerated bone loss known to occur in affected women (Chapter 243). The estrogen should be given sequentially with a progestin to prevent endometrial hyperplasia. Young women with ovarian failure may require twice as much estrogen as postmenopausal women for relief of signs and symptoms of hypoestrogenism. Inexplicably, women with premature ovarian failure may conceive while taking exogenous estrogen, even in the form of oral contraceptive agents, at the same rate as those not taking estrogen, so barrier contraception should be discussed if pregnancy is not desired. Women with hypergonadotropic amenorrhea are rarely able to become pregnant. It is not clear why pregnancy may rarely occur in such women, but

1592

CHAPTER 236  Reproductive Endocrinology and Infertility  

the pregnancy and delivery rate is 6 to 8%. Infertility treatment of young women with hypergonadotropic amenorrhea involves hormone replacement to mimic the normal menstrual cycle and embryo transfer by use of donor oocytes. Whether women with gonadal dysgenesis should be offered pregnancy by use of donor oocytes is now the subject of debate because a markedly increased incidence of aortic rupture during pregnancy secondary to medial necrosis has been documented. Women with Turner’s syndrome contemplating pregnancy should be counseled regarding the risks. The coordination of health care of adult women with Turner’s syndrome often falls to the endocrinologist because many of the complications of the disease are endocrinologic: hypothyroidism, diabetes, hypertension, obesity, osteoporosis, and hypogonadism. However, guidelines have been published about the surveillance of other multisystem conditions for which Turner’s syndrome patients are at risk, including significant psychosocial problems, congenital heart disease, deafness, and gastrointestinal and hepatic disorders.

CHRONIC ANOVULATION

Chronic anovulation, the most frequent form of amenorrhea encountered in women of reproductive age, implies that functional ovarian follicles remain and that cyclic ovulation can be induced with appropriate therapy (Table 236-4). The cause of the anovulation should be determined. The pathophysiologic bases for several forms of anovulation are unknown, but the anovulation can be interrupted transiently by nonspecific induction of ovulation in most affected women. Anovulation can result in either amenorrhea or irregular (generally less frequent) menses.

Hypothalamic Chronic Anovulation

DEFINITION

Hypothalamic chronic anovulation (HCA) represents a heterogeneous group of disorders with similar manifestations. Emotional and physical stress,

TABLE 236-4  CAUSES OF CHRONIC ANOVULATION Chronic anovulation of hypothalamic-pituitary origin Hypothalamic chronic anovulation Psychogenic Exercise associated Associated with diet, weight loss, or malnutrition Anorexia nervosa and bulimia Pseudocyesis Forms of isolated (idiopathic) hypogonadotropic hypogonadism (including Kallmann’s syndrome) Due to hypothalamic-pituitary damage Pituitary and parapituitary tumors Empty sella syndrome Following surgery Following irradiation Following trauma Following infection Following infarction Idiopathic hypopituitarism Hypothalamic-pituitary dysfunction or failure with hyperprolactinemia (multiple causes) Due to systemic diseases Chronic anovulation due to inappropriate feedback (i.e., polycystic ovary syndrome) Excessive extraglandular estrogen production (i.e., obesity) Abnormal buffering involving sex hormone–binding globulin (including liver disease) Functional androgen excess (adrenal or ovarian) Neoplasms producing androgens or estrogens Neoplasms producing chorionic gonadotropin Chronic anovulation due to other endocrine and metabolic disorders Adrenal hyperfunction Cushing’s syndrome Congenital adrenal hyperplasia (female pseudohermaphroditism) Thyroid dysfunction Hyperthyroidism Hypothyroidism Prolactin or growth hormone excess Hypothalamic dysfunction Pituitary dysfunction (microadenomas and macroadenomas) Drug induced Malnutrition

excessive exercise, nutritional deficiencies, weight loss, reduced body fat, and other unrecognized factors may contribute in varying proportions to the anovulation. Women with HCA have normal neuroanatomic findings.

ANOREXIA NERVOSA

Individuals with amenorrhea and significant weight loss should be examined for the possibility of anorexia nervosa (Chapter 219).

ISOLATED HYPOGONADOTROPIC HYPOGONADISM

Affected individuals have absence of spontaneous pubertal development. Most have functional GnRH deficiency, but some have abnormalities of gonadotropin deficiency localized to the pituitary gland. Kallmann’s syndrome is a familial disorder consisting of gonadotropin deficiency, anosmia or hyposmia, and color blindness in men or, more rarely, in women (Chapter 223). Partial or complete agenesis of the olfactory bulb is present on autopsy, accounting for use of the term olfactogenital dysplasia. Isolated gonadotropin deficiency in the absence of anosmia occurs as well. Sexual infantilism with a eunuchoid habitus is the clinical hallmark of this disorder, but moderate breast development may occur. Circulating LH and FSH levels are low but almost always detectable. Mutations in KAL1, FGFR1, FGF8, PROK2, ROKR2, HS6ST1, WDR11, or CHD7 have been identified in a minority of patients with Kallmann’s syndrome. Ovulation induction requires use of exogenous gonadotropins and HCG or pulsatile GnRH. Estrogen replacement therapy is indicated in these women until pregnancy is desired. It may not be possible to distinguish between partial isolated gonadotropin deficiency and functional HCA in all cases.

HYPOPITUITARISM

Hypopituitarism may be obvious on cursory inspection or sufficiently subtle to require endocrine testing (Chapter 224). The clinical presentation depends on the age at onset, the cause, and the nutritional status of the individual. Failure of development of secondary sex characteristics must always raise the question of hypopituitarism. Ovulation can be induced successfully with exogenous gonadotropins when pregnancy is desired and after the hypopituitarism is treated appropriately. Replacement therapy with estrogen is indicated.

HYPERPROLACTINEMIA

Galactorrhea associated with hyperprolactinemia, whatever the cause, almost always occurs together with amenorrhea caused by hypothalamic-pituitary dysfunction or failure. Many conditions can cause excess prolactin secretion (Chapter 224). A prolactinoma must be excluded. Hirsutism may be observed occasionally in association with amenorrhea-galactorrhea and hyperprolactinemia. Elevated levels of the adrenal androgens dehydroepiandrosterone and dehydroepiandrosterone sulfate may be observed and may account for the polycystic-type ovaries present in some hyperprolactinemic women.

FAILURE OF THE HYPOTHALAMIC-PITUITARY UNIT

The hypothalamic-pituitary unit may also fail to function normally in a number of stressful, debilitating, systemic illnesses that interfere with somatic growth and development. Chronic renal failure, liver disease, and diabetes mellitus are the most prominent examples.

DIAGNOSIS

Abrupt cessation of menses in women younger than 30 years who have no anatomic abnormalities of the hypothalamic-pituitary-ovarian axis and no other endocrine disturbances suggests a diagnosis of HCA. Affected individuals tend to be bright, educated, and engaged in intellectual occupations and may well give a history of psychosexual problems and socioenvironmental trauma. HCA is characterized by low to normal levels of gonadotropins and relative hypoestrogenism. Rarely, however, do affected women present with signs and symptoms of estrogen deficiency. It is important to rule out a central lesion as the cause of the hypogonadotropic hypogonadism in women who appear to have HCA.

TREATMENT  Psychological counseling or a change in lifestyle, especially for women engaged in strenuous exercise programs, may be effective in inducing cyclic ovulation and menses in women with functional HCA. Cognitive behavior therapy is effective in a proportion of women with functional HCA. For women

CHAPTER 236  Reproductive Endocrinology and Infertility  

1593

desiring pregnancy, ovulation can also be induced with clomiphene citrate (50 to 100 mg/day for 5 days beginning on the third to fifth day of withdrawal bleeding). Treatment with exogenous gonadotropins to induce follicular maturation followed by HCG to induce follicular rupture may be effective in women who do not ovulate in response to clomiphene. Because women with HCA have low circulating levels of leptin, investigators have given recombinant leptin and documented that ovulation may resume in some affected women. Given the heterogeneous nature of the disorder, it is not surprising that exogenous leptin is not effective in all women. Most physicians advocate the use of exogenous gonadal steroids to prevent osteoporosis. A regimen can consist of daily oral conjugated or esterified estrogens (0.625 to 1.25 mg), ethinyl estradiol (20 µg), or micronized estradiol-17β (1 to 2 mg) or transdermal estradiol-17β (0.05 to 0.10 mg) daily, with oral medroxyprogesterone acetate (5 to 10 mg) added for the first 12 to 14 days of each month. Sexually active women can be given oral contraceptive agents as an alternative. If steroid therapy is administered, patients must be informed that the amenorrhea will probably be present when therapy is discontinued. Other physicians believe that only periodic observation is indicated, with barrier methods of contraception recommended for fertility control. Adequate ingestion of calcium should be ensured regardless of therapy. Contraception is needed for sexually active women with HCA because the functional defect is mild in these disorders and may resolve spontaneously at any time, with ovulation occurring before any episode of menstruation.

Chronic Anovulation Related to Inappropriate Feedback

DEFINITION

PCOS is a heterogeneous disorder in which there is considerable clinical and biochemical variability among affected individuals. (See PCOS as the most common cause of heterosexual pubertal development in Chapter 235.) PCOS is currently considered to exist in women with any two of the following: (1) oligo-ovulation or anovulation, (2) hyperandrogenism, or (3) polycystic ovaries on ultrasound, and in whom other etiologies have been eliminated. PCOS is the classic disorder in which the amenorrhea or oligomenorrhea results from inappropriate feedback of gonadal steroids from the ovaries.

A

PATHOBIOLOGY

Current evidence suggests that the hypothalamic-pituitary unit is intact and that a functional derangement, perhaps involving insulin-like growth factors (IGFs) such as IGF-I within the ovary, results in abnormal gonadotropin secretion. PCOS is characterized by insulin resistance and compensatory hyperinsulinemia. (See association between PCOS, insulin resistance, and obesity in Chapter 220.) The insulin resistance has been found in affected women of many racial and ethnic groups, implying that it is a universal characteristic and that a common defect may be present. There is increasing evidence of specific genetic abnormalities in some women with PCOS.

CLINICAL MANIFESTATIONS

Although patients usually present with amenorrhea, hirsutism, and obesity, affected women may instead complain of irregular and profuse uterine bleeding, may not have hirsutism, and may be of normal weight (Fig. 236-5). Excess androgen from any source or increased extraglandular conversion of androgens to estrogens can lead to the typical findings of PCOS. Included are such diverse disorders as Cushing’s syndrome, mild congenital adrenal hyperplasia, virilizing tumors of adrenal or ovarian origin, hyperthyroidism and hypothyroidism, obesity, and primary PCOS with no other recognizable cause. In the primary syndrome, the irregular menses, mild obesity, and hirsutism begin during puberty and typically become more severe with time, although there is increasing evidence of improvement in the years just before menopause. Obesity alone can lead to a polycystic ovarian–like syndrome, with the degree of obesity required to cause anovulation varying widely. The increase in the prevalence of obesity is leading to an increased prevalence of PCOS. All such patients are well estrogenized regardless of whether they present with primary or secondary amenorrhea or dysfunctional bleeding. LH concentrations tend to be elevated, with relatively low and constant FSH levels, but both may be in the normal range for the follicular phase of the menstrual cycle. Levels of most circulating androgens, especially testosterone, tend to be mildly elevated.

DIAGNOSIS

After exclusion of other etiologies, two of the following three are required for diagnosis of PCOS: (1) hyperandrogenism (clinical or biochemical);

B FIGURE 236-5.  Adult with polycystic ovary syndrome (PCOS). This 28-year-old woman with documented PCOS had elevated luteinizing hormone levels, irregular menses, and hirsutism since puberty. Note the increased hair in the midline extending up to and above the umbilicus. Other findings (which are not necessarily abnormal) are periareolar hair and hypertrichosis of the arms.

(2) oligo-ovulation or anovulation; (3) polycystic ovaries on ultrasound examination or at surgery.12 This definition is confusing to clinicians because it implies that hirsute women with polycystic ovaries on ultrasound examination who ovulate regularly should be considered to have PCOS. Moreover, it is clear that

1594

CHAPTER 236  Reproductive Endocrinology and Infertility  

polycystic ovaries may be identified on ultrasound examination in normal women. In any case, the aim of the diagnostic evaluation is to rule out any causes (such as neoplasms) that require definitive therapy. Hirsutism should be evaluated as detailed in Chapter 442. A particularly severely affected subset of women present with marked obesity, anovulation, mild glucose intolerance with high levels of circulating insulin, acanthosis nigricans, hyperuricemia, severe hirsutism, and elevated circulating androgen levels. These women have hyperthecosis of the ovaries, in which the androgen-producing cells in the stromal, hilar, and thecal regions are increased greatly in number. Hyperthecosis should probably be viewed as a part of the spectrum of disorders constituting PCOS.

TREATMENT  Patients generally require therapy for hirsutism, for induction of ovulation if pregnancy is desired, and for prevention of estrogen-induced endometrial hyperplasia and cancer. No ideal therapy exists; the therapeutic approach must be individualized. The risks for metabolic syndrome, cardiovascular disease, and diabetes mellitus are increased in women with PCOS, at least in part because of the increased androgens and insulin resistance. Moreover, many women have elevated cholesterol levels.

Medical Therapy

In the anovulatory woman not desiring pregnancy who is not hirsute, therapy with intermittent progestin administration (e.g., medroxyprogesterone acetate, 5 to 10 mg orally for 10 to 14 days each month) or oral contraceptives can be provided to reduce the increased risk for endometrial carcinoma that is present in such a woman with unopposed estrogen. All women using intermittent progestin administration should be cautioned about the need for effective contraception if they are sexually active because these agents do not inhibit ovulation when they are administered intermittently. Improvements in insulin sensitivity in women with polycystic ovaries, either through lifestyle changes (i.e., exercise and diet) or through pharmacologic intervention, consistently result in improvements in the reproductive and metabolic abnormalities. Resumption of ovulation may occur in up to 60 to 70% of affected women. The longest and largest published experiences with any agent that improves insulin sensitivity in PCOS is with metformin, a biguanide that functions primarily by suppressing hepatic gluconeogenesis and also improves insulin sensitivity. A6  Its use in PCOS leads to reductions in insulin and androgen levels and resumption of menses in some women. Divided doses of 1500 to 2000 mg/ day have proved effective. Some clinicians advocate giving metformin to all women with polycystic ovaries, whereas others would administer such an agent only to those with documented insulin resistance. Some clinicians also advocate giving metformin first to women who desire pregnancy and then adding an agent to induce ovulation if the metformin proves ineffective. These agents are not approved for use in pregnant women or for the induction of ovulation.

Treatment Considering Pregnancy

Oral contraceptive agents are the first line of therapy for hirsute anovulatory woman not desiring pregnancy and offer protection from endometrial hyperplasia. In women with PCOS desiring pregnancy, clomiphene citrate or letrozole can be used to induce ovulation. A7  A8  Letrozole is not approved for this use by the FDA, but a large multicenter randomized trial has demonstrated its superiority to clomiphene in obese women with PCOS. A9  About 75 to 80% conceive with such therapy. In addition to insulin-sensitizing agents, other possible methods of inducing ovulation include use of exogenous gonadotropins and HCG, and laparoscopic ovarian surgery with multiple punctures of the ovary by diathermy or laser. A large clinical trial documented that clomiphene citrate is more effective than metformin in inducing ovulation and resulting in pregnancy; there was no further improvement when the two agents were used concurrently. A10  ,

Surgical Treatment

Laparoscopic ovarian surgery can achieve unifollicular ovulation or make it easier for medical ovulation induction but increases the risk for development of ovarian adhesions (themselves leading to infertility). It may be successful in a small subset of women with PCOS who are geographically removed from good medical care.

Chronic Anovulation Related to Other Endocrine and Metabolic Disorders Adrenal hyperfunction appears to cause chronic anovulation by inducing a polycystic ovarian-like syndrome secondary to increased adrenal androgen secretion. Both hyperthyroidism and hypothyroidism are associated with a

variety of menstrual disturbances, including dysfunctional uterine bleeding and amenorrhea as a result of alterations in the metabolism of androgens and estrogens. These metabolic changes in turn result in inappropriate steroid feedback and chronic anovulation.

Luteinized Unruptured Follicle Syndrome The luteinized unruptured follicle syndrome refers to the development of a dominant follicle without its subsequent disruption and release of the ovum. The abnormality can be diagnosed by ultrasonography or by the absence of evidence of ovulation when the ovary is viewed at laparoscopy. The disorder occurs infrequently and is not a significant cause of infertility. Menstrual cycles in which no ovum is released are characterized by presumptive evidence of ovulation, including biphasic basal body temperatures, secretory endometrium, normal LH surge, and normal progesterone production in the luteal phase.

Luteal Phase Dysfunction

PATHOBIOLOGY

Progesterone secretion in the luteal phase may be reduced in duration (termed luteal phase insufficiency) or in amount (termed luteal phase inadequacy). More rarely, the endometrium may be unable to respond to secreted progesterone because of the absence of progesterone receptors. These disorders represent causes of infertility (because of inability of fertilized ova to implant) in less than 5% of infertile couples. Abnormalities of the follicular phase, especially in the frequency of gonadotropin pulses, may account for most luteal phase defects. Luteal phase defects may also occur sporadically in normally ovulating women.

DIAGNOSIS

Luteal phase dysfunction may be associated with several clinical entities, including mild or intermittent hyperprolactinemia, strenuous physical exercise, inadequately treated 21-hydroxylase deficiency, and recurrent miscarriage. Luteal dysfunction occurs more commonly at the extremes of reproductive life and in the first menstrual cycles after full-term delivery, abortion, or discontinuation of oral contraceptives. It may also occur during ovulatory cycles induced with clomiphene citrate or exogenous gonadotropins and HCG.

TREATMENT  Treatment of luteal dysfunction is controversial. Any underlying defect should be treated. If subsequent luteal function depends on prior follicular development, modification of follicular development with either clomiphene citrate (25 to 100 mg daily by mouth for 5 days beginning on cycle day 3 to 5) or FSH (75 to 300 IU intramuscularly for 3 to 5 days beginning on cycle day 3 to 5) can be used; HCG (2500 to 5000 IU intramuscularly at 2- to 3-day intervals beginning with the shift in basal body temperature) or progesterone (12.5 mg intramuscularly in oil daily or 25 mg twice a day as rectal or vaginal suppositories) can be used as well. Bromocriptine may correct the abnormality in individuals with hyperprolactinemia. Synthetic progestational agents should not be used to treat luteal phase defects because of their possible association with congenital anomalies. Furthermore, the synthetic progestins produce an abnormal endometrium. None of these agents has been shown to increase the pregnancy rate.

INFERTILITY DEFINITION

The World Health Organization (WHO) has defined infertility as “a disease of the reproductive system defined by the failure to achieve a clinical pregnancy after 12 months or more of regular unprotected sexual intercourse.” Sterility is total inability to reproduce. More than 10% of couples in the United States seek medical assistance for infertility. The requirements for pregnancy to occur are several: • The male must produce adequate numbers of normal, motile spermatozoa. • The male must be capable of ejaculating the sperm through a patent ductal system. • The sperm must be able to traverse an unobstructed female reproductive tract. • The female must ovulate and release an ovum.

CHAPTER 236  Reproductive Endocrinology and Infertility  

TABLE 236-5  CAUSES OF INFERTILITY AND THEIR APPROXIMATE INCIDENCE (WHERE AVAILABLE)* Male factors (40%) Decreased production of spermatozoa Varicocele Testicular failure Endocrine disorders Cryptorchidism Stress, smoking, caffeine, nicotine, recreational drugs Ductal obstruction Epididymal (after infection) Congenital absence of vas deferens Ejaculatory duct (after infection) After vasectomy Inability to deliver sperm into vagina Ejaculatory disturbances Hypospadias Sexual problems (i.e., impotence), medical or psychological Abnormal semen Infection Abnormal volume Abnormal viscosity Immunologic factors Sperm-immobilizing antibodies Sperm-agglutinating antibodies Female factors Fallopian tube disease (20-30%) Pelvic inflammatory disease or puerperal infection Congenital anomalies Endometriosis Secondary to past peritonitis of nongenital origin Amenorrhea and anovulation (15%) Minor ovulatory disturbances (25% vertebral height loss) appear to better predict future fractures, as do nonvertebral fractures. When fractures are suspected, CT and MRI may be used, given that plain radiographs have a lower sensitivity acutely and with stress fractures. MRI also can be used to define a vertebral fracture with persistent swelling and edema based on T2 characteristics, potentially identifying patients who could benefit from vertebroplasty or kyphoplasty (see later). Finally, whole body bone scintigraphy is the most sensitive test for fracture but can be falsely positive because of inflammation, infection, or tumor and usually is positive for 6 to 12 months after a fracture event.

FIGURE 243-5.  Incidental vertebral compression fractures on chest radiograph. Lateral radiograph of the chest of a 74-year-old man studied for cough. No relevant pulmonary abnormality was noted on the frontal radiograph (not shown). Examination of the thoracic spine shows the presence of a mild anterior wedge compression fracture of T9 (thick arrow) and moderate anterior wedge fracture of T6 (thin arrow). Neither fracture was reported on the radiographic report.

DIAGNOSIS

1641

Although a recent fragility fracture is a reasonable basis for diagnosis of osteoporosis, other skeletal conditions should also be entertained, including inherited and acquired osteomalacias and pathologic fracture due to malignancy. These disorders can often be distinguished by history and physical examination, although additional investigations may be required. This distinction is critical because therapies may differ greatly between disorders. Most patients with osteoporosis are diagnosed on the basis of BMD measurement, generally by dual-energy x-ray absorptiometry (DXA). DXA is a low-radiation-based radiologic measurement of the areal bone density (g/ cm2) of the lumbar spine, proximal femur, and distal radius. Osteoporosis can be diagnosed if the BMD of a postmenopausal woman or man older than 50 years is more than 2.5 SD below young average normal (T score ≤ −2.5). A T score between −1.0 and −2.5 is considered low bone density or osteopenia, and a Z score (age-matched BMD) in premenopausal women and men younger than 50 years that is more than 2 SD below that of an a average age-matched individual is considered low bone density for age. BMD is an independent predictor of fracture risk, such that the relative risk for fracture increases by 1.5- to 2-fold for each 1 SD decrease in T score. In addition, fracture risk increases exponentially below a T score of −2.5. Furthermore, BMD of the femoral neck may be used in fracture prediction models such as FRAX to better define an individual’s risk for subsequent fracture (see later). In addition to DXA, other modalities are also used to diagnose osteoporosis, including quantitative CT of the spine (QCT) and wrist and tibia (pQCT), finger DXA, and ultrasound of the calcaneus or wrist. Measurement of BMD by all of these techniques has been shown to globally predict fractures, akin to DXA. QCT and pQCT provide additional information on cortical and trabecular bone compartments but are accompanied by higher radiation exposure and poorer reproducibility compared with DXA. Ultrasound is radiation free and easy to operate but is less sensitive in diagnosing osteoporosis and does not measure change in a reliable fashion in response to age or treatment, making it useful as a screening modality but not for longitudinal care. Although DXA is an effective diagnostic tool, several potential limitations and caveats need to be considered by the clinician. First, DXA cannot distinguish between low bone density and undermineralized bone matrix, the latter of which occurs in osteomalacia (Chapter 244). BMD may also be quite disparate between regions, perhaps in more than one third of individuals. This inconsistency results from a number of factors, including differences in bone composition (predominantly trabecular bone in the spine and cortical bone in the one-third radius) with resultant variations in rates of bone loss due to aging and disease (vertebral bone loss with menopause and glucocorticoid use versus cortical bone loss in hyperparathyroidism). Degenerative changes due to aging, such as facet osteoarthritis and aortic calcification, may artifactually raise spine BMD value. Given these considerations, the lowest skeletal site should be used for diagnosis. Finally, BMD should be measured longitudinally on the same DXA machine if possible, because of intermachine and intermanufacturer differences that may confound the ability to validly measure change over time. Despite these caveats, DXA remains the best method to diagnose and manage osteoporosis by bone density testing. Despite its utility, bone density has been limited historically in optimally predicting fracture risk in individual patients. In addition, BMD does not take into account clinical factors that independently predict fracture. Under this premise, fracture prediction models have been developed that combine BMD and risk factors to better stratify fracture risk. The best known and most widely used of these prediction models is FRAX. FRAX was developed by the World Health Organization in collaboration with national and international osteoporosis foundations as an Internet-based computer algorithm that defines a person’s 10-year risk for hip and major osteoporotic fracture (hip, clinical spine, forearm, and proximal humerus all combined). The model uses country-specific data on clinical risk factors and femoral neck BMD to calculate fracture probability and is available as a web-based tool that can be used by clinicians with their patients to assist in making informed decisions on osteoporosis management (http://www.shef.ac.uk/FRAX/). FRAX can be also be used to define country-specific recommended diagnostic and treatment thresholds. An example of this is the National Osteoporosis Foundation guidance that 10-year risks equal to or exceeding 3 and 20% for hip and major fracture risk, respectively, warrant consideration of pharmacologic treatment, which is based on cost-effective analyses in the United States. Furthermore, the number needed to treat (NNT) can be determined to best

1642

CHAPTER 243  Osteoporosis  

inform patients of their expected risk and benefit of treatment (e.g., bisphosphonate use roughly reduces hip fracture risk by half, or from 10 to 5%, with NNT of 1/0.05, or 20 patients treated to prevent one hip fracture). Despite its utility and ease of use, FRAX does have limitations. These include inability to use patients who are not treatment naïve, underestimation of fracture risk in those with disparately lower BMD in the spine than hip, absence of fall history/fall risk in the model, and the use of fixed clinical risk factors. Modifications have been suggested for the latest guidelines, including adjusting FRAX score up or down based on glucocorticoid dose. In addition, although fracture risk calculators that incorporate fall risk are available (e.g., from the Garvan Institute), they do not include the competing risk of mortality as FRAX does. As such, FRAX should be viewed, not as a perfect, but rather as a complementary tool to BMD in best defining a person’s risk for fracture and candidacy for pharmacologic intervention. Finally, all patients presenting with osteoporosis require an assessment for secondary causes of bone loss, given that 20 to 25% of women and perhaps an even greater portion of men will have identifiable additional etiologies that may contribute to bone loss (see Table 243-1). Most patients will have had routine chemistry, hematology, and thyroid studies as part of their annual examination. The 25(OH)D level should be measured in all patients for multiple reasons, as previously discussed. Additional investigations may also be considered, as directed by the clinical history and physical examination. In addition, a greater degree of BMD deficit (i.e., lower Z score) indicates a need for more extensive testing, given the greater likelihood of secondary causes being present. Bone turnover markers (BTMs) are serum and urinary products of bone formation or resorption that can also be used to assist in management. Available tests include bone-specific alkaline phosphatase, osteocalcin, type I procollagen amino-terminal propeptide, and type I procollagen carboxy-terminal propeptide as formation markers, and serum and urine C- and N-terminal peptides of type I collagen as resorption markers, among others. Their use is predicated on studies showing that high bone turnover increases fracture risk independent of BMD. In addition, fracture risk reduction correlates well with reduction in bone turnover based on clinical trials with anticatabolic agents. Nonetheless, their clinical utility has been tempered to date by several issues. First, there is significant biologic variability due to nonmodifiable (e.g., age, gender, underlying comorbid disease, medications) and modifiable (e.g., time of day, food intake, presence of fracture) factors that limit the ability to detect meaningful change over time in an individual patient. Second, optimal specimen processing is required for valid results and interpretation. Finally, and perhaps in part secondary to these issues and others, evidence to date does not demonstrate significant benefit of BTMs in individual patients in securely predicting bone density increase, fracture risk reduction, or cost effectiveness through patient feedback and improved adherence. Therefore, at present, BTMs should not be used in routine clinical practice, although they could help inform management in more complicated cases of metabolic bone disease.9

PREVENTION AND TREATMENT  Calcium

Adequate intake of calcium is critical to the optimal accumulation and maintenance of bone mineral density. Calcium supplementation has meaningful impact on BMD and fracture risk reduction, although the latter is much less certain. BMD is modestly improved by 1 to 2%, although evidence of a definitive reduction in hip and nonvertebral fracture risk when given without vitamin D is lacking at present, although previous studies have suggested a trend toward vertebral fracture risk reduction. Given the established increase in rate of nephrolithiasis and a possible, albeit unproven, potential increase in nonfatal cardiac events with higher dose calcium supplementation, it would seem prudent to recommend that adults with osteoporosis obtain 1200 to 1500 mg of calcium from a combination of supplements and dietary sources.

Vitamin D

Appropriate circulating levels of 25(OH)D are necessary for optimal intestinal absorption of calcium and skeletal accrual and maintenance. Despite this, a significant proportion of children and adults have vitamin D levels that would be deemed insufficient (i.e., 25[OH]D < 20 ng/mL). Data in adults with osteoporosis confirm a benefit of vitamin D supplementation for fracture risk reduction, although the effect is dependent on the patient population and the amount of supplementation. Doses of 400 to 800 IU of vitamin D combined with 1000 mg of calcium reduce the risk for hip fracture in postmeno-

pausal women and men aged 65 years and older, although the benefit is less certain for community-dwelling individuals than for those in assisted living centers. A1  In addition, it appears that a 25(OH)D level of at least 30 ng/mL is needed to reduce the risk for hip fracture, A2  although there is some controversy over this recommendation based on the results of other meta-analyses with likely different methodologies. In addition, a daily vitamin D intake of at least 800 IU also reduces the risk for falls, likely by improving muscle strength and reducing body sway. A3  It should be noted that the recent U.S. Preventive Services Task Force recommendation against the use of calcium and vitamin D10 was based on the general U.S. population and does not pertain to patients with osteoporosis. Finally, although activated vitamin D analogues such as calcitriol and α-calcidiol have been shown to reduce fracture risk, they are generally not indicated based on unacceptable risk for hypercalcemia. The exception to use of vitamin D analogues is possibly patients with stage 3 and 4 chronic kidney disease, wherein treatment of secondary hyperparathyroidism could provide skeletal benefit.

Exercise and Lifestyle

Physical activity is also a critical element of osteoporosis management, which can be indirectly inferred based on the known profound effects of decreased gravitational force (i.e., immobilization, paraplegia, weightlessness in space) on inducing bone loss. Physical activity likely also confers additional benefits through enhanced muscle strength, improved cardiovascular status, and reduction in fall risk. Meta-analysis confirmed a modest benefit of exercise on lumbar spine (mean difference = 0.85%) and trochanteric BMD (mean difference = 1.03%) in postmenopausal women compared with placebo, although it did not show significant changes in femoral neck or total hip BMD. A4  However, studies to date have not confirmed an improvement in bone strength with exercise in this patient group. Studies concerning middle-aged and older men are much more limited in number and quality, although preliminary evidence suggests that resistance training with or without impactloading activities has the greatest BMD benefit. Importantly, although none of the aforementioned studies have demonstrated a clear antifracture benefit from exercise, there are abundant data that multiple targeted exercise interventions do reduce either the risk for falling (Tai Chi) or both the rate and risk for falling (group and home-based exercise programs), A5  which is most likely an inciting event in older patients incurring an osteoporotic fracture. Finally, modification of aberrant lifestyles is also indicated in patients with osteoporosis, especially tobacco cessation and moderation of caffeine, carbonated beverage, and alcohol intake. Data are lacking, however, on whether these reduce overall fracture risk.

Medications

There is robust evidence that pharmacologic therapy significantly reduces the risk for osteoporotic fracture in a clinically meaningful and cost-effective manner.11 Medications approved for osteoporosis can be classified based on their mechanism of action: anticatabolic (i.e., antiresorptive) and anabolic (i.e., bone building).

Anticatabolic Agents

Anticatabolic medications, or antiresorptive as they were more commonly known, inhibit osteoclast recruitment, function, and/or survival, resulting in reductions in skeletal turnover and bone loss. These agents, depending  on the potency and persistence of bone effect, reduce the number of new activation sites (BMUs) and the bone remodeling space, thereby improving BMD while strengthening the skeletal microstructure and reducing fracture risk.

Bisphosphonates

Bisphosphonates (BPs) are the most widely prescribed and used medications for the treatment of osteoporosis, owing in large part to good tolerability and an ability to dose them infrequently (from once weekly to once yearly, depending on the drug). BPs are chemically engineered analogues of the naturally occurring molecule pyrophosphate in which a carbon is substituted for an oxygen. As a result, BPs have an extremely high affinity for hydroxyapatite crystals within bone. After incorporation into bone, BPs are taken up by osteoclasts and thereafter inhibit cellular attachment, function, and survival. The carbon side-chain molecules largely determine skeletal affinity and potency of BP effect. The first-generation BP etidronate, which is not approved in the United States for treatment of osteoporosis, is the least potent agent of the class. It must also be given in an interrupted fashion for 2 weeks every 3 months owing to the potential to cause focal osteomalacia, and it may cause lower gastrointestinal symptoms (i.e., abdominal pain and diarrhea). Nonetheless, it is has been shown to reduce the risk for vertebral but not nonvertebral nor hip fractures. Three oral bisphosphonates are approved by the U.S. Food and Drug Administration (FDA) and currently available in the United States: alendronate, risedronate, and ibandronate, in order of time since initial FDA approval. All three drugs are also available as generic preparations, although some differences do exist between the brand name and generic drugs in regard to the

CHAPTER 243  Osteoporosis  

TABLE 243-2  STRENGTH OF EVIDENCE FOR THE REDUCTION OF RISK FOR FRACTURE TYPES WITH PHARMACOTHERAPY IN WOMEN WITH POSTMENOPAUSAL OSTEOPOROSIS FRACTURE SKELETAL SITES VERTEBRAL

NONVERTEBRAL

HIP

WRIST

Alendronate

•••

•••

•••



Ibandronate

•••

••



ı

Risedronate

•••

•••

•••



Zoledronate

•••

•••

•••

ı

Denosumab

•••

•••

•••

ı

Teriparatide

•••

••



ı

Raloxifene

•••

ı

ı

ı

Strength of evidence symbol legend: ı = insufficient strength of evidence; • = low strength of evidence; • • = moderate strength of evidence; • • • = high strength of evidence. (Adapted with permission from Levis S, Theodore G. Summary of AHRQ’s comparative effectiveness review of treatment to prevent fractures in men and women with low bone density or osteoporosis: update of the 2007 report. J Manag Care Pharm. 2012;18[4 Suppl B]:S1-S15, discussion S13.)

inactive excipients. The oral BPS may be administered once weekly (alendronate and risedronate) or once monthly (risedronate and ibandronate), fasting in the morning with water only, and the patient must remain fasting in a sitting or standing position for 30 to 60 minutes after the dose. Recently, a delayedrelease formulation of risedronate (Atelvia) was approved that may be taken immediately after breakfast. The most common side effect is precipitation or aggravation of gastroesophageal reflux, although most patients tolerate the drugs without difficulty. In light of this side effect and a potential risk for esophageal irritation and ulceration, these drugs are contraindicated in patients with functional or anatomic disorders of esophageal transit (i.e., esophageal stricture, achalasia). All three drugs significantly reduce the risk for vertebral fractures, although high-strength evidence for hip and nonvertebral fracture risk reduction exists for alendronate and risedronate but not ibandronate (Table 243-2). A6  In addition, studies confirm a persistent BMD and likely antifracture benefit after 5 years of therapy. Parenteral BPs are also approved and available for osteoporosis treatment, although they should be considered second line to oral BPs based on overall risk-benefit assessment in most osteoporotic patients. They may be considered for use in patients with contraindications to oral BPs (e.g., esophageal disease, inability to sit upright and/or fast after dose), documented or expected poor adherence to oral BPs, or failure to respond to oral BPs or other FDAapproved therapies (recurrent fractures, declining BMD). Zoledronic acid, 5 mg once yearly, and ibandronate, 3 mg quarterly, may be given, although highstrength evidence would favor the use of zoledronic acid, given its unequivocal effect on spine, hip, and nonvertebral fracture risk reduction. A6  Intriguingly, zoledronic acid has also been shown to reduce mortality in women and men following a low-trauma hip fracture, A7  although the mechanism of the mortality benefit is unknown. Finally, BMD remains stable and the antifracture effect likely persists for 3 years after three annual doses of zoledronic acid. A8  Both drugs are associated with an approximately 15 to 20% likelihood of a flu-like reaction, typically consisting of fever, arthralgias, and myalgias, usually limited to the first infusion, and generally lasting 24 to 48 hours, although symptoms lasting weeks to months have rarely been reported to the FDA. Both drugs confer a higher risk as well for delayed healing of exposed bone in the oral cavity compared with oral BPs (see later). Rare, considerably more serious adverse effects have been associated with both oral and intravenous BPs. Osteonecrosis of the jaw, which is defined as exposed bone within the oral cavity for more than 8 weeks following an invasive dental procedure (e.g., tooth extraction, dental implant) or spontaneous tooth loss, occurs in roughly 1 in 10,000 to 100,000 patients treated with oral BPs, although it likely occurs in 1 in 1000 to 10,000 in intravenous BP−treated patients with osteoporosis. Current evidence suggests that microbial biofilm formation on an acellular bone surface, perhaps facilitated by BPs and the non-BP drug denosumab (see later), may be operative in the development of this disorder. As such, patients on intravenous BPs should maintain optimal oral hygiene and consider a BP holiday or delay in dose if invasive oral procedures are planned. Atypical femoral fractures have also been recently described in patients on long-term bisphosphonate therapy, generally after 5 years or more of treatment. Patients will typically have prodromal thigh or groin pain, which is referable to a stress fracture of a thickened lateral femoral cortex, inferior to the greater trochanter. These fractures can be bilateral in

1643

nature and may be identified radiographically with plain films, MRI, or CT. These patients are at risk for low-trauma, severe, oblique, “chalk-stick” fractures, which often represent orthopedic repair and healing challenges. Fortunately, the estimated prevalence of atypical femoral fractures is low (~1 in 5,000 to 10,000). Nonetheless, the severe manifestations of osteonecrosis of the jaw and atypical femoral fractures make it prudent for clinicians to consider a BP drug holiday, particularly given strong evidence of continued benefit on discontinuation.

Selective Estrogen Receptor Modulators

Selective estrogen receptor modulators (SERMs) are compounds that bind to the estrogen receptor and thereby influence bone and reproductive biology. As with estrogen (see later), SERMs are anticatabolic agents in bone, acting through a reduction in cytokines (RANKL, tumor necrosis factor-α) that engender osteoclast activation and function. Raloxifene is the only FDA-approved drug for prevention and treatment of osteoporosis in menopausal women, although the breast cancer drug tamoxifen likely has skeletal benefits as well. Both drugs have antiestrogenic effects in the breast and are FDA approved  for the prevention of breast cancer in high-risk patients. Raloxifene does reduce the risk for vertebral fractures by approximately 30 to 50% but does not reduce the risk for hip and nonvertebral fractures. This antifracture profile positions it as an alternative to bisphosphonates in postmenopausal women with osteopenia and a relatively low risk for hip and other nonspine fractures. The most common side effects include hot flushes and leg cramps in about 10 to 15% and about 5% of patients, respectively. SERMs also increase the risk for deep vein thrombosis, with an absolute risk of roughly 1 in 400, akin to that seen with oral estrogen replacement therapy (ERT). Raloxifene has also been associated with an increased risk for fatal stroke in women at higher baseline risk for stroke, likely precluding its general consideration in women older than 65 years.

Estrogen

ERT, either alone or in combination with a progestin in women with an intact uterus, had historically been a frontline agent in the management  of osteoporosis in postmenopausal women (Chapter 240). ERT prevents  bone loss if administered to women at menopause and significantly increases BMD by approximately 3 to 5% in woman who are well into their menopausal years. Although lower doses of estrogen may have skeletal benefits, more standard doses of estrogen (0.625 mg of conjugated equine estrogen and 1.0 mg of ethinyl estradiol) have been proved efficacious. Long-term estrogen therapy reduces the risk for all clinical fractures by about 27%, based  on the available moderate-quality evidence. A9  ERT is also the most efficacious agent available for treatment of vasomotor symptoms. These data not­ withstanding, ERT is associated with an increased risk for stroke (34% increase), A9  and the use of continuous combined hormone replacement therapy (HRT) confers an unacceptable greater global risk than benefit in woman initiating HRT, based on the results of the Woman’s Health Initiative. These results, however, may not be applicable to the younger postmenopausal population, based on differences in cardiovascular risk, although data confirming this are currently lacking. Both ERT and HRT are also associated with a two- to three-fold increase in the risk for venous thromboembolic disease. Therefore, ERT/HRT is recommended only for postmenopausal women at significant risk for fracture for whom other antifracture therapies are unsuitable.

Denosumab

As detailed previously, increased osteoclast activation through the RANKL pathway is a key mechanism through which bone loss occurs in menopause and other osteoporotic conditions. Intuitively, a therapy that targets this process directly would be desirable. Denosumab is a fully human monoclonal antibody to RANKL that is FDA approved for the treatment of osteoporosis in postmenopausal women and in men, as well as for individuals with breast and prostate cancer to reduce bone loss associated with hormonal deprivation therapy. It is administered twice yearly as a subcutaneous injection in the clinic and clearly reduces the risk for spine, hip, and nonvertebral fractures. A10  Denosumab does not undergo hepatic or renal metabolism and thus can potentially be used in patients with more advanced renal dysfunction, unlike BPs. In contrast to BPs, it is reversible, such that robust bone loss ensues once the medication is stopped. Denosumab is well tolerated in clinical studies, although a higher incidence of skin conditions (eczema and erysipelas) and infections, including serious infections that required hospitalization, were observed in drug- versus placebo-treated subjects. Therefore, the drug is likely not suitable for patients on immunosuppressant therapy who are at higher baseline risk for infection.

Anabolic Agents

Although anticatabolic drugs are effective at retarding bone loss and reducing fracture risk, anabolic or “bone-building” drugs would be preferred. Teriparatide (TPTD) is a recombinant human parathyroid hormone analogue that encompasses amino acids 1 to 34 and was approved by the FDA in 2002. Given

1644

CHAPTER 243  Osteoporosis  

as a self-administered once-daily subcutaneous injection, TPTD is truly anabolic based on robust increases in bone density (~10% over 2 years in the lumbar spine) and bone formation as determined by bone biopsies and other sophisticated imaging studies. More important, TPTD significantly reduces the risk for vertebral and nonvertebral fractures by approximately two thirds and one half, respectively. Because bone resorption increases along with bone formation, bone loss generally ensues on cessation of therapy, necessitating the initiation of an anticatabolic bone drug to preserve the increase in BMD facilitated by TPTD. Finally, although it is plausible to consider that a combination of TPTD and an anticatabolic drug is more beneficial than either drug alone, evidence from randomized controlled trials to date has failed to confirm this. Recent studies, however, suggest that the combination of TPTD and denosumab may have a truly synergistic effect on BMD. TPTD is more expensive than other treatments for osteoporosis, although it is generally covered by insurance in patients who have severe osteoporosis (based on BMD and/or fracture risk) and who cannot tolerate or have contraindications to other antifracture agents. The drug is generally well tolerated, with the most common adverse effects being dizziness and leg cramps. TPTD has a black box warning, based on the fact that toxicology studies in rats revealed an increase in risk for osteosarcoma in animals treated with suprapharmacologic doses of the drug, particularly in growing animals. Given this, the drug is contra-indicated for patients who are at a higher baseline risk for osteosarcoma, including patients with Paget disease and previous therapeutic radiotherapy, as well as younger individuals with open epiphyses. Fortunately, there has not been an observed increase in the rate of osteosarcoma in teriparatide treated patients above that expected in the general population to date.

Other Therapies and Treatment Considerations Currently Available and Emerging Therapies

Nasal calcitonin is FDA approved and available at the time of this writing for treatment of postmenopausal osteoporosis, although it is widely considered the weakest antifracture agent based on marginal vertebral fracture benefit. In addition, recent human studies have suggested a possible link to cancer, potentially further limiting its clinical utility and future availability in the United States. Strontium ranelate is approved in Europe for the treatment of osteoporosis and may have a dual proformation-anticatabolic effect on bone. It has been shown to reduce the risk for vertebral and nonvertebral fractures, as well as clinical osteoporotic fractures. A11  It is not available for use in the United States, and alternative forms of strontium salts cannot be assumed to be effective as well. In addition, BMD by DXA cannot be followed in patients on strontium because of artifactual increases in BMD related to the incorporation into bone of the strontium salt. Emerging therapies on the horizon will likely provide additional tools to treat this debilitating disease, including new anticatabolic agents (e.g., cathepsin K inhibitors) and new anabolic therapies (e.g., sclerostin antibody). Odanacatib is an oral, small molecule that reversibly inhibits cathepsin K, which is produced by activated osteoclasts and primarily is responsible for the breakdown of type 1 collagen. Odanacatib also does not significantly suppress bone formation, perhaps “uncoupling” bone turnover in a favorable fashion to potentiate improvements in BMD. Sclerostin is a naturally occurring inhibitor of the Wnt pathway and bone formation, and preliminary clinical studies do confirm a significant anabolic effect and BMD increase with intermittent administration of a monoclonal antibody to sclerostin. A12  Interestingly, unlike TPTD, inhibition of sclerostin does not appear to stimulate bone resorption, potentially affording greater and more persistent gains in BMD. Ongoing and future clinical studies are needed to confirm an antifracture benefit of this compound.

Glucocorticoid-Induced and Male Osteoporosis

As detailed previously, glucocorticoids are a major cause of and the most common etiology of medication-related secondary osteoporosis. Glucocorticoids are prescribed for a number of common inflammatory conditions, often in a chronic, long-term manner. They are potent suppressors of bone formation and at higher doses likely increase bone resorption, principally through central suppression of sex steroid production. This resultant “uncoupling” of bone turnover can result in dramatic declines in BMD within the first 6 months of starting therapy. In addition to bone loss, there is good evidence to support that individuals on glucocorticoids may fracture at a higher level on BMD compared with non-glucocorticoid-treated patients. Fracture rates are increased as well with doses of prednisone as low as 2.5 mg per day, although the increase in risk appears to attenuate with glucocorticoid discontinuation. The treatment approach to glucocorticoid-induced osteoporosis is similar to osteoporosis in general, with the exception that attempts should be made to reduce the steroid dose to as low as the underlying treated disease will permit.12 Calcium and vitamin D are important adjuncts but are insufficient to prevent bone loss or fractures. Although not clearly evidence-based, replacement of deficient sex steroids is a reasonable strategy in younger individuals who are at lower risk for fracture. The BPs alendronate, risedronate, and zoledronic acid are FDA approved for glucocorticoid-induced osteoporosis in

women and men, although the established benefit is based primarily on BMD improvement. A more logical and indeed superior treatment of glucocorticoidinduced osteoporosis is TPTD, which as an anabolic drug more directly addresses the primary mechanism of bone loss in glucocorticoid-induced osteoporosis: osteoblast inhibition. TPTD is FDA approved for treatment of glucocorticoid-induced osteoporosis in women and men and is superior to alendronate in improving BMD and vertebral fracture risk reduction. A13  Although the drug was used for 36 months in this head-to-head trial, treatment is advised for no more than 24 months based on previously mentioned safety considerations. Male osteoporosis historically has been under-recognized and underappreciated by primary care clinicians and patients alike, although the current data support a significantly more prevalent and clinically significant disorder. More than 2 million men in the United States have osteoporosis, and one in four men older than 50 years will suffer a fragility fracture in their remaining lifetime. Roughly 30% of vertebral and hip fractures combined occur in men, and these are the more common fractures in older men. In addition, men have a substantially higher mortality after hip fracture compared with women. As in women, aging, low body weight, and prior fragility fractures are independent predictors of fracture. In some contradistinction to women, however, osteoporosis in men is more commonly multifactorial in etiology, with the most common secondary causes being excess glucocorticoids, hypogonadism, and alcohol overuse. Despite these associations and others (current smoking, history of falls), there is not at present sufficient evidence to warrant use of a specific testing or screening strategy to identify men at higher risk for fracture. The laboratory work-up of male osteoporosis is similar to that for women, with the exception of a morning fasting testosterone level. Idiopathic osteoporosis may also occur, particularly in younger men with no discernable cause. Genetic factors may well be important in these men, with studies suggesting an association with lower production and circulating levels of estrogen. As in women, primary treatment of male osteoporosis is targeted at lifestyle changes, adequate nutrition (calcium and vitamin D), and exercise. Bisphosphonates (oral and intravenous), denosumab, and TPTD are all effective at improving BMD in men. Although more limited in scope, antifracture efficacy is evident for denosumab in men with prostate cancer on androgen deprivation therapy. True antifracture efficacy for the other agents is either less convincing or absent, based on the paucity of randomized controlled trial data, although this should not be construed as a reason not to treat. Testosterone replacement of men with significant biochemical hypogonadism (total T score < 200 ng/dL) does improve bone density, although data on fracture risk reduction are lacking. In older men (>50 years) at a substantial risk for fracture based on history and risk factors, androgen replacement should be considered second line behind the aforementioned other therapies, based on overall risk-benefit and lack-of-fracture data.

Vertebroplasty and Kyphoplasty and Low-Intensity Vibration

Although often clinically silent, vertebral fractures may cause acute and severe back pain. In addition, up to one third of vertebral fractures remain chronically painful, perhaps related to incomplete healing or instability of  the fracture. Over the past decade, vertebroplasty and kyphoplasty have been developed and advanced to reduce the morbidity associated with  acute spine fractures. These invasive procedures introduce, through the spinal pedicles, a cement-like substance (polymethylmethacrylate) to the compressed vertebral body, with (kyphoplasty) or without (vertebroplasty) use of saline-infused balloon tamps that permit a few millimeters of elevation of the vertebral end plates. Initial randomized trials suggested a benefit of vertebroplasty over conservative management in patients with acute vertebral fractures, although a recent meta-analysis of patient-level data from  two randomized controlled trials did not confirm this finding. A14  Additionally, there may be a concern about fracture of adjacent vertebrae following the procedure, reinforcing the need for further, adequately powered and designed clinical trials. Low-intensity vibration is also under active investigation as an anticatabolic and possibly anabolic intervention for osteoporosis. Animal studies using lowintensity vibration appears to show enhanced osteoblast and hindered osteoclast development, thereby “coupling” bone remodeling. Clinical studies suggest a modest but significant BMD benefit in postmenopausal women and other groups (children with cerebral palsy, adults on prolonged bed rest), although further studies are needed to confirm a true clinical and ideally antifracture benefit of this intervention.

PROGNOSIS

It stands to reason that, based on the information and data discussed previously, the burden incurred by individual patients and society as a whole can be significantly lessened through a combination of diagnostic, preventive, and therapeutic interventions. Although there is no true “cure” for osteoporosis, current pharmacotherapies reduce the risk for fracture roughly by half.

CHAPTER 244  Osteomalacia And Rickets  

This reduction is critical because there is robust evidence to suggest an independent increase in mortality after an osteoporotic fracture, including fractures of the spine, humerus, tibia, and pelvis, as well as the proximal femur. Moreover, available data, primarily from randomized controlled trials with bisphosphonates, confirm a statistically significant reduction in death with pharmacologic treatment of osteoporosis, although the mechanism of this effect is not known.13 These data further underscore the importance of identifying and treating patients with osteoporosis.

Grade A References A1. Avenell A, Gillespie WJ, Gillespie LD, et al. Vitamin D and vitamin D analogues for preventing fractures associated with involutional and post-menopausal osteoporosis. Cochrane Database Syst Rev. 2014;14:CD000227. A2. Bischoff-Ferrari HA, Willett WC, Wong JB, et al. Prevention of nonvertebral fractures with oral vitamin D and dose dependency: a meta-analysis of randomized controlled trials. Arch Intern Med. 2009;169:551-561. A3. Bischoff-Ferrari HA, Dawson-Hughes B, Staehelin HB, et al. Fall prevention with supplemental and active forms of vitamin D: a meta-analysis of randomised controlled trials. BMJ. 2009;339:b3692. A4. Howe TE, Shea B, Dawson LJ, et al. Exercise for preventing and treating osteoporosis in postmenopausal women. Cochrane Database Syst Rev. 2011;7:CD000333. A5. Gillespie LD, Robertson MC, Gillespie WJ, et al. Interventions for preventing falls in older people living in the community. Cochrane Database Syst Rev. 2012;9:CD007146. A6. Levis S, Theodore G. Summary of AHRQ’s comparative effectiveness review of treatment to prevent fractures in men and women with low bone density or osteoporosis: update of the 2007 report. J Manag Care Pharm. 2012;18:S1-S15. A7. Lyles KW, Colon-Emeric CS, Magaziner JS, et al. Zoledronic acid and clinical fractures and mortality after hip fracture. N Engl J Med. 2007;357:1799-1809. A8. Black DM, Rein IR, Boonen S, et al. The effect of 3 versus 6 years of zoledronic acid treatment of osteoporosis: a randomized extension to the HORIZON-Pivotal Fracture Trial (PFT). J Bone Miner Res. 2012;27:243-254. A9. Marjoribanks J, Farguhar C, Roberts H, et al. Long term hormone therapy for perimenopausal and postmenopausal women. Cochrane Database Syst Rev. 2012;7:CD004143. A10. Cummings SR, Ensrud K, Delmas PD, et al. Denosumab for prevention of fractures in postmenopausal women with osteoporosis. N Engl J Med. 2009;361:756-765. A11. Kanis JA, Johansson H, Oden A, et al. A meta-analysis of the effect of strontium ranelate on the risk of vertebral and non-vertebral fracture in postmenopausal osteoporosis and the interaction with FRAX(®). Osteoporos Int. 2011;22:2347-2355. A12. McClung MR, Grauer A, Boonen S, et al. Romosozumab in postmenopausal women with low bone mineral density. N Engl J Med. 2014;370:412-420. A13. Saag KG, Zanchetta JR, Devogelaer JP, et al. Effects of teriparatide versus alendronate for treating glucocorticoid-induced osteoporosis: thirty-six-month results of a randomized, double-blind, controlled trial. Arthritis Rheum. 2009;60:3346-3355. A14. Staples MP, Kallmes DF, Comstock BA, et al. Effectiveness of vertebroplasty using individual patient data from two randomised placebo controlled trials: meta-analysis. BMJ. 2011;343:d3952.

1645

TABLE 244-1  CAUSES OF OSTEOMALACIA VITAMIN D DEFICIENCY Dietary deprivation and lack of sunlight exposure VITAMIN D MALABSORPTION Postgastrectomy Gastric bypass for obesity Gluten enteropathy Small bowel disease or resection Pancreatic insufficiency Cholestyramine therapy for cholestatic liver disease Laxative abuse IMPAIRED 1-HYDROXYLATION OF 25-HYDROXYVITAMIN D Vitamin D−dependent rickets type I X-linked hypophosphatemia Autosomal dominant hypophosphatemic rickets/osteomalacia Oncogenic osteomalacia IMPAIRED TARGET-ORGAN RESPONSE TO 1,25-DIHYDROXYVITAMIN D Vitamin D−dependent rickets type II HYPOPHOSPHATEMIA X-linked hypophosphatemia Autosomal dominant hypophosphatemic rickets/osteomalacia Sporadic hypophosphatemia Fibrous dysplasia Oncogenic osteomalacia Antacid-induced osteomalacia Chronic metabolic acidosis Paraproteinemia Saccharated ferric oxide Tenofovir Cadmium INHIBITORS OF MINERALIZATION Etidronate Fluoride Aluminum Iron MISCELLANEOUS Hypophosphatasia Axial osteomalacia Fibrogenesis imperfecta ossium

GENERAL REFERENCES For the General References and other additional features, please visit Expert Consult at https://expertconsult.inkling.com.

244  OSTEOMALACIA AND RICKETS ROBERT S. WEINSTEIN

DEFINITION

Rickets refers to impaired mineralization of the cartilaginous growth plate and abnormal endochondral bone formation and therefore cannot occur in adults after epiphyseal closure.1 Osteomalacia, literally meaning softening of bone, refers to defective or delayed mineralization of the organic matrix of bone, or osteoid, at the interface between calcified bone and osteoid, and may occur at any age. Both rickets and osteomalacia may be present in a growing child, but defective mineralization can cause only osteomalacia in adults; therefore, this chapter will focus on osteomalacia. Despite advances in our understanding of vitamin D metabolism and the increased sensitivity of measurements of serum 25-hydroxyvitamin D, osteomalacia remains a common and frequently overlooked disorder in the world. Optimal therapy requires precise identification of the etiology of the abnormal mineralization, which may present a problem because there are numerous causes (Table 244-1). However, after a correct diagnosis is made, therapy is usually gratifying and often spectacular. Early recognition of osteomalacia depends on familiarity

with the typical clinical manifestations and settings. It is helpful to appreciate that the bone disease almost always manifests in the same manner regardless of the cause of the osteomalacia.

EPIDEMIOLOGY

About 20% of North American women receiving treatment for osteoporosis have 25-hydroxyvitamin D levels below 20 ng/mL (adequate values are greater than 30 ng/mL), and 8% have levels below 15 ng/mL. This indicates that, at the least, impaired bone mineralization could be a confounding factor in their osteoporosis treatment and, at worst, osteomalacia is the correct diagnosis (a defect in mineralization) rather than osteoporosis (a reduced amount of normally mineralized bone). An inadequate response to the bisphosphonate treatment commonly used for postmenopausal osteoporosis is four times more likely when 25-hydroxyvitamin D levels are subnormal than when the levels are above 30 ng/mL.2 Vitamin D deficiency is more common in elderly people, especially in nonaffluent people during the winter, at higher latitudes, and with low sun exposure. Vitamin D deficiency is also commonly found in medical inpatients, institutionalized patients, and postmenopausal women with acute hip fracture.3 The prevalence of osteomalacia due to vitamin D deficiency varies with the referral source. The disorder is far more frequent when patients are referred from geriatricians, gastroenterologists (osteomalacia may be found in up to 30% of patients with gastric surgery or bypass for obesity), nursing homes, or orthopedists concerned about symmetrical or nonhealing fractures. The most common hypophosphatemic osteomalacia is the inherited disease X-linked hypophosphatemia (XLH), but affected adults infrequently present to internists and then only when troubled by severe bone pain or nonunion of fractures.

CHAPTER 243  Osteoporosis  

GENERAL REFERENCES 1. Griffith JF, Genant HK. New advances in imaging osteoporosis and its complications. Endocrine. 2012;42:39-51. 2. Leslie WD, Morin SN. Osteoporosis epidemiology 2013: implications for diagnosis, risk assessment, and treatment. Curr Opin Rheumatol. 2014;26:440-446. 3. van den Bergh JP, van Geel TA, Geusens PP. Osteoporosis, frailty and fracture: implications for case finding and therapy. Nat Rev Rheumatol. 2012;8:163-172. 4. Richards JB, Zheng HF, Spector TD. Genetics of osteoporosis from genome-wide association studies: advances and challenges. Nat Rev Genet. 2012;13:576-588. 5. Nguyen TV, Eisman JA. Genetic profiling and individualized assessment of fracture risk. Nat Rev Endocrinol. 2013;9:153-161. 6. Kawai M, de Paula FJ, Rosen CJ. New insights into osteoporosis: the bone-fat connection. J Intern Med. 2012;272:317-329.

1645.e1

7. Bonewald LF, Kiel DP, Clemens TL, et al. Forum on bone and skeletal muscle interactions: summary of the proceedings of an ASBMR workshop. J Bone Miner Res. 2013;28:1857-1865. 8. Gerdhem P. Osteoporosis and fragility fractures: vertebral fractures. Best Pract Res Clin Rheumatol. 2013;27:43-55. 9. Naylor K, Eastell R. Bone turnover markers: use in osteoporosis. Nat Rev Rheumatol. 2012;8:379-389. 10. Moyer VA. Vitamin D and calcium supplementation to prevent fractures in adults: U.S. Preventive Services Task Force recommendation statement. Ann Intern Med. 2013;158:691-696. 11. Andreopoulou P, Bockman RS. Management of postmenopausal osteoporosis. Annu Rev Med. 2014;66:329-342. 12. Rizzoli R, Biver E. Glucocorticoid-induced osteoporosis: who to treat with what agent? Nat Rev Rheumatol. 2015;11:98-109. 13. Grey A, Bolland MJ. The effect of treatments for osteoporosis on mortality. Osteoporos Int. 2013;24:1-6.

1645.e2

CHAPTER 243  Osteoporosis  

REVIEW QUESTIONS 1. A 60-year-old woman presents to her physician to discuss her recent bone density results and management options. She has treated hypertension and a history of atrial fibrillation, as well as rheumatoid arthritis, for which she takes infliximab. Family history is notable for a hip fracture in her mother after a fall at age 65 years. She drinks socially and does not smoke. Her physical examination is unremarkable, including only 1 inch of height loss from her young adult maximum. Her spine examination reveals normal curvature, no kyphosis, normal rib-pelvis distance of 3 fingerbreadths, and 0 fingerbreadth wall-occiput distance. Laboratory studies are normal, including 25(OH)D level. Duel-energy x-ray absorptiometry (DXA) bone density reveals lumbar spine, femoral neck, and total hip T scores of −2.5, −3.0, and −2.7, respectively. Her FRAX 10-year estimates of hip and major osteoporotic fracture are 5.6 and 29%, respectively. What is the best management recommendation for this woman? A. Calcium and vitamin D supplementation alone B. Hormone replacement therapy C. Raloxifene D. Oral bisphosphonate E. Denosumab Answer: D  This patient has osteoporosis based on T score at the lumbar spine and proximal femur. In addition, she has an absolute fracture risk that supports pharmacologic intervention on a cost-effectiveness basis. Oral bisphosphonates are effective and generally frontline therapy. Active use of immunosuppressive therapy increases her risk for infection, which was seen more frequently in denosumab-treated patients in randomized controlled trials. Her cardiovascular history increases her risk for stroke, which has been observed more frequently in raloxifene- and estrogen-treated patients. Finally, calcium and vitamin D are important adjuncts to her management but should not be considered adequate alone for fracture risk reduction in this woman. 2. A 65-year-old woman presents with 6 months of progressive lower extremity pain and describes difficulty ascending stairs because of pain and weakness. She also brings a recent outside DXA bone density test, which shows total hip T and Z scores of −4.5 and −2.5, respectively. She does not have a history of fragility fractures. Past medical history is notable for hypertension and long-standing irritable bowel syndrome. She takes 600 mg of calcium and 400 IU of vitamin D daily, but no other medications. Family history is negative for osteoporosis or parental hip fracture. She does not smoke or drink alcohol. Physical examination is notable for tenderness to palpation over the mid-tibia bilaterally. She also has a wide-based, nonantalgic gait. What is the next best choice for her skeletal management? A. Double her calcium and vitamin D daily intake. B. Measure serum 25(OH)D level. C. Start an oral bisphosphonate. D. Measure fasting serum C telopeptide level (CTx) E. Start teriparatide therapy. Answer: B  This patient has lower bone density than expected for age based on Z score of −2.0 or less. This suggests a secondary cause for low bone mineral density (BMD) besides menopause. Her clinical presentation is consistent with osteomalacia, which cannot be distinguished from osteoporosis based solely on BMD. In addition, the prevalence of vitamin D deficiency or insufficiency and established hip fracture efficacy warrant its identification and treatment as the next best step in patient management. The recommended increase in calcium and vitamin D would be insufficient to treat vitamin D deficiency−related osteomalacia in this woman, and bisphosphonate and teriparatide therapy would be inappropriate until the vitamin D deficiency is corrected. Finally, bone turnover markers cannot be used independently for diagnosis in patients with metabolic bone disease.

3. An 81-year-old man is admitted after a fall and low-impact fracture of the proximal femur. His history is notable for Parkinson’s disease treated with dopamine agonist therapy. On physical examination, patient has a mild resting tremor while lying in bed. Laboratory investigations do not reveal secondary causes of osteoporosis, including a 25(OH)D level of 30 ng/ mL and testosterone of 300 ng/dL. The patient has normal renal function and calcium. The patient undergoes successful operative repair. After discharge, the patient undergoes 4 weeks of inpatient rehabilitation. What is the most definitive choice for this patient’s metabolic bone management? A. Start calcium and vitamin D supplementation. B. Increase dopamine agonist therapy for Parkinson’s disease. C. Give zoledronic acid, 5 mg intravenously. D. Start testosterone replacement therapy. E. Prescribe ergocalciferol 50,000 IU once weekly. Answer: C  This gentleman has incurred a low trauma femur fracture, which greatly increases his risk for subsequent fractures. Furthermore, the fracture confers a mortality rate up to 30% within 1 year of the event. Zoledronic acid has been proved not only to reduce the risk for subsequent fracture but also to reduce mortality by 27% Calcium and vitamin D are important adjuncts to treatment but not definitive. Given his level of 25(OH)D, additional antifracture and/or fall risk reduction would not be anticipated from pharmacologic vitamin D treatment. There is no evidence that testosterone treatment of either eugonadal or hypogonadal men reduces fracture risk. Finally, optimal management of his Parkinson’s disease, which does not appear undertreated in this patient, might reduce his risk for falls but not his risk for fracture. 4. A 55-year-old white woman presents to her physician with intense midback pain after a fall onto her backside while walking. Her history is notable for a recent diagnosis of polymyalgia rheumatica, for which she has taken prednisone 10 mg daily for the last 6 months. She does have known osteopenia, with lumbar spine, femoral neck, and total hip T scores of −1.5, −1.2, and −0.8, respectively. Physical examination is notable for tenderness to palpation and percussion over the lower thoracic spine. Plain films of the thoracolumbar spine reveal a new moderate (35%) anterior compression fracture at T11. Laboratory work-up, including 25(OH), is within normal limits. Other than continuing the calcium and vitamin D that the patient is currently taking, what is the most appropriate pharmacologic choice for treatment at this time? A. Vitamin D 50,000 IU once weekly B. Alendronate 70 mg once weekly C. Risedronate 150 mg once monthly D. Teriparatide 20 mcg subcutaneously daily E. Denosumab 60 mg subcutaneously every 6 months Answer: D  This woman presents with an acute vertebral fracture following a low trauma event. Her level of BMD is not osteoporotic, underscoring the independent contribution of glucocorticoids to fracture risk such that fractures may occur at a higher (i.e., better) level of BMD. The recent nature of her fracture further underscores the need for initiation of treatment now. There is no evidence that increasing this patient’s vitamin D level will further reduce her risk for fracture. Denosumab will likely reduce her risk for subsequent fracture, although the association with infection and use of concurrent prednisone in this woman preclude its consideration at present. Alendronate and risedronate are approved for the prevention and treatment of glucocorticoid-induced osteoporosis (GIO). Despite this, teriparatide has been proved more effective that bisphosphonates in patients with GIO and is indicated for this woman, assuming she has no contraindications to treatment.

CHAPTER 243  Osteoporosis  

5. A 70-year-old man presents with acute, mid-back pain after lifting a bag of topsoil while gardening 1 week ago. His medical history is notable for known male hypogonadism due to mumps orchitis in his 20s, although he does not take testosterone because of severe benign prostatic hypertrophy. He does take 600 mg of calcium twice daily and 1000 IU of vitamin D once daily. On physical examination, he has tenderness to palpation and percussion over his mid-thoracic spine. Laboratory investigations are unrevealing, including a 25(OH) level of 45 ng/mL. DXA bone density study confirms osteoporosis with lumbar spine T score of −3.0, although femoral neck and total hip T scores are normal. Radiographs of the thoracic spine reveal a new severe (50%) biconcave compression fracture at T8 without apparent widening of the pedicles. In addition to prescribing alendronate, which of the following is best adjunct management for this patient? A. Analgesic therapy and referral to physical therapy for post-fracture consultation B. Prescription for a spinal brace that patient should wear fro 6 months C. Percutaneous vertebroplasty D. Nasal calcitonin use for 1 year E. No additional interventions are required at this time.

1645.e3

Answer: A  This gentleman has osteoporosis secondary to long-standing hypogonadism, exhibited by predominantly great bone density loss and deficit in cancellous bone (i.e., spine). He has incurred by definition a lowtrauma fracture and is in need of a pharmacologic antifracture therapy (i.e., alendronate). Although he has significant pain that must be addressed, there is no evidence that conservative therapy with analgesics is inferior to vertebroplasty in regard to pain management. In addition, his radiographic findings do not suggest fracture instability that might benefit from vertebroplasty. Physical therapy is also helpful both short term as an adjunct to pain management and long term regarding modification of lifestyle and exercise to maximally reduce future fracture risk. A supportive spine brace may be used, although its use should be limited to only 4 to 6 weeks because of the necessary induction of paraspinal muscle weakness associated with long-term use. Similarly, nasal calcitonin may have some analgesic benefit in the acute, postfracture period based on limited data, although there is no evidence of a long-term analgesic benefit with continued use of the drug.

1646

PATHOBIOLOGY

CHAPTER 244  Osteomalacia And Rickets  

A review of normal bone remodeling and the mineralization of osteoid (bone matrix) serves as a background to understand the abnormal mineralization characteristic of osteomalacia. Bone remodeling or turnover is carried out by teams of juxtaposed osteoclasts and osteoblasts, comprising temporary anatomical structures known as basic multicellular units (BMUs). In cortical bone, the BMUs drill tunnels or “cutting cones” through the compact tissue; whereas in spongy, cancellous bone, they gouge across the trabecular surface, forming serpiginous trenches. Bone turnover begins by conversion of a quiescent skeletal surface to a remodeling site, a process referred to as activation. Activation involves proliferation of new blood vessels needed to bring recruited osteoclast progenitors to the remodeling site and retraction of the flat, pavement-like bone-lining cells that cover the quiescent surfaces to expose the mineralized bone surface. The recruited cells become multinucleated osteoclasts, which attach to the newly exposed bone surface with a ring of contractile proteins sealing off a subosteoclastic resorption compartment. Lysosomal enzymes, hydrogen ions, and collagenase are secreted through the microvilli of the ruffled underside border of the osteoclasts, and these chemicals begin to excavate a resorption cavity. The osteoclasts remove both the bone mineral and matrix. It is a misunderstanding to attribute to these cells or to metabolic acidosis the ability to remove only the mineral, leaving behind demineralized osteoid. Demineralized bone in vivo is a misnomer. Demineralized or decalcified bone only occurs when bones are placed in acid (1N HCl) or chelating solutions (ethylenediaminetetraacetate, or EDTA). Osteoclasts are motile cells, capable of resorbing more than just the cavity within which they are identified. After an osteoclast digs a cavity, it may detach from bone and move on to a new resorption site or die by apoptosis and be quickly removed by phagocytes. When the osteoclasts have moved on, osteoblasts assemble to reconstitute the previously resorbed cavity with new bone. In any established BMU, both events are happening at the same time; bone formation begins to occur while bone resorption advances. Between the end of bone resorption and the beginning of bone formation is the reversal phase, when mononuclear phagocytes smooth out the jagged erosion bays. During this phase, the old bone is coated by a thin layer of cement substance, a collagen- and mineral-poor matrix rich in glycosaminoglycans, glycoproteins, and acid phosphatase, to which the new osteoblasts attach. In adults, new osteoblasts assemble only at sites where osteoclasts have recently been eroding bone; a phenomenon referred to as coupling. The arrival of the osteoblasts in the right place at the right time and in sufficient numbers to reconstitute the cavity is referred to as remodeling balance and is likely due to proportional production of osteoblasts and osteoclasts in the bone marrow, release of osteoblast-recruiting substances from the resorbed bone, and chemotaxis by the cement substances. As osteoblasts complete their bone matrix synthesis and move away from the cement line, they gradually flatten. Some osteoblasts become bone-lining cells, and some become osteocytes, but as many as 65% of the osteoblasts that originally assembled at the remodeling site die by apoptosis. It is the balance between cell proliferation and apoptosis that determines the amount of work performed by these cells. Normally, up to 70% of the mineralization of the osteoid deposited by the osteoblasts starts within 4 to 12 days and proceeds at about 1 µm per day; but in osteomalacia, mineral deposition in the osteoid slows or stops completely, while the osteoblasts continue to make osteoid, which then accumulates in excessive amounts. Therefore, normal osteoid width is about 4 to 12 µm, but in osteomalacia, the osteoid width may become dramatically augmented. Depending on the extent of the delay in mineralization, overt osteomalacia may take many years to develop. In normal subjects, further mineralization proceeds slowly over months to years and at the cost of displacement of the water in the hydroxyapatite crystals, resulting in a modest increase in brittleness and the eventual need for another round of remodeling. Even though 1 million BMUs are undergoing remodeling every day, bone mass in a healthy adult is preserved thanks to a remarkably tight balance between the amount of bone resorbed and the amount formed during each cycle of remodeling. By this means, the adult skeleton is almost completely regenerated every 10 years. Mineralization requires the availability of sufficient calcium and phosphorus at the remodeling site, the presence of a normal bone collagen matrix, the absence of inhibitors of mineralization, and an adequate amount of skeletal alkaline phosphatase activity. Defects in these requirements are the cause of most forms of osteomalacia. Deficiency of vitamin D per se has traditionally been incriminated as the cause of the osteomalacia, but today, considerable evidence indicates that the abnormal mineralization associated with vitamin

D deficiency depends more on the deficiency of calcium and phosphorus than the absence of a direct effect of vitamin D on bone cells. The primary function of vitamin D is to provide adequate levels of calcium and phosphorus by increasing their intestinal absorption. Chronic metabolic acidosis has also been identified as a cause of osteomalacia, but evidence suggests that the bone disease associated with chronic metabolic acidosis is primarily due to the associated hypophosphatemia.

CLINICAL MANIFESTATIONS

The clinical presentation of osteomalacia depends on three overlapping manifestations: those due to the underlying disorder, such as gastrointestinal disease or surgery (especially troublesome are gastric resection, stapling or bypass for obesity, and intestinal malabsorption); those due to hypocalcemia or hypophosphatemia; and those directly due to the bone disease. The most common symptoms and signs are bone pain, muscle weakness, and bone tenderness. The bone pain is usually nonspecific and poorly localized. Because of the paucity of findings, the pain is often attributed to rheumatism or neurosis. It may be worse at night and after sudden movements such as turning in bed or the change from sitting to standing. Most often, the pain is in the lower back, pelvis and legs and is worse on weight bearing, resulting in a characteristic flat-footed, springless, waddling gait made worse by proximal muscle weakness. The gait has been referred to as “mother penguin’s walk.” Patients may complain that they can only climb stairs by pulling themselves up with the hand rail or rise from sitting in a chair or on the toilet by using their hands to push off. The decrease in strength is usually far greater than the degree of muscle wasting. Fasciculations are absent, and both reflexes and sensation remain normal. The bulbar, facial, and ocular muscles are always spared. However, muscle weakness is conspicuously absent when the osteomalacia is due to X-linked hypophosphatemia (see Table 244-1). Often, bone tenderness can be elicited by rib cage compression or pressing on the tibiae, wrists, pubic rami, or iliac crests. Hypocalcemia is usually mild to moderate but, rarely, can be severe enough to present with paresthesias, muscle cramps, a positive Chvostek’s sign, or seizures. If the osteomalacia is mistaken for osteoporosis and treatment is started with a bisphosphonate, the patient may experience new-onset paresthesias, muscle cramps, and palpitations. This not uncommon scenario occurs because the antiresorptive treatment interferes with the compensatory secondary hyperparathyroidism and aggravates the hypocalcemia.

DIAGNOSIS

Biochemical changes depend on the stage of the disease and its etiology. In vitamin D deficiency, hypophosphatemia precedes and is more severe than the hypocalcemia because of the secondary or compensatory hyperparathyroidism (Chapter 245) that almost invariably accompanies the disorder by the time that osteomalacia has occurred. In malabsorption, hypomagnesemia may contribute to the hypocalcemia, and hypoalbuminemia may lead to a spurious diagnosis of hypocalcemia. Increased serum alkaline phosphatase activity is classically associated with osteomalacia due to vitamin D deficiency but is not an early or reliable clue because some patients may have normal or borderline levels. The serum 25-hydroxyvitamin D levels are often less than 10 to 15 ng/mL. In contrast, serum 1,25-dihydroxyvitamin D levels are usually elevated because of the concomitant hyperparathyroidism and do not contribute to the diagnosis of osteomalacia except in the rare abnormalities of vitamin D resistance (when 1,25-dihydroxyvitamin D levels may be extraordinarily high) or when 1-hydroxylation is defective (and 1,25dihydroxyvitamin D levels are low). Quite a different pattern occurs with the inherited disease hypophosphatasia: serum 25-hydroxyvitamin D and calcium are normal, phosphorus and vitamin B6 levels are high normal or frankly elevated, and alkaline phosphatase activity is below the normal range.4 Radiographic findings may be absent with early osteomalacia, and only blurred margins of the cancellous bone with thin cortices may be noted. The presence of bilateral, thin (2 to 3 mm), radiolucent bands known as pseudofractures (Fig. 244-1) found perpendicular to the periosteal surface in ribs, pubic and ischial rami, the neck of the femur, and metatarsals and below the glenoid fossa on the outer border of the scapulae are generally considered to be pathognomonic of osteomalacia, but this classical radiographic sign is infrequent today. Rarely, it may be seen in disorders lacking excessive osteoid. These pseudofractures (sometimes called Looser’s zones or Milkman’s fractures) show increased uptake on bone scans (Fig. 244-2) and may lead to an inappropriate search for a primary malignancy. Bone mineral density T scores are often −3 or −4, with the radial diaphyseal density lower than that of the lumbar spine or total proximal femur.

CHAPTER 244  Osteomalacia And Rickets  

1647

FIGURE 244-3. An undecalcified bone biopsy specimen shows the characteristic abundant osteoid and flattened osteoblasts of osteomalacia (normally mineralized bone is blue and osteoid is red).

FIGURE 244-1.  Radiographic evidence of a pseudofracture of the femoral neck is suspicious for osteomalacia (arrow).

FIGURE 244-2. In osteomalacia, focal increased uptake of radionuclide on a bone scan may erroneously suggest metastatic disease (arrows).

Although characteristic clinical, radiographic, and biochemical findings may suggest osteomalacia, the absence of these findings cannot exclude the diagnosis. Quantitative histologic examination of undecalcified bone is, therefore, required to establish the unequivocal presence of osteomalacia (Fig. 244-3). Rigorous kinetic criteria for the histologic recognition of osteomalacia are necessary to preserve the traditional clinical, biochemical, and therapeutic connotations of the term. Therefore, a review of the quantitative bone histologic findings or histomorphometry in osteomalacia is useful.5 The histomorphometric diagnosis of osteomalacia requires the simultaneous presence of three findings: (1) excessive osteoid (osteoid area >10% of the cancellous bone area; normal is 15 µm; normal is 4 to 12 µm), (3) and prolongation of the mineralization lag time (>100 days; normal is 9 to 20 days), as determined by the osteoid width divided by the distance between and linear extent of double tetracycline labels observed in the bone after the patient receives two time-spaced courses of oral tetracycline. Tetracycline is deposited early in the course of hydroxyapatite crystal formation and generates bright stripes at the interface of mineralized bone and osteoid when viewed with fluorescent microscopy. If the two time-spaced courses of tetracycline (1 g/day for 3 days) are separated by a 14-day interval, the rate of mineralization (µm/day) can be calculated by measuring the average distance between the double labels divided by the number of days between the two courses. When the double labels are numerous and widely spaced, mineralization is intact and excess osteoid must

be due to increased bone turnover. A paucity of tetracycline labels that are narrowly spaced indicates that if excessive osteoid is present, it must be due to the delayed or ceased mineralization of osteomalacia (Fig. 244-4). Therefore, it follows that excessive osteoid can occur from two distinct mechanisms. Osteomalacia is the consequence of defective mineralization, while osteoid production continues. However, osteoid will also accumulate with accelerated bone formation if the rate of osteoid deposition exceeds the rate of mineralization, as occurs in states of greatly increased bone turnover, such as hyperparathyroidism (Chapter 245), Paget’s disease (Chapter 247), or thyrotoxicosis (Chapter 226). Even though osteoblasts in osteomalacia are usually sparse and flattened, whereas they are numerous, plump, and cuboidal with high bone turnover, these two groups of disorders can only be reliably distinguished with the use of tetracycline markers. The treatment of increased bone turnover and of defective mineralization is completely different, which is why the three histomorphometric criteria are necessary. Additionally, evaluation of each of the criteria in isolation has limitations. Regarding the first requirement, a small increase in the osteoid area relative to the total bone area may occur in osteoporosis, with a decrease in the amount of mineralized bone. In the second requirement, wide osteoid seams may be seen in some specimens obtained from patients with severe secondary hyperparathyroidism, such as those on maintenance hemodialysis therapy (Chapter 131). In the third requirement, reduced mineral appositional rate and increased mineralization lag time are nonspecific indices of impaired matrix synthesis by osteoblasts, as is often found in patients with involutional osteoporosis. Only when all three requirements are fulfilled is the diagnosis of osteomalacia irrefutable. Several presumed causes of osteomalacia (anticonvulsant drugs, metabolic acidosis without hypophosphatemia, pseudohypoparathyroidism, and chronic renal failure) have not fulfilled all of these requirements and primarily represent secondary hyperparathyroidism. Patients with the nephrotic syndrome lose albumin and vitamin D metabolites in the urine, but evidence indicates that serum ionized calcium and parathyroid hormone levels are normal and metabolic bone disease in adults with the nephrotic syndrome is absent. Muscle weakness and bone pain are significantly more common in patients in whom the rigorous histologic diagnosis of osteomalacia has been proved. However, bone biopsy is not always necessary to be reasonably certain of the diagnosis. When biopsy is necessary, the local pathologist must be familiar with the processing of undecalcified bone specimens and plastic embedding; otherwise, the best solution is to refer the patient to a histomorphometry center for biopsy. This ensures satisfactory communication between the clinician, operator, and pathologist and is the best insurance against the incomplete, broken, fragmented, or accidentally decalcified bone specimens. Such referral may be indispensable in the evaluation of a patient with unusually painful disease or progressive loss of bone mineral density, particularly when the results of the physical examination, radiographs, and biochemical findings are ambiguous. Biopsy may also be indicated in patients with unexplained chronic hypophosphatemia. The best approach is to avoid overlooking the diagnosis of osteomalacia by maintaining a high degree of suspicion in the typical clinical settings.6 This is especially important because osteomalacia can usually be successfully

1648

CHAPTER 244  Osteomalacia And Rickets  

A

B

FIGURE 244-4.  Histomorphometric diagnosis of osteomalacia by fluorescence imaging of double tetracycline labeling. A, Tetracycline double labels are numerous, discrete, and widely spaced, as is typical of intact mineralization. B, The tetracycline labels are mostly single despite the administration of two time-spaced doses of oral tetracycline, indicating that mineralization must be delayed or ceased, as is typical of osteomalacia.

treated. An investigation for osteomalacia is indicated in elderly patients with bone pain and muscle weakness, in patients with gastric surgery and low bone mineral density or bone pain, and in patients with persistent hypophosphatemia. Unexplained elevations of the serum alkaline phosphatase activity are usually due to drugs (e.g., anticonvulsants, anabolic steroids, phenothiazines, or antibiotics) or Paget’s disease of bone (Chapter 247) but rarely may be the only biochemical clue to osteomalacia in a patient with variable skeletal discomfort. Bilateral or slowly healing fractures also warrant an investigation for osteomalacia.

TREATMENT  Understanding of the treatment of osteomalacia is facilitated by dividing the disease into four subgroups. The first subgroup is osteomalacia due to disorders of vitamin D absorption or metabolism; the second is osteomalacia due to chronic hypophosphatemia. Most patients with osteomalacia will be in these first two subgroups. Treatment of osteomalacia caused by these two subgroups is discussed in detail in the next two sections. The third subgroup includes osteomalacia caused by inhibitors of mineralization, such as etidronate (the first oral bisphosphonate, now rarely used in North America);  high doses of fluoride; accumulation of a skeletal burden of aluminum from water used for dialysis or as a contaminate in solutions used for parenteral nutrition (now rarely seen); iron overload as in thalassemia; and cadmium, which induces the proximal tubular lesion of Fanconi’s syndrome and causes osteomalacia due to the resultant hypophosphatemia. The fourth subgroup includes miscellaneous causes of osteomalacia that lack specific therapy but are fortunately quite rare. This last subgroup includes the variable forms of the heritable disorder hypophosphatasia, caused by a deficiency of the tissuenonspecific (liver, bone, kidney) isoenzyme of alkaline phosphatase (although, therapeutic trials have shown that enzyme replacement is effective); axial osteomalacia, a sporadic osteosclerotic disorder primarily affecting middleaged men and presenting with mild to moderate pain in the spine and pelvis (but without fractures), apparently due to the production of an abnormal and poorly mineralized bone matrix by osteoblasts; and fibrogenesis imperfecta ossium, another sporadic disorder presenting with intractable bone pain and fractures, mainly in middle-aged men and women and apparently also due  to production of an abnormal bone matrix lacking the normal collagen birefringence by osteoblasts. In axial osteomalacia and fibrogenesis imperfecta ossium, serum calcium, phosphorus, and vitamin D levels are normal, but serum alkaline phosphates activity may be increased. General measures for this last subgroup include routine nutritional advice and avoidance of further bone loss due to postmenopausal or involutional osteoporosis. High-dose vitamin D therapy in these disorders has caused nephrocalcinosis, nephrolithiasis, and renal insufficiency and must be avoided.

Osteomalacia Due to Vitamin D Disorders

Iron deficiency anemia, hypocalcemia, weight loss, glossitis, or pruritic rash and bone discomfort in a patient with low bone mineral density point to celiac disease even without gastrointestinal symptoms. These signs suggest the need to test for immunoglobulin A antiendomysial and antitissue transglutaminase antibodies. Cholestyramine therapy for cholestatic liver disease may increase malabsorption of vitamin D by binding bile salts. Laxative abuse may cause osteomalacia and severe resistance to vitamin D supplementation, including treatment with calcitriol. Advice on nutrition and sun exposure, discontinuation of offending drugs, adherence to a gluten-free diet, and pancreatic enzyme replacement may cure the mineralization defect in mild cases without the need for additional treatment. Patients with severe disease will usually require vitamin D and calcium supplementation. Replacement doses depend on the serum 25-hydroxyvitamin

TABLE 244-2  VITAMIN D PREPARATIONS FOR TREATMENT OF OSTEOMALACIA VITAMIN D3 (CHOLECALCIFEROL)

CALCITRIOL (1,25[OH]2D3)

Trade names

Calciferol, BIOTECH†

Calcitriol

Dosage form

Caps: 50,000 units = 1.25 mg

Caps: 0.25 and 0.50 µg

Dosage:

0.50-2.0 µg/day

If serum 25(OH)D = 20 to 30 ng/mL

50,000 units once a week × 10 weeks* and once a month thereafter

If serum 25(OH)D = 10 to 20 ng/mL

50,000 units twice a week × 10 weeks* and twice a month thereafter

If serum 25(OH)D = less than 10 ng/mL

50,000 units three times a week × 10 weeks* and three times a month thereafter

Dosage in resistant cases

Up to 50,000 units per day

5-20 µg/day

Time to reach maximum effects

4-10 weeks

3-7 days

Persistence of effects after 6-30 weeks cessation

3-7 days

Cost in U.S.

$130/100 capsules of 0.25 µg $150/100 capsules of 0.5 µg

$40/100 capsules of 50,000 units

*If not >30 ng/mL after 10 weeks, exclude malabsorption, gluten enteropathy, and noncompliance. High-quality cholecalciferol, free from gluten, dairy, egg, fish nuts, soy, or artificial colors can be obtained from †BIOTECH at 1-800-345-1199. Weekly tanning bed treatments may be used if oral vitamin therapy fails or a switch to the more costly calcitriol may be necessary. 25(OH)D = 25-hydroxyvitamin D.

D level, as shown in Table 244-2. Because pharmacologic doses of any vitamin D preparation carry the risk for vitamin D intoxication, increases in the dose must be made carefully. The interval between increments in dosage should  be at least the time required to reach maximal effects plus about 50%. However, experience with the doses given in Table 244-2 indicates that serum 25-hydroxyvitamin D levels rarely reach 80 to 100 ng/mL. Vitamin D intoxication is unlikely even with of levels of 200 to 250 ng/mL. The goal is to raise the serum 25-hydoxyvitamin D level well above 30 ng/mL and restore the elevated parathyroid hormone concentration to normal without hypercalcemia or hypercalciuria. Urinary calcium excretion should be monitored when treatment has normalized the serum calcium level. The urinary calcium-to-  creatinine ratio (mg/mg) should be kept below 0.22. Approximately 1 to 1.5 g/ day of oral elemental calcium is a reasonable initial dose. Frequent small doses (three times a day) are more effective and tolerable than fewer larger ones, and the absorbability of calcium supplements is enhanced with meals. Most patient do well using the calcium preparations used for osteoporosis, such as calcium carbonate (40% calcium as in Os-Cal or the equivalent) or calcium citrate (21% calcium as in Caltrate or the equivalent). Some patients who cannot tolerate calcium carbonate or citrate experience fewer adverse gastrointestinal symptoms with the use of the chocolate or coffee-flavored formulations known as Viactiv (500 mg calcium per tablet). The vitamin D content of these calcium supplements is trivial in the treatment of osteomalacia.

CHAPTER 245  The Parathyroid Glands, Hypercalcemia and Hypocalcemia  

In patients with malabsorption, vitamin D requirements may increase during periods of increased diarrhea, and calcitriol may be easier for these patients to absorb. Its rapid onset of action and disappearance after cessation add to the safety of treatment, albeit at far greater cost. Calcium, phosphorus, potassium, magnesium, multivitamins, and gonadal steroids may also be beneficial in patients with malabsorption. Some patients do not tolerate any form of oral vitamin D, and the parenteral ergocalciferol preparations in North America are ineffective. These patients can be improved, although not restored to normal, by the use of weekly tanning bed treatments to areas of their bodies not normally exposed to the sun, an attempt to minimize solar-induced skin cancer. Calcitriol is the drug of choice in patients with the autosomal recessive disease, vitamin D−dependent rickets type I, in which the 1α-hydoxylase enzyme necessary to convert 25-hydroxyvitamin D to 1,25-dihydroxyvitamin D is deficient. In vitamin D−dependent rickets type II, another rare autosomal recessive disease presenting with alopecia, diminished target sensitivity to 1,25-dihydroxyvitamin D may require extraordinarily high doses of calcitriol.  If oral treatment fails, nocturnal infusions of calcium and phosphorus have been successful, providing additional evidence that the osteomalacia is due to inadequate calcium and phosphorus rather than the defect in vitamin D metabolism. An increase in the serum alkaline phosphatase activity (the healing “flare”) and a small increase in the serum and urine calcium levels are the earliest signs of effective treatment. Thereafter, the serum alkaline phosphatase activity level falls progressively as healing occurs. At the start of therapy, serum calcium levels should be measured at weekly intervals. If hypoalbuminemia is present, serum ionized calcium determinations are more useful. When therapy appears stabilized, biweekly or monthly intervals are usually sufficient for the first 3 or 4 months, but even with long-term therapy, measurements should be at least two to three times a year. In some patients with severe osteomalacia, bone pain and paresthesias may increase and the serum calcium levels decrease during the first few weeks of therapy. This is due to the increased skeletal avidity for mineral during healing and indicates the need for additional calcium supplementation.

Osteomalacia Due to Hypophosphatemia

Therapy of chronic hypophosphatemia is aimed at maintaining normal concentrations of serum phosphorus without inducing secondary hyper­ parathyroidism or nephrocalcinosis. This considerably difficult task requires divided doses of phosphorus supplements (1 to 3 g/day) and calcitriol (1 to 4 µg/day) to increase the absorption of phosphorus and try to prevent the increase in parathyroid hormone (Tables 244-2 and 244-3). If phosphorusinduced secondary hyperparathyroidism develops, the phosphorus supplements are rapidly excreted, and therapy thus is not only futile but also causes the additional bone disease of hyperparathyroidism. Baseline and yearly renal ultrasound examinations are necessary to recognize early nephrocalcinosis or nephrolithiasis. X-linked hypophosphatemia (XLH) is the most common cause of chronic hypophosphatemia, and the presence of a positive family history, pediatric onset, and bowed legs usually substantiates the diagnosis. Treatment with an anti-FG23 antibody can raise serum phosphorus and 1,25 vitamin D levels, A1  although the long-term benefits are not yet known. However, some hypophosphatemic patients have an autosomal dominant family history and present in adulthood with osteomalacia but without lower extremity deformities. Like patients affected with XLH, these patients may present with bone pain, pseudofractures, and high-normal or frankly elevated levels of fibroblast growth factor 23 (FGF23), a phosphaturic protein that interferes with 1-hydroxylation of 25-hydroxyvitamin D.7 Patients with autosomal dominant hypophosphatemic osteomalacia (ADHR) appear to acquire the renal phosphate losses in adolescence or adulthood, whereas other patients with ADHR may lose the

1649

defect as they age. A diagnostic problem arises when a patient with chronic hypophosphatemia presents without a positive family history or bowed legs because this presentation resembles that of patients with oncogenic osteomalacia. This disorder is associated with a variety of small, hard to find, benign mesenchymal tumors that secrete FGF23.8,9 The muscle pain, weakness, fractures, and osteomalacia characteristic of this syndrome are due to hypophosphatemia made worse by inappropriately low levels of 1,25-dihydroxy vitamin D. Oncogenic osteomalacia is also treated with phosphorus supplementation and calcitriol until the offending tumor can be located and resected. Improved tumor localization has been reported with positron emission tomography and computed tomography.9-11 Surgical correction of deformities should be postponed until medical management achieves persistently normal levels of calcium, phosphorus, and alkaline phosphatase activity. An exception to this rule is an acute fracture of the femoral neck. Prompt surgical repair may be essential to avoid osteonecrosis. Oncogenic osteomalacia rarely occurs with malignant tumors that secrete FGF23, but unless surgical resection is complete, the osteomalacia will persist. Recent evidence suggests that tumors in surgically difficult locations may be treated by radiofrequency ablation.12 Antacid-induced hypophosphatemia due to the ingestion of large quantities of phosphate-binding antacids has become rare with the increased availability of proton pump inhibitors but still occurs occasionally.

PREVENTION

Advice about vitamin D supplementation should help to prevent osteomalacia caused by vitamin D deficiency, but this has proved to be difficult because routine supplements may be inadequate and compliance with nutritional supplements is poor. The optimal vitamin D supplementation dosage is not clear, but most bone and mineral problems are avoided by 50,000 units of ergocalciferol given once monthly. Notable exceptions occur in patients with celiac disease, gastric surgery, or bypass for obesity, who often require much larger amounts (see Table 244-2). In patients with osteomalacia due to hypophosphatemia, the need for phosphorus supplementation may be lifelong. Rare exceptions occur in oncogenic osteomalacia if complete surgical removal or destruction of the tumor is accomplished.

PROGNOSIS

The response to appropriate treatment in most forms of osteomalacia is usually excellent. Improvements in bone pain and muscle weakness usually occur within 2 or 3 months, and healing of skeletal lesions occurs within 6 to 18 months. Depending on the quantity of excess osteoid, repeat bone mineral density determinations may show as much as 20% gains at the lumbar spine and total proximal femur. However, bone density at the radial diaphysis may not improve because of the irreversible loss of cortical bone resulting from prolonged secondary hyperparathyroidism. Furthermore, if decreased bone volume is present in addition to excess osteoid, skeletal recovery may be incomplete, and the risk for fractures may remain increased.

Grade A Reference A1. Carpenter TO, Imel EA, Ruppe MD, et al. Randomized trial of the anti-FGF23 antibody KRN23 in X-linked hypophosphatemia. J Clin Invest. 2014;124:1587-1597.

GENERAL REFERENCES

TABLE 244-3  PHOSPHATE PREPARATIONS FOR TREATMENT OF OSTEOMALACIA TABLET MARKINGS PREPARATION AND SHAPE Neutra-Phos

0

Neutra-Phos-K

0

K-Phos Neutral

“Beach 11-25” oblong

K-Phos Original

“Beach 1111” round

SODIUM CONTENT (mEq)

POTASSIUM CONTENT (mEq)

AMOUNT THAT CONTAINS 1 GRAM OF ELEMENTAL PHOSPHORUS

28.5

28.5

4 unit dose caps* 4 unit dose caps*

0

57.0

50.4

4.6

4 tabs

0

33.0

9 tabs

*Each unit dose cap is reconstituted with 75 mL of water, fruit juice, or cola, and this formulation is preferred by children. The unit dose cap contains the powder concentrate and is not to be swallowed undiluted. Adults prefer the K-Phos Neutral tablets.

For the General References and other additional features, please visit Expert Consult at https://expertconsult.inkling.com.

245  THE PARATHYROID GLANDS, HYPERCALCEMIA AND HYPOCALCEMIA RAJESH V. THAKKER

CALCIUM METABOLISM

A healthy adult body has a total of 1 kg of calcium; about 99% of this is present within the crystal structure of bone mineral, and less than 1% is in soluble form in the extracellular and intracellular fluid compartments. In the extracellular fluid compartment (ECF), about half of the total calcium is

CHAPTER 244  Osteomalacia And Rickets  

GENERAL REFERENCES 1. Elder CJ, Bishop NJ. Rickets. Lancet. 2014;383:1665-1676. 2. Peris P, Martinez-Ferrer A, Monegal A, et al. 25 Hydroxyvitamin D serum levels influence adequate response to bisphosphonate treatment in postmenopausal osteoporosis. Bone. 2012;51: 54-58. 3. van Schoor NM, Lips P. Worldwide vitamin D status. Best Pract Res Clin Endocrinol Metab. 2011;25:671-680. 4. Berkseth KE, Tebben PJ, Drake MT. Clinical spectrum of hypophosphatasia diagnosed in adults. Bone. 2013;54:21-27. 5. Kulak CA, Dempster DW. Bone histomorphometry: a concise review for endocrinologists and clinicians. Arq Bras Endoocrinol Metab. 2010;54:87-98. 6. Priemel M, von Domarus C, Klatte TO, et al. Bone mineralization defects and vitamin D deficiency: histomorphometric analysis of iliac crest bone biopsies and circulating 25-hydroxyvitamin D in 675 patients. J Bone Miner Res. 2010;25:305-312. 7. Quarles LD. “Dem bones” are made for more than walking. Nat Med. 2011;17:428-430.

1649.e1

8. Manger B, Schett G. Paraneoplastic syndromes in heumatology. Nat Rev Rheumatol. 2014;10: 663-670. 9. Chong WH, Molinolo AA, Chen CC, et al. Tumor-induced osteomalacia. Endocr Relat Cancer. 2011;18:53-77. 10. Chong WH, Andreopoulou P, Chen CC, et al. Tumor localization and biochemical response to cure in tumor-induced osteomalacia. J Bone Miner Res. 2013;28:1386-1398. 11. Clifton-Bligh RJ, Hofman MS, Duncan E, et al. Improving diagnosis of tumor-induced osteomalacia with gallium-68 DOTATATE PET/CT. J Clin Endocrinol Metab. 2013;98:687-694. 12. Jadhav S, Kasaliwal R, Shetty NS, et al. Radiofrequency ablation, an effective modality of treatment in tumor-induced osteomalacia: a case series of three patients. J Clin Endocrinol Metab. 2014;99: 3049-3054.

1649.e2

CHAPTER 244  Osteomalacia And Rickets  

REVIEW QUESTIONS 1. Defective mineralization in osteomalacia is due to lack of one or more of the following except which one? A. Adequate calcium and phosphorus at the remodeling site B. Adequate amount of skeletal alkaline phosphatase C. Normal pH at the site of calcification D. Presence of a normal bone collagen matrix E. Adequate level of fluoride Answer: E  Fluoride is an inhibitor of mineralization. See Pathobiology. 2. The correct sequence of steps involving bone remodeling is represented by which one of the following? A. Reversal, activation, bone resorption by osteoclasts, osteoblast assembly and new bone formation B. Activation, bone resorption by osteoclasts, reversal phase, osteoblast assembly and new bone formation C. Bone resorption by osteoclasts, activation, osteoblast assembly and new bone formation, reversal D. Activation, osteoblast assembly and new bone formation, bone resorption by osteoclasts, reversal E. None of the above Answer: B  See Pathobiology. 3. All of the following may be signs or symptoms of osteomalacia except which one? A. Nonspecific and poorly localized bone pain B. Bone pain after sudden movements C. Fasciculations and absent reflexes D. Flat-footed waddling gait E. Muscle weakness Answer: C  See Clinical Manifestations.

4. Excessive osteoid due to osteomalacia can be reliably distinguished from that caused by increased bone turnover by which one of the following? A. Serum alkaline phosphatase B. Presence of pseudofractures C. Increased radionuclide on a bone scan D. Phosphate level E. Bone histomorphometry using tetracycline labels Answer: E  See Diagnosis. 5. A 30-year-old woman presents to her primary care physician with fatigue, generalized bone pains, weight loss, and a pruritic rash on her elbows and back. Laboratory studies show: hemoglobin 10 (male, 14-17 g/dL; female, 12-16 g/dL); serum iron 20 (60-160 µg/dL); serum calcium 8 (9-10.5 mg/dL). What is the next appropriate diagnostic test? A. Bone mineral density scan B. Hemoglobin electrophoresis C. Colonoscopy D. Antiendomysial immunoglobulin A antibodies E. Skin biopsy Answer: D  The patient has gluten enteropathy, which may occur even without gastrointestinal symptoms. See Treatment.

1650

CHAPTER 245  The Parathyroid Glands, Hypercalcemia and Hypocalcemia  

Dietary Ca++, VitD

+

Parathyroid

ECF

Ca++

Ca++

+

Intestine

Bone

PTH

+

UV Light Skin

7-dehydrocholesterol

Vit D + Ca++

+ Kidney

1,25(OH)2 VitD 25(OH) VitD Liver

1αOHase

24OHase 1,24,25(OH)3 VitD

FIGURE 245-1. Regulation of extracellular fluid (ECF) calcium (Ca2+) by parathyroid hormone (PTH) action on kidney, bone, and intestine. A decrease in ECF Ca2+ is sensed by the calcium-sensing receptor (see Fig. 245-2), and this leads to an increase in PTH secretion and a reduction in PTH degradation. The increased circulating PTH predominantly acts directly on kidney and bone that possess the PTH receptor (PTHR, Fig. 245-2). The skeletal effects of PTH are to increase (+) osteoclastic bone reabsorption. However, because osteoclasts do not have PTHRs, this action is mediated by the osteoblasts, which do have PTHRs and in response release cytokines and factors in turn that activate osteoclasts. In the kidney, PTH stimulates (+) the 1α-hydroxylase (1αOHase) to increase the conversion of 25-hydroxyvitamin D [25(OH)VitD] to the active metabolite 1,25-dihydroxyvitamin D [1,25(OH)2VitD]. In addition, PTH increases (+) the reabsorption of Ca2+ from the renal distal tubule and inhibits the reabsorption of phosphate from the proximal tubule, thereby leading to hypercalcemia and hypophosphatemia. PTH also inhibits Na+, H+ antiporter activity and bicarbonate reabsorption, thereby causing a mild hyperchloremic acidosis. The elevated 1,25(OH)2VitD acts on the intestine to increase (+) absorption of dietary calcium and phosphate. It is important to note that PTH does not appear to have a direct action on the gut. Thus, in response to hypocalcemia and the increase in PTH secretion, all of these direct and indirect actions of PTH on the kidney, bone, and intestine will help to increase ECF Ca2+, which in turn will act through the calcium-sensing receptor to decrease PTH secretion. (From Thakker RV, Bringhurst FR, Jüppner HH. Regulation of calcium homeostasis and genetic disorders that affect calcium metabolism. In: Jameson JL, De Groot LJ, giudice LC, et al., eds. Endocrinology: Adult & Pediatric. 7th ed. Philadelphia: Saunders; 2016.)

ionized, and the rest is principally bound to albumin or complexed with counter-ions. Ionized calcium in the ECF plays an important role in many physiologic pathways, including muscle contraction, secretion of neurotransmitters and hormones, and coagulation pathways. Ionized serum calcium concentrations range from 4.65 to 5.25 mg/dL (1.16 to 1.31 mmol/L), and the total serum calcium concentration ranges from 8.5 to 10.5 mg/dL (2.12 to 2.62 mmol/L).1 However, the usual 2 : 1 ratio of total to ionized calcium may be disturbed by disorders such as metabolic acidosis, which reduces calcium binding by proteins, or by changes in protein concentration, caused by cirrhosis, dehydration, venous stasis, or multiple myeloma1. In view of this, total serum calcium concentrations are adjusted, or “corrected,” to a reference albumin concentration: the actual total serum calcium value is adjusted by adding or subtracting 0.8 mg/dL (0.016 mmol/L) for every 1 g/ dL (1 g/L) of albumin below or above a reference albumin concentration of 4 g/dL (40 g/L), respectively. The control of body calcium involves a balance between the amounts that are absorbed from the gut, deposited into bone and into cells, and excreted from the kidney (Fig. 245-1).2 This fine balance, involving three organs, is chiefly under the control of parathyroid hormone (PTH), which is synthesized and secreted by the parathyroid glands. Hypocalcemia leads to an increased secretion of PTH, whereas hypercalcemia results in diminished PTH secretion. Regulation of extracellular calcium takes place through complex interactions (Fig. 245-2) at the target organs of the major calciumregulating hormone, PTH, and vitamin D and its active metabolites, 1,25-dihydroxyvitamin D (1,25[OH]2D).

PARATHYROID GLANDS, PARATHYROID HORMONE, PTH GENE, AND PARATHYROID HORMONE ACTIONS

Parathyroid Glands

There are usually four parathyroid glands, which are located in close proximity to the superior and inferior poles of the lobes of the thyroid gland. The superior parathyroids are derived from the endoderm of the embryonic fourth pharyngeal pouches, and the inferior parathyroids are derived with the thymus from the endoderm of the third pharyngeal pouches. Extra parathyroid glands are commonly found in aberrant locations along this migrating

path and also within the thymus and thyroid. Parathyroid cells express a G protein−coupled receptor (GPCR), referred to as the calcium-sensing receptor (CaSR), that detects changes in extracellular calcium and leads to alterations in PTH secretions.3 For example, activation of the CaSR, which is also expressed in renal tubular cells, as a result of elevated extracellular calcium concentrations causes G protein−dependent stimulation of phospholipase C activity through Gαq and Gα11, which leads to accumulation of inositol 1,4,5-trisphosphate and an increase in intracellular calcium concentrations.4 These changes, in turn, lead to reduced circulating PTH concentrations and increased urinary calcium excretion. Disorders of the parathyroid glands may cause hypercalcemia or hypocalcemia, and these can be classified according to whether they arise from an excess of PTH, its deficiency, or insensitivity to its effects (Table 245-1; see Fig. 245-2).

Parathyroid Hormone and PTH Gene

The mature PTH peptide is encoded by the PTH gene and secreted from the parathyroid chief cells as an 84−amino acid peptide; however, when the PTH mRNA is first translated, it is as pre-proPTH peptide. The “pre” sequence consists of a 25−amino acid signal peptide (leader sequence) that is responsible for directing the nascent peptide into the endoplasmic reticulum to be packaged for secretion from the cell. The “pro” sequence is 6 amino acids in length and, although its function is less well defined than that of the “pre” sequence, is also essential for correct PTH processing and secretion. After the 84−amino acid mature PTH peptide is secreted from the parathyroid cell, it is cleared from the circulation with a short half-life of about 2 minutes, by nonsaturable hepatic uptake and renal excretion.

Parathyroid Hormone Actions

PTH shares a receptor with PTH-related peptide (PTHrP); this PTH/ PTHrP receptor (see Fig. 245-2) is a member of a subgroup of the G protein−coupled receptor family.5 PTH/PTHrP receptors are expressed in kidney and bone, where PTH is its predominant agonist, and thus PTH acts directly on kidney and bone cells and indirectly on intestinal cells (see Fig. 245-1) to enhance renal calcium reabsorption, release stored calcium in bones into the ECF, and increase gut calcium absorption, respectively. Expression of the PTH/PTHrP receptor also occurs in the brain, heart, skin, lung, liver, and testis, where it mediates the actions of PTHrP. Mutations

CHAPTER 245  The Parathyroid Glands, Hypercalcemia and Hypocalcemia  

PTH is an 84−amino acid peptide encoded by the PTH gene, which is located on chromosome 11p15 and consists of three exons (transcribed regions) that are separated by two introns. Exon 1 of the PTH gene is 85 base pairs (bp) in length and is untranslated, whereas exons 2 and 3 code for the 115−amino acid pre-proPTH peptide. Exon 2 is 90 bp in length and encodes the initiation (ATG) codon, the prehormone sequence, and part of the prohormone sequence. Exon 3 is 612 bp and encodes the remainder of the prohormone sequence, the mature PTH peptide, and the 3′ untranslated region. The 5′ regulatory sequence of the human PTH gene contains a vitamin D response element 125 bp upstream of the transcription start site, which downregulates PTH messenger RNA (mRNA) transcription in response to vitamin D receptor binding. PTH gene transcription (as well as PTH peptide secretion) is also dependent on the extracellular calcium concentration, although the presence of a specific upstream “calcium response element” has not yet been demonstrated.

1650.e1

CHAPTER 245  The Parathyroid Glands, Hypercalcemia and Hypocalcemia  

Ca2+ CaSR

Loss of function FHH1, NSHPT

Gain of function ADH1

FHH2

ADH2, hypoparathyroidism

1651

Parathyroid cell

PLC

Gi

Gq/11

AC

AP2σ

ATP

PIP2

MELAS, KSS MTPD5

cAMP

DAG + IP3

TBCE

[Ca2+]i PKC Proto-oncogenes and tumor-suppressor genes

Transcription factors (e.g., GATA3, GCM2, AIRE, FAM 111A)

PTHrP

Kenny-Caffey type 1 Sanjad-Sakati Parathyroid tumors (e.g., MEN 1, HPT-JT, CCND1, Rb, CDC73)

PTH

Target cell (e.g., kidney, bone, cartilage)

FHH3

Hypoparathyroidism DiGeorge syndrome, HDR, hypoparathyroidism, APECED, Kenny-Caffey type 2

PTH

PTH/PTHrP receptor

Gs

Gq/11

Blomstrand’s lethal chondrodysplasia

Jansen’s metaphyseal chondrodysplasia

Pseudohypoparathyroidism

McCune-Albright syndrome

AC

PLC PIP2 IP3 + DAG

ATP cAMP

Gq/11

ADH2, hypoparathyroidism FBHH, NSHPT

ADH1

CaSR

FIGURE 245-2. Schematic representation of some of the components involved in calcium homeostasis. Alterations in extracellular calcium are detected by the calcium-sensing receptor (CaSR), which is a 1078−amino acid G protein−coupled receptor. The PTH/PTHrP receptor, which mediates the actions of PTH and PTHrP, is also a G protein−coupled receptor. Thus, Ca2+, PTH, and PTHrP involve G protein−coupled signaling pathways, and interaction with their specific receptors can lead to activation of Gs, Gi, and Gq, respectively. Gs stimulates adenylcyclase (AC), which catalyzes the formation of cyclic adenosine monophosphate (cAMP) from adenosine triphosphate (ATP). Gi inhibits AC activity. cAMP stimulates protein kinase A (PKA), which phosphorylates cell-specific substrates. Activation of Gq stimulates phospholipase C (PLC), which catalyzes the hydrolysis of the phosphoinositide (PIP2) to inositol triphosphate (IP3), which then increases intracellular calcium, and diacylglycerol (DAG), activating protein kinase C (PKC). These proximal signals modulate downstream pathways, which result in specific physiologic effects. Loss of function in several genes, shown with their respective sites of action on the right, has been identified in specific disorders of calcium homeostasis (also see Table 245-1). (From Thakker RV, Bringhurst FR, Jüppner H. Regulation of calcium homeostasis and genetic disorders that affect calcium metabolism. In: Jameson JL, De Groot LJ, giudice LC, et al., eds. Endocrinology: Adult & Pediatric. 7th ed. Philadelphia: Saunders; 2016.)

involving the genes that encode these proteins and receptors in this calciumregulating pathway (see Fig. 245-2) are associated with hypercalcemic and hypocalcemic disorders (see Table 245-1).

Renal Actions

Calcium is absorbed at multiple sites and by different mechanisms, which include passive paracellular or active transcellular transport, along the renal tubule.2 The renal actions of PTH are to (1) stimulate activity of the proximal tubular cell 1α-hydroxylase; (2) increase reabsorption of calcium by the cells of the distal tubule, connecting tubules and the thick ascending loop of Henle (TAL); and (3) inhibit phosphate reabsorption by proximal tubular cells (see Fig. 245-1). PTH increases the formation of biologically active 1,25(OH)2D from its precursor 25-OH-D by stimulating the activity of the renal 1α-hydroxylase and inhibiting the 24-hydroxylase, which metabolizes 1,25(OH)2D to the inactive 24,25(OH)2D form (see Fig. 245-1). PTH regulates calcium reabsorption by distal tubular cells by upregulating expression

of the transient receptor potential vanilloid 5 (TRPV5), thereby promoting calcium entry into the cell, and increasing calbindin-D28K expression to enhance transcellular calcium reabsorption by increased buffering of subapical Ca2+ ions. In the TAL, PTH may increase active transcellular transport of calcium, as well as paracellular calcium transport, by augmenting the transepithelial voltage gradient. Phosphate transport in proximal tubular cells is mediated by the luminal membrane sodium-phosphate cotransporters 2a and 2c (NPT2a and NPT2c), and PTH actions lead to internalization and degradation of NPT2a and NPT2c, thereby resulting in decreased reabsorption of phosphate.

Skeletal Actions

PTH acts directly on osteoblasts and indirectly on osteoclasts to increase their numbers and activity, thereby enhancing bone turnover and release of stored calcium. Thus, PTH increases the size of the osteoblast precursor pool, increases the bone-forming activity of mature osteoblasts, and stimulates

1652

CHAPTER 245  The Parathyroid Glands, Hypercalcemia and Hypocalcemia  

TABLE 245-1  PARATHYROID DISEASES AND THEIR CHROMOSOMAL LOCATIONS METABOLIC

CHROMOSOMAL

ABNORMALITY

DISEASE

INHERITANCE

GENE/GENE PRODUCT

LOCATION

HYPERCALCEMIA Multiple endocrine neoplasia type 1 Multiple endocrine neoplasia type 2 Hereditary hyperparathyroidism and jaw tumors (HPT-JT) Sporadic hyperparathyroidism

Autosomal dominant Autosomal dominant Autosomal dominant

MENIN RET PARAFIBROMIN

11q13 10q11.2 1q31.2

Sporadic

Parathyroid carcinoma

Autosomal dominant or sporadic

PRAD1/CCND1 Retinoblastoma Unknown PARAFIBROMIN

11q13 13q14 1p32-pter 1q31.2

Retinoblastoma

13q14

CaSR Gα11 AP2S1 CaSR

3q 21.1 19p13 19q13 3q21.1

PTHR/PTHrP receptor Elastin, LIMK (and other genes) CYP24A Gsα

3p21.3 7q11.23 20q13.2-q13.3 20q13.3

Autosomal dominant Autosomal recessive X-linked recessive Autosomal dominant Autosomal dominant Autosomal recessive

PTH, GCMB PTH, GCMB SOX3 CaSR Gα11 AIRE-1

11p15* 11p15*, 6p24.2 Xq26–27 3q21.1 19p13 21q22.3

Maternal

Mitochondrial genome

Autosomal dominant Autosomal dominant Autosomal recessive Autosomal dominant

TBX1 GATA3 PTHR/PTHrP receptor TBCE

22q11.2/10p 10p15 3p21.3 1q42.3

Autosomal recessive Autosomal recessive† Autosomal recessive Autosomal dominant† Autosomal dominant Autosomal dominant parentally imprinted Autosomal dominant parentally imprinted

FAMIIIA Unknown Unknown Unknown Unknown? GNAS exons 1-3

11q12.1 ? ? ? ? 20q13.3

GNAS Upstream deletion

20q13.3

Familial benign hypercalcemia (FBH)   FBH1   FBH2   FBH3 Neonatal severe hyperparathyroidism (NSHPT) Jansen’s disease Williams syndrome Infantile hypercalcemia McCune-Albright syndrome

Autosomal dominant Autosomal dominant Autosomal dominant Autosomal recessive or autosomal dominant Autosomal dominant Autosomal dominant Autosomal recessive Mutations during early embryonic development?

HYPOCALCEMIA Isolated hypoparathyroidism Autosomal dominant hypocalcemia type 1 (ADH1) Autosomal dominant hypocalcemia type 2 (ADH2) Hypoparathyroidism associated with polyglandular autoimmune syndrome (APECED) Hypoparathyroidism associated with Kearns-Sayre and MELAS Hypoparathyroidism associated with complex congenital syndromes   DiGeorge syndrome   HDR syndrome   Blomstrand’s lethal chondrodysplasia   Kenney-Caffey syndrome type 1, Sanjad-Sakati syndrome   Kenney-Caffey syndrome type 2   Barakat syndrome   Lymphedema   Nephropathy, nerve deafness   Nerve deafness without renal dysplasia Pseudohypoparathyroidism (type 1a) Pseudohypoparathyroidism (type 1b)

HDR = hypoparathyroidism, deafness, and renal dysplasia; MELAS = mitochondrial encephalopathy, stroke-like episodes, and lactic acidosis; ? = location not known. *Mutations of PTH gene are identified only in some families. † Most likely inheritance.

osteoblasts to release cytokines such as colony-stimulating factor 1 and receptor activator of nuclear factor-κB (NF-κB) ligand (RANKL), which stimulate the formation of new osteoclasts and activate mature osteoclasts. PTH also inhibits osteoblast production of osteoprotegrin (OPG), which is a soluble decoy receptor for RANKL that inhibits osteoclast development. Calcium transport involves TRPV4 and TRPV5 in bone cells; TRPV4 regulates intracellular calcium concentrations in osteoblasts and osteoclasts, whereas TRPV5, expressed in osteoclasts, participates to remove the mineral bone matrix.2 The net result of persistent elevations of PTH is linked to an increase in osteoclast activity more than osteoblast activity, hence liberating the stores of calcium to the ECF (see Fig. 245-1).

Intestinal Actions

Calcium is absorbed throughout the intestine by passive paracellular routes and active transcellular routes, which involve TRPV6 and calbindin D9K. PTH exerts indirect actions on intestinal calcium absorption by increasing the circulating 1,25(OH)2D concentrations (see Fig. 245-1). The increased 1,25(OH)2D concentrations increase TRPV6 expression, which facilitates enhanced calcium entry into the cell from the lumen, and cytosolic calbindin D9K expression, which facilitates transcellular transport of calcium.

HYPERCALCEMIA DEFINITION

Hypercalcemia is defined as a serum calcium concentration greater than 2 standard deviations above the normal mean, and this is usually a total serum calcium above 10.5 mg/dL (2.62 mmol/L) and an ionized serum calcium of above 5.25 mg/dL (1.31 mmol/L). There is no formal grading system for defining the severity of hypercalcemia, but mild, moderate, and severe hypercalcemia is generally considered for total serum calcium concentrations less than 12 mg/dL (3 mmol/L), between 12 and 14 mg/dL (3 to 3.5 mmol/L), and greater than 14 mg/dL (3.50 mmol/L), respectively.

PATHOBIOLOGY

Hypercalcemia may arise through one of three mechanisms: increased bone resorption, increased gastrointestinal absorption of calcium, and decreased renal calcium excretion (see Fig. 245-1). For example, lytic bone metastases cause increased bone resorption; thiazide diuretics lead to a decrease in calcium excretion; and excessive PTH will either directly or indirectly, by increasing 1,25(OH)2D production, stimulate bone resorption and calcium

CHAPTER 245  The Parathyroid Glands, Hypercalcemia and Hypocalcemia  

absorption from the gut and renal tubules.6 The causes of hypercalcemia may be classified according to whether serum PTH concentrations are elevated (i.e., primary or tertiary hyperparathyroidism due to parathyroid tumors) or reduced (i.e., not due to parathyroid tumors but instead to an excessive production of PTHrP by a cancer; a defect in the PTH receptor, for example, the PTH/PTHrP receptor; an excess production of downstream mediators, for example, 1,25(OH)2D; or an altered set point in the calcium-sensing receptor) (Table 245-2; see Fig. 245-2). Primary hyperparathyroidism and malignancy are the most common causes and account for more than 90% of patients with hypercalcemia. Detailed clinical history and examination will usually help to differentiate between these two diagnoses. In primary hyperparathyroidism, the hypercalcemia is often less than 12 mg/dL (3 mmol/L), asymptomatic, and may have been present for months or years. If symptoms, such as nephrolithiasis, are present, then they have usually been present for several months. However, in malignancy, the patients are usually acutely ill, often with neurologic symptoms; the hypercalcemia is more than 12 mg/dL (3 mmol/L); and the cancer (e.g., lung, breast, or myeloma) is often readily apparent. Hypercalcemia from causes other than primary hyperparathyroidism or malignancy may also occur (see Table 245-2), and a careful history (e.g., for vitamin D ingestion, drugs, renal disease) and examination (e.g., for thyrotoxicosis, adrenal disease, granulomatous diseases), together with appropriate investigations (Table 245-3; Fig. 245-3), are essential for establishing the diagnosis.

CLINICAL MANIFESTATIONS AND DIAGNOSIS

The clinical presentation of hypercalcemia varies from a mild, asymptomatic, biochemical abnormality detected during routine screening to a lifethreatening medical emergency. In general, the presence or absence of symptoms correlates with the severity and rapidity of onset of the hypercalcemia.

TABLE 245-2  CAUSES OF HYPERCALCEMIA

1653

Thus, symptoms do not usually develop when serum calcium is below 12 mg/dL (3 mmol/L) and are invariably present when the hypercalcemia exceeds 14 mg/dL (3.5 mmol/L). However, there is a considerable variability, and some patients may be symptomatic with mild hypercalcemia. Although there are many causes of hypercalcemia (see Table 245-2), the signs and symptoms of hypercalcemia are similar, regardless of etiology. Indeed, the clinical manifestations of hypercalcemia involve several organ systems that include the renal, musculoskeletal, gastrointestinal, neurologic, and cardiac systems (Table 245-4), and many of these have been referred to as “moans, groans, pains, and stones.” Investigations should be directed at confirming the presence of hypercalcemia and establishing the cause (Table 245-5; see Table 245-3).

TABLE 245-3  PRELIMINARY INVESTIGATIONS FOR HYPERCALCEMIA BLOOD × 2-3 estimations of serum calcium, phosphate, albumin, urea and electrolytes, creatinine, alkaline phosphatase, liver function tests Parathyroid hormone Complete blood count Electrophoretic protein strip or serum protein electrophoresis 25-OH-D3 (and if indicated, 1,25[OH]2D3) Thyroid function tests Magnesium Parathyroid hormone−related peptide (if malignancy suspected) URINE × 2-3 estimations of 24-hr urinary calcium and creatinine clearance, and clearance ratios Imaging Chest radiograph Radiograph of hands Ultrasound of kidneys

HIGH PARATHYROID HORMONE LEVELS Primary hyperparathyroidism* (adenoma, hyperplasia, or carcinoma): nonfamilial or familial, e.g., MEN 1, MEN 2, HPT-JT, FIHP Tertiary hyperparathyroidism (hyperplasia or adenoma in chronic renal failure) LOW PARATHYROID HORMONE LEVELS Malignancy* Primary • Parathyroid hormone−related peptide (PTHrP): carcinoma of lung, esophagus, renal cell, ovary, and bladder • Excess production of 1,25(OH)2D (lymphoma) Secondary • Lytic bone metastases* (multiple myeloma* and breast carcinoma*) • Other location, ectopic factors (e.g., cytokines) Excess vitamin D Exogenous vitamin D toxicity by parent D compound, 25(OH) vitamin D3, or 1,25(OH)2 vitamin D3 in vitamin preparations, cod liver oil, herbal medicines Endogenous production of 25(OH) vitamin D3—Williams syndrome Endogenous production of 1,25(OH)2 vitamin D3, e.g., granulomatous disorders (sarcoidosis, HIV, TB, histoplasmosis, coccidioidomycosis, leprosy), lymphoma, and infantile hypercalcemia Drugs Thiazide diuretics Lithium Total parenteral nutrition Estrogens/antiestrogens, testosterone Milk-alkali syndrome Vitamin A toxicity Aluminum intoxication (in chronic renal failure) Aminophylline Nonparathyroid endocrine disorders Thyrotoxicosis Pheochromocytoma Acute adrenal insufficiency Vasoactive intestinal polypeptide hormone producing tumor (VIPoma) Immobilization INAPPROPRIATE PARATHYROID HORMONE LEVELS DUE TO ALTERED SET POINT Familial benign hypocalciuric hypercalcemia (FBH or FHH) *Most common causes. FIHP = familial isolated hyperparathyroidism; HIV = human immunodeficiency virus; HPT-JT = hyperparathyroidism with jaw tumors; MEN = multiple endocrine neoplasia; TB = tuberculosis.

TABLE 245-4  CLINICAL FEATURES OF HYPERCALCEMIA Renal Stones (nephrolithiasis) and nephrocalcinosis, polyuria, polydipsia Musculoskeletal Bone pain, osteopenia, fractures, muscular weakness, especially proximal myopathy Gastrointestinal Nausea, vomiting, lack of appetite, constipation, peptic ulcers, and pancreatitis Neurologic Tiredness, lethargy, inability to concentrate, increased sleepiness, depression, confusion, coma Cardiac Bradycardia, first-degree atrioventricular block, arrhythmias, shortened QT interval

TABLE 245-5  SUMMARY OF GUIDELINES FOR PARATHYROID SURGERY IN PRIMARY HYPERPARATHYROIDISM PATIENTS Surgery* recommended if patient meets any one of the following criteria: • Serum calcium >1 mg/dL (0.25 mmol/L) above upper limit of normal • Any complication of primary hyperparathyroidism (e.g., nephrolithiasis† or bone erosions of osteitis fibrosa cystica) • An episode of acute primary hyperparathyroidism with life-threatening hypercalcemia • Significant reduction in creatinine clearance (i.e., 10.5 mg/dL (2.62 mmol/L) or ionized calcium > 5.25 mg/dL (1.31 mmol/L)

Confirm hypercalcemia and undertake clinical assessment for risk factors (e.g., malignancy, chronic renal failure, excessive vitamin D intake, drugs such as thiazides, family history of MEN or FHH)

Undertake appropriate tests as outlined in Table 245-3, and evaluate results

Suppressed PTH

Consider malignancy and outcome of clinical evaluations and serum tests (e.g., EPS or PTHrP), or radiology (e.g., chest radiograph)

25-OH-D elevated

Elevated (or normal) PTH

Normal 25-OH-D

Elevated serum creatinine and PO4 and EGFR 0.01

Normal serum creatinine and PO4, and UCCR 9 mmol/L per 24 hr) was included as a recommendation for parathyroid surgery in asymptomatic primary hyperparathyroidism by the Second International Conference (2002), but not by the Third International Conference (2008). However, some physicians still regard marked hypercalciuria as an indication for parathyroid surgery.

1661.e3

4. Which of the following statements regarding parathyroid tumors in multiple endocrine neoplasia (MEN) syndrome is false? A. In MEN 1, parathyroid tumors occur with equal frequency in men and women. B. In MEN 2b and MEN 3, parathyroid tumors are a rare occurrence. C. In MEN 1, minimally invasive surgery is not a suitable approach because of the high occurrence of multiple parathyroid tumors. D. Patients with MEN 2a have a higher risk for developing parathyroid carcinomas. E. Parathyroid tumors occur in more than 95% of MEN 1 patients. Answer: D  MEN 1 is an autosomal dominant disorder, and this generally affects men and women equally. Parathyroid tumors are found in more than 95% of patients with MEN 1, who usually have multiple parathyroid tumors, and hence minimally invasive surgery is not recommended. Parathyroid tumors rarely occur in patients with MEN 2b or MEN 3, and about 20% of MEN 2a patients will usually have parathyroid hyperplasia and not parathyroid carcinoma. However, patients with hyperparathyroidism with jaw tumors are at high risk for developing parathyroid carcinomas. 5. Which of the following is not a cause of hypocalcemia? A. Vitamin D−resistance disorders B. Secondary hyperparathyroidism C. Adrenal insufficiency D. Autoimmunity E. Acute pancreatitis Answer: C  Acute adrenal insufficiency is associated with hypercalcemia, not hypocalcemia. Hypocalcemia typically occurs in patients with secondary hyperparathyroidism and in those with vitamin D−resistance disorders, and it may be found in patients with acute pancreatitis. In addition, autoimmune destruction of the parathyroids, resulting in hypoparathyroidism, or autoantibodies to the calcium-sensing receptor may be associated with hypocalcemia.

1662

CHAPTER 246  Medullary Thyroid Carcinoma  

246  MEDULLARY THYROID CARCINOMA SAMUEL A. WELLS, JR.

MEDULLARY THYROID CARCINOMA DEFINITION

Medullary thyroid carcinoma (MTC) is an uncommon cancer that arises from the neural crest—derived C cells of the thyroid gland.

EPIDEMIOLOGY

MTC accounts for 5% of thyroid cancers, and there will be approximately 3000 new cases in the United States in 2015. MTC occurs either sporadically (75% of cases) or as part of the multiple endocrine neoplasia (MEN) syndromes, MEN 2A (Online Mendelian Inheritance in Man [OMIM] #171400) or MEN 2B (OMIM #162300), or the related syndrome familial medullary thyroid carcinoma (FMTC) (OMIM #155240).1 The incidence of MEN 2A and FMTC combined is approximately 1 in 100,000 live births, whereas the incidence of MEN 2B is approximately 1 in 2,000,000 live births.

PATHOBIOLOGY

As the lateral thyroid complex closes during embryogenesis, the C cells are incorporated within the middle and upper portions of the thyroid lobes. Because of its anatomic location, MTC is classified as a thyroid tumor; however, considering its origin from the neural crest rather than the thyroid follicular cells, it is a neuroendocrine tumor. Sporadic MTC occurs as a solitary tumor in one thyroid lobe, whereas hereditary MTC develops in both thyroid lobes and is multicentric. In patients with hereditary MTC, the first manifestation of a C-cell disorder is C-cell hyperplasia (CCH) that progresses over time to microinvasive MTC and then to invasive MTC.2 The C-cell mass is much greater in the thyroid glands of men than in women, which accounts for the higher serum calcitonin levels seen in men compared with women. The C cells have diverse biosynthetic activity and secrete calcitonin (CTN) and carcinoembryonic antigen (CEA), which are excellent serum markers for the presence of a C-cell disorder. CTN was once thought to be important in calcium homeostasis; however, its physiologic importance has been called into question. The RET protooncogene encodes a single-pass transmembrane receptor of the tyrosine kinase family of proteins. At several stages of development, it is expressed in cells derived from the branchial arches (parathyroids), the neural crest (brain, parasympathetic and sympathetic ganglia, thyroid C cells, adrenal medulla, and enteric ganglia), and the urogenital system. Activating, germline point mutations in RET are present in virtually all hereditary MTCs, and somatic RET mutations are present in approximately half of sporadic MTCs.3,4 Recently, it was discovered that somatic mutations in HRAS, KRAS, and rarely NRAS are present in sporadic MTCs and are almost always mutually exclusive with the presence of somatic RET mutations.5 Approximately 75 RET mutations have been reported in association with MEN 2A, MEN 2B, and FMTC. The mutations for MEN 2A and FMTC are located in exons 5, 8, 10, 11, 13, 14, 15, and 16. The mutations for MEN 2A are mostly located in the extracellular, cysteine-rich region of exon 10 (including codons 609, 611, 618, and 620) and exon 11 (including codons 630 and 634). Approximately 85% of the mutations associated with MEN 2A involve RET codon 634, about half of which are C634R RET mutations. The RET mutations in MEN 2B cause constitutive activation, which alters substrate specificity, presumably owing to a conformational change in the binding pocket of the kinase. Approximately 95% of mutations causing MEN 2B are in codon M918T, and 5% are in codon A883F. Rare cases of MEN 2B are caused by double somatic RET mutations involving codon V804M and either codon Y806C, S904C, or E805K. In 50% of patients with MEN 2B and 10% of patients with MEN 2A and FMTC, the disease arises de novo. In such founder cases, the de novo mutation almost always derives from the paternal allele.6

CLINICAL MANIFESTATIONS

The peak incidence of sporadic MTC is in the fifth decade of life, and most patients present with a solitary thyroid nodule and lymph node metastases. Clinically, the tumors are more aggressive than papillary thyroid carcinoma

TABLE 246-1  CLINICAL MANIFESTATIONS OF MULTIPLE ENDOCRINE NEOPLASIA 2A, 2B, AND FAMILIAL MEDULLARY THYROID CARCINOMA MULTIPLE ENDOCRINE NEOPLASIA (MEN) 2A Medullary thyroid carcinoma (~100%) Pheochromocytoma (incidence of 50% in families with a RET codon 634 germline mutation but less in families with other RET codon mutations) Hyperparathyroidism (incidence of 30% in families with a RET codon 634 germline mutation but less in families with other RET codon mutations.) VARIANTS OF MEN 2A MEN 2A with cutaneous lichen amyloidosis (almost always associated with a RET codon 634 germline mutation.) MEN 2A with Hirschsprung disease (most common in families with RET germline mutation most commonly involving codon 620) FAMILIAL MEDULLARY THYROID CARCINOMA (FMTC) Since the original description of this syndrome, there has been confusion about the designation FMTC. Most clinicians now consider it a variant of MEN 2A. MEN 2B Medullary thyroid carcinoma (~100%) Pheochromocytoma (50%) Mucosal neuroma, ganglioneuromatosis, marfanoid habitus, colonic abnormalities, characteristic physical appearance (~100%)

and follicular thyroid carcinoma but less aggressive than anaplastic thyroid carcinoma (Chapter 226). The 10-year survival rate is 75%. In the expectation of detecting MTC at any early stage, clinicians in Europe evaluate serum CTN levels in patients with thyroid nodules who have no history of hereditary MTC. The detection rate of MTC is less than 0.5%, however, and clinicians in the United States have not adopted this practice. The clinical manifestations of MEN 2A, MEN 2B, and FMTC are listed in Table 246-1. MEN 2A (80% of cases), MEN 2B (5% of cases), and FMTC (15% of cases) are inherited as autosomal dominant traits with near-complete penetrance and, in the cases of MEN 2A and MEN 2B, variable expressivity. Approximately 50% of patients with MEN 2A (and a codon 634 mutation) develop pheochromocytomas (Chapters 228 and 231), the frequency being much lower in association with mutations in codons 609, 611, 618, and 620.7 Before the availability of biochemical and genetic screening in families with MEN 2A, the most frequent cause of death was pheochromocytoma, not MTC. The deaths occurred most often in patients during childbirth or interventional procedures. Thus, pheochromocytoma must be excluded in patients with a confirmed or presumptive diagnosis of hereditary MTC (Chapter 228). With rare exceptions, the pheochromocytoma should be excised first in patients who also have MTC. Parathyroid hyperplasia occurs in up to 30% of patients with MEN 2A and is usually associated with a RET codon 634 mutation. The disease is frequently asymptomatic, with the only abnormality being an elevated serum calcium concentration.8 Patients with MEN 2A may also develop cutaneous lichen amyloidosis (CLA) or Hirschsprung disease (HD).9 CLA occurs in about 25% of patients and involves the interscapular region of the back, corresponding to dermatomes T2 through T6. Pruritus, the dominant symptom, leads to repetitive scratching and secondary skin changes characterized by the deposition of amyloid. The lesion may be evident in infancy, thus serving as a precursor marker of MEN 2A. Cutaneous lichen amyloidosis is almost always associated with a RET codon 634 mutation. HD, manifested by the absence of intrinsic ganglion cells in the distal gastrointestinal tract, has been reported in 30 or more families with MEN 2A or FMTC and is associated with mutations in RET exon 10 involving codons 609 (15%), 611 (4%), 618 (30%), and 620 (50%). In functional studies, the cell surface expression of RET with these codon mutations is lower than that found with a codon 634 mutation. This suggests a novel mechanism whereby the specified RET mutations have low transforming activity, which is sufficient to trigger the development of MTC and pheochromocytoma, yet is insufficient to stimulate differentiation of intestinal ganglion cells. It is also of interest that 50% of patients with familial HD and 30% of patients with sporadic HD have germline RET mutations. Patients with MEN 2B develop mucosal neuromas, ganglioneuromatosis throughout the aerodigestive tract, hypotonia, skeletal malformations, and medullated corneal nerves. They also develop colonic dysfunction manifested by abdominal pain and occasionally intestinal obstruction. Patients

CHAPTER 246  Medullary Thyroid Carcinoma  

have a characteristic physical appearance, which may not be evident early in life. The failure to diagnose MEN 2B at a young age can be catastrophic because MTC is often evident soon after birth, and regional or distant metastases occur soon thereafter. The MTC associated with MEN 2B is much more aggressive than that occurring with MEN 2A or FMTC. The primary basis for the difference is that MEN 2B mutations are associated with significantly higher basal kinase activity compared with mutations in MEN 2A and FMTC. Patients with FMTC develop only MTC, which, relative to the tumors in patients with MEN 2A and MEN 2B, is slow growing. Many clinicians consider FMTC a variant of MEN 2A.

DIAGNOSIS

The measurement of serum levels of CTN, either in the basal state or following the intravenous administration of the secretagogues calcium, pentagastrin, or both, was initially the primary method of establishing the diagnosis of a C-cell disorder. With the discovery that MEN 2A, MEN 2B, and FMTC are caused by mutations in the RET protooncogene, direct DNA analysis became the method of choice for identifying affected family members who had inherited a mutated RET allele. At present, the determination of CTN is primarily used to detect persistent or recurrent MTC following thyroidectomy or to evaluate response to therapy in patients with regional or metastatic disease. As we have learned more about the variable clinical expression of MEN 2A in families with the identical RET mutation, however, the measurement of basal and stimulated serum CTN levels has assumed importance in timing early thyroidectomy in young family members with RET mutations.10 The two-site, two-step, chemiluminescent, immunometric assay that is highly specific for monomeric CTN is the preferred method for quantitating serum CTN levels. At present, direct DNA analysis of RET has become the preferred method of detecting RET mutations in families with hereditary MTC. The Gene Tests directory currently lists 63 laboratories that perform DNA analysis for RET mutations (http://www.genetests.org). Almost all laboratories use direct sequence analysis to evaluate mutations in exons 10, 11, 13, 14, 15, and 16, and some laboratories include exon 8. If no mutations are found in these exons, the entire coding region of RET can be sequenced. It is important to perform direct DNA analysis for RET mutations in all patients with presumed sporadic MTC because approximately 7% of them will have hereditary MTC. A diagnosis of hereditary MTC in this setting mandates a different treatment strategy for the patient, as well as his family members, who should be offered the opportunity for clinical evaluation and genetic testing.

TREATMENT  The primary treatment for patients with MTC, whether sporadic or hereditary, is total thyroidectomy. Resection of lymph nodes in the central compartment is included in all adults and in children with MEN 2B but is excluded in outwardly normal youngsters with MEN 2A and FMTC who are undergoing early thyroidectomy based on directed DNA analysis. If enlarged cervical lymph nodes are evident on preoperative ultrasound examination or at the time of thyroidectomy, the involved anatomic nodal compartment should also be resected. During the thyroidectomy, great care must be taken to preserve the parathyroid glands, the recurrent laryngeal nerves, and the external branch of the superior laryngeal nerve. Postoperatively, serum calcitonin is normal in only 10% of patients with node-positive disease compared with 60% of patients with node-negative disease. Many patients with regional lymph node metastases have a good prognosis, however, with 5- and 10-year survival rates of 80 and 70%, respectively. Repeat neck operation following initial thyroidectomy is indicated in patients with complications from recurrent tumor compressing or invading vital structures, such as the spinal cord, airway, or esophagus. Also, patients who have intractable diarrhea due to markedly elevated tumor hormone secretions, presumably CTN, may obtain symptom relief by tumor debulking. Patients who develop persistent or recurrent MTC following thyroidectomy, as indicated by elevated serum levels of CTN or CEA, are also candidates for reoperation; however, the benefit of such surgical procedures is open to question because there are no long-term data on quality of life and survival. Rarely, patients with MTC develop Cushing syndrome (Chapter 227) due to the inappropriate secretion of adrenocorticotrophic hormone (ACTH) or corticotropinreleasing hormone. Such patients have advanced disease, and bilateral adrenalectomy may be required if steroidogenesis inhibitors are ineffective. Inappropriate secretion of ACTH is a poor prognostic sign, associated with an average survival of 2 years. The treatment of pheochromocytoma is adrenalectomy, as described in Chapter 228. Hyperparathyroidism (Chapter 245) is managed by either 

1663

subtotal parathyroidectomy or total parathyroidectomy with heterotopic autotransplantation. For patients with locally advanced or metastatic MTC, single-agent or combined chemotherapeutic regimens have been minimally effective, being characterized by low response rates of short duration. External beam radiotherapy is indicated primarily for the treatment of localized metastases, primarily of the central nervous system or bone. With the demonstration that the tyrosine kinase inhibitor imatinib induced remissions in patients with chronic myelogenous leukemia and gastrointestinal stromal tumors, there was hope that similar molecular targeted therapeutics (MTTs) would be developed for other solid tumors, including MTC. In a recent prospective, randomized, placebo-controlled, double-blind, phase III trial, patients treated with the MTT vandetanib had a significantly prolonged progression-free survival compared with placebo. A1  On the basis of this study, the U.S. Food and Drug Administration (FDA) approved vandetanib for the treatment of patients with advanced MTC.11 The FDA also recently approved a second MTT, cabozantinib,12 based on similar results of a phase III trial. A2  Thus, effective systemic therapies are available for patients with advanced MTC, and additional studies of other MTTs have recently been initiated.

PREVENTION

In patients with a hereditary cancer syndrome, the removal of an organ destined to become malignant should be considered in the light of five factors. There should be (1) near-complete penetrance of the mutated gene, (2) a reliable method of detecting family members who have inherited a mutated allele, (3) minimal morbidity associated with removal of the organ at risk, (4) excellent replacement therapy for the function of the removed organ, and (5) a reliable method for determining whether the operative procedure has been curative. Few hereditary malignancies meet all of these criteria; fortunately, MEN 2A, MEN 2B, and FMTC meet each of them. Young members of kindred with hereditary MTC who are found to have a mutated RET allele on genetic screening have the greatest likelihood of being cured by early thyroidectomy. Surgeons in several countries have reported success with this operative procedure, and the question is no longer should it be done but at what age. The Consensus Committee of the 7th International Workshop on MEN, the National Comprehensive Cancer Network, and the American Thyroid Association have all proposed guidelines for the timing of prophylactic thyroidectomy in patients with MEN 2A, MEN 2B, and FMTC13. The recommendations of the three groups are similar, in that children with MEN 2B (or with mutations in codons 918 or 882) should have thyroidectomy at the time of diagnosis, even during the first months of life. Children with MEN 2A and mutations in codons 611, 618, 620, or 634 should have the thyroid removed at or before 5 years of age. In children with mutations in other RET codons, the recommended timing of thyroidectomy is less clear but is generally between 5 and 10 years of age.

PROGNOSIS

Several factors portend an adverse outcome in patients with MTC.14 Poor prognosis is associated with older age, advanced disease at the time of diagnosis of a large primary tumor, lymph node metastases, markedly elevated serum levels of CTN and CEA preoperatively, extrathyroidal invasion of the trachea or soft tissues, and distant metastases. Patients with MEN 2B and patients with MEN 2A who have RET mutations in codon 634 have a poorer prognosis than those with RET mutations in other codons. Also, in patients with sporadic MTC, the presence of a RET M918T mutation, compared with other codon mutations, is associated with a more aggressive tumor and a poor prognosis. Patients apparently cured by thyroidectomy are followed at 6-month intervals with measurement of serum levels of CTN and CEA. The doubling times of serum CTN are especially useful in predicting the course of the disease. CTN doubling times of less than 6 months (compared with those greater than 24 months) are associated with a very poor prognosis.

Grade A References A1. Wells SA Jr, Robinson BG, Gagel RF, et al. Vandetanib in patients with locally advanced or metastatic medullary thyroid cancer: a randomized, double-blind phase III trial. J Clin Oncol. 2012;30:134-141. A2. Elisei R, Schlumberger MJ, Müller SP, et al. Cabozantinib in progressive medullary thyroid cancer. J Clin Oncol. 2013;31:3639-3646.

1664

CHAPTER 247  Paget Disease of Bone  

GENERAL REFERENCES For the General References and other additional features, please visit Expert Consult at https://expertconsult.inkling.com.

247  PAGET DISEASE OF BONE

the rest of the world through emigration. Paget disease is strongly related to age; in the United Kingdom, the incidence is 0.3 to 0.5 per 10,000 person years in those aged 55 to 59 years but doubles in frequency each decade thereafter to reach an incidence of 5.4 per 10,000 person years in women and 7.6 per 10,000 person years in men in those 85 years and older. The prevalence and severity of Paget disease have diminished in most countries over the past 25 years.1 The causes are unclear, but suggested explanations include influx of migrants from low prevalence areas in some populations, improved nutrition, a more sedentary lifestyle with a reduction in skeletal injuries, and reduced exposure to infections.2

PATHOBIOLOGY

STUART H. RALSTON

DEFINITION

Paget disease of bone is a disorder of skeleton characterized by increased and disorganized bone remodeling affecting one or more skeletal sites. Affected bones enlarge, become deformed, and are at increased risk for pathologic fractures

EPIDEMIOLOGY

The population prevalence of Paget disease is about 1% in the United States and 2% in the United Kingdom. It is also common in Western Europe and in people of European descent who have migrated to other parts of the world. Paget disease is rare in Scandinavians, Africans, and Asians. These differences are thought to have a genetic basis and to be caused by founder mutations that occurred in Europeans many centuries ago with subsequent spread to

Susceptibility to Paget disease seems to be genetically determined, but environmental factors also play a key role in regulating onset and severity of the disease. The importance of genetics is emphasized by the fact that between 15 and 40% of patients have a positive family history and that the risk for developing Paget disease in a first-degree relative of a patient is about sevenfold higher than in the general population.3 In many families, the disease is transmitted in an autosomal dominant manner, although penetrance is incomplete. The most important susceptibility gene for classical Paget disease is SQSTM1. Mutations of SQSTM1 are present in up to 40% of patients with a family history and 5 to 10% of people with sporadic disease. The SQSTM1 gene encodes a protein called p62 that is involved in regulating signal transduction downstream of the receptor activator of nuclear factor κB (RANK), which plays a critical role in regulating osteoclastogenesis when activated by RANK ligand (RANKL) (Fig. 247-1). The disease-causing mutations cluster in the ubiquitin-associated domain and have the effect of upregulating nuclear factor κB (NFκB) signaling and stimulating osteoclastogenesis by complex mechanisms that are reviewed in detail elsewhere. Genome-wide association

Stem cell Stromal cell

M-CSF

OCL/M∅ precursor OPG RANK RANKL Ubiquitin

TRAF6 p62

CYLD

OCL precursor VCP DC-STAMP

IκB

NFκB

OPTN Nucleus

Osteoclast

NFκB

Gene expression

FIGURE 247-1.  Regulators of osteoclast dysfunction in Paget disease. Some of the key molecules that have been implicated in the pathogenesis of Paget disease are illustrated. Macrophage colony-stimulating factor (M-CSF) encoded by CSF1 is required for differentiation of stem cells to the osteoclast/macrophage (OCL/MØ) lineage. Osteoclast differentiation and activity are enhanced when RANK (encoded by TNFRSF11A) is activated by RANKL but inhibited by OPG (encoded by TNFRSF11B). Fusion of osteoclast precursors to form mature osteoclasts requires DC-STAMP (encoded by TM7SF4). Within the cell (inset), p62 (encoded by SQSTM1) is required for signal transduction downstream of the RANK receptor and is also involved in regulating autophagy. Both VCP (encoded by VCP) and OPTN (encoded by OPTN) also play a role in regulating NFκB signaling and autophagy.

CHAPTER 246  Medullary Thyroid Carcinoma  

GENERAL REFERENCES 1. Wells SA Jr, Pacini F, Robinson BG, et al. Multiple endocrine neoplasia type 2 and familial medullary thyroid carcinoma: an update. J Clin Endocrinol Metab. 2013;98:3149-3164. 2. Mete O, Asa SL. Precursor lesions of endocrine system neoplasms. Pathology. 2013;45:316-330. 3. Frank-Raue K, Rondot S, Raue F. Molecular genetics and phenomics of RET mutations: impact on prognosis of MTC. Mol Cell Endocrinol. 2010;322:2-7. 4. Agrawal N, Jiao Y, Sausen M, et al. Exomic sequencing of medullary thyroid cancer reveals dominant and mutually exclusive oncogenic mutations in RET and RAS. J Clin Endocrinol Metab. 2013;98:E364-E369. 5. Moura MM, Cavaco BM, Pinto AE, et al. High prevalence of RAS mutations in RET-negative sporadic medullary thyroid carcinomas. J Clin Endocrinol Metab. 2011;96:E863-E868. 6. Choi SK, Yoon SR, Calabrese P, et al. Positive selection for new disease mutations in the human germline: evidence from the heritable cancer syndrome multiple endocrine neoplasia type 2B. PLoS Genet. 2012;8:e1002420. 7. Welander J, Soderkvist P, Gimm O. Genetics and clinical characteristics of hereditary pheochromocytomas and paragangliomas. Endocr Relat Cancer. 2011;18:R253-R276.

1664.e1

8. Scholten A, Schreinemakers JM, Pieterman CR, et al. Evolution of surgical treatment of primary hyperparathyroidism in patients with multiple endocrine neoplasia type 2A. Endocr Pract. 2011;17:7-15. 9. Moore SW, Zaahl M. The Hirschsprung’s-multiple endocrine neoplasia connection. Clinics. 2012;67:63-67. 10. Rowland KJ, Moley JF. Hereditary thyroid cancer syndromes and genetic testing. J Surg Oncol. 2015;111:51-60. 11. Karras S, Anagnostis P, Krassas GE. Vandetanib for the treatment of thyroid cancer: an update. Expert Opin Drug Metab Toxicol. 2014;10:469-481. 12. Nix NM, Braun K. Cabozantinib for the treatment of metastatic medullary thyroid carcinoma. J Adv Pract Oncol. 2014;5:47-50. 13. Tuttle RM, Ball DW, Byrd D, et al. Medullary carcinoma. J Natl Compr Cancer Netw. 2010;8:512-530. 14. Ho AS, Wang L, Palmer FL. Postoperative nomogram for predicting cancer-specific mortality in medullary thyroid cancer. Ann Surg Oncol. 2014; [Epub ahead of print].

1664.e2

CHAPTER 246  Medullary Thyroid Carcinoma  

REVIEW QUESTIONS 1. Sporadic medullary thyroid carcinomas (MTCs) without somatic RET mutations most commonly have which of the following? A. Somatic RAS mutations B. Germline RET mutations C. Somatic BRAF mutations D. No additional genetic mutations E. Gene fusions involving the RET protooncogene Answer: A  About half of patients with sporadic MTC have somatic RET mutations, and about 70% of sporadic MTCs with no somatic RET mutations have HRAS, KRAS, or rarely NRAS mutations. (Moura MM, Cavaco BM, Pinto AE, Leite V. High prevalence of RAS mutations in RET-negative sporadic medullary thyroid carcinomas. J Clin Endocrinol Metab. 2011;96: E863-868.)

4. In patients with MEN 2B where the RET mutation arises de novo,the mutated allele comes from which parent(s)? A. The mother B. The father C. Either parent D. Neither parent Answer: B  In patients with de novo MEN 2B, where neither parent expresses the characteristic phenotype associated with the disease, the mutated RET allele is virtually always inherited from the father. The average age of males who transmit the disease is significantly greater than the average age of all fathers. This appears to be due to a selective advantage acquired by the testis stem cells, which increases the mutation’s frequency in the testis. (Choi SK, Yoon SR, Calabrese P, Arnheim N. Positive selection for new disease mutations in the human germline: evidence from the heritable cancer syndrome multiple endocrine neoplasia type 2B. PLoS Genet. 2012;8:e1002420.)

2. Patients with MEN 2A who develop pheochromocytomas and hyperparathyroidism, in addition to MTC, most commonly have mutations in which codon? A. RET codon 620. B. RET codon 918 C. RET codon 634 E. RET codon 791 E. RET codon 804 Answer: C  There is a clear relationship between genotype and phenotype in patients with MEN 2A. Virtually all patients with this syndrome develop MTC, but the development of pheochromocytoma and hyperparathyroidism is highly variable, occurring much more often in patients with RET 634 codon mutations and much less frequently in patients with other RET codons mutations. (Wells SA Jr, Pacini F, Robinson BG, Santoro M. Multiple endocrine neoplasia type 2 and familial medullary thyroid carcinoma: an update. J Clin Endocrinol Metab. 2013;98:3149-3164.)

5. In patients with hereditary MTC, thyroidectomy should be performed within the first 5 years of life in which cases? A. All patients with a germline RET codon mutation B. Only patients with MEN 2B C. Patients with MEN 2A who have a germline RET codon 634 mutation and patients with MEN 2B D. Patients with a germline RAS mutation E. Patients who only have an elevated serum calcitonin level Answer: C  The published guidelines for managing patients with hereditary MTC all discuss the timing of thyroidectomy in children who have inherited a mutated RET allele. There is uniform agreement that thyroidectomy should be performed before 5 years of age in patients with RET mutations in codon 634, and as soon as the diagnosis is made in patients with MEN 2B, even in the first months of life. (Kloos RT, Eng C, Evans DB, et al. Medullary thyroid cancer: management guidelines of the American Thyroid Association. Thyroid. 2009;19:565-612.)

3. After thyroidectomy, the most reliable indicator of prognosis is which of the following? A. The specific RET or RAS mutation associated with the MTC B. The serum levels of calcitonin (CTN) and carcinoembryonic antigen (CEA) immediately postoperatively C. The rate at which serum CTN and CEA double D. The completeness of the thyroidectomy E. The gender of the patient Answer: C  The presence of an elevated serum CTN or CEA in the immediate postoperative period or soon thereafter is diagnostic of persistent or recurrent MTC. Although this determination is valuable, it is not as significant an indicator of prognosis as the rate at which serum levels of CTN and CEA double. Doubling times of less than 6 months for either CTN or CEA are a very poor prognostic sign. (Meijer JA, le Cessie S, van den Hout WB, et al. Calcitonin and carcinoembryonic antigen doubling times as prognostic factors in medullary thyroid carcinoma: a structured meta-analysis. Clin Endocrinol. 2010;72:534-542.)

1665

CHAPTER 247  Paget Disease of Bone  

TABLE 247-1  GENES THAT PREDISPOSE TO PAGET DISEASE−LIKE SYNDROMES SYNDROME

GENE (PROTEIN)

GENE FUNCTION

Familial expansile osteolysis Early-onset familial Paget disease of bone Expansile skeletal hyperplasia

TNFRSF11A (RANK)

Enhances osteoclast differentiation and bone resorption

Autosomal dominant Insertion mutations exon 1

Onset during adolescence, extensive bone lesions, deafness, tooth loss.

Juvenile Paget disease

TNFRSF11B (OPG)

Inhibits osteoclast differentiation and bone resorption

Autosomal recessive Various loss of function mutations

Onset during childhood, extensive bone lesions, deafness, fractures, deformity, premature cardiovascular disease

Inclusion body myopathy, Paget disease, and frontotemporal dementia

VCP (p97)

Multiple cellular functions, including roles in NFκB signaling and autophagy

Autosomal dominant Loss of function mutations in UBA domain

Onset during 3rd-4th decades, with myopathy, Paget disease, and dementia occurring during 5th-6th decades

studies have identified seven other loci that predispose to Paget disease, which individually increase the risk between 1.4 and 1.7 fold.4,5 These loci have additive effects such that individuals who carry several predisposing alleles have a substantially increased risk for developing Paget disease. Many of these loci lie close to genes that play key roles in osteoclast function, including CSF1, which encodes macrophage colony-stimulating factor (M-CSF); TNFRSF11A, which encodes RANK; TM7SF4, which encodes DC-STAMP; and OPTN, which encodes optineurin (see Fig. 247-1). Several inherited diseases with clinical features overlapping with those of Paget disease have also been described (Table 247-1). These syndromes are also caused by mutations in genes that regulate osteoclast function. Under normal circumstances, bone is renewed and repaired in an orderly and tightly regulated fashion through the process of bone remodeling. The bone remodeling process is highly abnormal in Paget disease. Osteoclasts are increased in number, larger than normal, and hypernucleated. Some contain nuclear inclusion bodies. These were originally thought to be paramyxovirus nucleocapsids, but it has been suggested more recently that they may be aggregates of un-degraded proteins caused by defects in the autophagy pathway. Bone formation is also markedly increased, but the amount of new bone that is formed greatly exceeds that which has been removed by osteoclast activity, leading to enlargement and deformity of affected bones (Fig. 247-2). The bone that is formed is laid down in a disorganized fashion (woven bone) and has impaired mechanical strength. Other features include increased vascularity and marrow fibrosis. The focal nature of Paget disease remains a puzzle. Suggested explanations include the occurrence of somatic mutations in affected bones, which locally increase osteoclast activity, or excessive mechanical loading or skeletal injuries early in life, which by causing microdamage act as a focus for localized increases in bone remodeling.

CLINICAL MANIFESTATIONS

It has been estimated that between 7 and 16% of patients with Paget disease come to medical attention, and the presentation is highly variable.6 Many patients are asymptomatic, and Paget disease is detected as the result of a raised serum alkaline phosphatase (ALP) or an abnormal radiograph in patients who are being investigated for another reason. In those that do present clinically, symptoms that are attributable to Paget disease are observed in about 75% of cases. The most common is pain, which can be due to either increased bone turnover or a complication such as osteoarthritis, spinal stenosis, pseudofractures, or nerve compression syndromes. Deafness may occur in patients with skull involvement, but this is usually conductive rather than due to auditory nerve compression. Osteosarcoma occurs in less than 0.5% of cases but should be suspected in patients who experience a sudden increase in bone pain or swelling of an affected site. Other, rare complications include obstructive hydrocephalus, high-output cardiac failure, and hypercalcemia in patients who are immobilized. The risk for cardiovascular disease is increased in patients with Paget disease compared with age- and gendermatched controls, probably owing to an increased prevalence of vascular calcification. Most patients have no clinical signs but some present with bone deformity (see Fig. 247-2) or warmth of the skin overlying an affected bone.

DIAGNOSIS

The diagnosis can usually be made by radiograph, which shows the typical features of focal osteolysis with coarsening of the trabecular pattern, bone expansion, and cortical thickening (see Fig. 247-2). Occasionally, the disease

INHERITANCE/MUTATION

CLINICAL FEATURES

may be predominantly lytic in nature (see Fig. 247-2). The most sensitive way of defining the extent of Paget disease is a radionuclide bone scan in which tracer uptake is intensely increased at affected sites (see Fig. 247-2). Imaging with magnetic resonance imaging and computed tomography is not usually required unless complications such as spinal stenosis or osteosarcoma are suspected. Laboratory testing should include assessment of renal function, calcium, albumin, alkaline phosphatase (ALP), and 25(OH)D levels; liver function should be assessed to rule out the possibility that elevations in ALP are of hepatic origin. Typically, Paget disease presents with an elevation in ALP with otherwise normal biochemistries, but normal levels of ALP do not exclude the diagnosis. Vitamin D deficiency is a common finding but most likely reflects the fact that Paget disease predominantly affects older people in whom vitamin D deficiency is prevalent. Specialized markers such as bonespecific ALP or procollagen type 1 N-terminal propeptide can be useful in patients with coexisting liver disease but otherwise offer no advantage over total ALP in diagnosis and assessing treatment response. Susceptibility to Paget disease can be assessed in relatives of affected patients by genetic testing for SQSTM1 mutations, although this is not commonly performed in routine clinical practice. The differential diagnosis includes hyperostosis frontalis interna (a benign condition characterized by osteosclerosis of the frontal bones of the skull), fibrous dysplasia, pustulotic arthro-osteitis (which can present with mixed osteosclerotic and osteolytic lesions of the clavicle and ribs),7 and osteosclerotic metastases, particularly from carcinoma of the prostate. Usually, Paget disease can be distinguished from these conditions biochemically and through imaging, but occasionally, biopsy of an affected site may be required.

Treatment  The most common indication for medical treatment of Paget disease is bone pain localized to an affected site.8 Although such pain may be caused by increased metabolic activity, other causes may also be operative, including nerve compression syndromes, pseudofractures, secondary osteoarthritis, and other musculoskeletal conditions. Careful assessment of the patient is therefore necessary to decide on the most appropriate treatment. Bone pain caused by increased metabolic activity is localized to the affected site and is usually accompanied by a raised ALP level. It is common to encounter patients in whom pain occurs in the presence of coexisting osteoarthritis, bone deformity, or other musculoskeletal conditions. In such cases, it can be difficult to be sure about the origin of the pain, and many clinicians give a therapeutic trial of bisphosphonates. If the pain responds, then one can assume it was due to increased metabolic activity; if it does not, further evaluation should be undertaken to identify the cause and treat the patient appropriately. Pseudofractures represent a distinct management problem. These are areas of focal osteolysis that traverse the lateral cortex of weight-bearing bones of the lower limbs. Some remain stable for prolonged periods without causing symptoms; others regress spontaneously; and others progress to pathologic fracture, often in association with a localized increase in pain at the affected site.9

Bisphosphonates

Bisphosphonates are the drugs of first choice for the treatment of pain that is thought to be due to increased metabolic activity. Nowadays, nitrogencontaining bisphosphonates (aminobisphosphonates) are used in preference to older bisphosphonates because of their greater potency (Table 247-2).

1666

CHAPTER 247  Paget Disease of Bone  

B

A

C

D

FIGURE 247-2.  Radiographic and histologic features of Paget disease. A, Radionuclide bone scan image showing intense tracer uptake, typical of Paget disease of bone (PDB) in the right femur. B, Radiograph of an affected left femur showing bone expansion with mixed osteolytic/osteosclerotic areas and loss of normal trabecular pattern. A pseudofracture is visible in the lateral cortex (arrow). C, Histologic features from a hematoxylin and eosin stained section. A large osteoclast is visible (black arrow) close to an area of new bone formation (white arrows). There is extensive marrow fibrosis. Irregular cement lines typical of woven bone are apparent to the right of section. D, Predominantly lytic Paget disease of the left femur. The lytic area involves the intertrochanteric region and extends down the femoral shaft (white arrows).

Placebo controlled trials have shown that most bisphosphonates are effective at improving bone pain in Paget disease. Superiority of aminobisphosphonates over etidronate and tiludronate at suppressing ALP levels has been demonstrated, but with little difference in the response of pain. There are limited data comparing different aminobisphosphonates. In an open-label study comparing pamidronate 180 mg intravenously in unit doses of 30 mg weekly or 60 mg alternate weeks to oral alendronate 40 mg daily given in 3-monthly blocks over a 2-year period, there were no significant differences in the proportion of patients who achieved normal levels of ALP (86 and 91% respectively) or in symptomatic response. A1  Another study that compared a single infusion of 5 mg zoledronic acid with oral risedronate 30 mg daily for 2 months showed significant superiority of zoledronic acid in lowering ALP. Those randomized to zoledronic acid had greater improvement in some domains of health-related quality of life, but the differences between groups were small (1 to 2 points) and below the threshold of 5 points, which is considered clinically significant. A2  Another randomized trial compared the effects of giving repeated course of bisphosphonates (mainly risedronate) with the aim of normalizing ALP (intensive treatment), with therapy primarily aimed at

controlling symptoms (symptomatic therapy) in Paget disease. A3  This showed no difference in response of pain, quality of life, or complications between the groups, indicating that trying to restore ALP to normal confers no clinical advantage in most patients with Paget disease. After initiation of bisphosphonate therapy, levels of ALP start to fall within about 10 days and reach a nadir between 3 and 6 months. Levels of ALP can remain suppressed for many months or years thereafter, particularly with zoledronic acid. Symptoms can improve while ALP levels are still falling, and good clinical responses are often observed in patients whose ALP levels are not restored to normal. Intravenous bisphosphonates can cause transient bone pain, myalgia, headache, nausea, pyrexia, and fatigue within 1 to 3 days of the infusion in about 25% of cases (acute phase response). These symptoms can be ameliorated by acetaminophen given before and for a few days after the infusion, but they almost always subside within 7 days even without treatment. The acute phase response is much less common after second and subsequent infusions. Hypocalcemia may occur, particularly in patients with substantial elevations in bone turnover and vitamin D deficiency. The risk can be 

1667

CHAPTER 248  OSTEONECROSIS, OSTEOSCLEROSIS/HYPEROSTOSIS  

TABLE 247-2  BISPHOSPHONATES USED IN THE TREATMENT OF PAGET DISEASE DRUG

DOSE

COMMON ADVERSE EFFECTS

bisphosphonate therapy might impair fracture union and bone repair, but there is little evidence to suggest that this is a problem in clinical practice.10 Orthopedic surgery may also be required in patients who develop osteosarcoma, but the prognosis is poor even with aggressive operative treatment.

Oral Etidronate*

400 mg/day orally for 3-6 mo

Diarrhea, nausea, abdominal pain

Tiludronate

400 mg/day orally for 3 mo

Diarrhea, nausea, dyspepsia

Risedronate

30 mg/day orally for 2 mo

Dyspepsia, esophagitis

Alendronic acid†

40 mg/day orally for 6 mo

Dyspepsia, esophagitis

Pamidronate

180 mg IV in unit doses of 30 mg weekly or 60 mg alternate weeks

Acute phase response, hypocalcemia

Zoledronic acid

5 mg IV

Acute phase response, hypocalcemia

Intravenous

*Now seldom used. † Not licensed in the United Kingdom or Europe for Paget disease. Etidronate, pamidronate, tiludronate, and risedronate should be avoided if estimated glomerular filtration rate (eGFR) 24 hr

20 yr

20 ng/mL* *Criterion not valid if there is an associated clonal myeloid disorder. From Horny HP, Metcalfe DD, Bennett JM, et al. Mastocytosis. In: Swerdlow SH, Campo E, Harris NL, et al, eds. WHO Classification of Tumours of Haematopoietic and Lymphoid Tissues. Lyon, France: IARC Press; 2008:54-63.

sections (Fig. 255-3A). Such clusters are frequently observed around blood vessels and next to bone trabeculae in bone marrow biopsy sections. Immunohistochemical staining for tryptase is the recommended method for visualization of mast cells. Routine hematoxylin and eosin or metachromatic stains such as toluidine blue are not sufficiently sensitive to demonstrate subtle mast cell infiltrates or abnormal morphologic features of mast cells within the infiltrates in decalcified bone marrow biopsy sections. Mast cell morphology in bone marrow provides important clues to the diagnosis of systemic mastocytosis. Bone marrow mast cells in systemic mastocytosis often display atypical morphology, such as an elongated (spindle) shape, hypogranularity, and an eccentric or lobulated nucleus (Fig. 255-3B). These atypical mast cells are usually observed in close association with bone

1709

CHAPTER 255  Mastocytosis  

marrow spicules in the aspirate smear. Mast cells in mast cell leukemia (MCL) may be very sparsely granulated. Flow cytometric analysis of the mast cells in a bone marrow aspirate, when it is performed appropriately, is a sensitive diagnostic aid. The mean percentage of mast cells in a healthy bone marrow aspirate is approximately 0.02%, and it does not exceed 1% in most patients with mastocytosis. Therefore, to visualize the mast cell population correctly, the total cell numbers analyzed by flow cytometry should be significantly higher than those in other, more routine evaluations (e.g., leukemia phenotyping). The characteristic flow cytometric finding of systemic mastocytosis is the aberrant expression of CD25 or CD2 on CD117+ mast cells. CD25 is more sensitive than CD2 as CD2 may be absent or weakly expressed in some cases of advanced mastocytosis. Aberrant CD25 expression can also be demonstrated by immunohistochemical staining of bone marrow biopsy specimens.6 Serum tryptase level may be elevated in patients with mastocytosis.7 Currently available commercial tryptase immunoassays measure levels of total tryptase, the sum of mature tryptase, and tryptase precursors. Mature tryptase enzyme is a serine protease stored in mast cell granules and is transiently elevated in serum or plasma after mast cell degranulation episodes, such as anaphylaxis. In contrast, tryptase precursor proenzymes (α and β protryptases) are constitutively secreted outside the cell, and their serum levels at baseline correlate with mast cell burden. The median serum tryptase level in a healthy population is approximately 5 ng/mL. A serum tryptase level higher than 20 ng/mL raises suspicion for systemic mastocytosis in the appropriate clinical setting. A normal tryptase level does not rule out a diagnosis of mastocytosis, and increased tryptase levels can be seen in other conditions, such as myelodysplastic syndromes, acute myeloid leukemias, chronic eosinophilic leukemia, and chronic renal insufficiency. Metabolites of histamine, such as N-methylhistamine, and prostaglandin D2 can be elevated in a 24-hour urine specimen but are neither more sensitive nor more specific than the baseline serum tryptase measurement in mastocytosis. Demonstration of a codon 816 KIT mutation (D816V) may be necessary to fulfill the diagnostic criteria in patients lacking the major criterion (see Pathobiology).8 Examination of lesional tissues, such as skin and bone marrow, affords the highest sensitivity. Codon 816 KIT mutations have been detected in a variety of other neoplastic diseases, such as core binding factor acute myeloid leukemias, sinonasal lymphomas, and seminomas, in addition to mastocytosis. A rare histologic variant with clustering of mature round mast cells without CD25 expression termed well-differentiated systemic mastocytosis has been described. These patients generally have a history of childhood-onset mastocytosis without the D816V KIT mutation and therefore may respond to imatinib as opposed to those with typical systemic mastocytosis carrying the D816V mutation (see Treatment).

World Health Organization Disease Categories

Each patient diagnosed with mastocytosis should be assigned a category of disease according to the WHO classification (see Table 255-1). Cutaneous mastocytosis in the absence of bone marrow and internal organ involvement is the most common category in patients with pediatric-onset disease. Systemic mastocytosis is divided into the categories of indolent systemic mastocytosis, systemic mastocytosis with associated clonal hematologic non–mast cell lineage disease (SM-AHNMD), aggressive systemic mastocytosis, and MCL. An algorithm for classification of systemic mastocytosis is presented in Figure 255-4. Indolent systemic mastocytosis is the most common category in adults. Patients in this category usually have a normal life expectancy compared with age-matched general populations, although they experience symptoms related to release of mast cell mediators.9 Indolent systemic mastocytosis follows a persistent course, and progression to a more advanced category is unusual ( 10% or bone marrow aspirate mast cells > 20% No

Yes

Non–mast cell clonal hematologic disorder (e.g., MPD or MDS)

Mast cell leukemia

No

Yes

Evidence of end-organ dysfunction (e.g., ascites, malabsorption)

SM-AHNMD

No

Yes

ISM

ASM

FIGURE 255-4.  An algorithm for classification of systemic mastocytosis. ASM = aggressive systemic mastocytosis; ISM = indolent systemic mastocytosis; MDS = myelodysplastic syndromes; MPD = myeloproliferative disorders (neoplasms); SM-AHNMD = systemic mastocytosis with associated clonal hematologic non–mast cell lineage disease.

circulation or 20% or more mast cells in bone marrow aspirate smears, or both. To diagnose MCL, the mast cell percentage in bone marrow aspirate smears should be assessed in an area of the slide that is sufficiently distant from the spicules. Mast cell sarcoma and extracutaneous mastocytoma are rare diagnoses characterized by malignant and benign solid mast cell collections, respectively. There is a subset of patients with recurrent idiopathic or hymenoptera venom–induced anaphylaxis who have evidence of clonal mast cells carrying the D816V KIT mutation or aberrantly expressing surface CD25, without fully meeting the WHO diagnostic criteria and without displaying urticaria pigmentosa skin lesions. Such patients are provisionally referred to as having a monoclonal mast cell activation syndrome.

TREATMENT  The major goal of treatment for all categories of mastocytosis is symptom control. A reduction in mast cell numbers is considered only in disease categories with a poor prognosis (i.e., SM-AHNMD, aggressive systemic mastocytosis, MCL, and mast cell sarcoma).10,11 Current treatment modalities have not been shown to change the natural course of the disease.12

Medical Therapy

Patients with cutaneous and indolent systemic mastocytosis are treated symptomatically. Pruritus in mastocytosis usually responds to scheduled doses of histamine1-receptor blocker antihistamines, such as fexofenadine or cetirizine. Sedating antihistamines, such as hydroxyzine or diphenhydramine, may be used before bedtime. Photochemotherapy (oral psoralen plus ultraviolet A) or phototherapy may be helpful in patients with refractory pruritus; it results in symptomatic improvement and temporary fading of the pigmented skin lesions in up to 50% of patients. The side effects of phototherapy, including increased risk of skin cancer, should be taken into account when this treatment is considered. Histamine2-receptor blocker antihistamines, such as ranitidine or famotidine, are usually prescribed as a first-line treatment for patients with gastrointestinal complaints, such as heartburn, nausea, and abdominal pain. Proton pump inhibitors may be added in patients whose abdominal symptoms are refractory to histamine2-receptor blockers. Oral cromolyn sodium (adult dose, 200 mg four times daily) has been effective in reducing abdominal pain, diarrhea, nausea, vomiting, and pruritus in various studies, although the beneficial effects are variable among patients. Finally, low to moderate doses of systemic glucocorticoids can be beneficial in unusual cases of aggressive mastocytosis presenting with recalcitrant diarrhea associated with malabsorption or hepatomegaly with ascites.

1710

CHAPTER 255  Mastocytosis  

Cysteinyl leukotrienes, such as LTC4, that are produced after mast cell activation are thought to contribute to symptoms in mastocytosis. Therefore, drugs targeting the synthesis or receptor binding of leukotrienes are usually added to the treatment regimens of patients who derive suboptimal relief of itching and abdominal pain from histamine receptor–blocking therapy. However, there have been no controlled studies evaluating the clinical efficacy of this class of drugs in patients with mastocytosis. Self-administered epinephrine should be considered for all patients even if they do not have any history of hypotensive or anaphylactic episodes resulting in presyncope or syncope from acute mast cell degranulation. These episodes should be treated like systemic anaphylaxis (Chapter 253). Cytoreductive therapy, considered in aggressive disease variants associated with poor prognosis, has yielded disappointing results thus far. Some patients with recurrent life-threatening episodes of mast cell mediator release unresponsive to conventional therapy may also be candidates for cytoreductive therapy after careful consideration of risks and benefits. Approaches to cytoreductive treatment of mastocytosis have included interferon alfa-2b and the nucleoside analogue 2-chlorodeoxyadenosine. Interferon alfa-2b (0.5 to 5 million units, three to five times per week), alone or with prednisone, has been reported to partially improve clinical and laboratory abnormalities in approximately 50% of patients with aggressive systemic mastocytosis, patients with osteoporosis and pathologic fractures, and patients with recalcitrant recurrent anaphylaxis, although complete histopathologic and molecular remissions appear to be rare. Interferon alfa is difficult to tolerate because of its many side effects, including influenza-like symptoms, bone pain, and depression. A regimen of 2-chlorodeoxyadenosine (0.10 to 0.17 mg/kg/day for 5 days, repeated at intervals of 4 to 8 weeks) has been reported to result in partial and transient responses in patients with advanced categories of disease in case reports and small series. MCL usually is treated with polychemotherapy as acute myeloid leukemia (Chapter 183), although a successful treatment regimen has not yet been identified. Imatinib, a tyrosine kinase inhibitor with activity against wild-type KIT, PDGFR, and abl, has been effective in a small number of patients without D816V KIT mutation or with the FIP1L1-PDGFRA fusion gene, who present with chronic eosinophilic leukemia (Chapter 170) with a modest increase in bone marrow mast cells. However, most patients with mastocytosis have the D816V KIT mutation, which confers resistance to imatinib, and therefore are not appropriate candidates for this therapy.13

Ancillary and Other Therapies

Avoidance of the triggers of mast cell degranulation is an important adjunct to the pharmacologic treatment of symptoms. These show remarkable individual variation among patients (see Pathobiology), and the individual medical history can be helpful in identifying such triggers. General anesthesia and surgery impose an additional risk to patients with mastocytosis because several agents that are used perioperatively, such as muscle relaxants, opioid analgesics, and nonsteroidal anti-inflammatory drugs, can induce acute mast

cell degranulation. Prior surgical and anesthesia records should be obtained if available, and an appropriate strategy for the anesthetic management of the patient should be determined, with close communication involving the patient, anesthesiologist, surgeon, and an allergist. Non–mast cell clonal hematologic disorders associated with mastocytosis should be treated according to the standard-of-care guidelines for those disorders, regardless of the presence of mastocytosis. Bone marrow transplantation (Chapter 178) has yielded variable results for the treatment of mast cell disease, and occasional cases resulting in complete remission have been reported. Venom immunotherapy is recommended for those with a history of systemic reactions to hymenoptera who have evidence of IgE-mediated sensitization (by blood or skin allergy testing). Most experts recommend the duration of the therapy to be indefinite as fatalities have been reported after discontinuation of immunotherapy. Because of the high prevalence of osteoporosis and pathologic bone fractures in mastocytosis, bone densitometry should be considered a standard diagnostic procedure in adult patients with mastocytosis. If osteoporosis is detected, it should be treated per standard recommendations (Chapter 243).

PROGNOSIS

The prognosis for mastocytosis varies by the category of disease. At least 50% of patients with pediatric-onset cutaneous mastocytosis have complete resolution of the disease by adolescence, and the great majority of the rest of those patients experience improvement or fading of the skin lesions. Indolent systemic mastocytosis is a persistent disease but has a good prognosis without a decrease in life expectancy, and progression to a more aggressive disease category is rare.14 Factors associated with poorer prognosis have been reported as the absence of urticaria pigmentosa, older age at onset of symptoms, elevated serum lactate dehydrogenase or alkaline phosphatase, thrombocytopenia, anemia, peripheral blood smear abnormalities, and detectability of the D816V KIT mutation in peripheral blood. The prognosis for SM-AHNMD is determined by the prognosis for the associated hematologic disorder. Aggressive systemic mastocytosis and MCL have poor prognoses, with median survival times of less than 3 years and less than 1 year, respectively. GENERAL REFERENCES For the General References and other additional features, please visit Expert Consult at https://expertconsult.inkling.com.

CHAPTER 255  Mastocytosis  

GENERAL REFERENCES 1. Carter MC, Metcalfe DD, Komarow HD. Mastocytosis. Immunol Allergy Clin North Am. 2014;34:181-196. 2. Molderings GJ. The genetic basis of mast cell activation disease—looking through a glass darkly. Crit Rev Oncol Hematol. 2015;93:75-89. 3. Alvarez-Twose I, Bonadonna P, Matito A, et al. Systemic mastocytosis as a risk factor for severe Hymenoptera sting–induced anaphylaxis. J Allergy Clin Immunol. 2013;131:614-615. 4. Akin C. Anaphylaxis and mast cell disease. What is the risk? Curr Allergy Asthma Rep. 2010;10:34-38. 5. van der Veer E, van der Goot W, de Monchy JG, et al. High prevalence of fractures and osteoporosis in patients with indolent systemic mastocytosis. Allergy. 2012;67:431-438. 6. Morgado JM, Sánchez-Muñoz L, Teodósio CG, et al. Immunophenotyping in systemic mastocytosis diagnosis: “CD25 positive” alone is more informative than the “CD25 and/or CD2” WHO criterion. Mod Pathol. 2012;25:516-521. 7. Valent P, Sperr WR, Sotlar K, et al. The serum tryptase test: an emerging robust biomarker in clinical hematology. Expert Rev Hematol. 2014;7:683-690.

1710.e1

8. Valent P, Escribano L, Broesby-Olsen S, et al. Proposed diagnostic algorithm for patients with suspected mastocytosis: a proposal of the European Competence Network on Mastocytosis. Allergy. 2014;69:1267-1274. 9. Rabenhorst A, Christopeit B, Leja S, et al. Serum levels of bone cytokines are increased in indolent systemic mastocytosis associated with osteopenia or osteoporosis. J Allergy Clin Immunol. 2013;132:1234-1237. 10. Siebenhaar F, Akin C, Bindslev-Jensen C, et al. Treatment strategies in mastocytosis. Immunol Allergy Clin North Am. 2014;34:433-447. 11. Pardanani A, Tefferi A. Systemic mastocytosis in adults: a review on prognosis and treatment based on 342 Mayo Clinic patients and current literature. Curr Opin Hematol. 2010;17:125-132. 12. Cardet JC, Akin C, Lee MJ. Mastocytosis: update on pharmacotherapy and future directions. Expert Opin Pharmacother. 2013;14:2033-2045. 13. Ustun C, DeRemer DL, Akin C. Tyrosine kinase inhibitors in the treatment of systemic mastocytosis. Leuk Res. 2011;35:1143-1152. 14. Brockow K. Epidemiology, prognosis, and risk factors in mastocytosis. Immunol Allergy Clin North Am. 2014;34:283-295.

1710.e2

CHAPTER 255  Mastocytosis  

REVIEW QUESTIONS 1. Which of the following markers is aberrantly expressed by mast cells in patients with systemic mastocytosis? A. CD25 B. CD117 (Kit) C. High-affinity IgE receptor D. CD45 Answer: A  CD25 (alpha chain of the interleukin-2 receptor) is aberrantly expressed and is a sensitive diagnostic marker in systemic mastocytosis. All others are expressed by both normal and pathologic mast cells. (Valent P, Horny HP, Escribano L, et al. Diagnostic criteria and classification of mastocytosis: a consensus proposal. Leuk Res. 2001;25:603-625.) 2. A 25-year-old woman has recently been diagnosed with systemic mastocytosis by a bone marrow biopsy. There was no other hematologic disease noted. She experiences pruritus of the skin in hot weather and occasional abdominal cramping but is otherwise asymptomatic. KIT mutational analysis reveals the D816V mutation in bone marrow. Which of the following is the most appropriate treatment? A. Imatinib B. H1 and H2 antihistamines C. Prednisone D. Interferon alfa E. Bone marrow transplantation Answer: B  This patient has indolent systemic mastocytosis and should be treated symptomatically. Cytoreductive therapies such as interferon alfa are indicated in patients with aggressive mastocytosis. The patient is not a candidate for imatinib because of indolent disease and presence of D816V KIT mutation. The patient’s symptoms do not warrant bone marrow transplantation or use of systemic glucocorticoids, which are associated with long-term adverse effects. (Valent P, Akin C, Escribano L, et al. Standards and standardization in mastocytosis: consensus statements on diagnostics, treatment recommendations, and response criteria. Eur J Clin Invest. 2007;37:435-453.) 3. A 35-year-old farmer with a 10-year history of mastocytosis who is otherwise healthy has recently experienced anaphylaxis with brief loss of consciousness after being stung by a honeybee. Allergy skin testing confirmed honeybee allergy. What is the most appropriate next step in management of honeybee allergy in this patient? A. Venom immunotherapy is not recommended because of increased risk of reactions to immunotherapy due to mastocytosis. B. Recommend venom immunotherapy for 5 years. C. Recommend venom immunotherapy indefinitely. D. Confirm skin test results by blood testing before recommending immunotherapy. Answer: C  Patients with mastocytosis are at increased risk for life-threatening reactions to hymenoptera stings. Fatalities have been reported in patients after discontinuation of venom immunotherapy. Therefore, most experts recommend immunotherapy when IgE-mediated sensitization is found by either skin or blood testing in these patients. Whereas the patients are also at increased risk of having a systemic reaction during immunotherapy, the riskbenefit ratio is often considered favorable to initiate immunotherapy. The patients should always continue to carry a self-injectable epinephrine as the protection is lower than for those without mastocytosis. (Niedoszytko M, de Monchy J, van Doormaal JJ, et al. Mastocytosis and insect venom allergy: diagnosis, safety and efficacy of venom immunotherapy. Allergy. 2009;64:12371245; and Alvarez-Twose I, Bonadonna P, Matito A, et al. Systemic mastocytosis as a risk factor for severe Hymenoptera sting–induced anaphylaxis. J Allergy Clin Immunol. 2013;131:614-615.)

4. Which of the following patients is a candidate for therapy with imatinib? A. 30-year-old with adult-onset indolent systemic mastocytosis with D816V KIT mutation who remains symptomatic despite symptomatic therapy B. 30-year-old with adult-onset systemic mastocytosis with occasional symptoms on symptomatic therapy; peripheral blood analysis negative for D816V KIT mutation C. 60-year-old with systemic mastocytosis, splenomegaly, and pancytopenias; peripheral blood and bone marrow positive for D816V KIT mutation D. 40-year-old with childhood-onset cutaneous mastocytosis that persisted into adulthood; patient recently developed progressive splenomegaly and anemia; mutational analysis of bone marrow shows an exon 8 KIT mutation Answer: D  Patients with D816V KIT mutation regardless of the category of mastocytosis are resistant to therapy with imatinib. Patients with cutaneous or indolent systemic mastocytosis with symptoms manageable by symptomatic therapy are not candidates for cytoreductive therapy regardless of the mutational status. Patients with well-differentiated systemic mastocytosis without codon 816 (exon 17) KIT mutations have a high likelihood of response to imatinib. (Akin C, Fumo G, Yavuz AS, et al. A novel form of mastocytosis associated with a transmembrane c-kit mutation and response to imatinib. Blood. 2004;103:3222-3225.) 5. Which of the following mast cell mediator measurements raises the greatest suspicion for mastocytosis? A. Tryptase level of 50 ng/mL in a patient with perioperative anaphylaxis, obtained 1 hour after the event B. Baseline tryptase level of 90 ng/mL in a patient who experiences recurrent anaphylactic episodes of unclear etiology C. Elevated urinary N-methylhistamine in a patient with recurrent unexplained flushing D. Elevated urinary prostaglandin D2 in a patient with chronic diarrhea and osteoporosis Answer: B  Urinary metabolites of mast cell mediators are neither more sensitive nor more specific than measurement of baseline tryptase in diagnosis of systemic mastocytosis. Tryptase levels increase (usually within 4 hours) after an anaphylactic event regardless of whether it is associated with mastocytosis. Some patients with recurrent unexplained anaphylaxis who may have been previously diagnosed with idiopathic anaphylaxis may have mastocytosis as an underlying disorder (especially if the anaphylactic episodes involve hypotension and syncope rather than urticaria or angioedema). The diagnosis should be confirmed by a bone marrow biopsy in the patient in option D. (Akin C, Scott LM, Kocabas CN, et al. Demonstration of an aberrant mastcell population with clonal markers in a subset of patients with “idiopathic” anaphylaxis. Blood. 2007;110:2331-2333; and Akin C, Metcalfe DD. Surrogate markers of disease in mastocytosis. Int Arch Allergy Immunol. 2002;127: 133-136.)

XXII

RHEUMATIC DISEASES 256 APPROACH TO THE PATIENT WITH RHEUMATIC DISEASE

257 LABORATORY TESTING IN THE RHEUMATIC DISEASES

258 IMAGING STUDIES IN THE RHEUMATIC DISEASES

259 CONNECTIVE TISSUE STRUCTURE AND FUNCTION

262 OSTEOARTHRITIS

270 THE SYSTEMIC VASCULITIDES

263 BURSITIS, TENDINITIS, AND OTHER

271 POLYMYALGIA RHEUMATICA AND

PERIARTICULAR DISORDERS AND SPORTS MEDICINE

264 RHEUMATOID ARTHRITIS

AND BONES

273 CRYSTAL DEPOSITION DISEASES

266 SYSTEMIC LUPUS ERYTHEMATOSUS

274 FIBROMYALGIA, CHRONIC FATIGUE

267 SYSTEMIC SCLEROSIS

261 THE SYSTEMIC AUTOINFLAMMATORY

268 SJÖGREN’S SYNDROME

DISEASES

272 INFECTIONS OF BURSAE, JOINTS,

265 THE SPONDYLOARTHROPATHIES

260 INHERITED DISEASES OF CONNECTIVE TISSUE

TEMPORAL ARTERITIS

(SCLERODERMA)

269 INFLAMMATORY MYOPATHIES

SYNDROME, AND MYOFASCIAL PAIN

275 SYSTEMIC DISEASES IN WHICH ARTHRITIS IS A FEATURE

276 SURGICAL TREATMENT OF JOINT DISEASES

1712

CHAPTER 256  Approach to the Patient with Rheumatic Disease  

256  APPROACH TO THE PATIENT WITH RHEUMATIC DISEASE VIVIAN P. BYKERK AND MARY K. CROW

Rheumatic diseases are common and an important cause of reduced quality of life, increased comorbidity, and reduced life expectancy. They incur a significant socioeconomic burden and warrant expertise on the part of all physicians who treat patients. This chapter provides a framework to approach the evaluation of patients who present with signs and symptoms suggesting a rheumatic disease. An algorithmic approach is provided that allows the physician to incorporate presenting features, patient characteristics, anatomic structures, along with diagnostic tests, to facilitate a diagnosis and treatment plan.

DEFINITION AND CATEGORIZATION

Rheumatic diseases are disorders of connective tissue in which general or localized inflammation frequently manifests as pain attributable to peripheral joints, the spine, or muscles. Systemic features such as stiffness, fever, or weight loss and a multitude of extramusculoskeletal features, ranging from skin rashes to renal dysfunction, often accompany rheumatic diseases. In most cases, the basic underlying pathology is understood, although this is less true of pain disorders appearing alone or accompanying a rheumatic disease (Chapter 30). For most rheumatic disorders, the underlying molecular mechanisms through which environmental triggers and genetic susceptibility factors collaborate to result in a particular rheumatic disease in an individual remain to be fully elucidated. These diseases can be broadly considered as those that are primarily degenerative, with inflammation occurring secondarily, and those in which inflammation is the primary mediator of the disease. In the latter, the pathogenesis can be mediated through aberrant immune responses or as a result of metabolic abnormalities.

Histopathology

Rheumatic diseases are often also termed connective tissue diseases, understandably, because connective tissue is the most abundant tissue in the body supporting and connecting other tissues and organs. Loose and dense connective tissues include cellular components and extracellular matrix. Loose connective tissue fills spaces between muscle sheaths, encases blood and lymphatic vessels, and holds fibroblasts that synthesize collagen fibers. It includes reticular fibers that provide the skeleton of muscle cells, nerves, and capillaries. Dense connective tissue supports the body’s soft tissues and includes more collagen fibers and fewer cells. It is found in the dermis, joint capsules, cartilage, bone, and fascia of muscles, and it forms tendons, ligaments, and points of connection where these insert into bone (aponeurosis). Cells included in connective tissue may be wandering, such as mast cells or macrophages, or resident cells, such as fibroblasts, fibrocytes, and reticular cells. Fibroblasts are responsible for synthesizing collagen, elastic reticular fibers, and ground substance of extracellular matrix, including tissue fluids and collagen fibers. Importantly, connective tissue is integrated with cells associated with the body’s defense system: lymphocytes, plasma cells, macrophages, dendritic cells, and eosinophils. The close proximity of connective tissue to blood vessels and cells of the immune system provides the setting for a group of disorders that are mediated by impaired immune system regulation and disruptions of the vascular system.

Classification of Rheumatic Diseases

More than 100 types of rheumatic diseases have been described. Although these can be considered to be based primarily on one of two degenerative or inflammatory overarching processes, one can further subdivide rheumatic diseases as follows (and also outlined in Table 256-1): (1) those associated with degeneration of connective tissues attributable to (a) trauma, (b) structural/mechanical imbalances, or (3) inherent early demise of cellular components; (2) those associated with systemic autoimmunity,1 often linked with measurable autoantibodies that can manifest primarily with (a) synovitis, (b) widespread organ involvement, (c) inflamed blood vessels, or (d) inflammation of muscle; (3) other inflammatory connective tissue diseases

involving more dense tissues, not associated with the formation of autoantibodies and hence termed seronegative rheumatic diseases or spondyloarthropathies; (4) diseases in which inflammation of the vasculature, particularly small, medium, or large arteries, is the predominant feature; (5) autoinflammatory diseases that can be associated with crystal deposition or genetic mutations involving cytokine pathways; and (6) pain syndromes that must often be considered in the context of these diseases, in which some appear to be comorbid and closely linked to the underlying rheumatic disease, such as diffuse pain associated with Sjögren’s syndrome, hypermobility of connective tissue, or those regional pain syndromes that are anatomically linked to mechanical disruption. Patients presenting with generalized pain syndromes require investigation to exclude a connective tissue disease. Increasingly, genotypes have been identified that are associated with diseases that fall into each of these categories, and in some cases specific immunologic pathways have allowed grouping of a set of rheumatic diseases previously considered more distinct. Mimics of rheumatic diseases exist, and clinicians need to be mindful of considering these when evaluating a patient for a rheumatic disease. For instance, arthropathies and syndromes resembling a rheumatic disease can occur in the settings of both infection and malignancy. Autoimmune phenomena are increasingly being recognized in the setting of malignancy.2 Red flags for each need to be considered in the assessment of a patient for possible rheumatic disease. No classification of rheumatic disease can completely explain its genesis. However, considering these in a classification schema can aid in the approach to a patient in whom these disorders are being considered (see Table 256-1).

EPIDEMIOLOGY

Although connective tissue diseases can generally be categorized as noted in Table 256-1, in adults there are six prototypical rheumatic diseases most often assessed and managed by rheumatologists: rheumatoid arthritis (RA), systemic lupus erythematosus (SLE), systemic sclerosis (SSc), spondyloarthropathies (SpA) (primarily ankylosing spondylitis [AS]), Sjögren’s syndrome (SS), and vasculitis. These diseases are ubiquitous throughout the world (Table 256-2) and for the most part have a similar incidence and prevalence throughout the globe. Each is associated with differing immune aberrations and mechanisms of inflammatory damage, although the cause and reasons for chronicity still remain unknown. Autoimmune rheumatic diseases are also among the leading causes of death and morbidity in the industrial world, in part related to associated comorbid diseases, particularly comorbid cardiovascular disease. They contribute to a significant socioeconomic burden. Increasing evidence points to risks for their genesis relating to environmental factors, socioeconomic factors, and exposure to infectious agents, ultraviolet radiation, and pollutants. Smoking, in particular, has been associated with an increased risk for SLE and RA in genetically susceptible individuals in Western cultures. Effects of migration elucidate some of these risks. For instance, Africans who migrate far away from their native environmental and cultural origins appear to have an increased susceptibility to SLE. Also, reports have linked occupational exposures, such as silica dust, mercury, pesticides, solvents, and metals, to an increased risk for SLE and RA. In some cases, geographic clusters of rare autoimmune disease argue for specific genetic determinants. For example, with SSc, higher incidence, prevalence, and mortality rates have been reported in African American populations compared with white populations, and the prevalence has been reported as higher in southern Europe, particularly Italy (prevalence of 7 to 33 per 100,000). Additionally, social and demographic factors may contribute to the epidemiology of rheumatic diseases. For example, the prevalence of SLE is reported as very high in Georgia, United States, whereas the prevalence of AS is rare in malaria endemic regions where HLA-B27 genotypes are rare. Inflammatory arthropathies, including RA and AS, have a higher prevalence in North American Native populations.

CLINICAL MANIFESTATIONS

It is important to understand the potential clinical manifestations and natural history of rheumatic diseases. Primary care and hospital-based health care providers are often the first to evaluate a patient with an evolving rheumatic disease, and they need to be attuned to the presenting features to make a timely diagnosis. In many cases, the presentation could signal a life- or organthreatening condition. Evaluation of constitutional, systemic and joint symptoms should always include rheumatic disease in the differential diagnosis.

1713

CHAPTER 256  Approach to the Patient with Rheumatic Disease  

TABLE 256-1  COMMON RHEUMATIC DISEASES BROADLY CLASSIFIED ACCORDING TO PATHOGENESIS DEGENERATIVE DISEASES OF BONES AND JOINTS

SYSTEMIC AUTOIMMUNE DISEASES

SERONEGATIVE SPONDYLOARTHROPATHIES

VASCULAR RHEUMATIC DISEASES

AUTOINFLAMMATORY DISEASES

PAIN DISORDERS

Osteoarthritis

Rheumatoid arthritis

Ankylosing spondylitis

ANCA- associated vasculitis

Adult-onset Still’s disease

Regional myofascial pain syndromes

DISH

Systemic lupus erythematosus

Psoriatic arthritis

Temporal artery vasculitis

Crystal diseases

Tendonitis/bursitis

Degenerative disc disease

Sjögren’s syndrome

Reactive arthritis

Polymyalgia rheumatica

Pediatric periodic fever syndromes

Adhesive capsulitis

Spinal stenosis

Inflammatory myopathies (polymyositis, dermatomyositis)

Enteropathic arthritis

Behçet’s disease

Osteoporosis

Systemic sclerosis

Reflex sympathetic dystrophy Pain with hypermobility syndromes Fibromyalgia*

*The only pain disorder that has not been associated primarily with inflammation. ANCA = antineutrophil cytoplasmic antibody; DISH = diffuse idiopathic sclerosing hyperostosis (also linked to metabolic factors, including elevated growth hormone).

TABLE 256-2  WORLDWIDE PREVALENCE* AND INCIDENCE OF RHEUMATIC DISEASES ASSOCIATED WITH AUTOIMMUNITY DISEASE

NORTH AMERICA

CENTRAL AMERICA

SOUTH AMERICA

EUROPE

MIDDLE EAST

ASIA

SUB SAHARAN AFRICA

100-500

200-900 (2-7)

200-1500

100-800 (40-90)

Rare - 900

2000

50-60 (5)

N/A

20-70 (2-7)

N/A

20-70 (3)

Rare

20-80 (11)

N/A

N/A

95%)

Rim

Anti-double-stranded DNA

Double-stranded DNA

SLE (50%)

Speckled

Anti-Sm Anti-U1-RNP Anti-Ro (SS-A) Anti-La (SS-B) Anti-Ku Anti-SCL-70

snRNP proteins U1 snRNP proteins Two proteins complexed to small RNAs Y1-Y5 Single protein plus RNA polymerase III transcript DNA binding protein DNA topoisomerase I

SLE (30%) SLE (30%); MCTD (>95%) SLE (30%); Sjögren syndrome (70-80%) SLE (15%); Sjögren syndrome (50-70%) SLE (10%) PSS (40-70%); CREST (10-20%)

Nucleolar

Anti-PM-Scl Anti-Mi-2 Anti-RNA polymerase

Nucleolar protein complex Nuclear protein complex Subunits of RNA polymerase I

PSS (3%); PM (8%) DM (15-20%) PSS (4%)

Dividing cell

Anticentromere Antiproliferating cell nuclear antigen

Centromere/kinetochore protein Auxiliary protein of DNA polymerase δ

CREST (80%); PSS (30%) SLE (3%)

Cytoplasmic

Anti-Jo-1 Anti-PL-7 Anti-PL-12 Anti-SRP Anti-ribosomal P

Histidyl tRNA synthetase Threonyl tRNA synthetase Alanyl tRNA synthetase Signal recognition particle Large ribosomal subunit

ILD in PM/DM (18-25%) PM/DM (3%) PM (4%) SLE (10%) PM/DM (3%)

CREST = calcinosis, Raynaud phenomenon, esophageal dysmotility, sclerodactyly, and telangiectasia; DM = dermatomyositis; ILD = interstitial lung disease; MCTD = mixed connective tissue disease; PM = polymyositis; PSS = progressive systemic sclerosis (diffuse scleroderma); SLE = systemic lupus erythematosus; snRNP = small nuclear ribonucleoprotein; tRNA = transfer RNA.

1722

CHAPTER 257  Laboratory Testing in the Rheumatic Diseases  

6

correlate. Nevertheless, for each assay, the dynamic range for testing is large. With treatment and disease quiescence, anti-DNA antibodies may essentially disappear; with flare, levels may increase dramatically. This property distinguishes anti-DNA antibodies from other ANAs in SLE, levels of which tend to be more consistent over time. As is the case for other ANAs, the appearance of anti-DNA antibodies in the serum may precede other manifestations of SLE, suggesting vigilance if these antibodies are present in patients who have symptoms that suggest a CTD but lack other evidence to establish a firm diagnosis.

Other Antinuclear Antibodies

Anti-Sm and anti-RNP antibodies are related specificities that commonly occur together in the sera of patients with SLE, a phenomenon called linkage. These antibodies bind proteins on subcellular particles called snRNPs (small nuclear ribonucleoproteins) that are composed of a set of proteins and uridine-rich RNAs. Anti-Sm and anti-RNP antibodies differ in protein specificity and in the ability to cause immunoprecipitation of the bound RNA molecules. Anti-Sm antibodies occur only in patients with SLE and represent a serologic marker in disease classification. In contrast, anti-RNP antibodies can appear in the sera of patients with other clinical presentations and, in the absence of anti-Sm, may characterize patients with overlapping CTD features, so-called mixed CTD or MCTD. In SLE, the frequencies of anti-Sm and anti-RNP antibodies vary among racial and ethnic groups, although a clear association with particular clinical manifestations has not been established. Anti-Ro and anti-La antibodies (or anti-SS-A and anti-SS-B), another set of linked ANAs, are directed to protein-RNA complexes that are involved in cellular metabolism of RNA. These antibodies are expressed more widely in patients with CTD and appear in the sera of patients with SLE, RA, and Sjögren’s syndrome, among others. Assessment of these antibodies is important because of their association with the neonatal lupus syndrome, which results from the transplacental passage of antibodies and causes congenital heart block as well as rash in the neonate. Although both Sm/RNP and Ro/ La are complexes of proteins and RNA, these antibodies appear to be expressed by different patient subsets, suggesting distinct mechanisms of induction and clinical associations.7 Although ANAs are directed to ubiquitous antigens, they nevertheless are expressed in disease-specific patterns and may show association with particular organ-specific manifestations. These associations include anti–ribosomal P antibodies with central nervous systemic involvement in SLE, antibodies to DNA topoisomerase 1 (anti-SCL-70) with progressive systemic sclerosis (diffuse scleroderma), antibodies to centromeres with CREST syndrome (calcinosis, Raynaud phenomenon, esophageal dysmotility, sclerodactyly, and telangiectasia), and antibodies to histidyl transfer RNA synthetase (antiJo-1) with interstitial lung disease in scleroderma (Chapter 267). In inflammatory myopathies, the presence of certain autoantibodies may be associated with particular patterns of disease, with antibodies to the enzyme 3-hydroxy3-methylglutaryl-coenzyme A (HMG-CoA) reductase present in a syndrome of necrotizing myositis; the syndrome can occur in patients treated with statins, which can inhibit the enzyme.8 In addition to their association with specific disease manifestations, antibodies to both DNA- and RNA-binding proteins such as Sm and RNP may contribute to overall immune dysregulation in patients with autoimmune disease because of their formation of immune complexes containing DNA or RNA. These complexes can stimulate the production of type 1 interferon by triggering both TLR and non-TLR nucleic acid sensors as well as other cellular receptors (e.g., Fc receptors). Because immunoassays of interferon with patient sera are limited, the presence of interferon is observed more clearly in the pattern of gene expression known as the interferon signature in peripheral blood cells. This signature can be assessed by both microarray assays and measurement of more limited sets of messenger RNA molecules that are induced by interferon. Because antibodies to RNA-binding proteins in particular may promote this pattern, the serologic assay of these ANAs may allow assessment of the likelihood of both nonspecific and specific immunologic disturbances.9

Antibodies to Phospholipids

Originally defined by their effects on in vitro clotting tests, antibodies to phospholipids (APLs) are associated with in vivo thrombosis and have been termed lupus anticoagulants (LACs). Patients with these antibodies display a clinical condition, termed the antiphospholipid antibody syndrome, which is characterized by arterial or venous thrombosis, thrombocytopenia, and firsttrimester spontaneous abortions (Chapter 176). This syndrome may occur by itself or in the context of SLE, where it may contribute to the acceleration

of atherosclerosis, premature stroke, and myocardial infarction. The laboratory evaluation of this condition involves specific assays of antibodies to phospholipids and related proteins as well as functional assays of clotting. Because expression of these antibodies may vary over time, testing must be performed on more than one occasion at least 6 weeks apart. Furthermore, the results of immunochemical and functional assays may not be congruent because they are likely related to the heterogeneity of antibodies. The serology of APLs is complicated because it is related to the nature of the antigenic targets as well as heterogeneity among patients.10,11 These antigens include phospholipids such as cardiolipin. Cardiolipin, however, can bind to the protein β2-glycoprotein 1, which is also a target for antibodies in this condition. Serologic evaluation thus involves assays with a complex of cardiolipin and β2-glycoprotein 1 as well as β2-glcycoprotein in an ELISA format using reagents to measure IgG and IgM. The association of antibodies with thrombosis appears strongest with IgG antibodies; determination of the IgA isotype may also be informative depending on the results of IgG and IgM assays. ELISAs for these antibodies are not yet standardized, making it important to specify assay features related to quantitation such as cutoff values used to define positivity and values that are considered significant. Functional assays for LACs involve tests directed at inhibition of in vitro clotting (e.g., activated partial thromboplastin time, dilute Russell viper venom time), recognizing the discordance between in vivo thrombosis and in vitro anticoagulation. Functional assays to detect lupus anticoagulants involve a mixing step in which patient plasma is mixed with normal plasma to determine the presence of an inhibitor (i.e., an antibody) as opposed to a deficiency state. The mechanisms by which antibodies to phospholipids and related proteins may cause thrombosis in vivo are unknown, although these antibodies may interact with the surface of cells (e.g., endothelium) to promote a prothrombotic state. Assessing the likelihood of the syndrome is best accomplished by considering results of both the immunoassays and functional assays in the context of the individual patient because both factors may promote thrombotic events.

Complement

Assessment of the complement system can provide valuable information on the activity of diseases in which immune complex deposition may promote inflammation and tissue injury (Chapter 50). This system involves a large number of proteins that function in enzyme cascades to generate degradation products that amplify immunologic reactions and promote the destruction or removal of foreign organisms as well as damaged cells. In the setting of SLE and in certain forms of vasculitis and glomerulonephritis, immune complexes activate complement to promote local inflammation. This activation can be measured in terms of the total complement level in the blood by functional assays of hemolytic activity; by measurement of individual complement components such as C3 and C4, whose levels are reduced by cleavage during activation; by measurement of split products of cleaved complement components; and by measurement of complement fragments bound to red blood cells during complement activation. Proteins of the complement system are acute phase reactants and can increase with inflammation, including active disease. Correspondingly, low levels may reflect inherited complement deficiency rather than consumption; genetic deficiency of C1q, for example, is highly associated with SLE.

Antineutrophil Cytoplasmic Antibodies

Antineutrophil cytoplasmic antibodies (ANCAs) are autoantibodies that react to determinants in the neutrophil and occur prominently in patients with certain forms of necrotizing vasculitis or rapidly progressive glomerulonephritis. Reflecting the serology, conditions have been called ANCAassociated vasculitis (AAV). Two main forms of ANCA have been distinguished on the basis of the target antigens and pattern of immunofluorescence staining of fixed neutrophils: PR3-ANCA (C-ANCA), which reacts with proteinase-3 (PR3), and MPO-ANCA (P-ANCA), which reacts with myeloperoxidase (MPO). By immunofluorescence, PR3-ANCA shows staining in the cytoplasm; staining by MPO-ANCA localizes in the perinuclear area. ANCAs to other proteins have also been identified, but these may also occur in conditions other than vasculitis. In the evaluation of severe, multisystem inflammatory disease, ANCA testing is important to evaluate diagnostic possibilities.12 ANCAs occur in association with varying clinical manifestations in patients with AAV and help define patterns of clinical involvement in terms of organ system involvement as well as histopathology (e.g., presence of granulomatous inflammation). PR3-ANCA occurs commonly in patients with granulomatosis with polyangiitis (GPA, formerly called Wegener granulomatosis) as well

CHAPTER 258  Imaging Studies in the Rheumatic Diseases  

eosinophilic granulomatosis with polyangiitis (EGPA, formerly called Churg-Strauss disease); MPO-ANCA marks the course of vasculitis caused by microscopic polyangiitis.13 Although there is overlap between serology and clinical features, PR3-ANCA occurs commonly in patients with upper airway disease, whereas MPO-ANA occurs commonly patients with patients with rapidly progressive renal disease (Chapter 270). In patients with ANCA-associated glomerulonephritis, the kidney lacks evidence of immune deposits, as indicated by the lack of staining for immunoglobulins or complement. Kidney disease of this kind is termed pauciimmune glomerulonephritis. Although ANCA testing is useful in initial diagnosis, its role for assessing disease activity is less certain. Occasionally, in patients who are desperately ill and cannot tolerate a lung or kidney biopsy, the presence of an ANCA can be used as preliminary evidence for diagnosis to allow the initiation of immunosuppressive therapy. ANCA testing is also useful for assessing the likelihood for relapse because patients who express PR3-ANCA appear at risk for recurrent disease.

Cryoglobulins

Cryoglobulins are serum immunoglobulins that precipitate in the cold and promote the pathogenesis of systemic inflammatory disease through tissue deposition. The presence of a cryoglobulin is detected by allowing blood, collected warm, to remain cool at 2° to 4° C for 1 or more days. After centrifugation, the amount of cryoprecipitate is measured and expressed as a cryocrit. In the preanalytical phase, it is important that the blood remain at a temperature of 37° C during all steps from drawing the blood from the patient to separation of the serum fraction after coagulation. Thermos flasks with preheated sand or water and other special devices are available to keep the blood tubes at 37° C during transport. If these steps are not taken, cryoprecipitation at even room temperature may already occur before separation of the serum from the blood cells, possibly resulting in false-negative results.14 Subsequent analysis of the cryoprecipitate by immunochemical assays allows determination of its components. Cryoglobulins can be classified into three main types on the basis of their composition: (1) single, or type I; (2) mixed, type II; and (3) mixed, type III. A type I cryoglobulin consists of only a monoclonal immunoglobulin that precipitates in the cold. A mixed-type cryoglobulin contains RFs bound to polyclonal IgG to form an immune complex. In type II cryoglobulins, the IgM RF is monoclonal, and in type III, the IgM RF is polyclonal. Type I cryoglobulins occur in patients with lymphoproliferative disorders such as Waldenström macroglobulinemia, multiple myeloma, or chronic lymphocytic lymphoma (Chapters 184 and 187). In contrast, patients with mixed cryoglobulins can present with a wide range of signs and symptoms resulting from vasculitis. These manifestations include purpura (a sign of leukocytoclastic vasculitis), weakness, arthritis, and neuropathy, representing a syndrome known as essential mixed cryoglobulinemia. Most patients with this condition have infection with hepatitis C virus, with viral components present in the complexes. These patients have serologic evidence of this infection as well as manifestations attributable to the underlying liver disease. As in the case of other CTDs and systemic inflammatory diseases, the evaluation of patients with essential mixed cryoglobulinemia demands attention to the entire patient and the impact of disease on multiple organs. GENERAL REFERENCES For the General References and other additional features, please visit Expert Consult at https://expertconsult.inkling.com.

258  IMAGING STUDIES IN THE RHEUMATIC DISEASES RONALD S. ADLER Historically, rheumatic disorders have been well characterized by conventional imaging. In as much as these disorders often manifest in characteristic distributions and present with specific alterations in the appendicular or axial skeleton and adjacent soft tissues, radiographic evaluation has been sufficient to both characterize the abnormalities as well as provide a relatively small

1723

number of differential possibilities as to the specific disease. The most wellstudied example is rheumatoid arthritis (RA) in which symmetric involvement of the metacarpophalangeal joints, uniform joint space narrowing, periarticular osteopenia, and juxta-articular erosions along the “bare areas” are pathognomonic. The development of new therapeutic alternatives for the inflammatory arthritides, so-called disease-modifying antirheumatic drugs (DMARDs), and chondroprotective strategies in the case of osteoarthrosis, require methods to diagnose these diseases at an earlier stage, characterize the degree of inflammation, and provide a useful metric to assess therapeutic response.1 Indeed, it has become necessary to assess for possible joint and soft tissue abnormalities before irreversible tissue damage, the latter often being the case when the radiographic findings are abnormal. Fortunately, the requirement to achieve earlier diagnosis has paralleled advances in imaging. Ultrasonography and magnetic resonance imaging (MRI) have largely supplanted conventional radiographic evaluation in the imaging work-up of patients with suspected rheumatologic disorders and negative radiographs.2,3 The term molecular imaging has been applied, particularly in the case of MRI and positron emission tomography (PET), in as much as these modalities reflect local tissue environment or metabolic activity.

  RADIOGRAPHIC EVALUATION

Radiographic evaluation is among the first studies ordered in patients with a suspected rheumatologic disorder. In the current digital era, conventional analog-based radiographs have been largely replaced by computed radiography. Images are usually displayed on workstations with high-resolution monitors within the context of a picture archiving system (PACS). Digital radiographs are of high spatial resolution but relatively poor soft tissue contrast. These images are amenable to a variety of image processing schemes, resulting in enhanced definition of the cortical surfaces and cancellous bone, which may be of value in displaying subtle erosions. It is important to recognize that radiographs are projection images. To detect an abnormality, it may be necessary to view a joint or other structure at a specific angle. For instance, subtle erosions may only be apparent when viewed tangentially as opposed to en face. It is therefore necessary to have specific image protocols in order to optimally display the joint, cortical surface, or soft tissue structure. Most radiographic evaluations contain at least two orthogonal projections. The addition of an oblique view or other specialized projection may be necessary to address a specific clinical question. The nature and distribution of joint space narrowing, presence of osteopenia, new bone formation, soft tissue swelling, soft tissue calcification, chondrocalcinosis, presence and nature of erosions, and assessment for joint malalignment may allow a specific diagnosis and help determine the severity of disease (Fig. 258-1). For instance, the presence of a juxta-articular erosion extending over an adjacent area of slightly hyperdense soft tissue swelling in the setting of normal bone mineralization with maintenance of the adjacent joint space is diagnostic of gout, in contrast to RA noted earlier. The seronegative arthritides, such as psoriatic arthritis, have a characteristic appearance in the small joints of the hand and feet, including a predilection for distal joints, asymmetry, and appositional new bone formation. Table 258-1 summarizes some of the features of several of the more common diseases that may be encountered in clinical practice. Finally, radiographs provide a direct means for needle localization during percutaneous procedures, predominantly joint injections, aspirations, and some biopsies. These are generally performed while imaging in real time (fluoroscopy) using short bursts of low-intensity x-rays enhanced through an image intensifier. Injection of joints under fluoroscopic guidance provides a convenient means to ensure intra-articular deposition of therapeutic agent or for diagnostic aspiration. Intra-articular location is verified by injection of a small amount of a standard iodinated contrast material. Arthrography using fluoroscopic guidance can be used as a primary diagnostic tool, but this application has largely been replaced by intra-articular injection of contrast followed by computed tomography (CT) or MRI. For some procedures, CT may be preferable, depending on the location of the abnormality. The principal disadvantages of fluoroscopy relate to the use of ionizing radiation and poor soft tissue contrast. The latter becomes important with needle placements near neurovascular structures that may be potentially compromised by poor position. CT allows greater control over needle placement at the cost of greater levels of radiation exposure. Ultrasonography has replaced fluoroscopy and CT for a large number of percutaneous procedures. MRI provides another method to perform a variety of procedures without the necessity of ionizing radiation. These options will be discussed in greater detail below.

CHAPTER 257  Laboratory Testing in the Rheumatic Diseases  

GENERAL REFERENCES 1. Ansar W, Ghosh S. C-reactive protein and the biology of disease. Immunol Res. 2013;56:131-142. 2. Van Venrooij WJ, Van Beers JBC, Pruijn GJM. Anti-CCP antibodies: the past, the present and the future. Nat Rev Rheumatol. 2011;7:391-398. 3. Aleteha D, Neogi T, Silman AJ, et al. 2010 Rheumatoid arthritis classification criteria: an American College of Rheumatology/European League against rheumatism collaborative initiative. Arthritis Rheum. 2010;62:2569-2681. 4. Satoh M, Chan EK, Ho LA, et al. Prevalence and sociodemographic correlates of antinuclear antibodies in the United States. Arthritis Rheum. 2012;64:2319-2327. 5. Bizzaro N, Villata D, Giavarina D, et al. Are anti-nucleosome antibodies a better diagnostic marker than anti-dsDNA antibodies for systemic lupus erythematosus? A systematic review and a study of metanalysis. Autoimmun Rev. 2012;12:97-106. 6. Pisetsky DS. Standardization of anti-DNA antibody assays. Immunol Res. 2013;56:420-424. 7. Ching KH, Burbelo PD, Tipton C, et al. Two major autoantibody clusters in systemic lupus erythematosus. PLoS ONE. 2012;7:e32001.

1723.e1

8. Hamann PD, Cooper RG, McHugh NJ, et al. Statin-induced necrotizing myositis—a discrete autoimmune entity within the “statin-induced myopathy spectrum.”. Autoimmun Rev. 2013;12: 1177-1181. 9. Rönnblom L, Eloranta M-L. The interferon signature in autoimmune disease. Rheumatology. 2013;25:248-253. 10. Favaloro EJ. Variability and diagnostic utility of antiphospholipid antibodies including lupus anticoagulants. Int J Lab Hematol. 2013;35:269-274. 11. Lakos G, Favaloro EJ, Harris EN. International consensus guidelines on anticardiolipin and antiβ2-glycoprotein I testing. Arthritis Rheum. 2012;64:1-10. 12. Lionaki S, Blyth ER, Hogan SL. Classification of antineutrophil cytoplasmic autoantibody vasculitides. Arthritis Rheum. 2012;64:3452-3462. 13. Lally L, Spiera R. Current landscape of antineutrophil cytoplasmic antibody-associated vasculitis: classification, diagnosis, and treatment. Rheum Dis Clin North Am. 2015;41:1-19. 14. Damoiseaux J. The diagnosis and classification of the cryoglobulinemic syndrome. Autoimmun Rev. 2014;13:359-362.

1724

CHAPTER 258  Imaging Studies in the Rheumatic Diseases  

A

C

B

FIGURE 258-1.  Three hands with different diagnoses. A, Gout. Radiograph of the left hand showing multiple dense soft tissue nodules (n) with multiple small erosions affecting the ulnar styloid, triquetrum and fifth ray. A large erosion (arrow) at the fifth distal interphalangeal (DIP) joint demonstrates bone formation extending circumferentially about the adjacent tophaceous deposit typical of an overhanging edge. Bone mineralization and joint spaces are preserved. B, Rheumatoid arthritis. There is ulnar deviation of the second through fifth metacarpophalangeal (MCP) joints with uniform joint space loss involving the MCP joints and the carpus. The DIP joints are spared. Periarticular demineralization is present with small erosions along the radiovolar aspect of the second (arrow) MCP joint. C, Osteoarthrosis. Soft tissue swelling affecting the third digit with joint space narrowing and bone production affecting the DIP joints, third and fifth proximal interphalangeal (PIP) joints, basal joint of the thumb, and scaphotrapeziotrapezoid joint. There are subchondral cystic changes at the third PIP joint having an erosive character (arrow). Mineralization is preserved, as are the radiocarpal and MCP joint spaces.

TABLE 258-1  DISTINGUISHING RADIOGRAPHIC FEATURES OF SEVERAL COMMON RHEUMATIC DISEASES CONDITION Rheumatoid arthritis

COMMON SITES Hands: MCP, PIP; wrists: intercarpal, DRUJ, ulnar styloid; feet: fifth MTP, cervical spine (atlantoaxial, apophyseal)

DISTRIBUTION Bilateral, symmetric, polyarticular

Osteoarthritis (primary) Hands (DIP), wrists (basal joint, STT),feet Symmetric, weight-bearing joints (first MTP), hips (superolateral), knees (medial), spine (discs, facet, apophyseal, uncovertebral)

RADIOGRAPHIC FEATURES Periarticular osteopenia, periarticular swelling, subluxations (e.g., ulnar, volar), uniform joint space loss, erosions (bare areas) Normal or increased density, nonuniform joint space loss, subchondral sclerosis, cysts, bone formation (osteophytes) Spine–disc space narrowing, end plate sclerosis and bone formation

Psoriatic arthritis

Hands (DIP, terminal tufts), feet (IP joints), entheses (calcaneus: plantar, posterior), spine, sacroiliac joints

Asymmetric (single ray), polyarticular, Normal or increased density, periosteal bone formation, segmental (intervertebral, apophyseal) soft tissue swelling, ankylosis (SI joints), thick hyperostosis spine (nonmarginal syndesmophytes), juxta- and periarticular erosions

Ankylosing spondylitis

Spine, SI joints, fibrous joints (pubic symphysis), entheses (adductor origin), rhizomelic joints (hips, shoulders)

Symmetric, continuous (may affect entire Normal or increased density, erosions (spine squaring, spine-bamboo spine) shining corner) with superimposed bone formation (ankylosis: SI), thin (marginal) syndesmophytes)

Gout

Feet (first MTP), other damaged joints, elbow, knee, hindfoot

Asymmetric, extensor surfaces (elbow), abnormal joints (e.g., osteoarthritic joints)

Normal joint space, normal or increased density, dense soft tissue nodules (tophi), para-articular and subchondral erosions with bone formation along tophi (overhanging edge)

Calcium pyrophosphate dihydrate crystal deposition disease

Hands (second, third MCP), wrists (radiocarpal), TFC, knees (lateral compartment and patella-femoral, menisci)

Symmetric, fibrocartilaginous joints

Normal or increased density, hypertrophic bone formation, subchondral or periarticular cysts, chondrocalcinosis (hyaline, fibrocartilage), periarticular, peritendinous, periligamentous calcification

Infection

Any joint, pyogenic, TB

Monoarticular (mostly), any joint

Pyogenic (osteopenia; 8-10 days), joint space widened (early), joint space loss (rapid development), soft tissue swelling, erosions (both sides of joint), sequestra, periostitis, TB (joint space and mineralization may be preserved), juxta-articular erosions, spine–disc space loss and end plate erosion

DIP = distal interphalangeal; DRUJ = distal radial ulnar joint; IP = interphalangeal; MCP = metacarpophalangeal; MTP = metatarsophalangeal; PIP = proximal interphalangeal; SI = sacroiliac; STT = scaphotrapezotraphezoid; TB = tuberculosis; TFC = triangular fibrocartilage.

CHAPTER 258  Imaging Studies in the Rheumatic Diseases  

  COMPUTED TOMOGRAPHY

Computed tomography provides a two-dimensional map of tissue attenuation obtained from external x-ray source(s) located on a rotating gantry, whose radiation is detected by a series of detectors opposite the source. The current generation of CT scanners uses multiple detectors (16, 32, 64, and so on), allowing rapid image acquisition that can be displayed in a single plane in real time (CT-fluoroscopy) or as extremely thin section contiguous or overlapping acquisitions in the axial plane. The acquired images can be reconstructed in multiple planes with equivalent (isotropic) resolution elements (voxels) or as a three-dimensional rendering. Some scanners use dual energy sources, taking advantage of differences in the attenuation characteristics of various tissues at different energies. This has received greatest attention in the setting of gout, enabling a definitive diagnosis as well as depicting tophaceous deposits in anatomic locations not conducive to radiographs or ultrasound.4 Computed tomography allows the best assessment of trabecular and cortical bone, providing an excellent means to assess fractures and erosions, the presence of new bone formation (e.g., fracture callus), and degenerative or inflammatory arthritis. Soft tissue mineralization can likewise be well characterized, providing important information as to its etiology. Joints that are difficult to assess on radiographs, including the sacroiliac, temporomandibular, wrist, and sternoclavicular joints, are well seen on CT (Fig. 258-2). Computed tomography generally has poor soft tissue contrast. Nevertheless, it is still very useful in performing a number of guided procedures because of its tomographic nature and rapid image acquisition capability. Improved soft tissue contrast can be obtained with use of iodinated contrast material. A number of soft tissue tumors, inflammatory synovitis, and infectious processes display pathologic enhancement after contrast administration. CT can likewise be used to produce angiographic displays (CTA) when used in combination with contrast, providing exquisite detail of central and peripheral vascular disease, including in patients with suspected vasculitis. These agents are typically administered intravenously following well-defined enhancement characteristics. CTA has become the method of choice in evaluating patients with suspected pulmonary embolism. Likewise, contrast agents may be used to improve intra-articular contrast (CT arthrography), currently the method of choice in assessing internal derangement in the postoperative shoulder, knee, and so on and in patients who are unable to undergo MRI (e.g., those with claustrophobia, aneurysm clips, or cardiac pacemakers). Imaging of cartilage and soft tissue abnormalities usually depends on pathologic imbibition of contrast material, indicative of degeneration or tearing. A limitation of this approach resides in the fact that some abnormalities may remain occult. An example is the inability to detect a bursal-sided rotator cuff tear after shoulder CT arthrography. The radiation dose from CT can be high, especially when using the newer scanners. This is most significant when one is looking to minimize exposure, such as in children, requiring protocols specifically designed for the pediatric

A

1725

population. Intravenous (IV) use of iodinated contrast agents are contraindicated in patients with impaired renal function or history of allergic reaction. Nonionic agents can diminish the associated risks but still should be used with caution.

  ULTRASONOGRAPHY

Ultrasound imaging takes advantage of the near uniform speed of sound and predictable attenuation characteristics of sound propagation in soft tissue. In general, anatomic images derive from specular surfaces whose dimensions exceed the ultrasound wavelength; inherent noise (speckle) within the image derives from small scatterers, smaller than the resolution element of the transducer. Modern ultrasound equipment contains various methods to reduce speckle in the image, resulting in a more anatomic rendition of the soft tissues. Rapid image acquisition and processing enables ultrasonography to be performed in real time (≈30 frames per second). Ultrasonography is also conducive to evaluation of blood flow from which estimates of flow velocity can be obtained through the Doppler equation. Doppler information is typically reported by either continuously estimating velocity at a specific depth (spectral Doppler) or through a color encoded two-dimensional map (color or power Doppler). There is great appeal for using ultrasonography in patients with rheumatic disorders. There is no ionizing radiation, and it is real time, inexpensive, relatively portable, and well tolerated. Historically, however, ultrasonography has played only a limited role in the diagnostic assessment and treatment of patients with suspected musculoskeletal abnormalities, being used to differentiate fluid-filled from solid masses. The detection of a Baker cyst in the knee or the presence of a joint effusion constituted two major applications. There has also been limited application of ultrasonography to perform imageguided aspirations and biopsies. Within the United States, in particular, the development of MRI further limited the musculoskeletal applications of ultrasonography. With the development of linear high-frequency small parts transducers; new imaging capabilities of ultrasound scanners; and the evolution of a new class of compact, portable (laptop) ultrasound units that have excellent image quality, the role of ultrasonography has dramatically changed in recent years.5,6 These new applications have paralleled the development of new classes of DMARDs for which diagnosis of inflammatory synovitis prior to joint destruction is optimal. The current generation of ultrasound scanners enables examination of the small joints of the hands and feet, allowing early detection of synovitis (Fig. 258-3). Typically, a 10-MHz or higher frequency linear transducer is used. The displacement of the joint capsule by hypoechoic (dark) soft tissue that displays vascularity on Doppler imaging or is incompressible with direct pressure by the transducer is characteristic, allowing differentiation of synovitis from an effusion. In addition to the detection of synovitis, ultrasonography has been shown to be more sensitive than conventional radiographs in the detection of erosions. Erosions appear as discrete irregular discontinuities

B

FIGURE 258-2.  Infectious sacroiliitis in a 12-year-old boy with a 2-week history of back and left hip pain. A, Axial computed tomography (CT) image of the pelvis at the level of the sacroiliac joints (SIs) photographed using window settings optimized for bone detail. There is clear asymmetry in the two SI joints, with the left appearing more irregular. The cortical margins of the left sacral ala are less distinct and there is an isolated bone fragment (arrow) surrounded by soft tissue suspicious for a sequestrum. B, CT-guided aspiration of the left SI joint confirmed an infectious origin.

1726

CHAPTER 258  Imaging Studies in the Rheumatic Diseases  

A

B

FIGURE 258-3.  Synovitis on ultrasonography in a female patient with normal hand radiographs. A, Gray-scale ultrasound image obtained along the dorsal aspect of the radioscaphoid joint shows hypoechoic soft tissue (arrows) distending the dorsal recess. The cortical margins of the scaphoid (S) and radius (R) appear as bright reflectors on ultrasonography. B, Power Doppler image depicts the marked vascularity (red color hues) of the soft tissue illustrating the level of disease activity.

in the normally smooth hyperechoic (bright), reflecting cortical surfaces, often seen in continuity with adjacent inflammatory soft tissue. There is some variation in the appearance of synovitis among various arthritides. The distribution, presence, or lack of symmetry and other concomitant findings may be necessary to obtain a specific diagnosis. The level of vascularity on color-flow imaging can reflect active inflammation, correlating with clinical and biochemical parameters. A parametric image encoding either mean Doppler shift (color Doppler) or amplitude (power Doppler) is typically used as a standard Doppler map. Both maps can be used to detect abnormal levels of vascularity. Whereas power Doppler provides an indirect measure of the number of moving scatterers within the region being scanned, color Doppler provides a velocity map, and therefore is more subject to artifact (angle dependence and sampling errors). When combined with color-flow imaging, the activity of the synovitis can be estimated. Ultrasound contrast agents can depict capillary flow, resulting in significantly improved detection sensitivity of synovial inflammation, and are used extensively in Europe. They constitute microbubble agents encased in a lipid or polysaccharide shell that can be instilled as either bolus or constant infusion, with the shell being metabolized in the liver and the gas exhaled in the lungs. These agents have biological half-lives on the order of minutes and are best suited to examining target joints. Contrast agents have received Food and Drug Administration approval for cardiovascular applications and therefore can only be used off label for the assessment of synovitis. Articular and fibrocartilage have characteristic appearances on ultrasonography. Whereas the former appears as a thin hypoechoic band paralleling the articular surface, fibrocartilage appears hyperechoic. Chondrocalcinosis appears as discrete hyperechoic foci with the substance of the cartilage, in which case its presence is suggestive of calcium pyrophosphate deposition disease. Calcification along the margin of the articular cartilage gives rise to the double-line sign seen in gout. Tendons and muscles have characteristic appearances on ultrasonography. The presence of tendinosis, tendon tears, muscle edema or inflammation, atrophy, and tears can be diagnosed. Ultrasonography is very sensitive, although not specific, for the detection of small amounts of calcification or ossification. It is an excellent method to assess for calcific peritendinitis and to provide guidance for treatment. Abnormal fluid distention of synovial lined structures can be assessed and treated under ultrasound guidance. Ultrasonography is an excellent modality to provide image guidance for therapeutic aspiration and injection of small and large joints, tendon sheaths, and cysts (e.g., bursae, ganglion, paralabral cysts, hematomas, abscesses,) (Fig. 258-4). The real-time capability of ultrasonography is useful to demonstrate the presence of subluxations or painful snapping, to document the distribution of injected material, and to assess adhesions. Ultrasonography is considered the method of choice to detect foreign bodies. Nerves also have a characteristic appearance on ultrasonography. In crosssection, a nerve often has a “cluster of grapes” appearance, with nerve fascicles appearing hypoechoic and surrounded by hyperechoic endo- and epineural fat. In long axis, nerves display a characteristic tram-track appearance. Ultrasonography has been shown to be useful in the diagnosis and treatment of carpal tunnel syndrome and cubital tunnel syndrome. It is an excellent modality to assess for the presence of posttraumatic or postsurgical and interdigital neuromas and to provide image guidance for treatment, including therapeutic injections and ablative therapy.

Although ultrasonography is well suited to the evaluation of superficial structures, it is less well suited to deep structures. Frequency and penetration are reciprocally related: the higher the frequency, the better the axial resolution but poorer the degree of penetration. A 15-MHz linear transducer would work well in the hand but not in the hip. Examination of a hip might require a 5-MHz transducer and curved transducer geometry with reduced image quality. Excessive abdominal fat can further limit acoustic penetration and distort the ultrasound beam, limiting image quality. Diagnostic ultrasonography does not penetrate bone, resulting in limited acoustic access to joint structures. In some instances, soft tissue contrast can be poor. An inexperienced scanner may find ligaments and tendinous insertions difficult to differentiate from adjacent fibrofatty structures.

  MAGNETIC RESONANCE IMAGING

The natural abundance of hydrogen in biological systems and an inherent property of hydrogen, called spin, form the basis of conventional MRI. When placed in a strong magnetic field, protons tend to align themselves along the direction of the field. Magnetic field strengths are specified as Tesla and can be variable between clinical scanners. The majority of scanners in clinical usage vary between 1 and 3 Tesla. Application of a radiofrequency (RF) pulse to the system of protons induces the spins to rotate away from the direction of the field, during which time they precess about the direction of the magnetic field at a characteristic frequency, called the Larmor frequency. When the RF pulse is turned off, the spins relax toward their initial state determined by two tissue-dependent relaxation times, T1 and T2, which vary with field strength. T1 (also known as the spin-lattice relaxation) and T2 (or the transverse relaxation time) along with proton density are the principal determinants of signal intensity. The image can emphasize either the T1 or T2 characteristics of the tissue, impacting tissue contrast. Different tissues have varying appearance often based of levels of fat and water content, reflected by their inherent T1 and T2 relaxation times. Tissue morphology is often characterized by their appearance on T1-weighted or proton density images: tendon, muscle, fat, marrow, cortical bone, articular, and fibrocartilage have characteristic appearances. Many pathologic states, alternatively, are characterized by increased mobile water or effective T2 lengthening. Examples include soft tissue edema, inflammatory infiltrates, and neoplasm (Fig. 2585). Images that emphasize T2 contrast are therefore helpful to display most pathologic states. Selective maps of T2 have been used to characterize the state of articular cartilage in early degenerative disease.7 Other cartilage specific properties that relate to water content, glycosaminoglycan (GAG) content, and integrity of collagen architecture can be assessed using T2 and other parametric maps that can be derived from the MR data (Fig. 258-6). The widely used contrast for MRI studies is a neutral, hydrophilic salt of the gadolinium chelate, gadolinium diethylenetriamine-penta-acetic acid (Gd-DTPA). Gadolinium can be injected intravenously or directly into the joint. IV injection (indirect magnetic resonance arthrography) carries the contrast in the vascular system to areas of hyperemia and inflammation (Fig. 258-7). It can be used for assessment of synovial activity in inflammatory joint diseases.8 Gadolinium is taken up in inflamed synovium and is able to demonstrate thickened pannus. The slope of the early time-signal intensity curve provides a measure of tissue perfusion and can quantify inflammatory activity. Contrast material excreted into the synovial fluid provides excellent depiction of intra-articular structures and can be used in lieu of arthrographic

CHAPTER 258  Imaging Studies in the Rheumatic Diseases  

B

A

C

1727

D

FIGURE 258-4.  Ultrasound-guided therapy in the first metatarsophalangeal (MTP) joint of a patient with pain and swelling. A, Longitudinal gray scale image of the dorsal recess of the first MTP joint. Fluid and soft tissue distend the joint capsule (arrows). The metatarsal (m) and proximal phalanx (p) are labeled. Note that a thin hypoechoic (dark) band parallels the surface of the metatarsal head, corresponding to the overlying articular cartilage. B, Increased vascularity (red color hues) demonstrated on power Doppler imaging within the dorsal recess reflects the level of disease activity. C, Transverse gray scale ultrasound image shows a needle (N) within the distended dorsal recess from which several drops of synovial fluid were aspirated followed by therapeutic injection. D, Postinjection transverse ultrasonography depicts low level echoes (small echogenic foci within dorsal recess) and microbubbles (arrows) within the distended joint capsule from injected material. Whereas microbubbles aggregate along the nondependent portion of the distended joint capsule, injected material tends to settle to the deep portion of the recess.

A

B

FIGURE 258-5.  Magnetic resonance image of the right wrist in a female patient with advanced rheumatoid arthritis. A, Proton density coronal image shows loss of normally bright marrow signal within the scaphoid and lunate bones (arrows). The proximal scaphoid is eroded, and the lunate appears deformed and translocated and volarly tilted (not shown), giving rise to its triangular appearance. The distal ulna (u) is poorly visualized because of a large erosion. Intermediate-intensity material (appears dark gray) within the carpus and distal radioulnar joint is difficult to separate from the distal ulna, lunate, and scaphoid. The triquetrum (t), hamate (h), trapezium (tr1) and trapezoid (tr2), capitate (c), and radius (r) are labeled. B, Fluid-sensitive coronal image emphasizing T2 relaxation demonstrates increased signal intensity (bright) within the inflammatory pannus, compatible with increase in mobile water associated with inflammation. Increased signal intensity is evident within the lunate, scaphoid, and distal ulna, including focal areas within the distal row of carpal bones, corresponding to small erosions. Diffuse increased signal within the distal radius likely reflects reactive marrow edema (asterisk).

1728

CHAPTER 258  Imaging Studies in the Rheumatic Diseases  

A

B

C

D

FIGURE 258-6.  Imaging of the soft tissues in a patient with retrocalcaneal pain demonstrating complementary nature of magnetic resonance imaging and ultrasonography. Whereas the proton-density sagittal image (A) emphasizes anatomic detail, the fluid-sensitive image (B) depicts thickening and increased signal intensity within the distal Achilles tendon, reflecting tendinosis, retrocalcaneal bursitis, or a tear of the deep surface of the tendon. Surrounding increased signal intensity (bright areas) within the adjacent soft tissue reflects adjacent soft tissue edema. Long-axis gray scale (C) and power Doppler (D) images of the same patient obtained when the patient presented for ultrasound-guided therapeutic injection. The tendon (arrows) is inhomogeneous. A prominent hypoechoic collection deep to the tendon is compatible with retrocalcaneal bursitis (b). There is prominent increased vascularity on power Doppler imaging at the margin of the bursa and tendon. The calcaneus (calc) is labeled.

direct techniques. In glycosaminoglycan (GAG) -depleted cartilage, there can be delayed uptake of contrast into the cartilage, which would normally be inhibited by the negatively charged GAG molecules. Patients with renal disease who receive IV injection of gadolinium can develop nephrogenic systemic fibrosis (NSF). (Also see the section Nephrogenic Systemic Fibrosis in Chapter 267.) When the kidney cannot sufficiently clear out the gadolinium, it produces fibrosis of many tissues, including the skin, muscle, heart, nerves, and pleura. To date, NSF has been seen only in patients who have been given IV gadolinium with acute or chronic renal insufficiency. The changes in the skin with NSF are usually bilateral and symmetrical, primarily involving the extremities and the trunk. These changes can mimic systemic sclerosis but, unlike that disease, the face is usually spared. If renal function improves, the skin lesions may stabilize or get better, although in some patients, the process progresses, affecting mobility and causing severe pain. Injection of dilute gadolinium into the joint (direct magnetic resonance arthrography) is helpful for outlining structures to determine whether there is morphologic damage. Injection is usually performed either under fluoroscopic or ultrasound guidance. This technique is particularly effective for visualization of small structures such as the labrum of the hip or shoulder if there is no joint effusion. It is also helpful for demonstrating breakdown of soft tissue structures that normally prevent communication between joint compartments such as the rotator cuff, triangular fibrocartilage of the wrist, and ligaments in the various joints. Newer techniques that enable image acquisition in near real time as well as the development of MR-compatible needles now permit a variety of percutaneous procedures to be performed directly under MR guidance.

  SCINTIGRAPHY

Scintigraphy by its nature represents physiologic imaging because it derives from labeling physiologically occurring substances with a gamma-emitting

radionuclide and uses detectors in the form of gamma cameras arranged in a planar or circumferential configuration to determine the distribution of radionuclide within the tissue. Scintigraphy can provide a global assessment of abnormal tracer update or can be performed using a targeted approach (Fig. 258-8). Images often provide high tissue contrast but are of relatively poor spatial resolution. Commonly used agents vary from tagged red blood cells to assess blood flow; agents that reflect bone metabolism (technetium99m methylene diphosphate [Tc-MDP]); agents that reflect glucose metabolism (18-fluorine deoxy-glucose [18-FDG]), in the case of PET ; and agents that concentrate at sites of inflammation, such as autologous white blood cells labeled with 111In (Indium) and 67Ga-citrate (gallium). Clinical applications include detection of a variety of malignancies, osteomyelitis, vascular graft infection, multifocal infectious disease, inflammatory diseases such as RA, vasculitis, inflammatory bowel disease, sarcoidosis, fever of unknown origin, and infection of joint prostheses. Traditional nuclear medicine involves use of single gamma photon emissions as a product of nuclear decay. The information can be displayed using planar imaging through a single (or multiple) pinhole camera or displayed tomographically in a manner similar to CT (single-photon emission computed tomography [SPECT]). Bone scintigraphy uses Tc-MDP as the radioactive tracer. The isotope goes to areas of high bone turnover and vascular flow as well as areas of calcium or bone deposition. Three-phase bone scans are obtained at different intervals after injection, reflecting the early vascular phase, the intermediate blood pool phase, and the late phase. Each phase allows for further characterization of the disease process. Abnormal tracer uptake is seen in areas of inflammation, infection, neoplasm, osteonecrosis, and fracture. The scan is most useful to identify the location of lesions within the skeleton but is nonspecific. Positron emission tomography scans use the appearance of two simul­ taneously produced 511-KEV gamma rays after annihilation of a positron and electron pair to localize the distribution of radionuclide. The

CHAPTER 258  Imaging Studies in the Rheumatic Diseases  

1729

A 0

(min–1)

0.38

0

0.57

0

0.05

100 90 80 70 60 50 40 30 20 10

B ms

A

B

FIGURE 258-7.  Functional magnetic resonance image. A, Parametric image derived from fitting a twocompartment model of soft tissue and synovial enhancement after intravenous administration of contrast material. Three parameters extracted from the time-intensity curves are displayed as parametric images: Ktrans (left) provides a measure of contrast exchange into the extravascular soft tissues; Ve (center) and Vp (right) reflect the relative distribution volumes for the extravascular space and plasma, respectively. The arrows depict a region of increased synovial volume and enhancement at the second MCP joint. Increased values of Ktrans and Ve illustrate increased vascular permeability at the site of inflammation. (Printed with permission of Dr. Luis Beltran.) B, Sagittal T2 map of the knee in which relative T2 relaxation is color encoded, showing regions of higher cartilage T2 values at the femoral condyle and tibial plateau. This reflects alterations in cartilage collagen architecture and water content and possibly early osteoarthritis. (Printed with permission of Dr. Gregory Chang.)

C

D

FIGURE 258-8.  Rectilinear bone scan in a patient with back pain. Anterior (A) and posterior (B) delayed images of the axial and appendicular skeleton demonstrate increased tracer uptake in the region of the sacral ala, left ankle, and right midfoot (arrows). Follow-up radiographs confirmed the presence of bilateral sacral ala fractures. Note that the central pooling of tracer in the expected location of the urinary bladder is normal. Bone scans provide a sensitive but nonspecific method to evaluate the appendicular and axial skeletal. Increased uptake in the feet in this patient was attributed to degenerative change.

1730

CHAPTER 259  Connective Tissue Structure and Function  

near-simultaneous detection of the photons (coincidence counting) provides an estimate of source tracer concentration. Newer PET scanners are often used in combination with either CT or MRI to achieve improved spatial registration, allow accurate estimates of soft tissue attenuation, provide highquality anatomic images, and quantify metabolic activity.9 Combined PET-CT or PET-MRI provides high-resolution images of abnormal metabolic activity and may ultimately provide the most definitive maps of inflammatory activity in patients with rheumatic disease.10 Early results to date have been promising and are expected to provide sensitive evaluation of the response to DMARDs in patients with inflammatory arthritis. GENERAL REFERENCES

In addition to their structural role, connective tissues are involved in storage and activation of growth factors, cytokines, and morphogens. Some connective tissue molecules mediate inflammation when released as intact molecules, or may acquire new properties as proteolytically derived fragments (matrikines).1 Connective tissues such as bone, cartilage, and tendon harbor mesenchymal stem-like cells (MSC) that undergo expansion and differentiation during growth, repair, and regeneration and are therapeutically valuable. Bone contributes to calcium and phosphate homeostasis. A broad perspective on connective tissue is thus crucial for understanding the complex clinical presentation of inherited connective tissue disorders, pathways of tissue repair and regeneration, and pathogenesis of degenerative and autoimmune conditions.

For the General References and other additional features, please visit Expert Consult at https://expertconsult.inkling.com.

259  CONNECTIVE TISSUE STRUCTURE AND FUNCTION SUNEEL S. APTE Connective tissues are those having a primary mechanical or structural role. They are typically skeletal or osteoarticular tissues such as bone, cartilage, tendon, ligament, fascia, intervertebral disc, and joint-associated structures such as synovium and fibrocartilaginous menisci, although adipose tissue is also included (Table 259-1). However, all viscera, glands, and vascular and nervous tissue contain varying amounts of connective tissue elements that maintain proper spatial relationships between their cells, organize them into functional units, and provide an internal fibrous skeleton or external protective capsule. These connective tissue elements primarily comprise extracellular matrix (ECM), which is organized as a basement membrane or an interstitial matrix. Examples include elastic fibers in the aorta and lung that mediate stretch and recoil, glomerular basement membrane that participates in filtration, and the fibrous endoskeleton that connects the heart valves and transmits myocardial contraction. Indeed, heart valves and chordae tendinae have regional structural similarities to cartilage and tendons.

  CONNECTIVE TISSUE CELLS, THEIR EXTRACELLULAR MATRIX AND   TISSUE-SPECIFIC BIOMECHANICS

Connective tissues comprise cells, typically derived from mesoderm, or in the craniofacial region, derived from the neural crest, and the ECM they secrete and assemble around them. Unique morphologic and biosynthetic properties of the differentiated cells specify osteoblasts, osteoclasts, and osteocytes (in bone); chondrocytes (in cartilage); and tenocytes (in tendons), as well as mixed phenotypes such as cells of fibrocartilage (in the intervertebral disc, some tendon and ligament insertions, and joint menisci). Chondrocytes alter their biosynthetic profile as they terminally differentiate into hypertrophic chondrocytes in growth plate cartilage, downregulating aggrecan and collagen II expression and synthesizing collagen X as a specialized product.2 As the growth plate transitions into bone, the hypertrophic cells die, blood vessels invade the cartilage, and the cartilage matrix is gradually replaced by bone. Fibroblasts, which secrete collagens as a major product, are the dominant cells in tendons, fascia, ligaments, and dermis. The abundant ECM of connective tissues is their major defining characteristic, but in other tissues and organs, fibroblasts are relatively quiescent, and the interstitial ECM is less organized and not as abundant. However, under appropriate stimuli, fibroblasts transition to highly contractile, biosynthetically active myofibroblasts, which are associated with hypertrophic scars and fibrosis. During embryonic development, undifferentiated connective tissue cells (mesenchymal cells) produce a hydrated, loose, provisional ECM optimal for migration, branching, and folding of individual cells and cell collectives, such as epithelial sheets. As embryogenesis progresses, provisional ECM is remodeled by matrix-degrading proteinases and replaced by specialized connective tissues whose mechanical properties3 are better suited to weight bearing, locomotion, and increased circulatory stress. This requirement for increased mechanical strength is not met in the severest forms of inherited connective

TABLE 259-1  DIVERSITY OF CONNECTIVE TISSUES STRUCTURE

FUNCTION

CELLS

KEY MATRIX COMPONENTS

Adipose tissue

Energy metabolism Physical protection

Adipocytes, fibroblasts, endothelial cells

Collagen I, collagen III, microfibrils, collagen IV, laminin

Basement membranes

Epithelial support Cell polarity Filtration barrier Cell barrier Transparency (lens)

Epithelial and endothelial cells, myotubes, adipocytes, lens fibers

Collagen IV, laminin, nidogens, perlecan

Bone

Structural support Hematopoiesis Mineral storage

Osteoblasts, osteoclasts, osteocytes

Collagen I, osteocalcin, bone sialoprotein, hydroxyapatite

Cartilage

Bone growth Joint motion Load transmission

Chondrocytes

Collagen II, aggrecan

Dermis

Elasticity and resiliency

Fibroblasts

Collagen I, elastin

Ligament

Connects bone to bone

Fibroblasts

Collagen I, SLRPs

Tendon

Connects muscle to bone

Fibroblasts

Collagen I, SLRPs, fibrillins

Visceral stroma

Internal scaffold and capsule

Fibroblasts

Collagen I, collagen III, versican, fibronectin

Synovium

Produces synovial fluid

Fibroblasts, macrophages

Collagen I, collagen III

Vessel wall

Barrier, elastic recoil

Endothelium, vascular smooth muscle cells

Collagen III, collagen IV, elastin, fibrillins, fibulins, versican

SLRP = small leucine repeat-rich protein.

CHAPTER 258  Imaging Studies in the Rheumatic Diseases  

GENERAL REFERENCES 1. Haavardsholm EA, Lie E, Lillegraven S. Should modern imaging be part of remission criteria in rheumatoid arthritis? Best Pract Res Clin Rheumatol. 2012;26:767-785. 2. Kang T, Horton L, Emery P, et al. Value of ultrasound in rheumatologic diseases. J Korean Med Sci. 2013;28:497-507. 3. Sofka CM. Tracking rheumatic disease through imaging. Rheum Dis Clin North Am. 2013;39: 633-644. 4. Desai MA, Peterson JJ, Garner HW, et al. Clinical utility of dual energy CT for evaluation of tophaceous gout. Radiographics. 2011;31:1365-1375. 5. Mandl P, Kurucz R, Niedermayer D, et al. Contributions of ultrasound beyond clinical data in assessing inflammatory disease activity in rheumatoid arthritis: current insights and future prospects. Rheumatology (Oxford). 2014;53:2136-2142.

1730.e1

6. Rowbotham EL, Wakefield RJ, Granger AJ. The technique and application of ultrasound in the diagnosis and management of inflammatory arthritis. Semin Musculoskel Radiol. 2012;16:360-366. 7. Palmer AJR, Brown CP, McNally EG, et al. Non-invasive imaging of cartilage in early osteoarthritis. Bone Joint J. 2013;95-B:738-746. 8. McQueen FM. MRI in rheumatoid arthritis: a useful tool for the clinician? Postgrad Med J. 2014;90:332-339. 9. Yamashita H, Takahashi H, Kubota K, et al. Utility of fluorodeoxyglucose positron emission tomography/computed tomography for early diagnosis and evaluation of disease activity of relapsing polychondritis: a case series and literature review. Rheumatology (Oxford). 2014;53: 1482-1490. 10. Gotthardt M, Bleeker-Rovers CP, Boerman OC, et al. Imaging 0f inflammation by PET, conventional scintigraphy and other imaging techniques. J Nucl Med Technol. 2013;41:157-169.

1730.e2

CHAPTER 258  Imaging Studies in the Rheumatic Diseases  

REVIEW QUESTIONS 1. With regard to imaging of gout, the following statement(s) is (are) true: A. The presence of periarticular articular erosions and soft tissue nodules are pathognomonic. B. Osteopenia is a hallmark of the disease. C. Dual-energy computed tomography can be of value to assess disease extent. D. It has a characteristic appearance on ultrasound in these patients. E. C and D are correct. Answer: E  Dual-energy CT is well suited to assess the distribution of tophaceous deposits in patients with gout because of differences of the absorption characteristics of sodium urate crystals compared with other types of deposition diseases. On ultrasonography, calcification along the superficial margin of the cartilage is characteristic of gout, giving rise to a double-line sign. 2. A patient presents with radial-sided wrist pain that extends proximally into the forearm. What imaging test(s) would assess possible etiologies? A. Radiographs alone should be adequate B. MRI C. Ultrasonography D. B and C are correct E. A and B are correct Answer: D  The clinical history suggests DeQuervain’s tenosynovitis. Radiographs would allow assessment of the osseus structures but not the adjacent tendons. Although MRI would provide the most complete examination of the radial-sided structures, ultrasonography is well suited to evaluating the soft tissues, including the first dorsal compartment tendons. One also could perform ultrasound-guided therapy at the time of diagnosis. 3. A patient is suspected of having sacroiliitis. What imaging study would be appropriate to initially evaluate the patient? A. Radiographs of the SI joints B. Computed tomography C. Nuclear scintigraphy (bone scan) D. MRI E. Ultrasonography Answer: A  Dedicated radiographic views of the sacroiliac joints should be the first study ordered to assess for possible erosions and may be sufficient to establish the diagnosis. Computed tomography would provide a more sensitive evaluation for subtle erosions as well as the adjacent soft tissues, particularly with the addition of intravenous contrast. Computed tomography provides an ideal method to perform guided therapy or aspiration. A bone scan would provide a sensitive evaluation of the SI joints but findings would be likewise abnormal for trauma, inflammatory, or degenerative etiologies. A bone scan would allow a global assessment of the axial and appendicular skeleton to determine whether other sites are potentially affected. MRI allows the best assessment of the bone, adjacent marrow space, and soft tissues.

4. A patient has shoulder pain and gives a history of prior dislocation. There is an equivocal abnormality on the humeral head on radiographic evaluation. Which additional study should be considered? A. Ultrasonography to rule out a rotator cuff tear B. Noncontrast computed tomography C. Additional specialized radiographs to evaluate the scapula D. Direct MR arthrography E. Nuclear scintigraphy Answer: D  Although ultrasonography could evaluate the rotator cuff, it does not provide adequate assessment of the capsular labral complex. Crosssectional imaging with intra-articular contrast would best accomplish this. MR arthrography would be optimal. Direct arthrography has been the method of choice in assessing the glenoid labrum, surrounding ligaments, and capsule. CT arthrography is of value, particularly in the postoperative shoulder, and provides indirect imaging of internal structures by coating them with contrast material. Noncontrast CT would be very limited in assessing the labroligamentous complex, even in the presence of a joint effusion caused by poor soft tissue contrast. 5. A patient with early rheumatoid arthritis is being considered for placement on a DMARD. All of the imaging studies below could assess the level of disease activity and response to therapy except A. 18FDG scan. B. radiographs of the hands and wrists. C. gray-scale ultrasonography with power Doppler. D. MRI with gadolinium. E. parametric MR imaging of distribution volumes of contrast in the extravascular space. Answer: B  Radiographic findings in rheumatoid arthritis usually occur when there has already been irreversible joint damage. Ideally, therapy would be instituted on a radiographically negative patient. The remaining examinations can provide sensitive evaluation of disease activity before the development of either bone or cartilage erosion.

CHAPTER 259  Connective Tissue Structure and Function  

tissue disorders affecting the vasculature, bone, or cartilage, with neonatal, juvenile, or early adult mortality resulting. Connective tissues continually sense and adapt to their mechanical environment. The anabolic response of many connective tissues to optimal levels of mechanical stress is now recognized as a crucial, remediable determinant of health. This concept is embodied in Wolff ’s law of bone remodeling, which states that bone remodels in response to the mechanical loads imposed on it. The composition of specific connective tissues reflects adaptations in response to their mechanical environment or requirements for other specialized functions. For example, articular cartilage and other hyaline cartilages comprise proteoglycan aggregates that exert a swelling pressure and a network of collagen II fibrils that restrain them, contributing to compressive strength and shock absorption.4 The superficial zone of articular cartilage, in contrast, in enriched in collagenous fibrils that are arranged parallel to the surface to resist shear forces. A mucinous glycoprotein, lubricin, present on the joint surfaces, ensures the low coefficient of friction of synovial joints. In tendons, the major ECM component is collagen I, which has high tensile strength, but in bone, a composite of collagen I and calcium hydroxyapatite provides tensile and compressive strength. The contractile apparatus of skeletal muscle is connected by the dystrophin–glycoprotein complex to the muscle basement membrane and via hierarchical assemblies of interstitial matrix (endomysium, perimysium, and epimysium) to tendons and ultimately to bone, thus efficiently transmitting muscle contraction forces.5 In the intervertebral disc, the nucleus pulposus is rich in aggrecan, which exerts a swelling pressure that is constrained by the surrounding concentric lamellae of fibrillar collagen in the annulus fibrosus. This composite structure absorbs vertical loads on the spine while limiting deformation of the nucleus pulposus. The interfaces between different connective tissues show structured transitions, highlighted by fibrocartilage present at tendon insertion sites and by Sharpey fibers, which seamlessly integrate perimysial collagen of muscle fibers with collagen in bone. In knee menisci, the cells and ECM of the inner third are cartilaginous, with abundant proteoglycans. Whereas the cells of the outer third, which is vascular and connected to ligaments, are fibroblastic, with corresponding predominance of fibrillar collagens, the cells and ECM in the middle third have an intermediate fibrochondrocyte phenotype.

  MAJOR EXTRACELLULAR MATRIX COMPONENTS

Extracellular matrix comprises collagens; the glycosaminoglycan (GAG) hyaluronan (HA); proteoglycans; a variety of glycoproteins; phosphoproteins (especially in bones and teeth); and matricellular proteins such as thrombospondin, tenascin, and periostin, which regulate cellular functions via ECM but do not have a structural role. The distribution of ECM molecules and macromolecules is tightly regulated to achieve specific microenvironments that control proliferation, differentiation, polarity, and migration of cells and provide niches for postnatal stem cells. Collagens, the most abundant proteins of the body, comprise several distinct molecules, each containing at least one triple-helical domain composed of αchains having a repeating Gly-X-Y sequence. Twenty-eight different collagen types are formed from the products of more than 40 genes encoding collagen α chains. The presence of glycine, the smallest amino acid, at every third position, permits triple-helix formation; the amino acid proline, which is frequently modified to form hydroxyproline at the Y position, ensures stability of the triple helix. Fibrillar collagens (e.g., types I-III, V, XI) have long triple-helical (collagenous) regions, so they form rodlike structures. Whereas collagen I, the most widely distributed and abundant, is a heterotrimer comprising two α1(I) chains and an α2(I) chain, collagen II and III are homotrimers of α1(II) and α1(III) chains, respectively. Mutations of either collagen I chain cause the majority of cases of osteogenesis imperfect (Chapter 260) and infrequently can also cause rare subtypes of Ehlers-Danlos syndrome (EDS) (Chapter 260). The triple helices of nonfibrillar collagens are shorter and interspersed with interruptions of the Gly-X-Y sequence or noncollagenous domains that introduce flexible regions. Collagen XIII, XVII, XXIII, and XXV are transmembrane proteins. Fibrillar collagens are synthesized as procollagens having bulky terminal propeptides. The folding of the triple helix occurs intracellularly and is propagated from the C- to the N-terminus to form homo- or heterotrimeric triple helices. The propeptides are subsequently excised by specific proteinases, resulting in rodlike tropocollagen that can be assembled into tightly packed, quarter-staggered fibrils. Failure to remove the N-propeptide of collagen I impairs fibril assembly and leads to a specific type of EDS with severe skin fragility (type VIIc or dermatosparactic type). Whereas collagen I is the major component of bone, dermis, tendons, ligaments, and the sclera of the eye, collagen III is abundant in the skin, lung, and vasculature, explaining

1731

the association of genes encoding these collagens with osteogenesis imperfecta and vascular EDS (also known as type IV). Collagen II, together with minor amounts of collagen IX and collagen XI, predominates in cartilage, ocular vitreous and the nucleus pulposus of the intervertebral disc, and mutations of either of these collagen genes can cause the Stickler syndrome (triad of cartilage, eye, and hearing anomalies). Collagen fibrils can consist of more than one collagen type; for instance, collagen V is found in heterotypic fibrils with collagen I and is required for nucleation of fibrillogenesis to form large collagen I fibrils. Basement membranes are formed from collagen IV, with significant content of laminins, nidogens, and the heparin sulfate proteoglycan perlecan, which is also an important constituent of cartilage. Basement membranes have crucial roles in regulating molecular transport, such as in glomerular filtration, and in establishment and maintenance of epithelial polarity. Collagen VI is widely distributed as beaded microfibrils that form pericellular matrices in fibroblasts and, alongside collagen VII anchoring fibrils, connect basement membranes to interstitial ECM. Hyaluronan, a polymer with repeating disaccharide units, can achieve a molecular weight in the millions of Daltons. It is a key component of cartilage matrix, pericellular matrix of many cell types, and a mediator of inflammation. Proteoglycans are characterized by covalent attachment of glycosaminoglycans, such as chondroitin, heparan, and keratan sulfate, to a core protein. Aggrecan and versican are large chondroitin sulfate proteoglycans that form giant aggregates with hyaluronan in chondrocytes or nucleus pulposus cells and fibroblasts, respectively. On the other hand, some small leucine repeat-rich proteins (SLRPs) such as decorin, biglycan, and lumican are proteoglycans that may have only one or two glycosaminoglycan chains. This class of molecules, which also includes fibromodulin, interacts with collagen fibrils to cross-link them and regulate fibril diameter. They have been shown to bind transforming growth factor β (TGF-β) and, when released from connective tissue, to provide danger signals to the immune system. During connective tissue healing, fibrin and fibronectin provide a transitional ECM permissive for cell migration, differentiation, and other aspects of the repair process. Elastic fibers are formed by coacervation of a soluble precursor named tropoelastin. Elastic fiber assembly is guided by tissue microfibrils that are formed from three large glycoproteins named fibrillins.6 Fibrillin-1 is abundant in the aorta, ocular zonule, and perichondrium of bone and is mutated in Marfan syndrome (Chapter 260), resulting in aneurysms, ectopia lentis, and skeletal overgrowth. Fibrillin-2 mutations cause Beals syndrome, with skeletal overgrowth and limb contractures as major features but typically, not severe cardiac or eye problems. The elastic fiber-microfibril network additionally contains versican, microfibril-associated glycoproteins, latent TGF-β–binding proteins, and fibulins. Cutis laxa, which primarily affects the skin but can also involve vasculature and internal organs, can be caused by mutations affecting elastin and fibulin-5. Most ECM molecules undergo one or more posttranslational modifications. These include enzymatic formation of intra- and interchain disulfide bonds, phosphorylation, and several kinds of glycosylation, including addition of N- and O-linked sugars or glycosaminoglycans such as chondroitin, keratan, or heparan sulfates. Whereas N-linked sugars on some proteins are essential for protein folding and secretion, glycosaminoglycan (GAG) chains provide crucial biophysical properties (such as of aggrecan) and mediate intermolecular interactions. During its biosynthesis, collagen undergoes lysyl and prolyl hydroxylation. Some lysyl and hydroxylysl residues are modified extracellularly by lysyl oxidase and form stable cross-links between adjacent collagen molecules, which strengthens bone, tendons, and skin.7 Gene defects affecting collagen-modifying enzymes or subunits of the molecular complexes they operate in lead to various recessive forms of osteogenesis imperfecta or EDS. l-Ascorbic acid is a cofactor for lysyl hydroxylase and prolyl hydroxylase and stimulates procollagen synthesis. Its nutritional deficiency leads to scurvy by reducing collagen synthesis, triple-helical stability, and tropocollagen cross-linking. Lathyrism (a now rarely seen neurotoxic disease) is caused by excess ingestion of β-aminopropionitrile, which inhibits lysyl oxidase–mediated formation of lysine aldehydes, which are the precursors of the major collagen and elastin cross-links, and leads to connective tissue fragility. Copper is a required cofactor for lysyl oxidase, and its deficiency can also lead to lathyrism.

CELL-MATRIX INTERACTIONS IN CONNECTIVE TISSUE REGULATION

Cellular ECM receptors mediate cell-matrix interactions that ensure force transmission from cells to ECM8; the dynamic reciprocity between cells and

1732

CHAPTER 259  Connective Tissue Structure and Function  

ECM is crucial for environmental sensing and adaptive responses. Receptors provide feedback to the cells regarding the quality, content, or mechanical properties of the matrix (outside-in signaling) to generate appropriate responses, including cell proliferation or migration, or lead to altered ECM synthesis or degradation. Alternatively, cytoplasmic signals may alter cellmatrix interactions (inside-out signaling). Integrins are a large group of heterodimeric receptors with distinct binding preferences that comprise α and β subunits having short cytoplasmic domains. Integrins are crucial for fibronectin assembly, which in turn substantially influences assembly of collagen, fibrillins, and TGF-β activation. Intracellular signals activate integrins to promote high-affinity binding to ECM proteins, and binding of ECM proteins via specific integrin-binding motifs such as Arg-Gly-Asp initiates clustering of cell adhesion complexes and intracellular signaling that, among diverse effects, may elicit the production of inflammatory cytokines. Integrin αvβ6 transmits cellular traction to matrix-bound latent TGF-β, exposing the active growth factor. Because osteoclast attachment to and spreading on bone surfaces is dependent on αvβ3, it is being targeted for treatment of osteoporosis (Chapter 243) using chemical antagonists or blocking antibodies. Syndecans are transmembrane heparan sulfate proteoglycans that work as coreceptors alongside high-affinity ECM receptors such as integrins. Discoidin domain receptors are receptor tyrosine kinases activated by binding to native triple-helical collagen.9 DDR1 and DDR2 bind to fibrillar collagens I-III and collagen V. Collagen IV activates DDR1 but not DDR2, whereas collagen X activates DDR2. With their extended structure, extensive posttranslational modification, and exposure of numerous binding sites, ECM molecules such as collagens, fibrillins, and the classical cell-binding protein fibronectin bind to multiple receptor types. Among several hyaluronanbinding molecules on the cell surface, including hyaluronan synthases, CD44 is a major hyaluronan receptor with a role in assembling the pericellular matrix and in inflammation.

EXTRACELLULAR MATRIX NETWORKS AND CONNECTIVE TISSUE DISORDERS

Extracellular matrix molecules and macromolecules are assembled to form higher order (supramolecular) complexes and networks whose varied composition and geometry further diversify connective tissues. These are exemplified by parallel arrays of extensively cross-linked collagen fibrils conferring high tensile strength to tendons, ligaments, and bone, by the multidirectional or “basket-weave” arrangement of collagen in dermis, both permitting and constraining multiaxial mobility, and by the concentric elastic lamellae and crimped collagen in the aorta that regulate hemodynamics. In the eye, orthogonally oriented collagen fibrils in corneal stroma and the lattice-like structure of collagen VIII in the Descemet membrane allow transparency and are crucial for normal vision. Cartilage and nucleus pulposus ECM comprises large aggregates of hyaluronan and aggrecan, with hundreds of aggrecan molecules attached to each hyaluronan polymer via their N-terminal domain, leaving the C-terminal domain free for interactions with fibrillins, fibulins, and other ECM networks. Each aggrecan molecule has approximately 100 chondroitin sulfate chains and several keratan sulfate chains. Their high fixed-charge density creates an osmotic environment that favors water retention and restricts water flux in cartilage. Through the attachment of hyaluronan to cell-surface receptors, the aggrecan–hyaluronan network is retained by the chondrocyte as a conspicuous pericellular matrix (or glycocalyx). In connective tissue cells other than chondrocytes, pericellular matrices use versican or heparin sulfate proteoglycans instead of aggrecan. The pericellular matrix occupies the crucial interface between cells and their environment and influences cell behavior. It is a provisional ECM in microcosm because it is metabolically more active than further removed matrix, which is more stable. Some interstitial collagen and elastin can be stable for decades. Connective tissue disorders arise from inborn defects affecting cells or ECM, as well as cellular dysfunction caused by repetitive injury, inflammation, or metabolic or aging processes. Because ECM is such a crucial component of connective tissues, most inherited and acquired connective tissue disorders result from mutant ECM molecules, insufficient but essentially normal matrix (e.g., osteoporosis), or excess or inappropriately deposited matrix (e.g., adhesions, fibrosis, and scleroderma). Osteoarthritis (OA) involves all structures that form the joint, including the synovium and subchondral bone, and appears to result from a combination of genetic and nongenetic factors (Chapter 262). OA of the hip and hands has a stronger genetic component than OA of the knee. Because the function of cartilage is to absorb impact and distribute it to bone without aberrant loading of joint

structures, variations in genes that affect joint congruity and alignment or cartilage or meniscal integrity could predispose to OA. For instance, variations in genes encoding cartilage ECM components mutated in chondrodysplasias (Chapter 205), such as collagen II, IX, cartilage oligomeric protein, or matrilins, have been associated with early-onset OA, suggesting a genetic link and a spectrum extending from OA predisposition at the milder end to severe chondrodysplasia. Another genetic link is with the mechanisms that influence the formation of joints, correlating with the association between joint malalignment and OA. Variations in genes associated with the TGF-β superfamily pathway, including GDF5 (encoding a morphogen required for joint development), ASPN (encoding an ECM protein that binds TGFβ), and SMAD3 (encoding a cytoplasm-nucleus signaling intermediary), are associated with OA.10 Factors that affect load distribution across joints, such as ligament and meniscal tears or hip dysplasia, predispose to OA. Cartilage breakdown in arthritis is not caused by wear and tear but by altered cellmatrix interactions that lead to enzymatic digestion. Loss of aggrecan, followed by loss of SLRPs and other molecules from the surface of collagen fibrils, are initial changes that render collagen fibrils susceptible to destruction by collagenases. The loss of articular cartilage collagen II is thought to constitute an irreversible change in joint disease. Whatever the original insult, the response of chondrocytes or synovium can lead to a vicious cycle of joint destruction because the cells may respond by producing excess ECM and ECM-degrading proteases, with release of both intact and fragmented molecules that may further potentiate inflammation. Rodent immune arthritis models have revealed a role for the alternative complement pathway in arthritis, and cartilage proteins and their fragments released by proteolytic breakdown can have complex effects on the complement pathways that lead to both activation and suppression. Cell-matrix interactions are profoundly affected in muscular dystrophies (Chapter 421), revealing a continuum that is essential for force transmission from the cytoskeleton to interstitial matrix. Mutations affecting components of the dystrophin–glycoprotein complex including dystroglycan, a receptor that connects muscle cytoskeleton to laminin in the muscle basement membrane, muscle basement membrane (laminin and collagen IV), and collagen VI filaments in ECM, cause a variety of muscular dystrophies. Specific adhesion complexes such as hemidesmosomes in the epidermis mediate anchorage to basement membranes and underlying interstitial ECM. Autoantibodies against collagen XVII and VII, or laminin 5 mutations, affect major components of these complexes and lead to blistering skin diseases (Chapter 439).

PROTEASES AND CONNECTIVE TISSUE TURNOVER

Reflecting continuous adaptation to their environment, connective tissues have intrinsic mechanisms that ensure ECM renewal (i.e., through coupled synthesis and degradation). ECM molecules are substrates for several proteinase classes, chiefly matrix metalloproteinases (MMPs), astacin MMPs such as BMP1 and tolloids, cathepsins, and A disintegrin-like and metalloproteinase domain with thrombospondin type 1 repeats (ADAMTS).11 Although these proteases are typically considered to be catabolic, some are also essential for the maturation of precursor proteins, such as ADAMTS2, which excises the amino-propeptide of procollagen I, II, and III and is defective in EDS, dermatosparactic type. BMP1, which excises the C-propeptides of procollagen I, II, and III and cleaves lysyl oxidase and several other proteins, is deficient in a type of recessive osteogenesis imperfecta. Although most MMPs are secreted, a class of membrane-type MMPs is cell-surface bound. Most MMPs require proteolytic activation by other MMPs or serine proteinases such as plasmin and furin, and are inhibited by tissue inhibitors of MMPs (TIMPs) and α2-macroglobulin. Whereas MT1-MMP is crucial for collagen I proteolysis in bone, MMP-13 has been implicated as a major collagen II degrading enzyme in arthritis. Cell surface MMPs such as MT1-MMP and A disintegrin-like and MMPs (ADAM) are responsible for ectodomain shedding of ECM receptors, cell-surface cytokines and cytokine receptors, together regulating a variety of inflammatory and oncogenic situations. ADAMTS4 and ADAMTS5, also known as aggrecanase-1 and -2, respectively, are principal aggrecan-degrading proteases implicated in OA and, together with MMP-13, are potential drug targets in this disorder. However, MMPs, ADAMs, and ADAMTSs have structurally similar catalytic domains and zinc and calcium-dependent proteolytic mechanisms, which renders selective inhibition of any single proteinase from these classes challenging. Indeed, a major side effect of MMP inhibitors used in clinical trials for cancer was connective tissue stiffness and inflammation, presumably resulting from reduced physiological turnover of collagen and other

CHAPTER 260  Inherited Diseases of Connective Tissue  

ECM proteins. Among other proteinases, cathepsin K produced by osteoclasts is active at acidic pH, unlike MMPs, and therefore efficiently digests bone collagen in the acidic osteoclast–bone interface. Neutrophil elastase (a serine proteinase), MMP-9, and MMP-12 (metalloelastase) can degrade elastin. Proteolysis of ECM proteins can release bioactive fragments (matrikines) such as endostatin (from collagen XVIII) or endorepellin (from perlecan), which are antiangiogenic; some fibronectin fragments released in OA are proinflammatory. Hydroxyproline antibodies that recognize ECM-fragments with specific cleaved ends (neoepitope antibodies), as well as cross-linked collagen fragments released by proteolytic activity, are useful biomarkers of bone and cartilage turnover.

  AGING OF CONNECTIVE TISSUE

Some visible hallmarks of aging result not only from cellular senescence but also reduced ECM synthesis and increased catabolism, as well as greater connective tissue fragility. This leads, for example, to reduced bone mass or osteoporosis; thinning and loss of elastic properties of dermal ECM, which are visible as wrinkles and sagging skin; reduced volume of intervertebral discs; and increased capillary fragility. Loss of HA and GAGs reduces tissue hydration and of collagen and elastin reduces tensile strength and elasticity, respectively. Products of ECM proteolysis stimulate cells to release free radicals and cytokines that further accelerate ECM breakdown or lead to cell death. Extrinsic factors such as sunburn and ultraviolet irradiation induce cytokines that accelerate this process. In addition, aging collagen can be cross-linked by the Maillard reaction, especially when glucose levels are high, resulting in its modification by advanced glycation end products (AGE), which renders it inflexible, alters the rate of turnover, and affects binding to matrix receptors.12 AGE binding to a cellular receptor has been shown to lead to cellular dysfunction, oxidative stress, and inflammation. In summary, connective tissue provides the framework of the musculoskeletal system and a structural scaffold for internal organs while serving as a reservoir for growth factors, cytokines, and stem-like precursor cells. Inherited connective tissue disorders have the potential to disturb these functions in specific ways. Acquired disorders such as OA and fibrosis reflect perturbation of diverse physiological networks and pathways, with altered cell-matrix interactions at the center. GENERAL REFERENCES For the General References and other additional features, please visit Expert Consult at https://expertconsult.inkling.com.

260  INHERITED DISEASES OF CONNECTIVE TISSUE REED E. PYERITZ

  MUCOPOLYSACCHARIDOSES DEFINITION

Proteoglycans are ubiquitous components of the extracellular matrix (ECM) and the surfaces of cells, and they are among the largest and most complex of human molecules. Proteoglycans consist of a protein core to which are covalently bound glycosaminoglycans (GAGs; formerly called mucopolysaccharides) of several types: dermatan sulfate, heparan sulfate, keratan sulfate, and chondroitin sulfate. These four polymeric molecules are cleaved from their protein core in lysosomes; then they, plus hyaluronan (a GAG lacking a protein core), are catabolized further in lysosomes in a stepwise fashion by more than a dozen enzymes. Genetic defects in any one of these enzymes lead to the accumulation of GAG metabolites in lysosomes, with profound disruption of cellular physiology. The phenotypes resulting from deficiencies of these catabolic enzymes are termed mucopolysaccharidoses (MPSs) and are classified into seven types (Table 260-1). Several additional storage disorders, termed mucolipidoses (MLs), are caused by a genetic

1733

defect in posttranslational modification of lysosomal enzymes and share features with the MPSs.

EPIDEMIOLOGY

All MPS disorders are rare, each with an incidence of one or fewer cases per 100,000 births, and are without ethnic predilection.

PATHOBIOLOGY

With the exception of MPS II (Hunter syndrome), which is X-linked, each of these disorders is autosomal recessive. All MPSs are caused by deficiency of a single lysosomal enzyme responsible for a specific step in GAG metabolism. Catabolism of GAG proceeds normally until the step requiring the defective enzyme, when further normal metabolism halts. Although a minor degree of nonspecific breakdown occurs, resulting in urinary excretion of cleaved GAG that can be useful diagnostically, the accumulation of GAG within lysosomes of cells of mesenchymal origin; endothelium; and, in most cases, neurons causes widespread, progressive cellular dysfunction and clinical effects. Lysosomal enzymes are targeted to lysosomes by posttranslational addition of mannose 6-phosphate. Deficiency of the phosphotransferase that catalyzes the first step in this reaction results in an inability to catabolize any GAG molecules. The catabolic enzymes, which normally would be transported into lysosomes, instead are secreted from the cell and are found in unusually high concentrations in plasma, providing one diagnostic test for MLs.1

Pathology

All pathologic manifestations of MPS and ML disorders worsen with age, and some are present from early developmental stages. Gross anatomic hallmarks are hepatosplenomegaly, marked skeletal alterations (termed dysostosis multiplex)2 that result in short stature and thoracic cage deformity, thickening and narrowing of airways and arteries, and coarsening of facial features. Although mental retardation is a prominent feature of some of these conditions, the brain may show only ventriculomegaly secondary to communicating hydrocephalus. On microscopy, mesenchymal cells show a cytoplasm full of apparently empty vacuoles; these are lysosomes from which GAG has been removed by fixation. Cells cultured from patients show greatly enlarged lysosomes filled with granular material. In the severe form of ML, dense inclusions are present, which gave rise to the common name, I-cell disease.

CLINICAL MANIFESTATIONS

Each of the disorders in Table 260-1 shows a wide spectrum of clinical severity. This wide spectrum has led to a classification that gives the impression of separate disorders within some of the MPS and ML types, but these represent the apparent ends of the continuum. Some of the disorders result in death by adolescence (Hurler syndrome, severe Hunter syndrome, ML II), but others are commonly compatible with survival to adulthood. The latter group of disorders is emphasized here. The milder end of the MPS I spectrum, Scheie syndrome, may not be diagnosed until adulthood; patients present with stiffened joints, corneal clouding and glaucoma, carpal tunnel syndrome, and aortic valvular disease. Stature and intelligence are not affected. The main health risks are valvular involvement, thickening of meninges that can produce a myelopathy, and thickening of the upper airways that can produce obstructive symptoms and sleep apnea. The milder form of MPS II, Hunter syndrome, is distinctive because it is X-linked (affecting males almost exclusively), and the cornea shows little overt clouding. Cervical myelopathy, obstructive airway disease, and cor pulmonale are important concerns. A combined conductive and neurosensory hearing loss is common. Neither MPS IV (Morquio syndrome) nor MPS VI (Maroteaux-Lamy syndrome) affects intelligence. Both syndromes often are associated with severe skeletal changes, which are distinct radiographically but produce similar problems of kyphoscoliosis, pectus carinatum, restrictive lung disease, severe short stature, and joint degeneration. Cervical myelopathy resulting from a thickened dura is common to both disorders and is accentuated by odontoid hypoplasia in MPS IV. Thickening of the aortic and mitral valves may produce severe dysfunction necessitating their replacement. General anesthesia is especially hazardous because of the narrow upper and middle airways and cervical instability. Patients with ML III (pseudo-Hurler polydystrophy) resemble patients with MPS VI but often have mild to moderate mental retardation. Aortic regurgitation is common.

CHAPTER 259  Connective Tissue Structure and Function  

GENERAL REFERENCES 1. Frey H, Schroeder N, Manon-Jensen T, et al. Biological interplay between proteoglycans and their innate immune receptors in inflammation. FEBS J. 2013;280:2165-2179. 2. Fosang AJ, Beier F. Emerging frontiers in cartilage and chondrocyte biology. Best Pract Res Clin Rheumatol. 2011;25:751-766. 3. Nandadasa S, Foulcer S, Apte SS. The multiple complex roles of versican and its proteolytic turnover by ADAMTS proteases during embryogenesis. Matrix Biol. 2014;35:34-41. 4. Heinegard D, Saxne T. The role of the cartilage matrix in osteoarthritis. Nat Rev Rheumatol. 2011;7:50-56. 5. Carmignac V, Durbeej M. Cell-matrix interactions in muscle disease. J Pathol. 2012;226: 200-218. 6. Baldwin AK, Simpson A, Steer R, et al. Elastic fibres in health and disease. Expert Rev Mol Med. 2013;15:e8.

1733.e1

7. Byers PH, Pyott SM. Recessively inherited forms of osteogenesis imperfecta. Annu Rev Genet. 2012;46:475-497. 8. Ross TD, Coon BG, Yun S, et al. Integrins in mechanotransduction. Curr Opin Cell Biol. 2013; 25:613-618. 9. Fu HL, Valiathan RR, Arkwright R, et al. Discoidin domain receptors: unique receptor tyrosine kinases in collagen-mediated signaling. The J Biol Chem. 2013;288:7430-7437. 10. Sandell LJ. Etiology of osteoarthritis: genetics and synovial joint development. Nat Rev Rheumatol. 2012;8:77-89. 11. Gargiulo S, Gamba P, Poli G, et al. Metalloproteinases and metalloproteinase inhibitors in agerelated diseases. Curr Pharm Des. 2014;20:2993-3018. 12. Semba RD, Nicklett EJ, Ferrucci L. Does accumulation of advanced glycation end products contribute to the aging phenotype? J Gerontol A Biol Sci Med Sci. 2010;65:963-975.

1734

CHAPTER 260  Inherited Diseases of Connective Tissue  

TABLE 260-1  MUCOPOLYSACCHARIDOSES AND MUCOLIPIDOSES TYPE

EPONYM OR COMMON NAME

INHERITANCE

OMIM*

MPS IH

Hurler syndrome

DM and short stature; MR; corneal clouding; HS; heart disease; death in childhood

AR

252800

α-l-iduronidase

MPS IS

Scheie syndrome

Coarse facies; stiff joints, corneal clouding; aortic valve disease; normal intelligence and lifespan

AR

252800

α-l-iduronidase

MPS II

Hunter syndrome

Severe form: coarse facies, DM and short stature, HS; MR; no corneal clouding; death by late adolescence Mild form: coarse facies, short stature; normal intelligence; survival to adulthood

XL

309900

Iduronate sulfatase

CLINICAL FEATURES

ENZYMATIC DEFECT

MPS IIIA

Sanfilippo A

Severe MR and hyperactivity; mild somatic changes

AR

252900

Heparan N-sulfatase

MPS IIIB

Sanfilippo B

Same as MPS IIIA

AR

252920

α-N-acetylglucosaminidase

MPS IIIC

Sanfilippo C

Same as MPS IIIA

AR

252930

Acetyl-coenzyme A: α-glucosaminide acetyltransferase

MPS IIID

Sanfilippo D

Same as MPS IIIA

AR

252940

N-acetylglucosamine 6-sulfatase

MPS IVA

Morquio A

Short stature and distinct skeletal dysplasia with odontoid hypoplasia and myelopathy; corneal clouding; normal intelligence; valvular heart disease

AR

253000

Galactose 6-sulfatase

MPS IVB

Morquio B

Same as MPS IVA

AR

253010

β-Galactosidase

MPS VI

Maroteaux-Lamy

DM and short stature; corneal clouding; normal intelligence; aortic stenosis; leukocyte inclusions; hydrocephalus in severe form

AR

253200

N-acetylgalactosamine

MPS VII

Sly syndrome

DM; HS; widely variable, including MR

AR

253220

β-Glucuronidase

MPS IX



Short stature; periarticular soft tissue masses

AR

601492

Hyaluronidase

ML II

I-cell disease

Similar to but more severe than MPS IH but with cellular inclusions; no mucopolysacchariduria

AR

252500

UDP-N-acetylglucosamine: lysosomal enzyme N-acetylglucosaminyl-1- phosphotransferase

ML III

Pseudo-Hurler polydystrophy

Short stature and mild DM; stiff joints, mild MR; survival to adulthood

AR

252500

Same as ML II arthropathy, coarse facies; variable but milder

*Entries in Online Mendelian Inheritance in Man, OMIM. McKusick-Nathans Institute of Genetic Medicine. Baltimore: Johns Hopkins University. http://omim.org. AR = autosomal recessive; DM = dysostosis multiplex; HS = hepatosplenomegaly; MR= mental retardation; XL = X-linked.

DIAGNOSIS

Differential Diagnosis

Diagnosis of these conditions is difficult in young children, before most of the clinical features have progressed, but should be considered in any person with hepatosplenomegaly and coarsening of the facial features. Evaluation requires a pedigree analysis, ophthalmologic examination, skeletal radiographic survey, echocardiography, and analysis of the urine for excretion of GAGs. Often the specific MPS is evident from radiographs, the presence or absence of corneal clouding, and the pattern of mucopolysacchariduria. Enzymatic analysis of leukocytes confirms the diagnosis. Patients with MLs do not show mucopolysacchariduria but have marked elevation of all the GAG catabolic lysosomal enzymes in plasma.

TREATMENT  Ventriculoperitoneal shunting is necessary if intracranial pressure is elevated. Close attention to hearing and visual problems is essential throughout life. Many adults with MPS or ML require surgery for carpal tunnel syndrome. Cardiovascular surgery for valvular or coronary disease may be necessary. All use of anesthesia is high risk because of the narrow airways and, in the case of MPS IV, atlantoaxial instability. For patients who remain ambulatory, selective joint replacement can be beneficial. Because of the morbidity associated with thoracic cage deformity, consideration should be given to stabilizing the spinal deformity before it becomes severe. Replacement of the deficient enzyme via intravenous infusion is being studied for most of the MPS disorders. Laronidase (Aldurazyme) has been approved in the United States for treatment of MPS I. An infusion every 2 weeks for 1 year in adolescent and adult patients resulted in substantial reduction in hepatosplenomegaly and modest improvement in pulmonary function, sleep apnea, and joint mobility. Whether early institution of therapy in young children modulates mental retardation in the Hurler variant of MPS I is uncertain. Galsulfase (Naglazyme) has been approved for the treatment of MPS VI, in which somatic rather than neurologic problems predominate. Bone marrow transplantation has been attempted in many of the MPS disorders, with mixed success. The earlier transplantation occurs, the better the outcome

in terms of somatic problems, but prevention of mental retardation has not occurred. Current recommendations based on consensus in Europe calls for hematopoietic stem cell transplantation for patients with Hurler syndrome before the age of 2.5 years. Enzyme replacement should be started in all patients when diagnosed.3

  MARFAN SYNDROME DEFINITION

Marfan syndrome is an autosomal dominant, pleiotropic disorder caused by defects in the principal component of the extracellular microfibril, the large glycoprotein fibrillin-1. The disease manifestations occur in multiple systems, especially the eye, skeleton, heart, aorta, lung, and integument. Notable features include dislocation of the ocular lens, tall stature with particularly long limbs and digits, deformity of the thoracic cage from pectus carinatum or excavatum with abnormal curvature of the spine, mitral and tricuspid valve prolapse, dilation of the sinuses of Valsalva and predisposition to aortic dissection, spontaneous pneumothorax, abnormal skin stretch marks, hernias, and dural ectasia. If untreated, patients often die before 30 or 40 years of age from aortic dissection or congestive heart failure.

EPIDEMIOLOGY

Marfan syndrome is a common Mendelian disorder, with an estimated incidence of about one per 5000 births. Marfan syndrome is found throughout the world, without ethnic or geographic predilection.

PATHOBIOLOGY

Pathogenesis

Mutations in FBN1, which maps to human chromosome 15q21.1 and encodes fibrillin-1, cause Marfan syndrome and related connective tissue disorders. More than 1000 distinct mutations have been found, and few occur in more than one family. Patients are heterozygous for mutations in FBN1, leading to autosomal dominant inheritance. Extracellular microfibrils are

CHAPTER 260  Inherited Diseases of Connective Tissue  

polymers of many fibrillin-1 molecules and are ubiquitous in the ECM of most tissues. Latent transforming growth factor β (TGF-β) binding protein, which keeps the cytokine inactive, bears striking homology to regions of fibrillin. Abnormalities of either the quality or the quantity of microfibrils disrupt normal signaling by TGF-β, especially during embryonic development and postnatal growth. Studies in mice engineered to harbor human mutations in FBN1 showed that excessive TGF-β signaling causes abnormal lung septation (the precursor to pneumothorax), mitral valve prolapse, muscular hypoplasia, and aortic dilatation. This fundamental shift in understanding of the pathogenesis of Marfan syndrome has suggested novel therapies, such as with small molecules that affect the activity of TGF-β or its downstream signaling. The features of Marfan syndrome are highly variable, even among relatives who share the same mutation in FBN1. This variability persists after accounting for the effects of age. Men tend to be affected more severely, for unclear reasons.

Pathology

The features of Marfan syndrome are age dependent. Some severely affected infants have flagrant features and often die of mitral regurgitation and heart failure despite aggressive management. At the other end of the clinical spectrum, Marfan syndrome merges with several related disorders, and patients may not come to medical attention, let alone receive a definitive diagnosis, until adulthood. None of the gross or microscopic pathologic changes is specific for Marfan syndrome. The medial degeneration of the aortic wall, characterized by disarray and fragmentation of the elastic fibers and increased proteoglycan (often inappropriately termed cystic medial necrosis) also can be seen in other disorders and in older people with hypertension. Aortic dissection (Chapter 78) usually begins just superior to the aortic valve (type A) and often progresses to the bifurcation. Death usually results from retrograde dissection and hemopericardium. About 10% of dissections begin in the descending thoracic aorta (type B).

CLINICAL MANIFESTATIONS

The lens tends to be displaced superiorly, and usually the zonules remain intact. The retina is at increased risk of detachment, especially in patients who are highly myopic. Tubular bones overgrow, accounting for the disproportionate tall stature (dolichostenomelia), long digits (arachnodactyly), and sternal deformity. Ligaments may be lax, causing scoliosis and joint hypermobility. Alternatively, congenital contractures are common, especially of the elbows. The palate typically is highly arched, and the dentition can be crowded and maloccluded. Mitral valve prolapse occurs in about 80% of cases, and the valve leaflets become progressively thickened (myxomatous on histopathology) (Chapter 75). The mitral annulus may dilate and calcify. Aortic root dilation begins in the sinuses of Valsalva and progresses with age, albeit at highly variable rates (Chapter 78).4 Most males with Marfan syndrome have an aortic root dimension above the upper limit of normal for their body surface area by adolescence. Some females show a slower progression and may have a root diameter near the upper limit of normal well into adulthood. The dilation usually does not involve the distal ascending aorta. Spontaneous pneumothorax, resulting from rupture of apical blebs, occurs in about 5% of patients. Stretch marks (striae atrophicae) occur over areas of flexural stress, such as the shoulders, breasts, and lower back. The neural canal in the lumbosacral region is enlarged in most people with Marfan syndrome; this may be visible on plain radiographs, especially if the neuroforamina are widened. Imaging by computed tomography or magnetic resonance imaging is diagnostic and should be used in patients with back pain and radicular symptoms. Dural ectasia progresses with age; large anterior meningoceles in the pelvis are a severe manifestation.5 Simple cysts in the liver and kidneys are common, increase with age, and seldom cause clinical problems. Sleep apnea is of increased frequency in adults.6

DIAGNOSIS

Differential Diagnosis

The conditions that overlap clinically and genetically with Marfan syndrome include familial aortic aneurysm, familial ectopia lentis, mitral valve prolapse, mild aortic dilation, striae, skeleton (MASS) phenotype (which includes many families with mitral valve prolapse syndrome), and Loeys-Dietz syndrome. Most of these conditions are diagnosed clinically, so differentiating among them is arbitrary. A careful family history is essential to this process. Molecular genetic testing has a limited role. However, if the mutation in

1735

FBN1 is known in a family, analysis of DNA can be used effectively for presymptomatic or prenatal diagnosis. Loeys-Dietz syndrome, which is associated with generalized arterial tortuosity and susceptibility to dissection, is caused by mutation in either of two receptors for TGF-β, TGFBR1 and TGFBR2, and molecular analysis is clinically available. A question of Marfan syndrome arises most commonly in tall, lanky adolescents who have several minor skeletal features, nearsightedness, and athletic desires. A detailed ophthalmologic examination with full pupillary dilation and a transthoracic echocardiogram are essential components in the evaluation. If these test results are negative and no one in the family has a history of Marfan syndrome or aortic dissection, the patient probably can be reassured.7

TREATMENT  Life expectancy for those with Marfan syndrome has improved markedly, to the point that many patients can expect survival to advanced years. All patients should be seen at least annually by a physician who manages the overall care. Most patients require annual ophthalmologic and cardiologic consultation and orthopedic consultation as required by specific problems. Lens subluxation often requires surgical correction.8 A number of studies, but only one randomized clinical trial, support the prophylactic use of β-adrenergic blockade from an early age to slow the rate of aortic root dilation and protect against aortic dissection. Based on studies of the Marfan mouse, therapies that interfere with excess signaling through pathways mediated by TGF-β are being studied in human clinical trials. One large European trial suggested a benefit of the angiotensin receptor blocker losartan on aortic root dilatation rate, A1  but a large international trial found no benefit of losartan compared with atenolol. A2  Prophylactic surgical repair of the aortic root has had the greatest beneficial impact. The composite graft, involving a prosthetic valve in a Dacron tube and implantation of the coronary ostia into the graft, was the first approach to produce markedly improved survival in these patients. More recently, replacement of the aneurysm and preservation of the native aortic valve have shown promise and should be considered first.9,10 For adults, aortic root surgery should be strongly considered when the maximal aortic diameter reaches 45 mm, and a family history of aortic dissection should prompt earlier repair (Chapter 78).

  EHLERS-DANLOS SYNDROMES DEFINITION

The Ehlers-Danlos syndromes (EDSs) are clinically variable and genetically heterogeneous. Diagnoses still are based largely on the bedside examination. The unifying themes among these disorders are fragility of tissues, joint hypermobility, and skin hyperextensibility.11

EPIDEMIOLOGY

No accurate data exist, but an incidence of about one in 5000 births is a reasonable estimate of how many individuals qualify for one of the EDS diagnoses. Each type represents something of a clinical spectrum, with the mild end merging with what might be considered normal variation. Just as the diagnostic criteria are arbitrary, so would be any determination of prevalence based on phenotypic criteria. The extent to which normal variation in joint hypermobility, skin elasticity, and tissue fragility represents genetic variation at loci that encode collagen or other ECM genes requires considerable research.

PATHOBIOLOGY

Pathogenesis

Defects in collagen and other proteins in the ECM of various tissues underlie all forms of EDS that have been elucidated so far. The specific mutations occur in a variety of genes, with the effect of altering the structure, synthesis, posttranslational modifications, or stability of the collagens involved. The known molecular defects are listed in Table 260-2.

Pathology

Few findings in the routine pathologic evaluation distinguish among the various types of EDS or even distinguish individual types from normal. Thickness of the dermis is decreased in some forms, especially the vascular type, and the walls of arteries are reduced in thickness in this type. By electron microscopy, the classic, hypermobile, and kyphoscoliotic types have abnormal collagen fibers, especially when viewed in cross section (variable and often increased fiber diameter with an irregular outline). In the vascular type,

1736

CHAPTER 260  Inherited Diseases of Connective Tissue  

TABLE 260-2  EHLERS-DANLOS SYNDROMES TYPE

FORMER NAME

CLINICAL FEATURES*

INHERITANCE

OMIM†

MOLECULAR DEFECT

Classic

EDS I and II

Joint hypermobility; skin hyperextensibility; atrophic scars; smooth, velvety skin; subcutaneous spheroids

AD

130000 130010

Structure of type V collagen caused by mutations in COL5A1 or COL5A2

Hypermobility

EDS III

Joint hypermobility; some skin hyperextensibility, with or without a smooth, velvety texture

AD AR

130020 225320

? Tenascin-X (TNX)

Vascular

EDS IV

Thin skin; easy bruising; pinched nose; acrogeria; rupture of large- and medium-caliber arteries, uterus, and large bowel

AD

130050 (225350) (225360)

Deficient type III collagen (COL3A1)

Kyphoscoliotic

EDS VI

Joint hypermobility; congenital, progressive rupture; scoliosis; scleral fragility with globe rupture; tissue fragility, aortic dilation, MVP

AR

225400

Deficiency of lysyl hydroxylase

Arthrochalasis

EDS VII A

Joint hypermobility, severe, with subluxations, congenital hip dislocation; and skin hyperextensibility; tissue fragility

AD

130060

No cleavage of amino terminus of type I procollagen caused by mutations in COL1A1 or COL1A2

Dermatosparaxis

EDS VII C

Severe skin fragility; decreased skin elasticity, easy bruising; hernias; premature rupture of fetal membranes

AR

225410

No cleavage of amino terminus of type I procollagen caused by deficiency of peptidase

Unclassified types

EDS V EDS VIII EDS X EDS XI EDS IX EDS, progeroid form

Classic features Classic features and periodontal disease Mild classic features, MVP Joint instability Classic features; occipital horns Classic features and premature aging

XL AD ? AD XL AR

305200 130080 225310 147900 309400 130700

? ? ? ? Allelic to Menkes syndrome Deficiency of galactosyltransferase I

*Listed in order of diagnostic importance. † Entries in Online Mendelian Inheritance in Man, OMIM. McKusick-Nathans Institute of Genetic Medicine. Baltimore: Johns Hopkins University. http://omim.org. AD = autosomal dominant; AR = autosomal recessive; EDS = Ehlers-Danlos syndrome; MVP = mitral valve prolapse; XL = X-linked.

some patients have dilated endoplasmic reticulum consistent with aberrant secretion of type III collagen molecules.

CLINICAL MANIFESTATIONS

The major and minor features of each EDS are detailed in Table 260-2. Infants with classic EDS often are born prematurely by 4 to 8 weeks because of rupture of fetal membranes. Diagnosis of the vascular and kyphoscoliotic types is important because of their cardiovascular features. The vascular type, previously termed EDS IV, is characterized by spontaneous rupture of large arteries and hollow organs, especially the colon and uterus, and pneumothorax. Because these events carry considerable morbidity, life expectancy is reduced, on average, by more than half. During pregnancy, women with this form of EDS are especially vulnerable to rupture of major arteries and the uterus. In the kyphoscoliotic type, aortic root dilation and aortic regurgitation can develop. Patients with most forms of EDS are prone to develop mitral valve prolapse, and progression to mitral regurgitation (Chapter 75) occurs more often than in the common form of mitral valve prolapse.

DIAGNOSIS

Differential Diagnosis

By carefully adherence to the clinical features shown in Table 260-2 and judicious use of laboratory tests, the various defined types of EDS can be differentiated. Many specific non-EDS syndromes need to be excluded. The kyphoscoliotic type of EDS in infants shares some features with severe Marfan syndrome. Patients with Larsen syndrome may resemble patients with the arthrochalasis type of EDS. The skin redundancy and loss of elasticity of the dermatosparaxis type of EDS is reminiscent of autosomal dominant cutis laxa, which is not associated with easy bruising or tissue fragility. The most difficult decision is whether any diagnosis of EDS is warranted. Patients who have only joint hypermobility without skin changes should not be labeled with EDS; a diagnosis of familial joint hypermobility might be more appropriate. Familial joint instability involves a predisposition to dislocations of major joints that is rare in most types of EDS except for arthrochalasis.

TREATMENT  Management of most skin and joint problems should be conservative and preventive. Sutures need to be placed with careful attention to approximating the margins and avoiding tension; removable sutures should be left in place

for twice the usual time. Most instances of joint hypermobility and pain in EDS do not require surgical treatment. Benefit often is derived from physical therapy designed to strengthen the muscles that provide support for the loose ligaments. All patients should receive genetic counseling about the mode of inheritance and their risk of having children affected with EDS. The possibility of prenatal diagnosis exists for all of the EDS types with defined molecular or biochemical defects. The vascular type of EDS requires particular surgical care; the ruptured arteries are difficult to repair because of the pronounced vascular fragility. Experienced vascular surgeons are having some success with prophylactic repair of vessels deemed to be at risk of dissection or rupture.12 One clinical trial suggested improved outcomes with prophylactic β-adrenergic blockade. A3  Rupture of the bowel is a surgical emergency. Because the risk of uterine and vascular rupture is especially high during pregnancy in women with the vascular form, affected women should be advised that there is a substantial risk of death related to pregnancy and delivery. Patients should be advised to avoid contact sports and to treat blood pressure elevations aggressively. Arteriography and arterial lines should be avoided if possible. Biochemical and genetic screening holds the potential for reassuring relatives at risk that they do not have a defect in type III collagen. The kyphoscoliotic type of EDS may improve with large doses of vitamin C (1-4 g/day) because ascorbate is a cofactor for the enzyme that is deficient. No other metabolic or genetic therapy is effective in other forms of EDS.

  OSTEOGENESIS IMPERFECTA SYNDROMES DEFINITION

The heterogeneous group of disorders called osteogenesis imperfecta (OI) includes, at one end of the severity spectrum, a type that is lethal prenatally or in the neonatal period and, at the other, such mild features that distinguishing affected individuals from the general population is difficult. The unifying feature is hereditary osteopenia (insufficient bone), with primary defects in the protein matrix in bone and other tissues. The clinical syndromes all involve osteoporosis with liability to fracture (Chapter 243).

EPIDEMIOLOGY

No careful epidemiologic study has been performed, and the milder forms of type I OI merge with the phenotypes of familial osteoporosis, fracture

CHAPTER 260  Inherited Diseases of Connective Tissue  

1737

TABLE 260-3  OSTEOGENESIS IMPERFECTA TYPE

INHERITANCE

OMIM*

I

Fractures variable in number; little deformity; stature normal or nearly so; blue sclerae; hearing loss common but not always present; DI uncommon

CLINICAL FEATURES

AD

166200

Typically, one nonfunctional COL1A1 allele

BASIC DEFECTS

II

Lethal in utero or shortly after birth; many fractures at birth typically involving ribs (may appear “beaded”) and other long bones; little calvaria; pulmonary hypertension

AD

166210

AR

259400

COL1A1 or COL1A2: substitution of glycyl residues; occasionally deletions of a portion of the triple-helical domain Deletion in COL1A2 plus a nonfunctional allele

III

Fractures common, but long bones progressively deform starting in utero; stature markedly reduced; sclerae often blue but become lighter with age; DI and hearing loss common

AD AR (rare)

259420 259440

One single amino acid substitution Two mutations in COL1A1 and/or COL1A2 (rarely)

IV

Fractures common; stature usually reduced; bone deformity common but rarely severe; scleral hue normal to grayish; hearing loss variable; DI common

AD

166220 166240

Point mutations in COL1A1 or COL1A2 Exon skipping mutations in COL1A2

V

Similar to type IV without DI or blue sclerae; fractures develop hyperplastic callus; calcification of the interosseous membrane between the radius and ulna

AD

610967

?

VI

Similar to type IV without DI, blue sclerae or Wormian bones; excess osteoid present in bone

?

610968

?

VII

Similar to types II or III with fractures at birth, blue sclerae, no DI; presence of rhizomelic limb shortening and coxa vara

AR

610682

Mutations in CRTAP

VIII

Similar to types II or III with fractures at birth

AR

610915

Mutations in LEPRE1

IX

Similar to types II or III with fractures at birth

AR

259440

Mutations in PPIB

*Entries in Online Mendelian Inheritance in Man, OMIM. McKusick-Nathans Institute of Genetic Medicine. Baltimore: Johns Hopkins University. http://omim.org. AD = autosomal dominant; AR = autosomal recessive; DI = dentinogenesis imperfecta.

susceptibility, and joint hypermobility found in the general population. A crude estimate of the overall prevalence of OI is one to two per 20,000 births. The neonatal lethal form (type II), which is almost always caused by a new mutation in a parental gamete, has an incidence of about one in 50,000 births.

PATHOBIOLOGY

Pathogenesis

Most patients in whom mutations have been found usually have defects in the two genes that encode the procollagen chains of type I collagen, COL1A1 and COL1A2. Type I collagen is composed of two α1(I) and one α2(I) procollagen chains; the mature fiber requires considerable posttranslational modification, which occurs appropriately only if the three procollagen chains have intertwined to form a triple helix that is perfect and completed at the right speed. A mutation that affects formation of the triple helix, such as substitution of one of the mandatory glycine residues that occurs at every third position, also has adverse effects on the modifications that render the molecule capable of forming effective mature fibers. As a result, a single nucleotide change resulting in a missense mutation can have profound effects on the ECM and produce a severe condition.13 Alternatively, and at first glance paradoxically, a mutation that eliminates an entire allele, or at least production of any product capable of intertwining with normal procollagen chains, has a much milder effect on the ECM and on the severity of OI. Examples of the most common classes of mutations are shown in Table 260-3. Hundreds of mutations have been described. Patients with mutations in COL1A1 or COL1A2 are heterozygous, and thus the most common forms of OI are inherited as autosomal dominant traits. Several autosomal recessive forms of OI occur because of mutations in genes that encode enzymes that process type I collagen into mature fibrils.14

Pathology

Other than the gross pathology associated with the clinical manifestations, the most characteristic pathology is a primary reduction in bone matrix with secondary undermineralization.

CLINICAL MANIFESTATIONS

The major phenotypic features of OI are shown in Table 260-3. Among the most common forms, the most severe type is type II, followed in decreasing order by types III, IV, and I. In type II, infants either are stillborn or die soon after birth of pulmonary failure secondary to the small thorax, which usually is compromised further by myriad rib fractures. A few infants have

survived for at least a few years but require enormous attention to their medical needs. Type III OI may be confused with type II at birth, but survival alone helps make the distinction. Bony deformity is pronounced and not necessarily caused by fractures. Mobility is impaired, and most patients require a wheelchair at an early age. Stature may be severely compromised. Because of progressive vertebral column deformity and rib fractures, restrictive lung disease is a common problem as patients age; many die of pulmonary complications. Basilar impression causing compression of the brain stem and the craniocervical junction can produce central sleep apnea, headache, and upper motor neuron signs. Patients with type IV OI generally have reduced stature, some bony deformity, and abnormal teeth that are opalescent and wear easily (dentinogenesis imperfecta). As in type I OI, the tendency to fracture is highest in childhood and lessens with adolescence. A distinguishing characteristic of type IV OI is a normal scleral hue. Type I OI is probably the most common form and is associated with a bluish or blue-gray scleral hue. People with type I OI who also have dentinogenesis imperfecta tend to have more severe skeletal problems. The risk of fracture diminishes during adulthood but reemerges as a major concern for women after menopause. Hearing impairment in all forms of OI is common and age related, being rare before adolescence. The deficits are of either a mixed or a predominantly conductive form. The recessive forms of OI (types VI-IX) range in severity from type IV to type II and may have distinctive radiologic or histopathologic findings.

DIAGNOSIS

Differential Diagnosis

The range of diagnostic possibilities in a person with multiple fractures largely depends on age. In infancy, the genetic conditions hypophosphatasia, severe osteochondrodysplasias (e.g., achondrogenesis and forms of spondyloepiphyseal dysplasia), and Menkes syndrome need to be excluded when a diagnosis of type II or type III OI is considered. The radiographic features eventually become entirely diagnostic, but often the neonatologist has to arrive at a definitive answer in short order. Analysis of serum alkaline phosphatase and copper can be helpful. In childhood, the most common situation leading to consideration of a mild form of OI is child abuse. In this situation, the pattern of fracture is usually distinct, and bone mineralization should be normal if the child is the object of nonaccidental or repeated accidental trauma. Abnormal scleral hue, dentinogenesis imperfecta, and wormian bones (microfractures along the cranial sutures) all support the diagnosis of

1738

CHAPTER 260  Inherited Diseases of Connective Tissue  

OI. The legal and child-protective systems often request exclusion of OI by analysis of collagen production from cultured skin fibroblasts or analysis of DNA for a mutation. In older children, the disorder idiopathic juvenile osteoporosis should be considered in any patient seen initially with repeated fractures. Many osteochondrodysplasias are associated with short stature, skeletal deformity, and a tendency to fracture. Pyknodysostosis and osteopetrosis are associated with sclerotic bones rather than osteoporotic ones. In adulthood, early-onset osteoporosis may be confused with OI (Chapter 243). Mutations in type I collagen also cause familial osteoporosis, and the skeletal phenotypes merge; patients with true OI may have scleral, hearing, or dental abnormalities and a positive family history. Analysis of the specific enzymes defective in the recessive forms of OI is useful for establishing the diagnosis and enabling reproductive counseling and prenatal diagnosis if desired.

TREATMENT  Management of the skeletal complications largely depends on orthopedic, physical, and occupational therapy approaches. Risedronate (2.5 or 5 mg daily) increases bone mineral density and reduces both first and recurrent fractures in children with OI. A4  The long-term goals are for the patient to maintain function and independence as an individual. These goals can be advanced in some patients by judicious use of intramedullary rods in the long bones of the legs; if mobility and especially ambulation can be maintained, the demineralization associated with inactivity can be avoided. Unaffected parents of a child with OI and all affected individuals should have genetic counseling. For the parents of a child with type II OI, the possibility of germinal mosaicism (which has been well documented in this condition) should not be overlooked. If one parent has a “new” mutation in one of the type I procollagen genes and multiple gonadal cells carry this mutation,  the risk of recurrence in future children is not negligible. If the mutation in the affected child can be defined, the risk of recurrence can be quantified (through molecular analysis of sperm) if the mutation arose in the father.

  PSEUDOXANTHOMA ELASTICUM DEFINITION

Pseudoxanthoma elasticum (PXE) is a heritable disorder of connective tissue with pleiotropic manifestations wherever elastic fibers are found but primarily in the skin, eye, and vasculature.15 Life expectancy is reduced, on average, because of a predisposition to myocardial infarction and gastrointestinal hemorrhage.

EPIDEMIOLOGY

The exact frequency of PXE is unknown, but it is probably underdiagnosed. Rough approximations suggest a prevalence of one in 25,000 to 100,000 births. Males and females are equally affected, although women are more likely to seek medical attention out of concern for the skin changes.

PATHOBIOLOGY

Pathogenesis

In most families, PXE occurs as an autosomal recessive trait, which means, given relatively small sibships, that many patients will have no affected relatives. Apparent autosomal dominant inheritance may reflect expression in occasional heterozygotes. The gene for PXE maps to human chromosome 16 and encodes one of the adenosine triphosphate (ATP)-binding cassette transporters (ABCC6). Because of the prominent histopathologic feature of calcification of elastic tissue, this gene may be important in calcium homeostasis. It is unclear, however, whether calcification is a primary or a secondary phenomenon in PXE.

Pathology

The hallmark of PXE, and an important diagnostic clue, is the histopathologic finding of hyperproliferated elastic fibers in the mid-dermis; these fibers become fragmented, clumped, and calcified. An arteriolar sclerosis develops in the media of muscular arteries and arterioles; the lumen may become progressively and concentrically narrowed. Alternatively, microaneurysms can form. Thickening of the endocardium, especially in the atria, develops in some patients. In the eye, Bruch membrane becomes calcified and fragmented.

CLINICAL MANIFESTATIONS

Because of the pleiotropic nature of PXE, the diagnosis initially may be suspected by any of a variety of clinicians, especially dermatologists, ophthalmologists, cardiologists, and gastroenterologists. The condition gains its name from the dermatologic feature of yellowish papules that appear at areas of flexural stress, especially the neck, groin, and popliteal and cubital fossae; in periumbilical regions; and on the buccal mucosa. The appearance of affected skin has been likened to that of a “plucked chicken.” Over time, affected areas coalesce and become thickened. Changes in the eye begin as a generalized, subtle, mottled pattern in the retina (peau d’orange) and progress to the characteristic angioid streaks. The latter changes are not specific for PXE and can be seen in diabetes mellitus, sickle cell disease, and a variety of other conditions. Streaks represent breaks in Bruch’s membrane, an elastic lamina that lies between the retinal vasculature and the choroid. Spontaneous hemorrhages, especially those involving the macula, lead to progressive visual loss.16 Involvement of arteries of various calibers produces problems because of occlusion and hemorrhage.17 The lifetime risk of serious gastrointestinal hemorrhage from any site, but especially the stomach, is about 10%. Hypertension is relatively common, in part because of involvement of the renal vasculature. Progressive occlusion of peripheral arteries leads to absence of pulses; acral ischemia is rare because of the development of collaterals. The risk for stroke, myocardial infarction, abdominal angina, and intermittent claudication is increased independent of other risk factors. Impaired left ventricular function is common in adults.

DIAGNOSIS

Differential Diagnosis

Whole exome sequencing is an efficient and sensitive way to make the diagnosis.18 An acquired form of PXE has been reported and is also of unclear etiology. This form is difficult to differentiate from a sporadic case in a family because of heterozygosity in the parents, but it tends to affect only the skin. As suggested by the name, the cutaneous features of PXE need to be differentiated from those of true xanthoma, which results from a disorder of lipid metabolism (Chapter 206). The dermatologic manifestations need to be differentiated from those of Miescher elastoma, elastic tissue nevi (BuschkeOllendorff syndrome), and solar elastosis.

TREATMENT  No cure for or means of preventing PXE is known. In many instances, careful attention to the ocular features by a retinal specialist experienced in PXE can delay but not prevent loss of vision. The risk of gastrointestinal hemorrhage suggests that patients should avoid gastric irritants such as aspirin, nonsteroidal anti-inflammatory drugs, and excessive alcohol. Stool should be checked regularly for occult blood, and angiography may be necessary to detect the source of bleeding. All standard risk factors for atherosclerosis should be managed aggressively. Complaints of chest pain should prompt a rigorous investigation for coronary artery disease. Angioplasty has not been reported to be effective, and the coronary lesions tend to be diffuse. Coronary artery bypass graft surgery has been performed, but long-term results have not been reported. It may be theoretically advantageous to use vein grafts rather than the internal mammary artery for bypass. The excessive wrinkling and pseudoxanthoma in exposed areas can be ameliorated by plastic surgery.

FUTURE DIRECTIONS

Each of these disorders poses special considerations in clinical diagnosis, utility of molecular testing, genetic counseling, and management. For the storage disorders, the clinical utility of enzyme replacement therapy is actively being pursued by several pharmaceutical companies. For several of the other conditions, somatic stem cell therapy offers some promise but is years away from routine clinical use. In Marfan syndrome, clinical trials of drugs that modulate activity of TGF-β are underway. Additionally, close medical management for individuals detected as being at heightened risk for cardiovascular, skeletal, and ocular complications will remain a mainstay.

Grade A References A1. Groenink M, den Hartog AW, Franken R, et al. Losartan reduces aortic dilatation rate in adults with Marfan syndrome: a randomized controlled trial. Eur Heart J. 2013;34:3491-3500.

CHAPTER 261  The Systemic Autoinflammatory Diseases  

A2. Lacro RV, Dietz HC, Sleeper LA, et al. Atenolol versus losartan in children and young adults with Marfan’s syndrome. N Engl J Med. 2014;371:2061-2071. A3. Ong KT, Perdu J, De Backer J, et al. Effect of celiprolol on prevention of cardiovascular events in vascular Ehlers-Danlos syndrome: a prospective, randomized, open, blinded-endpoints trial. Lancet. 2010;376:1476-1484. A4. Bishop N, Adami S, Ahmed SF, et al. Risedronate in children with osteogenesis imperfecta: a randomised, double-blind, placebo-controlled trial. Lancet. 2013;382:1424-1432.

GENERAL REFERENCES For the General References and other additional features, please visit Expert Consult at https://expertconsult.inkling.com.

261  THE SYSTEMIC AUTOINFLAMMATORY DISEASES RICHARD M. SIEGEL AND DANIEL L. KASTNER

DEFINITION

The systemic autoinflammatory diseases (Table 261-1) are a group of illnesses characterized by seemingly unprovoked inflammation, without evidence of high-titer autoantibodies or antigen-specific T cells, thus distinguishing them from the more classic autoimmune diseases. The first conditions recognized as autoinflammatory were the hereditary recurrent fevers, a group of mendelian disorders characterized by episodic or fluctuating degrees of fever and localized inflammation. The scope of autoinflammatory disease has been broadened to include other heritable illnesses, including disorders in which purulent or granulomatous inflammation predominates, as well as inherited disorders of the complement system (Chapter 50).1-6 In addition, in numerous autoinflammatory conditions, some of which manifest in childhood and others that occur later in life, there is a complex interaction of genetic susceptibilities and environmental factors. These illnesses include systemic-onset juvenile idiopathic arthritis (Still’s disease), Behçet’s disease, and even the crystalline arthritides. Recent advances in the genetics and pathophysiology of the inherited autoinflammatory diseases suggest that these conditions are inborn errors of innate immunity, the phylogenetically more primitive part of the immune system that uses germline membrane and intracellular receptors expressed in granulocytes and macrophages to mount the body’s first line of defense against pathogens (Chapters 45 and 48).

  HEREDITARY RECURRENT FEVER SYNDROMES

Familial Mediterranean Fever

DEFINITION

Familial Mediterranean fever (FMF) is a recessively inherited illness that typically manifests with 12- to 72-hour episodes of fever and localized serosal, synovial, or cutaneous inflammation. Between attacks, patients usually feel completely well, although biochemical evidence of inflammation may remain, and some patients eventually develop systemic amyloidosis. Before the identification of the causative gene, FMF was defined purely clinically; clinical features remain an important part of the diagnosis, because some patients with typical disease have only one, or sometimes no, demonstrable mutation in MEFV, the only known causative gene.

EPIDEMIOLOGY

FMF is most common in individuals of Jewish, Arab, Armenian, Turkish, and Italian ancestry. The frequency of asymptomatic carriers of a single MEFV mutation in these populations is as high as 1 in 5, a finding that suggests a selective advantage for heterozygotes. With genetic testing, FMF is now frequently recognized in both Ashkenazi (eastern European) and non-Ashkenazi Jewish populations, as well as in Mediterranean populations previously thought not to be at risk. Mutation-positive individuals with typical symptoms have been documented worldwide. FMF usually manifests in childhood, sometimes even in infancy, although approximately 10% of patients

1739

experience their first attack as adults; infrequently, FMF first occurs in persons older than 40 years.

PATHOBIOLOGY

MEFV, the gene for FMF, was identified by positional cloning in 1997. It encodes a 781–amino acid protein denoted pyrin (or marenostrin) that is expressed in granulocytes, monocytes, and dendritic cells, as well as in peritoneal, synovial, and dermal fibroblasts. The N-terminal 92 amino acids of pyrin are the prototype for a motif, the PYRIN domain, that is involved in protein-protein interactions; this domain defines a family of more than 20 human proteins, including pyrin itself, involved in the regulation of cytokine production (particularly the interleukin-1 [IL-1] family), nuclear factor kappa B (NF-κB) activation, and apoptosis. More than 50 FMF-associated mutations in pyrin have been identified, many of which reside in the C-terminal domain encoded by exon 10 of MEFV. An even larger number of variants of unknown significance have been described in individual patients with a spectrum of inflammatory phenotypes.

CLINICAL MANIFESTATIONS

Episodes of FMF are more properly termed recurrent than periodic, and some patients associate attacks with psychological stress or physical exertion. Women of childbearing age sometimes experience their attacks with menses, with remissions during pregnancy. Some patients are unaware of fever during the attacks, but it is almost always observed when sought. Serosal involvement in FMF is usually peritoneal or pleural. Abdominal attacks are the most frequent, and they may vary from mild discomfort to frank peritonitis, with boardlike rigidity, direct and rebound tenderness, and air-fluid levels on upright films of the abdomen. Regardless of the severity of the abdominal attack, constipation is much more common than diarrhea. When a laparotomy or laparoscopy is performed during an attack, a small amount of sterile exudate rich in polymorphonuclear leukocytes is found. Except for serosal inflammation, the appendix is normal. Repeated abdominal attacks may cause peritoneal adhesions, but ascites is rare. Pleurisy, usually unilateral, may accompany abdominal pain, or it may occur independently. Physical findings, if present, may include diminished breath sounds and a pleural friction rub, whereas x-ray films may show a small effusion or atelectasis. With multiple attacks, pleural thickening may develop. Symptomatic nonuremic pericardial involvement in FMF has been reported but is unusual. In adults, the arthritis of FMF typically manifests as monoarticular involvement of the knee, hip, or ankle, and attacks of arthritis may persist for up to 1 week at a time. In children, oligoarticular or polyarticular joint involvement may occur. Large joint effusions are sometimes present, and the synovial fluid may have as many as 100,000 leukocytes/mm3. In approximately 5% of patients who are not treated with prophylactic colchicine, chronic arthritis (usually of the hip or knee) may develop, often necessitating joint replacement surgery. Regardless of colchicine treatment or a particular human leukocyte antigen (HLA-B27) status, some patients with FMF develop sacroiliitis. Arthralgia without frank arthritis is common in FMF. Cutaneous manifestations of FMF tend to be less common than serosal or synovial involvement. The characteristic skin lesion of FMF is erysipeloid erythema, a painful, demarcated erythematous area most often seen on the lower leg, ankle, or dorsum of the foot. This rash may occur independently, or it may accompany an episode of arthritis. Histologically, a mixed perivascular cellular infiltrate is seen. Other acute manifestations of FMF include unilateral scrotal inflammation (the tunica vaginalis is an embryologic remnant of the peritoneal membrane) and myalgia, either with fever or, especially in children, without fever and induced by vigorous exercise. Various forms of vasculitis also have been associated with FMF; Henoch-Schönlein purpura may occur in children with FMF; less frequently, polyarteritis nodosa is seen.

COMPLICATIONS

Before the widespread use of colchicine prophylaxis, systemic AA amyloidosis (Chapter 188) was a frequent complication of FMF, caused by the ectopic deposition of a misfolded fragment of serum amyloid A (SAA), an acute phase reactant, in the gastrointestinal tract, kidneys, spleen, lung, testes, and adrenals. Malabsorption and nephrotic proteinuria leading to renal failure are the most common manifestations of AA amyloidosis. Cardiomyopathy is less common, and neuropathy and arthropathy are rare. Several risk factors for amyloidosis development in FMF have been identified, including late diagnosis of FMF, colchicine noncompliance, male gender, and specific

CHAPTER 260  Inherited Diseases of Connective Tissue  

GENERAL REFERENCES 1. Campos D, Monaga M. Mucopolysaccharidosis type I: current knowledge on it pathophysiological mechanisms. Metab Brain Dis. 2012;27:121-129. 2. Stevenson DA, Steiner RD. Skeletal abnormalities in lysosomal storage diseases. Pediatr Endocrinol Rev. 2013;10(suppl 2):406-416. 3. De Ru MH, Boelens JJ, Das AM, et al. Enzyme replacement therapy and/or hematopoietic stem cell transplantation at diagnosis in patients with mucopolysaccharidosis type I: results of a European consensus procedure. Orphanet J Rare Dis. 2011;6:55. 4. Jondeau G, Detaint D, Tubach F, et al. Aortic event rate in the Marfan population. Circulation. 2012;125:226-232. 5. Sheikhzadeh S, Sondermann C, Rybczynski M, et al. Comprehensive analysis of dural ectasia in 150 patients with a causative FBN1 mutation. Clin Genet. 2014;86:238-245. 6. Rybczynski M, Koschyk D, Karmeier A, et al. Frequency of sleep apnea in adults with the Marfan syndrome. Am J Cardiol. 2010;105:1836-1841. 7. Pyeritz RE. Evaluation of the tall adolescent with some features of Marfan syndrome. Genet Med. 2012;14:171-177. 8. Miraldi Utz V, Coussa RG, Traboulsi EI. Surgical management of lens subluxation in Marfan syndrome. J AAPOS. 2014;18:140-146. 9. Song HK, Preiss LR, Maslen CL, et al. Valve-sparing aortic root replacement in patients with Marfan syndrome enrolled in the National Registry of Genetically Triggered Thoracic Aortic Aneurysms and Cardiovascular Conditions. J Heart Valve Dis. 2014;23:292-298.

1739.e1

10. Svensson LG, Blackstone EH, Alsalihi M, et al. Midterm results of David reimplantation in patients with connective tissue disorders. Ann Thorac Surg. 2013;95:555-562. 11. De Paepe A, Malfait F. The Ehlers-Danlos syndrome, a disorder with many faces. Clin Genet. 2012;82:1-11. 12. Lum YW, Brooke BS, Black JH III. Contemporary management of vascular Ehlers-Danlos syndrome. Curr Opin Cardiol. 2011;26:494-501. 13. Li Q, Jiang Q, Uitto J. Ectopic mineralization disorders of the extracellular matrix of connective tissue: molecular genetics and pathomechanisms of aberrant calcification. Matrix Biol. 2014;33: 23-28. 14. Byers PH, Pyott SM. Recessively inherited forms of osteogenesis imperfect. Annu Rev Genet. 2012;46:475-497. 15. Uitto J, Jiang Q, Varadi A, et al. Pseudoxanthoma elasticum: diagnostic features, classification, and treatment options. Expert Opin Orphan Drugs. 2014;2:567-577. 16. Ebran JM, Milea D, Trelohan A, et al. New insights into the visual prognosis of pseudoxanthoma elasticum. Br J Ophthalmol. 2014;98:142-143. 17. Campens L, Vanakker OM, Trachet B, et al. Characterization of cardiovascular involvement in pseudoxanthoma elasticum families. Arterioscler Thromb Vasc Biol. 2013;33:2646-2652. 18. Hosen MJ, Van Nieuwerburgh F, Steyaert W, et al. Efficiency of exome sequencing for the molecular diagnosis of pseudoxanthoma elasticum. J Invest Dermatol. 2014;[Epub ahead of print].

1739.e2

CHAPTER 260  Inherited Diseases of Connective Tissue  

REVIEW QUESTIONS 1. Osteogenesis imperfect syndromes demonstrate A. intergenic heterogeneity. B. intragenic heterogeneity. C. variable expression. D. pleiotropy. E. all of the above. Answer: E  All four of the choices are correct. Mutations in multiple genes can cause OI (intergenic heterogeneity). With a single locus, many different mutant alleles have been discovered (intragenic heterogeneity). Within a given type of OI, relatives with the same mutation can demonstrate different features of varying severity (variable expression). Multiple organ systems are affected in OI (pleiotropy). 2. Which of the following is not a common feature of Marfan syndrome? A. Pulmonary arteriovenous malformation B. Aortic root dilatation C. Mitral valve prolapse D. Pulmonic artery dilatation E. Aortic dissection Answer: A  All four of the latter choices are common features. Arteriovenous malformations are common in hereditary hemorrhagic telangiectasia but not in Marfan syndrome. 3. Hypertension is a relatively common feature in which syndrome? A. Osteogenesis imperfect B. Hurler syndrome C. Marfan syndrome D. Vascular Ehlers-Danlos syndrome E. Pseudoxanthoma elasticum Answer: E  Because of partial occlusion of the renal arteries, elevated blood pressure can occur in PXE. None of the other syndromes has a predisposition to hypertension.

4. Which of the following is not a currently accepted approach to management of Marfan syndrome? A. Prophylactic surgery of the aortic root B. Exercise restriction C. Annual echocardiography D. Gene therapy E. Chronic β-adrenergic blockade Answer: D  At the present time, no approach to correcting the specific mutation in FBN1 is feasible. All of the other approaches are thought to be beneficial. 5. If treatment is not offered, which of the following conditions is associated with the best prognosis? A. Ehlers-Danlos syndrome, hypermobility type B. Osteogenesis imperfect, type II C. Marfan syndrome D. Hurler syndrome E. Ehlers-Danlos syndrome, vascular type Answer: A  The hypermobility form of Ehlers-Danlos syndrome has little to no added mortality. Osteogenesis imperfect type II is usually lethal in infancy. Marfan syndrome has reduced life expectancy because of aortic dissection. Patients with Hurler syndrome rarely survive to their third decade. Patients with the vascular form of Ehlers-Danlos syndrome are at high risk of death from arterial or bowel rupture.

1740

CHAPTER 261  The Systemic Autoinflammatory Diseases  

TABLE 261-1  SYSTEMIC AUTOINFLAMMATORY DISEASES: A PARTIAL LISTING INHERITED AUTOINFLAMMATORY DISEASES

INHERITANCE/ETIOLOGY

GENES OR RISK FACTORS

OMIM*

HEREDITARY RECURRENT FEVER SYNDROMES Familial Mediterranean fever (FMF) Tumor necrosis factor receptor–associated periodic syndrome (TRAPS) Hyperimmunoglobulinemia D with periodic fever syndrome (HIDS) Familial cold autoinflammatory syndrome (FCAS) Muckle-Wells syndrome (MWS) Neonatal-onset multisystem inflammatory disease (NOMID)/chronic infantile neurologic cutaneous and articular (CINCA) syndrome

Autosomal recessive Autosomal dominant

MEFV† TNFRSF1A†

249100 142680

Autosomal recessive

MVK†

260920

Autosomal dominant Autosomal dominant Sporadic, autosomal dominant

NLRP3 (formerly CIAS1)† NLRP3 (formerly CIAS1)† NLRP3 (formerly CIAS1)†

120100 191900 607115

Autosomal dominant

NOD2/CARD15†

186580



GRANULOMATOUS DISORDERS Granulomatous inflammatory arthritis, dermatitis, and uveitis (Blau’s syndrome) Early-onset sarcoidosis Crohn’s disease

Sporadic, autosomal dominant Complex inheritance

NOD2/CARD15 NOD2/CARD15†

609464 266600

Autosomal dominant

PSTPIP1†

604416

Deficiency of interleukin-1 receptor antagonist (DIRA) Chronic recurrent multifocal osteomyelitis (CRMO)

Autosomal recessive Sporadic, autosomal recessive

612852 259680

Synovitis acne pustulosis hyperostosis osteitis syndrome (SAPHO)

Idiopathic

IL1RN LPIN2,† when associated with congenital dyserythropoietic anemia (Majeed syndrome) —

Autosomal dominant Autosomal dominant, sporadic Complex inheritance

C1NH HF1 (complement factor H) HF1 (complement factor H)

106100 235400 603075

Syndrome of periodic fever with aphthous stomatitis, pharyngitis, and cervical adenopathy (PFAPA) Autoinflammation, lipodystrophy, and dermatosis syndrome (Nakajo-Nishimura syndrome, JMP syndrome, CANDLE syndrome) Systemic-onset juvenile idiopathic arthritis (SOJIA) Adult-onset Still’s disease Schnitzler’s syndrome Behçet’s disease

Idiopathic



Autosomal recessive

PSMB8

256040

Complex inheritance Idiopathic Idiopathic Complex inheritance

604302 — — 109650

Crystalline arthropathies

Complex inheritance

IL-6, MIF polymorphisms — — HLA-B51, polymorphisms in IL10, IL23R, CCR1, STAT4, KLRC4, ERAP1, MEFV, TLR4 SLC2A9/GLUT9, ABCG2

PYOGENIC DISORDERS Syndrome of pyogenic arthritis with pyoderma gangrenosum and acne (PAPA) AUTOINFLAMMATORY DISORDERS OF SKIN AND BONE



COMPLEMENT DISORDERS Hereditary angioedema Hemolytic-uremic syndrome Age-related macular degeneration OTHER AUTOINFLAMMATORY SYNDROMES —



*Online Mendelian Inheritance in Man, an online catalogue of genetic disorders, available at http://www.ncbi.nlm.nih.gov/entrez/query.fcgi?db=OMIM. Accessed September 29, 2014. † An updated list of disease-associated mutations is available online at http://fmf.igh.cnrs.fr/infevers. Accessed September 29, 2014.

genotypes of the MEFV and SAA genes. Amyloidosis in FMF is less common in the United States than in the Middle East. Abdominal fat aspirates are much less sensitive than rectal or renal biopsy in detecting the amyloidosis of FMF. The latter procedure may be preferred, because of the increasing recognition of nonamyloid glomerular disease in FMF. With early diagnosis, aggressive suppression of the acute phase response with colchicine or adjunctive agents may lead to improvement, but for patients with renal failure, early renal transplantation is preferred.

DIAGNOSIS

Based on a simple recessive model of inheritance, two mutations in MEFV, in trans, should be identified to establish the genetic diagnosis of FMF. Nevertheless, the interpretation of genetic testing is complicated by complex alleles consisting of various combinations of mutations in cis, as well as by the observations that as many as one third of patients with clinically typical FMF have only one demonstrable mutation in MEFV, and a few patients with typical disease have no identifiable MEFV mutations. These latter two findings suggest that, under some circumstances, one MEFV mutation may be sufficient for symptoms or that additional genes for FMF exist. For these reasons, clinical data remain an essential part of the diagnosis of FMF, and genetic testing plays an adjunctive role in settings in which clinical experience is limited.7 Clinical criteria emphasize attack duration (12 to 72 hours); recurrence of symptoms (three or more episodes); documented fever (rectal temperature > 38° C); painful manifestations in the abdomen, chest,

joints, or skin; and the absence of other causative factors. The differential diagnosis includes the other hereditary recurrent fever syndromes (Table 261-2), as well as other conditions specific to the clinical setting. For patients with recurrent abdominal pain, considerations include gynecologic disorders, porphyria (Chapter 210; which can be distinguished by hypertension during attacks, dominant inheritance, and urine porphyrins), and hereditary angioedema (Chapter 252; which usually does not cause fever). The syndrome of periodic fever with aphthous stomatitis, pharyngitis, and cervical adenopathy is probably the most common cause of unexplained recurrent fever in children and is also included in the differential diagnosis. In patients presenting primarily with recurrent monoarthritis, joint aspiration for cultures and crystals may aid in excluding bacterial and crystalline arthritis, respectively. Still’s disease in children (systemic-onset juvenile idiopathic arthritis) and adults (adult-onset Still’s disease) is also considered in the differential diagnosis. Adult-onset Still’s disease8 (see Table 261-1) is an uncommon autoinflammatory condition of unknown cause that is not considered to be hereditary. It is characterized by spiking fever, an evanescent salmon-pink maculopapular rash, arthritis, and neutrophilic leukocytosis. It can be clinically distinguished from FMF by the pattern of fever (intermittent quotidian in Still’s disease vs. discrete episodes in FMF), the pattern of arthritis (chronic polyarthritis vs. intermittent monoarthritis), the characteristic skin involvement (evanescent rash vs. erysipeloid erythema), and the presence of lymphadenopathy (more common in Still’s disease).

1741

CHAPTER 261  The Systemic Autoinflammatory Diseases  

TABLE 261-2  CLINICAL FEATURES OF SELECTED HEREDITARY RECURRENT FEVER SYNDROMES CLINICAL FEATURE Typical ethnicity

FMF Arab, Armenian, Italian, Jewish, Turkish

TRAPS Any ethnicity

HIDS

FCAS

Dutch, other North European

European

MWS European

NOMID/CINCA Any

Attack duration

12-72 hr

Days to weeks

3-7 days

12-24 hr

1-2 days

Continuous, with flares

Abdominal attacks

Sterile peritonitis, constipation more often than diarrhea

Severe pain, vomiting, peritonitis

Sterile peritonitis, diarrhea, rarely constipation

Nausea

Abdominal pain

Not common

Pleural attacks

Common

Common

Rare

Not seen

Rare

Rare

Joint/bone involvement

Monoarthritis, rarely protracted arthritis in knee or hip

Arthritis in large joints, arthralgia

Arthralgia, symmetrical polyarthritis

Polyarthralgia

Polyarthralgia, oligoarthritis, clubbing

Epiphyseal overgrowth, contractures, intermittent or chronic arthritis, clubbing

Skin rash

Erysipeloid erythema on lower leg, ankle, foot

Migratory rash, underlying myalgia

Diffuse maculopapular rash, urticaria

Cold-induced urticaria-like rash

Urticaria-like rash

Urticaria-like rash

Lymphatic involvement

Splenomegaly, occasional lymphadenopathy

Splenomegaly, occasional lymphadenopathy

Cervical adenopathy in children

Not seen

Rare

Hepatosplenomegaly, adenopathy

Neurologic involvement

Aseptic meningitis?

Controversial

Headache

Headache

Sensorineural deafness

Sensorineural deafness, chronic aseptic meningitis, intellectual disability, headache

Ophthalmologic involvement

Rare

Conjunctivitis, periorbital edema, rarely uveitis

Uncommon

Conjunctivitis

Conjunctivitis, episcleritis

Uveitis, conjunctivitis, progressive vision loss

Vasculitis

Henoch-Schönlein purpura (HSP), polyarteritis nodosa

HSP, lymphocytic vasculitis

Cutaneous vasculitis common, rarely HSP

Not seen

Not seen

Occasional

Systemic amyloidosis

Risk depends on MEFV and SAA genotypes; more common in Middle East

Occurs in ~10%; risk increased with cysteine mutations

Rare

Rare

Occurs in ~25%

May develop in some, usually in adulthood

FCAS = familial cold autoinflammatory syndrome; FMF = familial Mediterranean fever; HIDS = hyperimmunoglobulinemia D with periodic fever syndrome; MWS = Muckle-Wells syndrome; NOMID/CINCA = neonatal-onset multisystem inflammatory disease/chronic infantile neurologic cutaneous and articular syndrome; TRAPS = tumor necrosis factor receptor–associated periodic syndrome.

TREATMENT  The mainstay of therapy for FMF is daily oral colchicine, which can prevent both acute attacks of FMF and the development of systemic amyloidosis. Colchicine probably works by several mechanisms, including its effects on inhibiting leukocyte adhesion and modulating cytokine production. In adults, the therapeutic dose is 1.2 to 1.8 mg/day, and nearly 90% of patients note significant improvement at this dose. The major side effects are gastrointestinal, and they can usually be minimized by gradually increasing the dosage and avoiding milk products in patients who develop lactose intolerance. Most experts continue to prescribe colchicine to patients during pregnancy, with the recommendation that amniocentesis be performed to exclude trisomy 21, for which there may be a slightly increased risk. Use of colchicine in lactating women is considered safe. Intravenous colchicine should be used with extreme caution, if at all, in FMF, because fatal toxicity has been reported in patients already receiving oral colchicine who are given the drug intravenously. IL-1 inhibitors9 may be effective in patients who are poorly responsive to colchicine or who cannot tolerate therapeutic doses.

Tumor Necrosis Factor Receptor–Associated Periodic Syndrome

DEFINITION

Worldwide, the tumor necrosis factor (TNF) receptor–associated periodic syndrome (TRAPS) is the second most frequently diagnosed hereditary recurrent fever syndrome, behind FMF. TRAPS is defined by recurrent episodes of fever and localized inflammation, in many ways resembling FMF, but differing in key details (noted later) and caused by mutations in the 55-kD receptor for TNF (TNFRSF1A, TNFR1, p55, CD120a). Whereas a positive genetic test is not necessary to diagnose FMF, the diagnosis of TRAPS requires the identification of a TNF receptor mutation. One of the first wellcharacterized families with what was later defined as TRAPS was of Irish ancestry, and the condition was termed familial Hibernian fever to emphasize

the ethnic background and clinical differences from FMF. However, with the discovery of TNF receptor mutations in families of other ancestries, the ethnically neutral TRAPS nomenclature was proposed.

PATHOBIOLOGY

The p55 TNF receptor is composed of four cysteine-rich extracellular domains, a transmembrane region, and an intracellular death domain. To date, nearly all of the more than 90 mutations described are in the extracellular domains and approximately one third are missense substitutions of cysteine residues that abolish highly conserved disulfide bonds. The initial description of TRAPS documented a defect in activation-induced ectodomain cleavage of the p55 receptor in patients with the C52F TNFRSF1A mutation, possibly leading to a defect in homeostasis by impaired downregulation of membrane receptors and diminished shedding of potentially antagonistic soluble receptor molecules. More recent studies indicate a more complex pathogenetic picture, because not all mutant receptors exhibit this shedding defect. Additional mechanisms by which p55 mutations may lead to autoinflammation include impaired leukocyte apoptosis and impaired intracellular receptor trafficking, with possible constitutive activation of mitogen-activated protein (MAP) kinases by intracellular aggregates of mutant receptors.10

DIAGNOSIS

Although genetic testing is necessary for the diagnosis of TRAPS, certain clinical clues can help distinguish TRAPS from FMF. These include ethnicity (FMF is seen predominantly in Mediterranean and Middle Eastern populations, whereas TRAPS has a more widespread distribution), mode of inheritance (autosomal recessive in FMF, dominant in TRAPS), and duration of attacks, which tends to be longer in TRAPS and sometimes approaches continuous symptoms. The rash of FMF is typically erysipeloid erythema on the lower extremity, whereas patients with TRAPS often have a distinctive erythematous rash, often with underlying myalgia, which may migrate on the trunk or centrifugally on the extremities. Ocular involvement, with periorbital edema, conjunctivitis, and occasionally even uveitis, is observed in

1742

CHAPTER 261  The Systemic Autoinflammatory Diseases  

TRAPS but not in FMF. Finally, whereas colchicine is much more effective than corticosteroids in FMF, the opposite is true in TRAPS. Nevertheless, aside from the difference in duration and susceptibility to pharmacologic intervention, the abdominal, pleural, synovial, and even scrotal manifestations of the two diseases are rather similar. The usual age of onset for TRAPS is also in childhood, and systemic AA amyloidosis is seen in approximately 10% of untreated patients with TRAPS. As in FMF, life expectancy in TRAPS is normal in patients whose disease is not complicated by amyloidosis. As noted earlier, the diagnosis of TRAPS is established by the identification of TNFRSF1A mutations in the appropriate clinical setting. One variant, the substitution of glutamine for arginine at residue 92 (R92Q), is present in more than 1% of whites and may be associated with a broader spectrum of symptoms than is typically seen in TRAPS, including early inflammatory arthritis or, in some cases, no symptoms at all. The substitution of lysine for proline at residue 46 (P46L) is common among African American patients with TRAPS and is associated with a receptor shedding defect, but it is also seen among healthy African American controls. These findings establish a “gray zone” for the diagnosis of TRAPS and emphasize the potential role of polymorphisms in the recurrent fever genes in other more common phenotypes.

TREATMENT  The treatment of TRAPS depends on the frequency and severity of attacks. Patients with relatively infrequent, mild episodes may respond to nonsteroidal anti-inflammatory drugs (NSAIDs). Patients with more severe attacks that occur infrequently may be treated with corticosteroids, although increasing doses may be required as the episodes become more frequent and toxicities may become limiting. For patients with severe attacks occurring once a month or more frequently, treatment with etanercept, the soluble p75 TNF receptor:Fc fusion protein, may be warranted. This may be a unique effect of etanercept, because there is anecdotal evidence that monoclonal antibodies against TNF may actually exacerbate TRAPS. Consistent with a model implicating multiple cytokines in the pathogenesis of TRAPS, IL-1 inhibitors have also been found effective in TRAPS.11

  INTERLEUKIN-1–ASSOCIATED AUTOINFLAMMATORY DISEASES

The IL-1–associated autoinflammatory diseases are linked by markedly increased expression or cellular responsiveness to this cytokine, and dramatic resolution of symptoms with IL-1 blockade. Interleukin-1α and Interleukin1β (IL-1α and IL-1β) are structurally related cytokines released from cells triggered by a number of inflammatory stimuli, such as lipopolysaccharide. They mediate inflammatory responses by binding to a common receptor that is present on the surface of a wide variety of cell types and signals to activate inflammatory genes through the nuclear factor kappa B (NF-KB) transcription factor complex. IL-1 is part of a larger family of cytokines including IL-18, IL-33, and IL-36, which bind to related receptors and share the property of not having a characteristic signal peptide that normally targets cytokines to secretory vesicles. Because of this, IL-1 family cytokines may be secreted only by dead or dying cells, functioning as molecular markers of cellular stress, which can trigger beneficial inflammatory responses to infection and injury. IL-1β and IL-18 are unique in that they are not biologically active until cleaved by the protease caspase-1. Caspase-1 is itself activated in cytoplasmic protein complexes containing various sensor proteins such as NLRP3, and the adapter protein ASC. These complexes are referred to as inflammasomes because of their ability to trigger IL-1–mediated inflammation. Autoinflammatory diseases caused by mutations in genes encoding proteins that process or sense IL-1 are described later.

Cryopyrin-Associated Periodic Syndromes: The Cryopyrinopathies Three rare, recurrent febrile disorders usually beginning early in life have been associated with mutations in NLRP3 (formerly CIAS1), the gene encoding a protein variously named cryopyrin, NLRP3, NALP3, PYPAF1, or CATERPILLER 1.1, a key component of the NLRP3 inflammasome that activates caspase-1. These disorders are referred to as cryopyrinopathies or cryopyrinassociated periodic syndromes (CAPS). The least severe clinical phenotype is familial cold autoinflammatory syndrome (FCAS; formerly called familial cold urticaria), which is dominantly inherited and is notable for day-long attacks of chills, fever, headache, diffuse urticarial skin rash, arthralgia, and

conjunctivitis, precipitated by generalized cold exposure. Amyloidosis is rare in FCAS. Of intermediate severity is Muckle-Wells syndrome (MWS), also dominantly inherited, in which 1- to 2-day episodes of chills, fever, urticarial rash, limb pain, and arthritis occur independently of cold exposure. Sensorineural hearing loss is common in MWS, and systemic amyloidosis also may occur. The most severe NLRP3-associated phenotype is neonatal-onset multisystem inflammatory disease (NOMID), known in Europe as chronic infantile neurologic cutaneous and articular (CINCA) syndrome. It is usually sporadic owing to the reduced reproductive fitness of most affected individuals. Fever and constitutional symptoms occur almost daily, often from birth, with generalized urticarial skin rash, a peculiar arthropathy characterized by epiphyseal overgrowth of the long bones, and central nervous system (CNS) involvement that includes chronic aseptic meningitis, uveitis, and cochlear inflammation, which may lead to intellectual disability, blindness, and deafness. In all three cryopyrinopathies, the rash is not true urticaria because there is a neutrophilic rather than a mast cell infiltrate and serum histamine levels are normal. The protein mutated in all three disorders is NLRP3, a critical component of the eponymous NLRP3 inflammasome, which serves as an intracellular scaffold for the processing of IL-1β. The alternative name of this protein, cryopyrin, refers to its aminoterminal PYRIN domain, the basis for a structural and functional relationship to the protein mutated in FMF. Diseaseassociated cryopyrin mutations are thought to decrease the threshold for inflammasome activation, thereby increasing IL-1β production. The discovery that the NLRP3 inflammasome is also necessary for IL-1β production in response to crystalline forms of monosodium urate and calcium pyrophosphate connected the pathophysiology of these rare autoinflammatory diseases to crystal-induced arthritis (Chapter 273), which shares some clinical features, such as episodic, self-limited attacks, with autoinflammatory diseases. Because there are patients with FCAS, MWS, and NOMID/CINCA without demonstrable NLRP3 mutations, these diagnoses remain clinical, although genetic testing serves as a valuable adjunct and has greatly increased the recognition of all three conditions. Deep sequencing has identified somatic NLRP3 mutations in some patients with symptoms consistent with CAPS who are negative for mutations by standard genetic testing. In addition, overlap syndromes that are intermediate between FCAS and MWS and between MWS and NOMID/CINCA have been reported.

TREATMENT  Blockade with anakinra, a recombinant IL-1 receptor antagonist, is effective in controlling fever and acute phase reactants in all three cryopyrinopathies, and longitudinal analysis of a large series of patients at the National Institutes of Health showed that long-term treatment with anakinra markedly decreased CNS inflammation and end-organ damage in NOMID/CINCA, which led to the regulatory approval of anakinra for the treatment of this condition in the United States and Europe. Recent studies have also documented the efficacy of rilonacept, another soluble IL-1 blocker, and canakinumab, a monoclonal antibody against IL-1β, in FCAS and MWS, although these agents may be less effective against NOMID/CINCA because of reduced penetration into the CNS. The efficacy of canakinumab suggests that the major biologic effect of cryopyrin in humans is mediated through IL-1β rather than by IL-1α or other distinct inflammatory pathways.

Deficiency of Interleukin-1 Receptor Antagonist Deficiency of the IL-1 receptor antagonist (DIRA) is characterized by the neonatal onset of a pustular skin rash, multifocal osteomyelitis, periostitis, and, rarely, vasculitis.12 Fever is not a prominent finding, although acute phase reactants are markedly elevated. DIRA is caused by recessively inherited lossof-function mutations in IL1RN, which encodes the IL-1 receptor antagonist (IL-1Ra). Patients usually present within the first 2 weeks of life with skin lesions ranging from discrete crops of pustules to generalized severe pustulosis or ichthyosiform lesions. Histologic examination demonstrates extensive neutrophilic infiltrates in the dermis and epidermis. Typical radiographic findings include multifocal osteolytic lesions, periosteal elevation of the long bones, heterotopic ossification of the proximal femurs, and widening of the anterior rib ends. Bone biopsies demonstrate sterile purulent osteomyelitis, fibrosis, and sclerosis. To date, five different IL1RN mutations have been identified, three of which are truncating point mutations that drastically reduce IL-1Ra messenger RNA and protein levels. The fourth is a 15bp in-frame deletion, and the fifth is a 175-kilobase genomic deletion in

CHAPTER 261  The Systemic Autoinflammatory Diseases  

chromosome 2q that subsumes IL1RN and five other genes in the IL-1 family. In DIRA, the lack of IL-1Ra leads to unopposed IL-1β and IL-1α signaling, whereas in the cryopyrinopathies, NLRP3 mutations lead to inflammasome activation and increased IL-1β production. DIRA patients respond dramatically to anakinra, a recombinant form of the protein they lack.

  OTHER INHERITED SYSTEMIC AUTOINFLAMMATORY DISEASES

Syndrome of Pyogenic Arthritis with Pyoderma Gangrenosum and Acne The syndrome of pyogenic arthritis with pyoderma gangrenosum and acne (PAPA) is a rare, dominantly inherited autoinflammatory disease charac­ terized by intermittent episodes of sterile pyogenic arthritis, pyoderma gangrenosum, and severe cystic acne. It is caused by mutations in prolineserine-threonine phosphatase–interacting protein 1 (PSTPIP1), also known as CD2BP1. PSTPIP1 is a cytoskeletal protein that interacts with certain other proteins involved in the immune response, including CD2; the WiskottAldrich syndrome protein (WASP); a phosphatase denoted PTP-PEST; and pyrin, the FMF protein. PAPA mutations abrogate the binding of PSTPIP1 to PTP-PEST, leading to hyperphosphorylation of PSTPIP1 and increased binding to pyrin. Both in patients and in cell lines, this finding is associated with markedly increased IL-1β production. Early in life, PAPA tends to present with monoarticular or pauciarticular pyogenic arthritis, sometimes induced by trauma. In the absence of treatment, arthritis may progress to severe joint damage and ankylosis. As patients reach puberty, skin manifestations begin to predominate, including disfiguring cystic acne. Pathergy also may develop, and extensive pyoderma gangrenosum may require opiates for pain control. The diagnosis of PAPA syndrome is made by documenting PSTPIP1 mutations in the appropriate clinical setting. High doses of corticosteroids have been used in PAPA, with varying success, and patients with arthritis sometimes require aspiration, intra-articular corticosteroids, or open drainage. Newer investigational approaches for PAPA syndrome focus on the use of targeted cytokine inhibitors. Anecdotal evidence supports the use of anakinra for the arthritis and monoclonal anti-TNF antibodies for the pyoderma gangrenosum of PAPA.

Granulomatous Inflammatory Arthritis, Dermatitis, and Uveitis (Blau’s Syndrome) Blau’s syndrome is a rare, dominantly inherited illness characterized by the following features: early-onset granulomatous synovitis often complicated by cyst formation and camptodactyly (flexion contractures of the fingers and toes); granulomatous anterior and posterior uveitis, sometimes causing retinal detachment, glaucoma, cataracts, and blindness; and an intermittent papular rash with noncaseating granulomas. Lung or other visceral involvement is generally not present. However, visceral involvement of the liver and spleen is observed in early-onset sarcoidosis (Chapter 95), which is phenotypically quite similar to Blau’s syndrome. Both Blau’s syndrome and some cases of early-onset sarcoidosis are caused by mutations in NOD2/CARD15. Distinct variants of NOD2/CARD15 have been associated with susceptibility to Crohn’s disease, which manifests as granulomatous inflammation of the gastrointestinal tract (Chapter 141). The protein encoded by this gene is thought to be an intracellular sensor of bacterial products. Crohn’s disease— associated mutations in the ligand-binding, leucine-rich repeat region of the protein may alter responses to bacterial products in the gastrointestinal tract to cause inflammation, whereas Blau’s syndrome mutations in the nucleotide binding domain may lead to constitutive extraintestinal inflammation. Topical and systemic corticosteroids are currently the mainstay of treatment of Blau’s syndrome. There are case reports of the efficacy of TNF and IL-1 inhibitors in this disease.

Hyperimmunoglobulinemia D with Periodic Fever Syndrome Hyperimmunoglobulinemia D with periodic fever syndrome (HIDS) was first described in 1984 as an FMF-like illness seen in six patients of Dutch ancestry. Besides the difference in ethnicity, a key distinction was the observation of extremely high levels of immunoglobulin D (IgD) in the serum of these patients, thus prompting the HIDS nomenclature. HIDS is now recognized in a broader ethnic distribution, although northern Europeans still predominate. Overall, HIDS is still quite rare. Family studies documented autosomal recessive inheritance. In 1999, patients with HIDS were found to have mutations in MVK, which encodes the mevalonate kinase enzyme

1743

involved in the biosynthesis of cholesterol and nonsterol isoprenes. Enzyme activity in patients is markedly reduced, but not absent. The elevated immunoglobulin D (IgD) levels seen in HIDS appear to be an epiphenomenon and do not correlate with disease severity either among patients or in a given patient over time, although IgD may contribute to the release of proinflammatory cytokines in vitro. Moreover, modest elevations of IgD are seen in several inflammatory conditions, including chronic infections, and can be observed in other hereditary recurrent fever syndromes. Up to 20% of patients (particularly young children) with typical recurrent fevers and MVK mutations have normal serum IgD levels. Current data suggest that isoprenoid deficiency may play a more important pathogenic role in the pathophysiology of HIDS. In vitro studies suggest that isoprenoid deficiency may lead to excessive IL-1β production, and increased body temperature can further decrease mevalonate kinase enzymatic activity, thereby creating a vicious circle in which infection or immunization can precipitate HIDS attacks. One of the well-recognized clinical characteristics of HIDS is the provocation of attacks by immunizations. Other distinguishing clinical features include a very early age of onset (average age, 6 months), a duration of attacks intermediate between FMF and TRAPS (3 to 7 days), prominent cervical lymphadenopathy during attacks, polyarticular joint involvement, a diffuse maculopapular rash, the predominance of diarrhea over constipation with abdominal attacks, and the infrequency of pleuritic attacks or systemic amyloidosis. The diagnosis of HIDS can be established in a patient with recurrent episodes of fever and typical associated findings by documenting either two mutations in MVK or elevated levels of mevalonic acid, the substrate for mevalonate kinase, in the urine during attacks. Approximately 10% of patients with otherwise typical disease have only a single identifiable MVK mutation. The significance of elevated IgD without genetic or biochemical findings remains unknown. NSAIDs or corticosteroids are sometimes useful in the treatment of the arthritic manifestations of HIDS. Colchicine is generally not effective. Numerous agents are investigational in HIDS, including the statins, TNF inhibitors, and IL-1 inhibitors. Patients with HIDS have a normal lifespan, and attacks may become somewhat less frequent in adulthood.

Proteasome-Associated Systemic Inflammatory Diseases Recently, a constellation of diseases have been described linked to recessive loss of function mutations in PSMB8, which encodes the β5i subunit of the proteasome, also known as LMP7. An autosomal recessive syndrome in adults characterized by recurrent fevers, progressive lipodystrophy, joint contractures, and cardiac manifestations was linked to homozygous missense mutations in PSMB8.13 Patients with a pediatric syndrome termed CANDLE (chronic atypical neutrophilic dermatosis with lipodystrophy and elevated temperature) were found to have homozygous missense and nonsense mutations in PSMB8, with some patients having only one known PSMB8 mutation. It is not yet clear whether these syndromes represent identical diseases related to loss of function of β5i. The β5i proteasome subunit is one of the subunits that that are induced in immune cells through immune stimuli such as interferons, altering the proteasome so that it more efficiently processes peptides for antigen presentation to T cells. However, there is no indication of a T cell component to this disease, and studies have shown that the β5i proteasome subunit can be expressed in nonimmune cells such as adipocytes. A striking interferon transcriptional signature, similar to that seen in systemic lupus erythematosus, was observed in circulating blood cells from patients with CANDLE. Defective degradation of proteins in cells lacking β5i may result in buildup of ubiquitinated proteins, which somehow triggers interferon production, or PSMB8 deficiency may enhance interferon signaling by stabilizing components of the interferon signal transduction machinery that are negatively regulated by ubiquitin-proteasome degradation. Whichever the mechanisms, the link to interferon hyperactivity suggests that blocking interferons with antibodies or inhibitors of interferon signal transcution may be effective in the therapy of CANDLE and possibly other PSMB8-associated syndromes.

New Autoinflammatory Syndromes and the Promise of Whole-Exome Sequencing Recent years have seen a dramatic acceleration in the pace of discovery of new mendelian inflammatory diseases as a result of the availability of wholeexome sequencing, which allows unbiased identification of disease-causing mutations in protein coding sequences, although it should be noted that accurate clinical description of these syndromes is as important as the genetic tools for identification of new syndromes. These discoveries have confirmed

1744

CHAPTER 262  Osteoarthritis  

the role of gene products in human inflammation that were identified in other animal model systems and identified new genes and proteins not previously thought to be involved in the regulation of inflammation. Early-onset, apparently sporadic, cases of inflammatory syndromes have often turned out to be due to de novo mutations in a child when screened against parental DNA. For example, inherited gain-of-function mutations in CARD14, encoding an adapter protein in innate immune sensing, cause dominantly inherited familial psoriasis, and a more severe gain-of-function de novo mutation in the same gene caused infantile-onset severe pustular psoriasis. Recessive mutations causing systemic autoinflammatory disease also have been identified by whole-exome sequencing from just a few families. Recent examples of novel diseases discovered through these methods include a syndrome characterized by fevers, early-onset strokes, and vasculopathy or frank vasculitis caused by autosomal recessive mutations in CECR1, encoding adenosine deaminase 2 (ADA2), a serum protein with newly recognized effects on macrophage differentiation and vascular development.14 Recessive mutations in HOIL1 that impair the addition of linear ubiquitin chains to receptor signaling complexes cause a complex syndrome marked by autoinflammation and immunodeficiency and intramuscular glycogen deposition. Gain-of-function mutations in PLCG2, encoding phospholipase Cγ2, an enzyme with essential functions in B-cell receptor and Fc Rceptor signaling, cause a dominantly inherited autoinflammatory syndrome characterized by blistering skin lesions, bronchiolitis, arthralgia, ocular inflammation, and enterocolitis in the absence of autoantibodies. GENERAL REFERENCES For the General References and other additional features, please visit Expert Consult at https://expertconsult.inkling.com.

262  OSTEOARTHRITIS JOEL A. BLOCK AND CARLA SCANZELLO

DEFINITION

Osteoarthritis (OA) is a heterogeneous disease that has many names, including degenerative joint disease and osteoarthrosis. It is a joint disease characterized clinically by pain and functional loss. Although local inflammation of involved joints is common, OA is not associated with a systemic inflammatory process, in contrast to other arthritides. It is the most common form of arthritis and accounts for the overwhelming majority of arthritis cases, and its prevalence is expected to rise dramatically during the next 20 years as global populations age. OA is often neglected either because it is not a fatal disease or because many physicians assume it is a normal part of aging and is not inherently treatable. Yet it results in vast direct medical costs and significant loss of work; it is the leading indication for total joint replacement and is a leading cause of work disability. Formal definitions of OA have evolved as our understanding of pathophysiology has progressed. Whereas it conventionally had been considered primarily a degenerative process of cartilage, it is now clear that OA involves the entire joint. Thus, a modern definition of OA is a painful degenerative process involving progressive deterioration of all joint structures and remodeling of subchondral bone that is not primarily inflammatory. It is important to distinguish true OA from asymptomatic structural degeneration of joints that is virtually universal during normal aging.

EPIDEMIOLOGY

As an age-related disease, OA prevalence has risen substantially with the aging population of the developed world; an estimated 27 million people had physician-diagnosed OA in the United States in 2005, increased from 21 million a decade earlier, and this number is expected to reach 67 million patients with clinically significant OA by 2030. For epidemiologic studies, OA is often defined radiographically by the presence of osteophytes, joint space narrowing, and subchondral sclerosis. However, a substantial number

of individuals have these x-ray changes but remain clinically asymptomatic. Thus, estimates of OA prevalence vary widely depending on whether one is assessing radiographic or symptomatic OA. In either case, the lifetime risk is exceedingly high. There is general concordance among epidemiologic studies across North American, Asian, and European populations that the prevalence of radiographic OA in the knees, hips, and hands is quite low before age 45 and increases dramatically with aging, with most people having x-ray evidence of OA in at least one joint by the seventh decade. Symptomatic knee OA affects between 7% and 17% of those older than 44 years, with rates increasing with age; women have higher prevalence than men, and African Americans have higher prevalence than white Americans. Symptomatic hip OA is less prevalent than knee OA, with overall rates between 6.7% and 9.7% among those over age 44; as with knee OA, prevalence is higher among the elderly, women, and African Americans.1 Symptomatic hand OA affects at least 6.8% of those older than 25, occurring in women more than two-fold more frequently than in men. Hand OA may be less common in African Americans than in white Americans. Risk factors for the development of OA and for specific joint involvement have been extensively studied. Among nonmodifiable risk factors, the strongest is aging. This is true both for radiographic changes of OA and for symptomatic involvement. In addition, female sex is a risk for prevalence and severity of OA, especially after menopause. There is a significant heritable component, particularly for hand OA and hip OA. This component is estimated at 48 to 65% for so-called generalized OA characterized by osteophytes of the distal interphalangeal joints (Heberden nodes) or the proximal interphalangeal joints (Bouchard’s nodes). Modifiable OA risk factors may provide clues for preventive strategies. The most important of these is obesity, which alone confers an approximately three-fold increased risk for incident OA. Occupational and lifestyle activities that involve repeated trauma or excessive loading may be associated with increased risk for OA. These include chronic squatting, bending, and lifting such as by warehouse workers and laborers, who have increased knee involvement, and, classically, pneumatic drill operators who develop OA of the wrist and elbow. Significant trauma, such as major knee or ankle injury, is strongly associated with subsequent development of OA in the injured joint. Aberrant loading of joints is an important risk factor for the development and progression of OA. For example, excessive loading of the knee has been observed to result in significantly increased risk for progression to advanced knee OA. In addition, joint alignment is an important parameter in OA, and malalignment at the knee is among the strongest predictors of OA progression.2

PATHOBIOLOGY

Although degeneration of articular cartilage is a central common pathway in OA, multiple joint and periarticular tissues are compromised and contribute to clinical manifestations. Pathologic changes in synovium, ligaments, supporting musculature, and fibrocartilagenous structures such as the menisci in the knee are common.3 Unlike autoimmune arthritides, OA does not affect extra-articular organs. However, the chronic pain of OA involves both the peripheral and central nervous system (CNS) and OA-related disability degrades the general physical and mental health of the patient. Appreciation of the global effects of OA has important implications for current and future treatment approaches.

Tissues Central to the Osteoarthritis Process Cartilage

The hallmark of osteoarthritis is progressive deterioration of articular cartilage. Normal articular cartilage distributes loads across joint surfaces and allows for almost frictionless joint motion. These functions are furnished by the extracellular matrix, which accounts for more than 90% of the tissue volume and is organized by a network of collagen type II fibers, which provides tensile strength, entrapping aggrecan complexes, a proteoglycan that confers compressive stiffness and resilience (Fig. 262-1A and C). There is one major cell type, the chondrocyte, that synthesizes these matrix components. In mature cartilage, turnover of extracellular matrix molecules, particularly collagen type II, is slow. Cartilage change in OA begins with swelling of the matrix, then progresses through stages of surface roughening, fibrillation, fissuring, and eventually full-thickness erosion. These are accompanied by activation of chondrocytes to increase synthesis of proteolytic enzymes that degrade matrix.4 Matrix metalloproteinase-13 (MMP-13; collagenase-3) plays a central role in collagen type II degradation, while ADAMTS-4 and -5 (a disintegrin and metalloproteinase with thrombospondin motifs-4 and -5) proteases are important

CHAPTER 261  The Systemic Autoinflammatory Diseases  

GENERAL REFERENCES 1. Broderick L, De Nardo D, Franklin BS, et al. The inflammasomes and autoinflammatory syndromes. Annu Rev Pathol. 2015;10:395-424. 2. Henderson C, Goldbach-Mansky R. Monogenic autoinflammatory diseases: new insights into clinical aspects and pathogenesis. Curr Opin Rheumatol. 2010;22:567-578. 3. Jacobs Z, Ciaccio CE. Periodic fever syndromes. Curr Allergy Asthma Rep. 2010;10:393-404. 4. Kastner DL, Aksentijevich I, Goldbach-Mansky R. Autoinflammatory disease reloaded: a clinical perspective. Cell. 2010;140:784-790. 5. Wurster VM, Carlucci JG, Edwards KM. Periodic fever syndromes. Pediatr Ann. 2011;40:48-54. 6. Infevers database. A compendium of mutations associated with known autoinflammatory syndromes with links to relevant publications. http://fmf.igh.cnrs.fr/ISSAID/infevers/. Accessed January 20, 2015. 7. Berkun Y, Eisenstein EM. Diagnostic criteria of familial Mediterranean fever. Autoimmun Rev. 2014;13:388-390. 8. Gerfaud-Valentin M, Jamilloux Y, Iwaz J, et al. Adult-onset Still’s disease. Autoimmun Rev. 2014;13: 708-722.

1744.e1

9. Jesus AA, Goldbach-Mansky R. IL-1 blockade in autoimflammatory syndromes. Annu Rev Med. 2014;65:223-244. 10. Bachetti T, Ceccherini I. Tumor necrosis factor receptor-associated periodic syndrome as a model linking autophagy and inflammation in protein aggregation diseases. J Mol Med (Berl). 2014;92: 583-594. 11. Ter Haar NM, Frenkel J. Treatment of hereditary autoinflammatory diseases. Curr Opin Rheumatol. 2014;26:252-258. 12. Aksentijevich I, Masters SL, Ferguson PJ, et al. An autoinflammatory disease with deficiency of the interleukin-1-receptor antagonist. N Engl J Med. 2009;360:2426-2437. 13. Liu Y, Ramot Y, Torrelo A, et al. Mutations in proteasome subunit beta type 8 cause chronic atypical neutrophilic dermatosis with lipodystrophy and elevated temperature with evidence of genetic and phenotypic heterogeneity. Arthritis Rheum. 2012;64:895-907. 14. Zhou Q, Yang D, Ombrello A, et al. Early-onset stroke and vasculopathy associated with mutations in ADA2. N Engl J Med. 2014;370:907-916.

1744.e2

CHAPTER 261  The Systemic Autoinflammatory Diseases  

REVIEW QUESTIONS 1. Which of the following is not commonly associated with the development of AA amyloidosis? A. Hyper-IgD syndrome (HIDS) B. Familial Mediterranean fever (FMF) C. Tumor necrosis factor receptor–associated periodic syndrome (TRAPS) D. Muckle-Wells syndrome Answer: A  All of the listed periodic fever syndromes are commonly associated with the development of AA amyloidosis with the exception of hyperIgD syndrome, which generally has a benign course. 2. Patients with which monogenic autoinflammatory disease often have a history of flaring after routine immunizations? A. FMF B. HIDS C. TRAPS D. FCAS Answer: B  Of these syndromes, only HIDS has a substantiated association of flares with routine immunizations. Psychological stress is often cited as a trigger for flares in FMF and TRAPS, and cold exposure triggers a systemic inflammatory flare and hivelike skin lesions in FCAS. 3. A 30-year-old woman of Armenian and Jewish ancestry presents with a life-long history of unexplained febrile episodes. These attacks are variable in length, with the shortest lasting a few days and the longest lasting over 1 month. The fevers are accompanied by severe abdominal pain or pleuritic chest pain, periorbital edema, arthralgia, and a painful migratory erythematous rash. Corticosteroids ameliorate her symptoms. During pregnancy 7 years ago she was totally free of fevers, but she developed a severe attack in the postpartum period. She is currently not experiencing an attack, but has an erythrocyte sedimentation rate of 85 (Westergren), C-reactive protein of 100 mg/L, urine protein-to-creatinine ratio of 5.3, and serum creatinine of 2.5. Which of the following is most likely true? A. The patient probably has familial Mediterranean fever, and should undergo genetic testing for MEFV mutations, have a rectal biopsy to rule out amyloidosis, and commence treatment with intravenous colchicine. B. The patient has chronic atypical neutrophilic dermatosis with lipo­ dystropy and elevated temperature (CANDLE syndrome), with the T75M mutation in PSMB8, encoding a component of the immunoproteasome. C. The patient has a mutation at a cysteine residue in the extracellular domain of the p55 TNF receptor and possible amyloidosis. She should undergo a rectal biopsy and should be treated with etanercept or anakinra in an effort to normalize her acute phase reactants. D. The patient has a mutation in the NLRP3 (CIAS1) gene, and could be treated with anakinra, rilonacept, or canakinumab for neonatal-onset multisystem inflammatory disease (NOMID). Answer: C  Despite the demographic information that may point toward FMF, the presence of longer disease flares of longer than 7 days, periorbital edema, painful migratory rash, and remission of symptoms with pregnancy are all characteristic of the TNFR1-associated periodic syndrome (TRAPS). Patients with TRAPS and structure-disrupting mutations in TNFR1 have an elevated risk for amyloidosis. TRAPS does not have a predilection for a specific ethnic group or geographic location.

4. A 3-year-old boy of northern European ancestry presents to the autoinflammatory disease clinic for an initial evaluation. Per his mother’s report, he began experiencing febrile attacks at the age of 3 months that last 4 to 5 days and occur every 1 to 2 months. Associated symptoms include a nonpruritic macular rash and oral ulcers, and on three occasions he has had genital ulcers. He tends to have flares approximately 2 days after immunizations and after routine viral illnesses. There is no family history of febrile illnesses. He is treated with ibuprofen and acetaminophen and has been given a couple of courses of prednisolone without complete resolution of symptoms. He had been well before his visit, but his mother thinks he is starting to have a flare. On physical examination, he has conjunctival injection, cervical lymphadenopathy but no rashes, oral or genital ulcers, or arthritis. Laboratory studies reveal an ESR of 57 mm/ hour, CRP of 55 mg/L, and IgD level at the top of the normal range. Which of the following is most likely true? A. The patient probably has periodic fever with aphthous stomatitis, pharyngitis, and cervical adenitis (PFAPA). He should be given either 1 mg/kg of prednisolone at the onset of symptoms or anakinra 2 mg/ kg SC for 1 to 2 days at the onset of symptoms. B. The patient has Behçet’s disease and should consider colchicine for his oral ulcers. He should undergo HLA typing to assess for the presence of HLA-B51 or other Behçet’s-associated MHC findings. C. The patient has hyperimmunoglobulinemia D syndrome and should initiate therapy with anakinra at the time of flares or weekly etanercept. He should be tested for mutations in the MVK gene. D. The patient has cyclic neutropenia and should be tested for mutations in the ELA-2 gene. He should also have weekly CBC drawn to assess for periodic neutropenia. Answer: C  Despite the oral and genital ulcers that might suggest Behçet’s disease, the history of flares after vaccinations or viral illness, the pattern and age of onset, and the ethnic background more strongly suggest that the patient has hyperimmunoglobulinemia D syndrome. IgD levels can be normal in this disease, and a normal IgD should not rule out this diagnosis, which can be confirmed by finding a heterozygous mutation in the mevalonate kinase gene. 5. A 16-year-old high school student is evaluated in the emergency department for 18 hours of abdominal pain, pleurisy, and fever. At that time, he had diffuse rebound tenderness, temperature of 102.5° F (39.2°degrees C), and WBC of 17,000/mm3. He was admitted to general surgery and after 12 hours of persistent symptoms, he was taken to the operating room for exploratory surgery. A normal-appearing appendix was laparascopically removed. Postoperatively, the fever persisted and he was noted to have a left pleural effusion. Additional history reveals that he has had similar episodes of febrile illness in the past. On physical examination, he has no rash but has moderate abdominal tenderness with guarding and a left pleural friction rub. Laboratory data reveal an ESR of 80 mm/hour and a negative ANA and RF. While waiting for the results of genetic testing, which of the following medications is most appropriate to start at this time? A. Indomethacin, 50 mg orally three times daily B. Prednisone, 40 mg orally daily C. Colchicine, 0.6 mg orally twice daily D. Anakinra, 100 mg subcutaneously daily E. Acetylsalicylic acid, 1000 mg orally daily Answer: C  The duration of fevers, association with pleural and peritoneal inflammation, and lack of other localizing symptoms are most consistent in this case with familial Mediterranean fever. Colchicine is effective in the prophylaxis of future attacks in FMF. Indomethacin and prednisone can be of some value in terminating attacks but should be reserved for refractory cases. Anakinra can be effective in the treatment of refractory FMF, but there is no need to use this agent in patients who respond to colchicine.

CHAPTER 262  Osteoarthritis  

Normal Joint

1745

Osteoarthritic joint

Synovial membrane hyperplasia, inflammation, fibrosis

Synovial membrane

Cartilage

Leukocyte

Osteophyte Collagen fibrils

Subchondral bone

Proteoglycan complexes

A

Osteophyte

Roughening and loss of cartilage matrix

Cartilage

B

Thickened subchondral bone

Normal

Osteoarthritis

Synovial membrane

Synovial membrane

Cartilage Cartilage

Bone

Bone

C

D

FIGURE 262-1.  Pathologic features of osteoarthritic joint tissues. A, Features of a normal adult synovial joint. Healthy adult articular cartilage is characterized by a smooth surface and extracellular matrix (ECM) composed of a collagen type II fibrillar network and large proteoglycan complexes. The ECM is produced and maintained by the cellular components of cartilage, chondrocytes. The subchondral bone consists of a thin cortical layer and underlying trabecular bone. The synovial membrane lines the joint capsule and attaches at the cartilage-bone interface. In the normal state, it consists of a lining layer 1 or 2 cells thick, with underlying vascularized loose connective tissue. B, Typical changes to tissues seen in osteoarthritis (OA). Enzymatic activities (ADAMTS-4,5 and MMP-13 in particular) cleave proteoglycan and collagen components of the ECM, leading to loss of these molecules  from the matrix. As the process advances, the articular cartilage thins and fibrillates and eventually fissures down to the underlying bone are seen. Simultaneously, a remodeling response in the bone is observed. Thickening of the cortical subchondral bone layer occurs, and new bone growth at the margins appears as osteophytes. The synovial membrane changes observed in OA patients include lining layer hyperplasia, inflammation in the form of leukocyte infiltration, and fibrosis which can be seen to varying degrees. Photomicrographs of human joint tissues showing these features are depicted in C (normal tissues) and D (OA tissues). (C and D courtesy of Edward F. DiCarlo, MD. Hospital for Special Surgery, New York, NY).

for loss of aggrecan, but other enzymes participate (see Fig. 262-1B and D). Concomitantly, the activated chondrocytes proliferate to form clonal clusters and produce inflammatory mediators, including interleukin-1α and β, (IL1α and IL-1β), IL-6, tumor necrosis factor-α (TNF-α) and nitric oxide (NO), which accelerate the degradative cycle and stimulate chondrocyte apoptosis. Thus, both cellular and molecular components of cartilage are lost as the process progresses.5

chanical loading. This may result in thinning (attrition) and reduced bone density, leading to subchondral cyst formation in early disease, but progresses to subchondral sclerosis as bone formation outpaces resorption (see Fig. 262-1B and D).6 Remodeling at joint margins and entheses results in osteophytes (bone spurs). An important role for the growth factors transforming growth factor-β (TGF-β) and bone morphogenetic protein-2 (BMP-2) in driving osteophyte formation has been demonstrated in animal models.

Bone

Synovium

The cortical bone underlying articular cartilage (subchondral bone) supports load-bearing and transmits mechanical signals to articular chondrocytes. In OA, there is increased remodeling, likely in response to abnormal biome-

Synovial involvement in OA is more variable than in rheumatoid arthritis, but low-grade synovitis, characterized by infiltration of macrophages and lymphocytes, increased vascularity, synovial lining hyperplasia, and fibrosis,

1746

CHAPTER 262  Osteoarthritis  

Pathogenic factors

Tissue reaction

Altered loading • obesity • joint injury/trauma • malalignment • overuse

Cartilage Chondrocyte activation • proteases • cytokines

Altered matrix • aging • metabolic derangement • genetic defects

Bone Aberrant bone remodeling • growth factors • cytokines

Secondary factors Inflammation • synovitis • adipok